Sei sulla pagina 1di 847

2009 PREP SA on CD-ROM

Question: 1
You are evaluating a 6-month-old child who has a ventricular septal defect and is scheduled for cardiac
surgery. The child’s weight is 6 kg (3rd percentile), length is at the 30th percentile, and head
circumference is at the 50th percentile. His mother states she prepares the formula by adding 1 scoop
of powder to 2 oz of water. She estimates that he drinks 24 oz of formula per day. You estimate the
baby’s intake is approximately 500 kcal per day of cow milk formula, which is the recommended dietary
allowance (RDA) for his age. According to his mother, he spits up three times a day and passes two
soft stools daily. On physical examination, you hear a 3/6 holosystolic murmur and palpate the liver 1
cm below the right costal margin.

Of the following, the BEST explanation for the child’s malnutrition is

A. caloric requirements exceeding the RDA

B. cow milk protein intolerance

C. incorrect preparation of the formula

D. pathologic gastroesophageal reflux

E. undiagnosed pancreatic insufficiency

page 1
2009 PREP SA on CD-ROM

Critique: 1 Preferred Response: A


Children who have large ventricular septal defects, such as described for the child in the vignette, have
increased pulmonary blood flow and may have ventricular hypertrophy and heart failure. Because their
hearts have to work harder, their caloric needs often are increased above the recommended dietary
allowance (RDA) for healthy children. The absence of significant vomiting, diarrhea, or rectal bleeding
suggests that the patient does not have significant gastroesophageal reflux, cow milk protein
intolerance, or pancreatic insufficiency. Incorrect preparation of the formula always should be
considered, but the mother's reported preparation is correct.
The RDA is defined as "a nutrient intake level that is ... sufficient to meet the nutrient requirements of
97% of healthy individuals" in a given group, categorized by sex and age. RDAs are developed by the
Food and Nutrition Board of the Institute of Medicine in collaboration with Health Canada. A full list of
RDAs may be found in many reference textbooks and at the Food and Nutrition Information Center on
the United States Department of Agriculture web site (http://fnic.nal.usda.gov). RDAs estimate energy
requirements of healthy children and do not adjust for chronic illness states that increase caloric needs,
such as cystic fibrosis or congenital heart disease. Children who have chronic illnesses often require
more calories than the RDA to grow and develop.
The estimated daily caloric requirement of a patient is the aggregate of the patient's basal metabolic
rate and physical activity. Various mathematic equations can be used to estimate the daily caloric
intake of children and adults of different ages. Such equations usually take into account the individual's
age, sex, physical activity level, and either the weight and height or, preferably, the body surface area.
For children who have chronic illnesses, more accurate estimates of daily caloric requirements can be
obtained by using a laboratory-based technique, such as indirect calorimetry.

References:

McDaniel NL. Ventricular and atrial septal defects. Pediatr Rev. 2001;22:265-270. Available at:
http://pedsinreview.aappublications.org/cgi/content/full/22/8/265

Sonneville K. Nutritional requirements: dietary reference intakes. In: Hendricks KM, Duggan C. Manual
of Pediatric Nutrition. 4th ed. Hamilton, Ontario, Canada: BC Decker; 2005:83-100

page 2
2009 PREP SA on CD-ROM

Question: 2
You are admitting a 750-g female infant to the neonatal intensive care unit (NICU) for treatment of
respiratory distress and presumed sepsis. The pregnancy was complicated by chorioamnionitis and
preterm labor. The infant’s trachea was intubated, a single dose of exogenous surfactant administered,
and both an umbilical venous catheter and umbilical arterial catheter were placed successfully in the
delivery room. In the NICU, the infant is placed on a radiant warmer. The nurse caring for the infant
asks if the infant will need to be transferred to an isolette incubator.

Of the following, the MOST likely reason for this infant to be relocated into an isolette incubator is

A. avoidance of light damage to the eyes

B. bronzing of the skin under the radiant warmer

C. inability to maintain core temperature on a radiant warmer

D. increased risk for infection under the radiant warmer

E. increased transcutaneous water loss under the radiant warmer

page 3
2009 PREP SA on CD-ROM

Critique: 2 Preferred Response: E


The extremely low gestational-age newborn (ELGAN) who has a birthweight of less than 1,000 g is at
great risk for transcutaneous evaporative water loss in an open air, nonhumidified environment such as
a radiant warmer. This is most concerning in the first 24 to 72 hours of postnatal life, diminishing
thereafter as the infant's skin becomes cornified. The use of a contained, convectively heated, and
humidified environment can reduce transcutaneous evaporative water loss best in the first few days of
the ELGAN's postnatal life.
Ambient light is not a contributing factor to retinopathy of prematurity, for which the ELGAN is also at
risk. Although excessive thermal warming can occur under the radiant warmer, bronzing of the skin is
related to the use of phototherapy lights in newborns who have conjugated hyperbilirubinemia. The
ELGAN's core temperature can be maintained using a radiant warmer, but evaporative and convective
heat losses remain a concern over lengthy periods of time, and differences in extremity, head, and core
temperatures may affect perfusion and acid-base status. There is no increased risk for infection on the
radiant warmer compared with the isolette incubator.

References:

Dollberg S, Hoath SB. Temperature regulation in preterm infants: role of the skin-environment interface.
NeoReviews. 2001;2:e282-e291. Available for subscription at:
http://neoreviews.aappublications.org/cgi/content/full/2/12/e282

Korones SB. An encapsulated history of thermoregulation in the neonate. NeoReviews.


2004;5:e78-e85. Available for subscription at:
http://neoreviews.aappublications.org/cgi/content/full/5/3/e78

Sedin G. The thermal environment of the newborn infant. In: Martin RJ, Fanaroff AA, Walsh MC, eds.
Fanaroff and Martin's Neonatal-Perinatal Medicine. 8th ed. Philadelphia, Pa: Mosby Elsevier;
2006:585-596

page 4
2009 PREP SA on CD-ROM

Question: 3
You are seeing a 1-month-old girl for follow-up after a hospitalization for acute gastroenteritis caused by
rotavirus. Her diarrhea had decreased in the hospital while taking oral rehydration solution, but when
her mother resumed her usual cow milk formula, the girl began to have an increased number of very
watery stools. She appears well hydrated, and findings on her abdominal examination are normal.

Of the following, the MOST appropriate approach to managing this infant’s diarrhea is to

A. change to a lactose-free formula for the next few days

B. dilute the cow milk formula with oral rehydration solution for the next few days

C. give her only oral rehydration solution until the diarrhea resolves

D. readmit her to the hospital for administration of intravenous fluids

E. repeat her stool studies to confirm the diagnosis of rotavirus infection

page 5
2009 PREP SA on CD-ROM

Critique: 3 Preferred Response: A


The infant described in the vignette most likely has lactase deficiency due to rotavirus infection. Lactase
is an enzyme found in the most superficial villous portion of the intestinal brush border, which
hydrolyzes lactose to glucose and galactose. Lactase deficiency may have several causes in children
and adults. Primary lactase deficiency, the most common type, is a genetically determined condition
that affects children and adults at different ages but is unusual before 5 years. Symptoms include
abdominal distention, bloating, flatulence, or nausea after the ingestion of lactose, with the amount of
lactose needed to cause such symptoms varying from person to person. The diagnosis is made by
breath hydrogen testing, and management consists of removing some or all lactose from the diet.
Congenital lactase deficiency is extremely rare.
Secondary lactase deficiency may develop after an infectious gastroenteritis, such as rotavirus,
giardiasis, or cryptosporidiosis. Other causes include celiac disease and enteropathy related to
immunodeficiency. Secondary lactase deficiency is suggested when a child who has a recent diarrheal
illness experiences worsening diarrhea or bloating after the reintroduction of lactose into the diet, as
described for the girl in the vignette. Most children who have gastroenteritis do not develop lactase
deficiency. For this reason, most infants can tolerate and should continue taking human milk or
standard lactose-containing formula throughout a diarrheal illness. For very young infants (eg, <3
months old), such as the one described in the vignette, or those who have significant fluid losses, a
lactose-free formula may be attempted until the diarrhea resolves. Infants who are breastfed should be
encouraged to continue breastfeeding, even if secondary lactase deficiency is suspected.
Giving full-strength formula or human milk is recommended to supply the child with sufficient calories
during the recovery phase of a diarrheal illness; therefore, diluting the formula or providing only oral
rehydration solution is inappropriate. If the child is not vomiting, oral hydration is optimal, and
intravenous hydration is not necessary. There is no need to confirm the diagnosis of rotavirus infection;
doing so would not alter management plans.

References:

Dalby-Payne J, Elliott E. Gastroenteritis in children. BMJ Clinical Evidence. 2007. Available for
subscription at: http://clinicalevidence.bmj.com/ceweb/conditions/chd/0314/0314.jsp

Heyman MB; Committee on Nutrition. Lactose intolerance in infants, children, and adolescents.
Pediatrics. 2006;118:1279-1286. Available at:
http://pediatrics.aappublications.org/cgi/content/full/118/3/1279

King CK, Glass R, Bresee, Duggan C. Managing acute gastroenteritis among children: oral rehydration,
maintenance, and nutritional therapy. MMWR Morbid Mortal Wkly Rep Recomm Rep.
2003;52(RR-16):1-16. Available at: http://www.cdc.gov/mmwr/preview/mmwrhtml/rr5216a1.htm

page 6
2009 PREP SA on CD-ROM

Question: 4
During a prenatal visit with expectant parents, they report that they are strict vegans. They ask you to
advise them on a healthy diet and any required supplements. The mother plans to breastfeed the
newborn exclusively for the first 6 months.

Of the following, you are MOST likely to tell them that their newborn may require supplemental

A. calcium

B. folate

C. iron

D. vitamin B6

E. vitamin B12

page 7
2009 PREP SA on CD-ROM

Critique: 4 Preferred Response: E


A vegan diet, by definition, excludes all foods derived from animal products. A lacto-ovo-vegetarian diet
may include milk and eggs. Although a vegan diet may be healthy, there is a risk for vitamin B12
deficiency because vitamin B12 is only found in foods of animal origin. Breastfeeding vegan mothers
may produce milk that is deficient in this vitamin and require supplementation that generally is achieved
by continuing the consumption of prenatal vitamins containing vitamin B12.
The recommended supplementation for breastfed vegan infants to prevent vitamin B12 deficiency is
0.4 mcg/day during the first 6 postnatal months and 0.5 mcg/day from 6 months to 1 year of age. Vegan
infants who are not breastfed should receive iron-fortified soy infant formula until 1 year of age to avoid
deficiencies in iron. Vegan infants require no other mineral or vitamin supplementation.
Vegan diets in older children and adolescents may be low in calcium (similar to the typical American
"teenage diet" that contains less than the recommended intake of dairy products), and the zinc
consumption may be relatively low due to the absence of phytate, which renders zinc more bioavailable.
Children who follow vegan diets may have relatively diminished overall energy intake because such
diets commonly are low in fat and high in fiber.
Review of nutrient intake and energy intake in conjunction with growth curves of children eating
vegan diets in both the United States and the United Kingdom demonstrate no significant health issues.
Height and weight measured in vegan populations may be slightly lower than average but not in the
range of failure to thrive or short stature. Adolescents eating vegan diets are more likely than
adolescents eating a typical American diet to meet nutritional goals, including recommended intake of
fruits and vegetables. Vegan adolescents are less likely to be obese because they consume fewer
foods high in fat. However, they remain at risk for vitamin B12 deficiency and should consume at least a
daily multivitamin. They are less likely to have anemia but just as likely to have low calcium intake as
their non-vegan peers. Adolescents who follow a lacto-ovo-vegetarian diet are less likely to have
deficiencies in vitamin B12, calcium, and iron.
Folate and vitamin B6 are not likely to be deficient in persons who consume vegan diets because
those nutrients are found in many legumes, fruits, and vegetables that are the mainstays of the diet.

References:

Kleinman RE. Nutritional aspects of vegetarian diets. In: Pediatric Nutrition Handbook. 5th ed. Elk
Grove Village, Ill: American Academy of Pediatrics; 2003:191-208

Mangels AR, Messina V. Considerations in planning vegan diets: infants. J Am Diet Assoc.
2001:101:670-677. Abstract available at: http://www.ncbi.nlm.nih.gov/pubmed/11424546

Messina V, Mangels AR. Considerations in planning vegan diets: children. J Am Diet Assoc.
2001:101:661-669. Abstract available at: http://www.ncbi.nlm.nih.gov/pubmed/11424545

Moilanen BC. Vegan diets in infants, children and adolescents. Pediatr Rev. 2004:25:174-176.
Available at: http://pedsinreview.aappublications.org/cgi/content/full/25/5/174

Perry CL, McGuire MT, Neumark-Sztainer D, Story M. Adolescent vegetarians: how well do their dietary
patterns meet the Healthy People 2010 objectives? Arch Pediatr Adolesc Med. 2002; 156:431-437.
Available at: http://archpedi.ama-assn.org/cgi/content/full/156/5/431

page 8
2009 PREP SA on CD-ROM

Question: 5
You are treating a child who has suffered a splenic injury and is being transfused with large volumes of
packed red blood cells for severe anemia. He weighs 10 kg and has received 4 units thus far.

Of the following, the finding on electrocardiography that is MOST likely to represent a serious
complication of his therapy is

A. atrial flutter

B. delta waves

C. prominent U waves

D. supraventricular tachycardia

E. tall-peaked T waves

page 9
2009 PREP SA on CD-ROM

Critique: 5 Preferred Response: E


Administration of fluids and blood products can be essential for resuscitation of the trauma patient but
may lead to potentially dangerous electrolyte imbalances. Recognition of these abnormalities, which
may be subtle, can be important and even lifesaving. When large volumes of blood are transfused, as
reported for the child in the vignette, hyperkalemia may occur, which is believed to be the result of
extravasation of potassium from the red blood cells that have been irradiated and stored over time.
Hyperkalemia can be associated with paresthesias, weakness, and tingling, although cardiac toxicity
typically precedes such symptoms. Severe cardiac rhythym changes may begin abruptly. The classic
electrocardiographic sign of hyperkalemia is tall, peaked T waves (Item C5A), particularly as the serum
potassium concentration approaches or exceeds 5.0 to 6.0 mEq/L (5.0 to 6.0 mmol/L). As hyperkalemia
progresses, other changes in the ECG, such as widening of the QRS complex, may be noted. The
rhythm changes that occur as a result of hyperkalemia, including ventricular arrhythmias, may develop
abruptly, leading to sudden changes in the patient's clinical condition.
Another potential electrolyte complication of fluid and blood administration is hypocalcemia, which
results from the citrate-containing anticoagulants that bind free calcium. Hypocalcemia may manifest on
ECG as a prolonged QT interval, which tends to widen as the ionized calcium concentrations decrease.
Atrial flutter, a primary disease of the atrial tissue, would not be expected in the patient described in
the vignette. The delta wave refers to the ECG finding of pre-excitation, seen in conditions such as the
Wolff-Parkinson-White syndrome that have an associated "bypass" tract allowing for excitement of the
His-Purkinje system without passage through the atrioventricular node (Item C5B).
The U wave may be seen in hypokalemia (Item C5C), particularly as concentrations decrease below
2.7 mEq/L (2.7 mmol/L), or hypercalcemia (calcium concentrations exceeding 12.0 mg/dL [3.0 mmol/L]),
which would not be expected in the patient described in the vignette. Hypercalcemia also may lead to a
diminished QT interval, and with more severe hypercalcemia, second- or third-degree heart block (Item
C5D) may develop. Supraventricular tachycardia (Item C5E) would not be expected to occur as a result
of a large-volume transfusion process.

References:

Galel SA, Naiman JL. Use of blood and blood products. In: Rudolph CD, Rudolph AM, eds. Rudolph's
Pediatrics. 21st ed. New York, NY: McGraw-Hill Medical Publishing Division; 2003:1576-1581

Vetter V. Arrhythmias. In: Moller JH, Hoffman JIE, eds. Pediatric Cardiovascular Medicine. Philadelphia,
Pa: Churchill Livingstone; 2000:833-884

page 10
2009 PREP SA on CD-ROM

Critique: 5

The electrocardiogram in hyperkalemia shows peaked T waves. (Courtesy of


A. Friedman)

page 11
2009 PREP SA on CD-ROM

Critique: 5

In Wolff-Parkinson-White syndrome, delta waves (arrows) are present that


represent pre-excitation depolarization of the QRS complex. (Courtesy of A.
Friedman)

page 12
2009 PREP SA on CD-ROM

Critique: 5

The U wave (arrow) may be observed in hypokalemia (shown here in which


there is also ST depression and flattening of the T wave) or hypercalcemia.
(Courtesy of A. Friedman)

page 13
2009 PREP SA on CD-ROM

Critique: 5

In third-degree heart block that may result from severe hypercalcemia, P


waves are completely dissociated from QRS complexes. (Courtesy of A.
Friedman)

page 14
2009 PREP SA on CD-ROM

Critique: 5

Narrow complex tachycardia that is consistent with supraventricular


tachycardia or paroxysmal atrial tachycardia. (Courtesy of A. Friedman)

page 15
2009 PREP SA on CD-ROM

Question: 6
A 15-year-old girl presents to the emergency department with a 4-week history of nasal drainage and
face pain and a 2-week history of frontal headaches and fatigue. Her mother complains that her
daughter has an "attitude" and has not been respectful or seemed to care about anything for the past 2
weeks. The daughter awoke this morning with a headache and vomited. On physical examination, the
adolescent is afebrile and has normal vital signs. She responds slowly to questions and is not oriented
to the date. She complains of pain to palpation of her cheeks and forehead. She has no nuchal rigidity
and no focal weakness. The remainder of the physical examination findings are normal.

Of the following, the BEST initial diagnostic procedure is

A. computed tomography scan of the head with intravenous contrast

B. emergent electroencephalography to rule out nonconvulsive status epilepticus

C. lumbar puncture to rule out meningitis

D. nasal swab for bacterial culture

E. urine drug screen for barbiturates, amphetamines, and cocaine

page 16
2009 PREP SA on CD-ROM

Critique: 6 Preferred Response: A


The subacute onset of mental status changes described for the adolescent in the vignette warrants an
emergency evaluation. In most cases, neuroimaging is indicated, along with appropriate laboratory
testing.
The relatively nonspecific pain and what her mother perceives as common emotional problems
(apathy in a teenager) probably represent early frontal lobe symptoms. The headache on awakening
and vomiting are concerning for increased intracranial pressure (ICP). Confusion and psychomotor
retardation on the mental status examination indicate involvement of the central nervous system. A
focal, ischemic, ictal, infectious/inflammatory, or toxic/metabolic process must be identified urgently. A
brain abscess is suggested by the prominent facial pain in this setting; the sinuses are a common
source of brain abscesses (Item C6). Brain abscesses often present only with nonspecific pain and not
with fever.
Head computed tomography (CT) scan is preferred for this patient because the constellation of pain,
confusion, and morning vomiting makes a focal intracranial mass a possibility. Increased ICP is
associated with morning vomiting because ICP is highest in the morning. Contrast is recommended
because of the insidious onset, which could indicate either a neoplasm or infectious process.
Intravenous contrast is not needed for all neuroimaging procedures. However, it increases the
diagnostic yield of imaging studies where either neoplasm or infection is suspected because both
typically involve some degradation of the blood-brain barrier or hypervascularity, resulting in contrast
enhancement at the site of the lesion. Magnetic resonance imaging (MRI) with contrast also is a good
choice. The advantage of MRI is higher spatial and soft-tissue resolution. Disadvantages of MRI
compared with CT include: 1) less availability for emergency department studies; 2) need for
pharmacologic sedation in agitated patients because sedation affects mental status, thereby masking
disease-related mental status; 3) longer time in the scanner, which could delay treatment decisions;
and 4) cost. Thus, in most cases, a head CT scan with contrast is preferred as the initial study in the
emergency department.
Electroencephalography (EEG) is an important test for assessment of a patient who has
encephalopathy of unclear cause to rule out nonconvulsive status epilepticus (NCSE), particularly if the
patient is known to have epilepsy. If an EEG cannot be obtained rapidly, intravenous administration of
0.1 mg/kg lorazepam can treat NCSE immediately, although this would not clear confusion about other
causes. For this adolescent, the facial pain makes the diagnosis of NCSE less likely than a brain
abscess.
A lumbar puncture may be needed to rule out meningitis or encephalitis, but the pain and morning
vomiting more strongly suggest the possibility of an intracranial mass. Lumbar puncture prior to head
CT is not advised in this case because it could reduce pressure below the foramen magnum and result
in herniation from the supratentorial mass. Toxicology screening and nasal swabs are reasonable but
not the preferred initial diagnostic tests because they will not affect emergency management of the
increased intracranial pressure.

References:

Goodkin HP, Harper MB, Pomeroy SL. Intracerebral abscess in children: historical trends at Children's
Hospital Boston. Pediatrics. 2004;113:1765-1770. Available at:
http://pediatrics.aappublications.org/cgi/content/full/113/6/1765

Haslam RHA. Brain abscess. In: Kliegman RM, Behrman RE, Jenson HB, Stanton BF, eds. Nelson
Textbook of Pediatrics. 18th ed. Philadelphia, Pa: Saunders Elsevier; 2007:2524-2525

page 17
2009 PREP SA on CD-ROM

Kan L, Nagelberg J, Maytal J. Headaches in a pediatric emergency department: etiology, imaging, and
treatment. Headache. 2000;40:25-29. Abstract available at:
http://www.ncbi.nlm.nih.gov/pubmed/10759899

page 18
2009 PREP SA on CD-ROM

Critique: 6

Brain abscess: Contrast-enhanced computed tomography scan of the head


reveals a rim-enhancing lesion extending to an area of bony destruction in an
opacified left frontal sinus. Vasogenic edema surrounding the abscess
creates a mass effect and shift of the midline to the right. (Courtesy of D.
Krowchuk)

page 19
2009 PREP SA on CD-ROM

Question: 7
A 5-year-old girl who is new to your practice presents to the clinic for a prekindergarten physical
examination. Her primary caretaker, the maternal grandmother, reports that the child’s mother used
multiple street drugs throughout her pregnancy as well as medications prescribed for seizure and
bipolar disorders. The grandmother is concerned that this child’s speech development is delayed. On
physical examination, you note that the girl has wide-spaced eyes, a short nose, and midface
hypoplasia.

Of the following, the substance that is MOST likely to be associated with this child’s dysmorphic
features is

A. lithium

B. lysergic acid diethylamide (LSD)

C. marijuana

D. methamphetamine

E. phenobarbital

page 20
2009 PREP SA on CD-ROM

Critique: 7 Preferred Response: E


The features described for the child in the vignette are most consistent with fetal anticonvulsant
syndrome, which can occur following exposure to numerous medications, including phenytoin,
carbamazepine, valproate, and phenobarbital. Multiple authors have observed a 10% to 20% incidence
of birth defects in infants exposed to phenobarbital in utero. Anomalies include midface hypoplasia,
ocular hypertelorism (Item C7A), nail hypoplasia (Item C7B), cleft lip+/-cleft palate, and heart defects as
well as developmental delay and pre- and postnatal growth failure.
Phenobarbital-exposed newborns may exhibit a withdrawal syndrome that is evidenced by
tremulousness and increased activity. Of interest, such infants are likely to have lower serum bilirubin
concentrations than nonexposed neonates.
Prenatal exposure to lithium is associated with an increased risk of cardiac malformations (eg,
Ebstein anomaly) in the fetus. If the mother takes lithium near term, the exposed neonate may have
cyanosis, hypotonia, abnormalities of cardiac rhythm, goiter, hypothyroidism, and nephrogenic diabetes
insipidus. Lithium exposure is not associated with dysmorphic features or developmental delays.
Despite popular belief, lysergic acid diethylamide (LSD) generally is not associated with birth defects
or withdrawal symptoms in prenatally exposed infants. Although there are isolated case reports of birth
defects in exposed neonates, an increased risk for anomalies is not borne out by epidemiologic studies.
It is important to note, however, that LSD users often abuse other substances, underscoring the
importance of taking an in-depth drug/substance abuse history in pregnant women.
Marijuana use during pregnancy is not known to be associated with an increased incidence of birth
defects, dysmorphic features, or developmental delay in exposed offspring, although further study is
needed in this regard. Some investigations have shown reduced fetal growth in exposed pregnancies,
but this is not confirmed. Prenatally exposed newborns may have tremulousness, increased irritability,
and abnormal visual response to light stimulus.
Although there does not appear to be an increase in congenital anomalies associated with
methamphetamine use during pregnancy, further study is necessary. There are reports of decreased
birthweight in exposed neonates. A neonatal withdrawal syndrome consisting of abnormal sleep
patterns, tremulousness, poor feeding, and increased tone has been observed commonly. Once again,
it is important to consider polydrug abuse in these instances.
As with all teratogens, the timing of exposure is critical, with the most vulnerable period of embryonic
development occurring between 18 and 60 days after conception, during organogenesis. Dosage of the
offending agent also is important, as are route of administration, modifying environmental factors, and
genetic background of the mother and fetus.

References:

Gallagher RC, Kingham K, Hoyme HE. Fetal anticonvulsant syndrome. In: Cassidy SB, Allanson JE,
eds. Management of Genetic Syndromes. 2nd ed. Hoboken, NJ: Wiley-Liss; 2005:239-250

Phenobarbital, lithium, LSD, marijuana, methamphetamine. Reprotox. Available for subscription at:
http://www.reprotox.org

Phenobarbital, lithium, LSD, marijuana, methamphetamine. Teris. Available for subscription at:
http://depts.washington.edu/terisweb/teris

page 21
2009 PREP SA on CD-ROM

Critique: 7

Wide-spaced eyes and a short upturned nose are facial features of the fetal
anticonvulsant syndrome. This child was exposed to phenytoin. (Courtesy of
M. Rimsza)

page 22
2009 PREP SA on CD-ROM

Critique: 7

Nail hypoplasia is observed in a child who has fetal anticonvulsant syndrome.


(Courtesy of the Media Lab at Doernbecher)

page 23
2009 PREP SA on CD-ROM

Question: 8
A 15-year-old girl comes to the urgent care clinic complaining of lower abdominal pain for 48 hours. She
is nauseated but has had no fever, vomiting, or diarrhea. She is afebrile and denies abdominal trauma.
She localizes the pain to the left lower quadrant and describes it as intermittent, stabbing pain episodes
separated by intervals of more continuous dull pain. She has never been sexually active. Her last
menstrual period was 1 week ago. She has had no vaginal discharge or itching. On physical
examination, she has left lower quadrant guarding and rebound tenderness. Her pelvic examination
shows no vaginal discharge or uterine tenderness, although there is an exquisitely tender mass in the
left adnexal area.

Of the following, the MOST likely diagnosis is

A. appendicitis

B. endometritis

C. ovarian torsion

D. sacroiliitis

E. splenic rupture

page 24
2009 PREP SA on CD-ROM

Critique: 8 Preferred Response: C


Torsion of any adnexal structure, including the ovary, can result in an acute pelvic mass (Item C8). The
most common causes of adnexal torsion in young women are cysts and neoplasms, with about 6% of
torsions in one series occurring in the setting of normal-appearing ovaries. Histopathology was benign
in more than 90% of cases in this series. Torsion occurs when masses cause the ovary to swing on its
vascular pedicle, and larger masses generally are associated with a greater potential for torsion until
the size of the mass impedes movement. The onset of pain associated with torsion can be abrupt,
sharp, and very severe. However, with intermittent or partial torsion, intense periods of intermittent pain
may be separated by generalized aching during detorsion, as described for the teenager in the vignette.
Pain can occur for several days or weeks prior to a complete torsion. Nausea or vomiting can
accompany severe pain. If adnexal torsion is diagnosed, prompt intervention (untwisting the adnexa
usually via laparoscopy) is indicated to preserve ovarian function.
Appendicitis is in the differential diagnosis for lower abdominal pain, but is less likely when pain
occurs in the left lower quadrant. Splenic rupture is a surgical emergency, like ovarian torsion, but the
finding of an adnexal mass on examination is unlikely. Endometritis is not very likely in a young woman
who is not sexually active and more commonly presents with midline uterine pain. An orthopedic cause
for this patient's pain (eg, sacroiliitis) is unlikely, but it remains in the differential diagnosis of pelvic pain.
Sacroiliitis usually presents with low back or hip pain.

References:

Adams Hillard PJ. Pelvic masses. In: Neinstein LS, eds. Adolescent Health Care: A Practical Guide. 5th
ed. Philadelphia, Pa: Lippincott Williams & Wilkins; 2008:706-713

Growdon WB, Laufer MR. Ovarian torsion. UpToDate Online 15.3. 2008. Available for subscription at:
http://www.utdol.com/utd/content/topic.do?topicKey=gyn_surg/5273

Laufer MR, Goldstein DP. Gynecologic pain: dysmenorrhea, acute and chronic pelvic pain,
endometriosis, and premenstrual syndrome. In: Emans SJH, Laufer MR, Goldstein DP, eds. Pediatric
and Adolescent Gynecology. 5th ed. Philadelphia, Pa: Lippincott, Williams & Wilkins; 2005:417-476

Varras M, Tsikini A, Polyzos D, Samara Ch, Hadjopoulos G, Akrivis Ch. Uterine adnexal torsion:
pathologic and gray-scale ultrasonographic findings. Clin Exp Obstet Gynecol. 2004;31:34-38. Abstract
available at: http://www.ncbi.nlm.nih.gov/pubmed/14998184

page 25
2009 PREP SA on CD-ROM

Critique: 8

Ovarian torsion: Color flow Doppler ultrasonography of the right ovary shows
abundant flow in the adjacent pelvic tissue (blue, red, and orange color seen
inferiorly) but none in the ovary (the area within the dashed line).

page 26
2009 PREP SA on CD-ROM

Question: 9
A 2-year-old boy who has chronic renal failure is brought to the emergency department for evaluation of
nausea, fatigue, and muscle weakness. On physical examination, the boy has a heart rate of 140
beats/min, decreased perfusion, and palpable pulses. You obtain electrocardiography (Item Q9).
Electrolyte measurements include a potassium concentration of 7.5 mEq/L (7.5 mmol/L) and a glucose
value of 72.0 mg/dL (4.0 mmol/L).

Of the following, the MOST appropriate initial treatment is administration of

A. calcium chloride

B. insulin

C. normal saline 20 mL/kg

D. sodium polystyrene sulfonate

E. verapamil

page 27
2009 PREP SA on CD-ROM

Question: 9

In hyperkalemia, electrocardiography demonstrates peaked T waves.


(Courtesy of A. Friedman)

page 28
2009 PREP SA on CD-ROM

Critique: 9 Preferred Response: A


Hyperkalemia can be asymptomatic or produce symptoms such as nausea, fatigue, and muscle
weakness, as reported for the boy in the vignette. The most concerning effect of hyperkalemia is on
cardiac membrane polarization. Electrocardiographic changes typically consist of peaked T waves and
progress to widening of the QRS complexes with dampening of P waves as the hyperkalemia
increases. Without treatment, the patient eventually develops ventricular fibrillation or asystole.
The electrocardiographic tracing of the boy described in the vignette demonstrates peaked T waves
(Item C9), and he has decreased perfusion. The immediate priority is to stabilize his cardiac membrane
potential and decrease his risk of arrhythmias, which is accomplished best by administration of
intravenous calcium chloride, an agent that has a rapid onset of action. Insulin causes potassium to
shift to the intracellular spaces, thereby decreasing serum concentrations, but it always should be
administered in conjunction with glucose to avoid hypoglycemia. Sodium bicarbonate administered
intravenously also produces intracellular shifts of potassium and may be useful in patients who have
metabolic acidosis.
Treatments to enhance the elimination of potassium often are indicated in the treatment of
hyperkalemia, but they have a slower onset of action and, therefore, should not be the first line of
treatment in life-threatening hyperkalemia. Sodium polystyrene sulfonate, which can be administered
orally or rectally, exchanges sodium for potassium, which subsequently is eliminated from the body.
Loop diuretics also can enhance potassium elimination. Dialysis may be indicated when ongoing
elevated potassium concentrations are expected, such as in renal failure, or with very high acute
concentrations, as seen with tumor lysis syndrome or rhabdomyolysis.
Administration of verapamil is not indicated in the treatment of hyperkalemia; it has been reported to
be associated with the development of complete heart block in this setting. Normal saline has a minimal
effect on hyperkalemia.

References:

Greenbaum LA. Electrolyte and acid-base disorders: potassium. In: Kliegman RM, Behrman RE,
Jenson HB, Stanton BF, eds. Nelson Textbook of Pediatrics. 18th ed. Philadelphia, Pa: Saunders
Elsevier; 2007:279-284

Hauser GJ, Kulick AF. Electrolyte disorders in the pediatric intensive care unit. In: Wheeler DS, Wong
HR, Shanley TP, eds. Pediatric Critical Care Medicine: Basic Science and Clinical Evidence. New York,
NY: Springer-Verlag; 2007:1156-1175

page 29
2009 PREP SA on CD-ROM

Critique: 9

In hyperkalemia, electrocardiography demonstrates peaked T waves.


(Courtesy of A. Friedman)

page 30
2009 PREP SA on CD-ROM

Question: 10
A 13-year-old girl who has just moved to the United States from Brazil comes to your office because her
mother is worried that she is not "developing yet." On physical examination, her height is 50 inches, and
she has a triangular face, a low hairline, high-arched palate, and a shield-shaped chest (Item Q10).
Breast tissue is not visible or palpable, but there is Sexual Maturity Rating 3 pubic hair. You obtain bone
age radiography and a karyotype and measure serum luteinizing hormone and follicle-stimulating
hormone.

Of the following, the MOST appropriate additional laboratory measurement is

A. adrenocorticotropic hormone

B. prolactin

C. 17-hydroxyprogesterone

D. testosterone

E. thyroid-stimulating hormone

page 31
2009 PREP SA on CD-ROM

Question: 10

Shield-shaped chest and lack of breast development, as described for the girl
in the vignette. (Courtesy of M. Rimsza)

page 32
2009 PREP SA on CD-ROM

Critique: 10 Preferred Response: E


The clinical findings described for the girl in the vignette are characteristic of Turner syndrome (gonadal
dysgenesis) associated with an abnormality of one X chromosome. Girls who have this disorder usually
are short (mean adult height, approximately 55 inches without growth hormone treatment); have poorly
developed ovaries; and often have dysmorphisms, including a triangular facies, low hairline,
high-arched palate, hypoplastic nipples, and an increased carrying angle. They may have left heart
disorders such as coarctation of the aorta as well as horseshoe kidney or other renal malformations.
Initial screening studies to diagnose Turner syndrome include a karyotype and measurement of
luteinizing hormone (LH) and follicle-stimulating hormone (FSH). Most girls who have Turner syndrome
do not initiate normal puberty. Concentrations of LH and FSH rise as they reach pubertal age range
because they have ovarian failure. Although concentrations of estradiol and other estrogens are low,
clinical estradiol assays are not designed to provide accurate values in the low-normal range expected
in early puberty. Therefore, physical findings such as breast development are a better marker of
estrogen effect than measurements of estrogen.
Adolescents who have Turner syndrome are at higher risk of developing chronic lymphocytic
thyroiditis and hypothyroidism than the general population. Approximately 20% of affected adolescent
girls have antibody-positive autoimmune chronic lymphocytic thyroiditis, and 5% to 10% develop overt
hypothyroidism. Accordingly, measurement of thyroid-stimulating hormone is an appropriate laboratory
test for patients such as the girl described in the vignette. An elevated value indicates primary
hypothyroidism and the need for confirmatory assessment of free thyroxine and antithyroid antibodies
(thyroperoxidase, antimicrosomal, or antithyroglobulin).
Abnormalities of the hypothalamic-pituitary-adrenal axis are unusual in patients who have Turner
syndrome. Therefore, measurement of adrenocorticotropic hormone is not useful. Measurement of
prolactin would be useful if the girl had a pituitary or hypothalamic problem, but her clinical findings
strongly point to Turner syndrome. A 17-hydroxyprogesterone value would be elevated in the presence
of an adrenal biosynthetic defect leading to the development of the most common form of congenital
adrenal hyperplasia (cyp21 or 21-hydroxylase deficiency) as well as some of the less common
disorders of adrenal biosynthesis. Measuring testosterone would be reasonable if there were evidence
of inappropriate masculinization, such as clitoromegaly and a growth spurt. Some girls who have Turner
syndrome have functioning Y chromosomal DNA and could have androgenization, but this is unusual.
The presence of Y chromosomal DNA does increase the risk of gonadal malignancy, and girls who
have significant Y chromosomal DNA on testing often require prophylactic gonadectomy.

References:

Chiovato L, Larizza D, Bendinelli G, et al. Autoimmune hypothyroidism and hyperthyroidism in patients


with Turner's syndrome. Eur J Endocrinol. 1996;134:568-575. Abstract available at:
http://www.ncbi.nlm.nih.gov/pubmed/8664977

Doswell BH, Visootsak J, Brady AN, Graham JM Jr. Turner syndrome: an update and review for the
primary pediatrician. Clin Pediatr. 2006;45:301-313. Abstract available at:
http://www.ncbi.nlm.nih.gov/pubmed/16703153

Frias JL, Davenport ML, Committee on Genetics and Section on Endocrinology. Health supervision for
children with Turner syndrome. Pediatrics. 2003;111:692-702. Available at:
http://pediatrics.aappublications.org/cgi/content/full/111/3/692

Matura LA, Ho VB, Rosing DR, Bondy CA. Aortic dilatation and dissection in Turner syndrome.

page 33
2009 PREP SA on CD-ROM

Circulation. 2007;116:1663-1670. Abstract available at: http://www.ncbi.nlm.nih.gov/pubmed/17875973

Sybert VP, McCauley E. Turner's syndrome. N Engl J Med. 2004;351:1227-1238. Extract available at:
http://content.nejm.org/cgi/content/extract/351/12/1227

Turner Syndrome Society Website. Available at: http://www.turnersyndrome.org

page 34
2009 PREP SA on CD-ROM

Question: 11
A 2-month-old infant has lost the vision in both of his eyes due to bilateral retinoblastoma. His
distressed parents ask how the infant’s blindness will affect his behavior and development.

Of the following, the child MOST likely will

A. begin saying single words at 16 to 20 months

B. begin walking between 18 and 22 months

C. display behaviors of an autism spectrum disorder

D. have a language-based learning disorder

E. have significant cognitive impairments

page 35
2009 PREP SA on CD-ROM

Critique: 11 Preferred Response: B


Legal blindness is defined as central visual acuity with corrective lenses of 20/200 or less in the
strongest eye or a limited visual field that extends to an angle of 20 degrees. Congenital blindness
occurs in 30 per 100,000 births. More than 50% of children who have visual impairment also have
developmental disabilities, such as cognitive-adaptive disability, seizures, hearing impairments, and
learning disorders. In many of these cases, the disabilities result from central nervous system
pathology. Postnatal blindness, which accounts for approximately 8% to 11% of all childhood blindness,
can be caused by infections, trauma, or tumors. Retinoblastoma is the most common primary malignant
intraocular tumor of childhood. The initial finding in most cases is a white pupillary reflex (leukokoria)
(Item C11). Advanced tumors may be treated with enucleation.
Children who have congenital or acquired (eg, due to retinoblastoma) blindness without associated
neurologic abnormalities should not be at increased risk for motor or cognitive impairment. They are not
at increased risk for language-based learning disabilities or autism spectrum disorders. However,
children who have significant visual impairment may begin to walk at an older age (18 to 22 months)
than sighted children due to different exposure to motor exploration. They typically develop language
skills at the same time (12 months) as sighted children. Children who have visual impairments should
be provided with much physical contact that includes hugging and comforting. They should be
encouraged to partake in self-help skills and exploration of their environment.

References:

Davidson PW, Burns CM. Visual impairment and blindness. In: Levine MD, Carey WB, Crocker AC,
eds. Developmental- Behavioral Pediatrics. 3rd ed. Philadelphia, Pa: WB Saunders Company;
1999:571-578

Msall ME. Visual impairment. In: Parker S, Zukerman B, Augustyn M. Developmental and Behavioral
Pediatrics: A Handbook for Primary Care. 2nd ed. Philadelphia, Pa: Lippincott Williams & Wilkins;
2005:366-369

Olitsky SE, Hug D, Smith LP. Disorders of vision. In: Kleigman RM, Behrman RE, Jenson HB, Stanton
BF, eds. Textbook of Pediatrics. 18th ed. Philadelphia, Pa: Saunders Elsevier; 2007:2573-2576

page 36
2009 PREP SA on CD-ROM

Critique: 11

Leukokoria may be observed in patients who have retinoblastoma. (Courtesy


of R.G. Weaver, Jr)

page 37
2009 PREP SA on CD-ROM

Question: 12
A 15-year-old boy presents to the clinic because of a persistent cough. According to his mother, his
cough has been present for approximately 2 weeks, but it seems to be getting worse. He does not
cough all the time, but the coughing episodes tend to come in bursts. This morning she became very
worried because he passed out during a coughing spell. Physical examination reveals a
healthy-appearing male in no apparent distress. He is afebrile, and his vital signs are normal. He has
petechiae on his face but no other skin lesions. His lungs are clear.

Of the following, the MOST appropriate antimicrobial agent to prescribe for this patient is

A. azithromycin

B. clarithromycin

C. doxycycline

D. erythromycin

E. trimethoprim-sulfamethoxazole

page 38
2009 PREP SA on CD-ROM

Critique: 12 Preferred Response: A


Any patient who has episodic coughing episodes that conclude in syncope or vomiting, such as the boy
described in the vignette, should be considered to have pertussis. Other complications from pertussis
described in adolescents include urinary incontinence, sleep interruption, rib fractures, and pneumonia.
Despite universal immunization of children against pertussis, a marked increase in disease incidence
has been demonstrated among adolescents of 11 to 18 years of age. In an attempt to address this
problem, the American Academy of Pediatrics recommends that adolescents in this age group receive
a single dose of tetanus toxoid, reduced diphtheria toxoid, and acellular pertussis (Tdap) for booster
immunization. For those who received only a tetanus toxoid and reduced diphtheria toxoid (Td) booster,
a booster with Tdap is suggested if the interval since the Td administration is at least 2 years or if the
adolescent is living in a setting of increased disease, has a risk of a complicated course if he or she
acquires the disease, or possibly can transmit infection to a vulnerable contact.
Treatment of pertussis with antimicrobial agents once the cough has started does not affect the
course of the illness but is recommended to limit the spread of disease to others. In the past, the
treatment of choice was erythromycin, but recent data have demonstrated that the effectiveness of
other macrolides such as azithromycin or clarithromycin is similar to that of erythromycin, and they have
fewer adverse effects. The most common complaints of patients who take erythromycin preparations
are gastrointestinal irritation, including epigastric distress, abdominal cramping, nausea, vomiting, and
diarrhea. The occurrence of these adverse effects can result in poor adherence to treatment regimens.
In addition, erythromycin cannot be used in children younger than 1 month of age due to its association
with the development of hypertrophic pyloric stenosis. Azithromycin and clarithromycin attain higher
tissue concentrations than erythromycin, have longer half-lives, and can be administered less often and
for a shorter total duration. With the advent of these newer macrolides, erythromycin rarely is indicated
as a first-line agent for any illness.
Azithromycin has become the agent of choice for treatment of pertussis because of the ease of
administration (once daily for 5 days) and the fact that it does not inhibit cytochrome P-450, as
erythromycin and clarithromycin do. Therefore, it does not interact with other medications that are
metabolized by this system (eg, digoxin, carbamazepine). Trimethoprim-sulfamethoxazole also is
effective against pertussis and is considered an alternative for patients who cannot tolerate a macrolide
or have a macrolide-resistant isolate if they are older than 2 months of age. Doxycycline is not
recommended for the treatment of pertussis.

References:

American Academy of Pediatrics. Pertussis (whooping cough). In: Pickering LK, Baker CJ, Long SS,
McMillan JA, eds. Red Book: 2006 Report of the Committee on Infectious Diseases. 27th ed. Elk Grove
Village, Ill: American Academy of Pediatrics; 2006:498-520

Committee on Infectious Diseases. Prevention of pertussis among adolescents: recommendations for


use of tetanus toxoid, reduced diphtheria toxoid, and acellular pertussis (Tdap) vaccine. Pediatrics.
2006;117:965-978. Available at: http://pediatrics.aappublications.org/cgi/content/full/117/3/965

Tiwari T, Murphy TV, Moran J. Recommended antimicrobial agents for the treatment and postexposure
prophylaxis of pertussis: 2005 CDC guidelines. MMWR Recomm Rep. 2005;54(RR14):1-16. Available
at: http://www.cdc.gov/mmwr/preview/mmwrhtml/rr5414a1.htm

page 39
2009 PREP SA on CD-ROM

Question: 13
A 15-year-old male presents for evaluation of a progressively enlarging lesion on his left forearm that
began 5 days ago. He explains that the lesion initially looked like a "spider bite" with a blister, but over
the last several days, a black scab has developed in the center of the lesion, and there is a large area
of redness around the scab. The lesion has been pruritic but not painful. Except for low-grade fevers for
the last 2 days, he has had no other systemic symptoms. He returned 1 week ago from a school trip to
Morocco, where he visited a leather tannery, went shopping in the large outdoor marketplace, visited
some historic sites, and took a camel ride in the desert. He states that the students stayed in a hostel in
Morocco, but there were no screens on the windows, and spiders, ants, and other insects were visible
in the rooms. On physical examination, the boy is afebrile, and his left forearm is edematous, with a
3x3-cm black eschar surrounded by a 5-cm area of erythema and induration (Item Q13). The lesion is
not tender to palpation, and there is no drainage. There are several 1.5-cm tender lymph nodes in his
left axilla. Findings on the remainder of his examination are within normal limits.

Of the following, the MOST likely cause of this patient’s lesion is

A. Bacillus anthracis

B. Francisella tularensis

C. Loxosceles laeta

D. methicillin-resistant Staphylococcus aureus

E. Yersinia pestis

page 40
2009 PREP SA on CD-ROM

Question: 13

Eschar, as desribed for the patient in the vignette. (Courtesy of the Centers for
Disease Control and Prevention, Public Health Image Library, James H. Steele)

page 41
2009 PREP SA on CD-ROM

Critique: 13 Preferred Response: A


Anthrax is a zoonotic disease caused by Bacillus anthracis, which is a gram-positive, encapsulated,
spore-forming rod that occurs in many areas of the world. B anthracis spores can remain viable in the
soil for decades (Item C13A), representing a major reservoir of infection for herbivorous livestock
through ingestion. Human infection occurs through contact with infected animals or contaminated
animal products, including carcasses, hides, hair, wool, meat, bone meal, and other contaminated
foodstuffs.
Depending on the route of infection, anthrax disease may manifest in three different forms:
cutaneous, inhalational, and gastrointestinal. Approximately 95% of all human anthrax cases are
cutaneous anthrax. The incubation period of cutaneous anthrax is 1 to 12 days. The initial skin lesion is
a pruritic papule that resembles an insect or spider bite, as described for the boy in the vignette. The
papule progresses to the development of a central vesicular or bullous lesion that becomes necrotic
and hemorrhagic and forms a central black painless eschar, which is the classic lesion of anthrax (Item
C13B). There is marked surrounding edema, swelling, induration, and erythema of the involved area but
no associated tenderness. Regional lymphadenopathy, fever, malaise, and headache also may be
present. In most cases, the eschar falls off in 1 to 2 weeks, and total resolution occurs in 6 weeks.
Infections caused by methicillin-resistant Staphylococcus aureus (MRSA), the bite of Loxosceles
laeta, Francisella tularensis, and Yersinia pestis are associated with pain, rapid progression, and in
most cases, systemic symptoms. Skin and soft-tissue infections due to MRSA have been described as
resembling a "spider bite" that is very tender, with progressive worsening of the erythema, warmth, and
tenderness (Item C13C). It is not associated with the development of a bullous, necrotic lesion with
eschar. The bite of Loxosceles laeta (the brown recluse spider) may resemble the cutaneous lesion of
anthrax, but it is very painful (Item C13D). Also, symptoms develop shortly after the bite and progress
rapidly within 24 hours. Francisella tularensis is the agent that causes tularemia. Signs and symptoms
develop within 3 to 5 days of exposure and include the abrupt onset of fever, chills, headache, malaise,
and fatigue as well as the development of a progressively enlarging, tender, localized lymphadenopathy
and a red, painful papule (Item C13E) in a region draining into the involved lymph nodes. The signs and
symptoms of Yersinia pestis (plague) develop abruptly and include fever; chills; weakness; headache;
and extremely tender, rapidly enlarging swelling of the lymph nodes of the groin, axilla, or neck.

References:

American Academy of Pediatrics. Anthrax. In: Pickering LK, Baker CJ, Long SS, McMillan JA, eds. Red
Book: 2006 Report of the Committee on Infectious Diseases. 27th ed. Elk Grove Village, Ill: American
Academy of Pediatrics; 2006:208-211

Butler T, Dennis DT. Yersinia species, including plague. In: Mandell GL, Bennett JE, Dolin R, eds.
Mandell, Douglas and Bennett's Principles and Practice of Infectious Diseases. 6th ed. New York, NY:
Elsevier Churchill Livingstone; 2005:2691-2700

Inglesby TV, Henderson DA, Bartlett JG, et al. Anthrax as a biological weapon: medical and public
health management. JAMA. 1999;281:1735-1745. Available at:
http://jama.ama-assn.org/cgi/content/full/281/18/1735

King MD, Humphrey BJ, Wang YF, Kourbatova EV, Ray SM, Blumberg HM. Emergence of
community-acquired methicillin-resistant Staphylococcus aureus USA 300 clone as the predominant
cause of skin and soft-tissue infections. Ann Intern Med. 2006;144:309-317. Abstract available at:
http://www.ncbi.nlm.nih.gov/pubmed/16520471

page 42
2009 PREP SA on CD-ROM

Penn RL. Francisella tularensis (tularemia). In: Mandell GL, Bennett JE, Dolin R, eds. Mandell, Douglas
and Bennett's Principles and Practice of Infectious Diseases. 6th ed. New York, NY: Elsevier Churchill
Livingstone; 2005:2674-2685

Swanson DL, Vetter RS. Bites of brown recluse spiders and suspected necrotic arachnidism. N Engl J
Med. 2005;352:700-707. Extract available at: http://content.nejm.org/cgi/content/extract/352/7/700

page 43
2009 PREP SA on CD-ROM

Critique: 13

Scanning electron micrograph at magnification 6408X demonstrates spores of


Bacillus anthracis. The spores may surive in soil for decades. (Courtesy of
the Centers for Disease Control and Prevention, Public Health Image Library,
Laura Rose)

page 44
2009 PREP SA on CD-ROM

Critique: 13

The classic lesion of cutaneous anthrax is a black eschar with surrounding


swelling, erythema, and induration. The affected area is not painful. (Courtesy
of the Centers for Disease Control and Prevention, Public Health Image
Library)

page 45
2009 PREP SA on CD-ROM

Critique: 13

Initial lesions of staphylococcal skin infection may be small tender papules


that often are attributed to spider bites (yellow arrows). Frequently, lesions
enlarge and develop an overlying pustule (black arrow). (Courtesy of Bernard
Cohen, MD, DermAtlas; www.dermatlas.org)

page 46
2009 PREP SA on CD-ROM

Critique: 13

An eschar with surrounding painful erythema and swelling is observed


following the bite of a brown recluse spider. (Courtesy of M. Smith)

page 47
2009 PREP SA on CD-ROM

Critique: 13

An ulcerated papule may appear at the site of inoculation of Francisella


tularensis. (Courtesy of the Centers for Disease Control and Prevention,
Public Health Image Library, Dr. Thomas F. Sellers)

page 48
2009 PREP SA on CD-ROM

Question: 14
A 3-month-old infant who has a history of renal dysplasia associated with obstructive uropathy has
marked polyuria. He is breastfeeding and receiving supplemental cow milk-based formula. In an effort
to reduce the high urine output, you consider reducing the renal solute load by changing feedings from
the milk-based formula currently being used.

Of the following, the MOST appropriate change is to

A. a hydrolyzed formula containing medium-chain triglycerides

B. a more concentrated (24-kcal) milk-based formula

C. human milk exclusively

D. soy milk-based formula

E. whole cow milk

page 49
2009 PREP SA on CD-ROM

Critique: 14 Preferred Response: C


The infant described in the vignette has polyuria caused by a urinary concentrating defect. The
concentrating defect is the result of tubular damage due to the obstructive uropathy. The inability to
concentrate the urine causes the kidneys to create an "excessive" volume of urine to excrete the solute
load presented to them.
One strategy to reduce polyuria is to reduce the solute burden placed on the kidneys. Potential renal
solute load is affected by intake of protein, sodium, potassium, chloride, and phosphorus. The protein
and phosphorus content are the most important variables when comparing infant feeding regimens.
Human milk possesses a lower potential renal solute load than cow milk or cow milk-based
formulas. Accordingly, the most appropriate change in feeding for the infant in the vignette is to
recommend that the mother stop cow milk formula supplementation and exclusively breastfeed. If
human milk is not available, a "low-solute" cow milk-based formula can be used. A low
calcium-phosphorus formula has the next lowest potential renal solute load compared with human milk.
Cow milk, soy milk-based formula, hydrolyzed formula with medium-chain triglycerides, and 24-kcal
milk-based formula all have greater renal solute loads than human milk.
Renal solute load should also be considered in nephrogenic diabetes insipidus.

References:

Fiorino KN, Cox J. Nutrition and growth. In: Robertson J, Shilkofski N, eds. Harriet Lane Handbook: A
Manual for Pediatric House Officers. 17th ed. Philadelphia, Pa: Elsevier Mosby; 2005:525-608

Hall RT, Carroll RE. Infant feeding. Pediatr Rev. 2000;21:191-200. Available at:
http://pedsinreview.aappublications.org/cgi/content/full/21/6/191

Linshaw MA. Congenital nephrogenic diabetes insipidus. Pediatr Rev. 2007;28:372-380. Available at:
http://pedsinreview.aappublications.org/cgi/content/full/28/10/372

Ziegler EE, Fomon SJ. Potential renal solute load of infant formulas. J Nutr. 1989;119 (12
suppl):1785-1788. Available at: http://jn.nutrition.org/cgi/reprint/119/12_Suppl/1785

page 50
2009 PREP SA on CD-ROM

Question: 15
A mother brings in her 13-month-old daughter for evaluation because her girl developed a perioral rash
and "hives" on two occasions last week. One episode occurred while eating yogurt and another
happened immediately after eating a bagel with cream cheese. She states that her daughter has eaten
other foods such as eggs and bread without problems but is breastfeeding and never has been given
milk-based formulas or cow milk. The infant has been given rice milk, but she became fussy and seems
to prefer breastfeeding. The mother is concerned that her daughter may be allergic to milk but would
like to stop breastfeeding.

Of the following, the BEST advice is to recommend

A. a cow milk food challenge in the clinic

B. avoidance of milk, egg, soy, and wheat products

C. breastfeeding until the child is 3 years old

D. switching to an amino acid-based formula

E. switching to soy milk

page 51
2009 PREP SA on CD-ROM

Critique: 15 Preferred Response: E


Milk protein allergy is an immunoglobulin (Ig) E-mediated food reaction that affects 2% to 3% of infants
within the first postnatal year. Typical symptoms include urticaria, angioedema, atopic dermatitis, and
anaphylaxis. With IgE-mediated reactions, the quantity of milk required to result in a reaction often is
minimal (eg, milk touching the face, a taste of ice cream). Taking a detailed history about the specific
food(s) involved, timing of the onset of symptoms, and type of symptoms is important to distinguish
IgE-mediated reactions, as described for the child in the vignette, from other adverse milk reactions,
such as milk protein enterocolitis and lactose intolerance. Once an IgE-mediated food allergy is
suspected, the clinician should consider allergy skin testing or serum IgE testing for the suspected food.
While awaiting results from either blood testing or allergy consultation for skin testing, the first
reasonable action is to switch to a soy-based formula. Approximately 10% to 15% of infants and
children who have IgE-mediated milk protein allergies may not tolerate soy formula, but this risk applies
to infants younger than 6 months of age. For infants older than 6 months, the risk is closer to 5%.
Nonetheless, the initial soy formula trial should be performed in the clinic. Other acceptable formula
options in this scenario include an extensively hydrolyzed or an amino acid-based formula, although the
unpleasant taste and significantly higher cost can be limiting for many families. Also, because almost all
affected infants can be fed successfully with a soy or extensively hydrolyzed formula, switching initially
to an amino acid-based formula is not required.
Food challenges often are used to assess adverse food reactions, but they generally are reserved
for foods that are unlikely allergens or if the clinical history is inconsistent or vague (eg, a patient who
eats a particular food and does not always have a reaction). Food challenges may result in anaphylaxis
and generally are avoided when the history and testing results support an IgE-mediated reaction.
Infants who have a specific food allergy sometimes are placed incorrectly on restricted diets that
avoid multiple foods. Without a specific history of other adverse food reactions, avoidance of other
foods such as egg or wheat is not recommended. However, parents should be counseled that children
can develop other food allergies and should monitor their children during ingestion of other common
food allergens.
Breastfeeding until age 3 years old is an option that is not preferred by the mother in the vignette.
Although most IgE-mediated cow milk allergies resolve by 3 years of age, milk protein allergy can
persist past 5 years of age in up to 20% of affected children.

References:

American Academy of Pediatrics Committee on Nutrition. Soy protein-based formulas:


recommendations for use in infant feeding. Pediatrics. 1998;101:148-153. Available at:
http://pediatrics.aappublications.org/cgi/content/full/101/1/148

Bhatia J, Greer F, and the Committee on Nutrition. The use of soy protein-based formulas in infant
feeding. Pediatrics. 2008;121:1062-1068. Available at:
http://pediatrics.aappublications.org/cgi/content/full/121/5/1062

Klemola T, Vanto T, Juntunen-Backman K, Kalimo K, Korpela R, Varjonen E. Allergy to soy formula and
to extensively hydrolyzed whey formula in infants with cow's milk allergy: a prospective, randomized
study with a follow-up to the age of 2 years. J Pediatr. 2002;140:219-224. Abstract available at:
http://www.ncbi.nlm.nih.gov/ pubmed/11865274

Saarinen KM, Pelkonen AS, Mäkelä MJ, Savilahti E. Clinical course and prognosis of cow's milk allergy
are dependent on milk-specific IgE status. J Allergy Clin Immunol. 2005;116:869-875. Abstract

page 52
2009 PREP SA on CD-ROM

available at: http://www.ncbi.nlm.nih.gov/pubmed/16210063

Sampson HA, Leung DYM. Adverse reactions to foods. In: Kleigman RM, Behrman RE, Jenson HB,
Stanton BF, eds. Nelson Textbook of Pediatrics. 18th ed. Philadelphia, Pa: Saunders Elsevier;
2007:986-989

page 53
2009 PREP SA on CD-ROM

Question: 16
A 16-year-old girl is brought to the emergency department after being found unresponsive in her
bedroom. Her parents report finding a note in which she wrote of "wanting to end the pain." In addition,
they found several empty, unlabeled pill vials on her dresser. On physical examination, the girl is
responsive only to painful stimuli. Her heart rate is 60 beats/min, respiratory rate is 16 breaths/min,
blood pressure is 90/60 mm Hg, and oxygen saturation is 92%. Her pupils are 3 mm, equal in size, and
sluggishly reactive. The remainder of findings on her physical examination are normal.

Of the following, the MOST important diagnostic test to obtain when evaluating this patient is a

A. carboxyhemoglobin concentration

B. complete blood count

C. serum acetaminophen concentration

D. serum ammonia concentration

E. serum osmolality

page 54
2009 PREP SA on CD-ROM

Critique: 16 Preferred Response: C


Treatment of a patient who has ingested an unknown substance or substances, such as the one
described in the vignette, should focus initially on stabilization of vital functions; treatment of the
patient's symptoms; and identification of agents that are potentially fatal, have delayed clinical toxicity,
or for which antidotal therapy is indicated. The evaluation should include a screening history to identify
circumstances surrounding the event, potential environmental exposures, available medications or
toxins, and previous medical or psychiatric history. The physical examination should focus on vital sign
abnormalities, pupillary size and reactivity, skin findings, and mental status. These components are
likely to yield the most useful clues to toxins that have identifiable symptom complexes (Item C16A).
Laboratory testing should assess the patient's acid/base status, oxygenation and ventilation, glucose
concentration, and anion gap. Qualitative urine drug testing may be obtained but has significant
limitations, including the small number of drugs tested (typically drugs of abuse) and screening
thresholds that may produce false-negative results. Therefore, the results of urine toxicologic screening
rarely affect treatment plans. Quantitative drug concentrations should be measured based on the
information gleaned from the initial history, physical examination, and screening laboratory testing,
although acetaminophen and salicylate concentrations should be measured for every patient who has
an unknown, mixed, or intentional ingestion. Acetaminophen is of particular concern because it causes
few initial symptoms, is a common agent in adolescent and adult intentional ingestions, and may
require antidotal therapy to prevent potential fatal liver damage. Abdominal radiographs may be
considered to examine the patient for radio-opaque substances (Item C16B).
A complete blood count is not likely to provide information leading to the identification of a toxin.
Measurement of serum ammonia may be indicated if acetaminophen is identified as the ingested agent
because toxicity can lead to hepatic damage. Carboxyhemoglobin should be measured in the patient
whose history is suggestive of carbon monoxide exposure. Increased serum osmolality may provide
indirect evidence of alcohol ingestion, although a quantitative serum test for alcohols is more useful in
this setting.

References:

Erikson TB, Thompson TM, Lu JJ. The approach to the patient with an unknown overdose. Emerg Med
Clin North Am. 2007;25:249-281. Abstract available at: http://www.ncbi.nlm.nih.gov/pubmed/17482020

Lavallee M, Olsson J Jr, Cheng TL. In brief: unknown poison. Pediatr Rev. 2004;25:370-371. Available
at: http://pedsinreview.aappublications.org/cgi/content/full/25/10/370

McKay CA Jr. Can the laboratory help me? Toxicology laboratory testing in the possibly poisoned
pediatric patient. Clin Pediatr Emerg Med. 2005;6:116-122

Valez LI, Shepherd JG, Goto CS. Approach to the child with occult toxic exposure. UpToDate Online
15.3. 2008. Available for subscription at:
http://www.utdol.com/utd/content/topic.do?topicKey=ped_tox/3023&selectedTitle=4~150&source=searc
h_result

page 55
2009 PREP SA on CD-ROM

Critique: 16

page 56
2009 PREP SA on CD-ROM

Critique: 16

page 57
2009 PREP SA on CD-ROM

Question: 17
You are following a 3-month-old infant who was born at 30 weeks’ gestation, underwent a distal ileal
resection for necrotizing enterocolitis at 2 weeks of age, and subsequently was placed on parenteral
nutrition for 2 months. The baby has residual cholestasis from the parenteral nutrition (total bilirubin, 5.0
mg/dL [85.5 mcmol/L]; direct bilirubin, 3.0 mg/dL [51.3 mcmol/L]). Currently, she is receiving a cow milk
protein hydrolysate formula concentrated to 24 kcal/oz (0.8 kcal/mL). You are considering adding a
dietary supplement to increase the caloric density of the formula.

Of the following, the supplement that is the MOST likely to be tolerated and cause less diarrhea in this
infant is

A. flaxseed oil

B. medium-chain triglyceride oil

C. olive oil

D. omega-3 polyunsaturated fatty acid (fish oil)

E. soybean oil

page 58
2009 PREP SA on CD-ROM

Critique: 17 Preferred Response: B


Infants who have chronic illnesses may have specialized nutritional requirements and often do not
tolerate the standard 20-kcal/oz formula given to healthy term infants. For example, children who have
some forms of congenital heart disease or renal disease may require a more concentrated formula
because the standard formula may lead to volume overload. Children who have intestinal disease or
malabsorption, such as the child described in the vignette, also may require a more concentrated
formula to decrease the likelihood of feeding intolerance or diarrhea.
Formula may be concentrated by increasing the concentration of protein, carbohydrate, or fat.
Perhaps the easiest method of increasing the caloric density of a formula is to mix more powder with
the same amount of water. For example, four scoops of most commercially available formulas mixed
with 8 oz of water yields standard 0.67-kcal/mL (20-kcal/oz) formula, but mixing five scoops in 8 oz
results in 0.83-kcal/mL (25-kcal/oz) formula. However, exceeding 25-kcal/oz formula by increasing the
amount of powder may yield too high a concentration of protein, which could result in an excessive
renal solute load. For this reason, carbohydrate and fat supplements are available to concentrate infant
and toddler feedings further. The most common adverse effect of carbohydrate supplements is
diarrhea, and lipid supplements may cause either diarrhea or delayed gastric emptying. For these
reasons, caution is recommended when increasing the caloric density of a formula, especially when the
caloric density is increased to greater than 1 kcal/mL (30 kcal/oz).
Because the patient in the vignette has had an ileal resection and cholestasis, the best fat
supplement for him is oil composed of medium-chain triglycerides (MCT oil). MCT oil can be absorbed
directly across the enterocyte and does not require intraluminal digestion by bile acids. In contrast, soy,
olive, flaxseed, and fish oils are long-chain fatty acids that require bile acids for digestion and might
cause diarrhea in a child who has cholestasis and ileal resection.

References:

Courtney E, Grunko A, McCarthy T. Enteral nutrition. In: Hendricks KM, Duggan C. Manual of Pediatric
Nutrition. 4th ed. Hamilton, Ontario, Canada: BC Decker; 2005:252-316

Suchy FJ. Neonatal cholestasis. Pediatr Rev. 2004;25:388-396. Available at:


http://pedsinreview.aappublications.org/cgi/content/full/25/11/388

page 59
2009 PREP SA on CD-ROM

Question: 18
You have admitted a 750-g male infant to the neonatal intensive care unit (NICU) for treatment of
respiratory distress and presumed sepsis. The Apgar scores were 1, 5, and 7 at 1, 5, and 10 minutes,
respectively. The infant received one dose of exogenous surfactant in the delivery room. In the NICU,
the infant is being cared for on a radiant warmer. At 4 hours after birth, physical examination reveals a
temperature of 97.0°F (36.1°C), heart rate of 180 beats/min, respiratory rate of 40 breaths/min (assisted
breaths on the ventilator), blood pressure of 45/27 mm Hg, mean arterial blood pressure of 30 mm Hg,
and pulse oximetry of 92%. The infant is receiving synchronized intermittent mechanical ventilation with
a peak inflation pressure of 18 cm H2O over a positive end-expiratory pressure of 4 cm H2O at a rate of
40 breaths/min and an FiO2 of 0.40. Umbilical catheters are present in the umbilical artery and vein. On
physical examination, you note a soft, flat anterior fontanelle. You auscultate equal mechanical breath
sounds bilaterally over the chest and note minimal subcostal retractions. The skin is thin and somewhat
moist, and many veins are visible through it. The ears are flattened against the cranium and lack any
cartilage or recoil. There is a small phallus and an empty scrotum. The infant is laying on the warmer
with legs and arms extended. The neuromotor tone is decreased, and the infant does not appear to be
very active, but he does respond to tactile stimuli with movement of the arms and legs in seemingly
random and purposeless activity.

Of the following, the MOST important next step is to

A. administer another dose of exogenous surfactant

B. consult the urologist for cryptorchidism

C. increase the inspired oxygen concentration

D. move the infant into an isolette incubator

E. obtain head ultrasonography to evaluate for any hemorrhage

page 60
2009 PREP SA on CD-ROM

Critique: 18 Preferred Response: D


The extremely low gestational-age newborn (ELGAN) whose birthweight is less than 1,000 g may
present with physical examination findings indicative of marked physical and neurodevelopmental
immaturity. These findings have been studied and correlated with gestational age in scoring systems
such as those published by Dubowitz in 1980 and Ballard in 1979 and (revised) 1991. Even the best
efforts to estimate the gestational age of very low-birthweight and immature infants at dates less than
28 completed weeks of gestation by maternal last menstrual period and obstetric ultrasonography may
be imprecise and tend to overestimate maturity (gestational age) by 2 weeks. Nevertheless, the thin,
moist, transparent skin; flattened thin ears without cartilage or recoil; and small phallus and empty
scrotum described for the infant in the vignette indicate a degree of physical immaturity consistent with
28 weeks' gestation or less. Similarly, the low neuromotor tone, absence of flexed posture, relative
inactivity, and random purposeless flailing of extremities with tactile stimulation indicate significant
neurologic immaturity.
Moving the infant into an isolette incubator is an important next step in the care of this ELGAN. The
absolute benefit of using a radiant warmer bed (access to the infant to conduct early examinations,
admission nursing care, and procedures such as placement of umbilical catheters) is eclipsed by its
associated increased ambient convection, evaporative heat loss, and transcutaneous water loss
compared with the contained, convectively heated, and humidified environment of the isolette. The
isolette environment still allows access, visibility, and treatment with phototherapy, but with less
insensible water loss or variability in patient temperature.
This ELGAN is being treated for respiratory distress syndrome and has had a good response to
exogenous surfactant administration, oxygen, and assisted ventilation. At 4 hours of postnatal age, it is
too early to administer a second dose of surfactant. Developmentally cryptorchid testes may yet
descend into the scrotal sac in coming weeks. An oxygen saturation of 92% is good for this infant, and
increasing the inspired oxygen concentration may be harmful, contributing to oxidative injury of
pulmonary tissues or retinopathy of prematurity. Cranial ultrasonography is indicated in this sick and
preterm infant, but without clinical indications of hemorrhage, shock, and seizure or encephalopathy,
such a study is performed best at 3 to 7 days of postnatal life.

References:

Donovan EF, Tyson JE, Ehrenkranz RA, et al. Inaccuracy of Ballard scores before 28 weeks' gestation.
National Institute of Child Health and Human Development Neonatal Research Network. J Pediatr.
1999;135:147-152. Abstract available at: http://www.ncbi.nlm.nih.gov/pubmed/10431107

Marín GMA, Martín Moreiras J, Llitera Fleixas G, et al. Assessment of the new Ballard score to estimate
gestational age [in Spanish]. An Pediatr (Barc). 2006;64:140-145. English abstract available at:
http://www.ncbi.nlm.nih.gov/pubmed/16527066

Sedin G. The thermal environment of the newborn infant. In: Martin RJ, Fanaroff AA, Walsh MC, eds.
Fanaroff and Martin's Neonatal-Perinatal Medicine. 8th ed. Philadelphia, Pa: Mosby Elsevier;
2006:585-596

Thilo EH, Rosenberg AA. The newborn infant. In: Hay WW Jr, Levin MJ, Sondheimer JM, Deterding
RR, eds. Current Pediatric Diagnosis & Treatment. 18th ed. New York, NY: The McGraw-Hill
Companies, Inc; 2007:chap 1

page 61
2009 PREP SA on CD-ROM

Question: 19
You are called to the newborn nursery to evaluate a 1-day-old girl whose hands and feet are blue. She
was born at term via a cesarean section, and there were no complications. Apgar scores were 9 at both
1 and 5 minutes. Her respiratory rate is 40 breaths/min, heart rate is 140 beats/min, and blood pressure
is normal. Pulse oximetry is 98% on room air. Her lungs are clear, and there is no murmur. Her lips are
pink, but her hands and feet are cyanotic (Item Q19), and capillary refill is less than 2 seconds.

Of the following, the MOST likely cause of her peripheral cyanosis is

A. cold environment

B. polycythemia

C. retained fetal lung fluid

D. sepsis

E. transposition of the great arteries

page 62
2009 PREP SA on CD-ROM

Question: 19

Cyanosis of the feet, as exhibited by the infant in the vignette. (Courtesy of the
Media Lab at Doernbecher)

page 63
2009 PREP SA on CD-ROM

Critique: 19 Preferred Response: A


Cyanosis is a common finding in newborns. The first step in the evaluation of an infant who exhibits
cyanosis is to determine whether it is central or peripheral (acrocyanosis). The infant described in the
vignette has clinical features suggestive of acrocyanosis, a bluish discoloration of the hands and feet
(Item C19A) in response to vasomotor instability or cold environment. In this condition, the lips and
mucous membranes are spared, although the perioral area may be affected. The extremities may be
cool to touch. Acrocyanosis is believed to be due to vasoconstriction of small arterioles and resolves in
the first few postnatal months. Infants who have acrocyanosis require no further evaluation; parental
reassurance is all that is needed.
Central cyanosis is the bluish discoloration of the tissues best observed in the lips, tongue, mucous
membranes, and occasionally nail beds (Item C19B). It occurs when the concentration of deoxygenated
(reduced) hemoglobin exceeds 4.0 to 6.0 g/dL (40.0 to 60.0 g/L) within the capillary bed. In an individual
who has normal hemoglobin, cyanosis usually becomes apparent when the oxygen saturation drops
below 75%-80%. Causes of central cyanosis in the neonatal period typically are related to cyanotic
heart disease, such as transposition of the great arteries, or respiratory disease, such as pneumonia
and retained fetal lung fluid (transient tachypnea of the newborn). The absence of central cyanosis, a
cardiac murmur, and respiratory distress for the infant in the vignette makes these diagnoses less likely
than acrocyanosis of infancy.
Polycythemia, defined as a venous hematocrit of at least 65% (0.65), causes a deep reddish-purple
discoloration of the skin and mucous membranes. Infants may be otherwise asymptomatic or may
exhibit lethargy, anorexia, or respiratory distress. Polycythemia does not cause either peripheral or
central cyanosis per se, but it does affect the percent saturation at which cyanosis becomes clinically
apparent. Cyanosis in infants who have polycythemia occurs at higher oxygen saturations than in those
who have anemia. For example, if an infant has a hemoglobin of 24.0 g/dL (240.0 g/L), he is likely to
appear cyanotic when his oxygen saturation reaches 87%. In an infant who has a hemoglobin of 12.0
g/dL (120.0 g/L), cyanosis may not be seen until the oxygen saturation falls to 75%. Newborns who
have sepsis may develop cyanosis as a component of multisystem involvement, but other signs, such
as respiratory distress, poor peripheral perfusion, lethargy, or poor feeding, also are evident.

References:

Bernstein D. Evaluation of the cardiovascular system: history and physical examination. In: Kliegman
RM, Behrman RE, Jenson HB, Stanton BF, eds. Nelson Textbook of Pediatrics. 18th ed. Philadelphia,
Pa: Saunders Elsevier; 2007:1857-1863

Sasidharan P. An approach to diagnosis and management of cyanosis and tachypnea in term infants.
Pediatr Clin North Am. 2004;51:999-1021. Abstract available at:
http://www.ncbi.nlm.nih.gov/pubmed/15275985

page 64
2009 PREP SA on CD-ROM

Critique: 19

Acrocyanosis is a blue discoloration of the hands and feet. (Courtesy of the


Media Lab at Doernbecher)

page 65
2009 PREP SA on CD-ROM

Critique: 19

Central cyanosis, as exhibited by this infant who has tetralogy of Fallot, is


observed best in highly vascularized areas, such as the lips, tongue, mucous
membranes, and nail beds. (Courtesy of M. Rimsza)

page 66
2009 PREP SA on CD-ROM

Question: 20
A frustrated mother requests referral of her 15-month-old child to an allergy and asthma specialist
because the boy never seems to have stopped coughing and wheezing over the 6 months of the past
respiratory virus season. During the history taking, the mother states that the baby only occasionally is
exposed to wood smoke at the family’s barbecue restaurant and to cosmetic chemicals used at the
grandmother’s hair salon. Both the father and grandfather smoke cigarettes in the home.

Of the following, the environmental exposure that is MOST likely to be causing the child’s respiratory
symptoms is exposure to

A. cigarette smoke

B. cleaning fluids

C. dust mites

D. hairspray

E. wood smoke

page 67
2009 PREP SA on CD-ROM

Critique: 20 Preferred Response: A


The child described in the vignette may have multiple triggers for his wheezing exacerbations, although
tobacco smoke probably is the one to which he is exposed most often that should be diminished.
Well-known tobacco toxins include nicotine, carbon monoxide, formaldehyde, hydrogen cyanide, sulfur
dioxide, nitrogen oxide, ammonia, polycyclic aromatic hydrocarbons, and the nitrosamines. These
substances produce both irritant and immunologic effects on the respiratory tract. It is important to note
that smoking cessation is difficult, and most pediatricians have little training in the process. However,
there is evidence that a brief discussion of smoking cessation in the context of medical visits is
effective.
Wood smoke is another significant irritant to the respiratory tract; in addition to chemical irritants,
wood smoke contains particulate matter. Assuming that the family's restaurant kitchen meets current
standards for air filtration/exhaust, the child's exposure should be minimal. However, outdoor barbecue
pits that discharge smoke directly into the environment should be avoided.
Exposure to cooking spray/oils at the restaurant and cosmetic chemicals such as hairspray at the
salon also should be avoided, although the use of volatile organic compounds, including
chlorofluorocarbons in hairspray, permanent wave solution, cleaning solutions, and cooking sprays, has
decreased sharply since passage of the Clean Air Act and other Environmental Protection Agency
measures in the 1990s. Material Safety Data Sheets for most hairsprays, as well as other cosmetics
and cleaning fluids, reveal little inhalation toxicity, except in the case of prolonged exposure under
improper ventilation.
Although dust mite exposure may contribute to asthma exacerbation, there is no evidence that
environmental control can reduce the concentration of mites or exacerbations related to them. Finally,
serial respiratory virus infections themselves may contribute significantly to this child's history of
wheezing and cough.
The child in the vignette may benefit from therapeutic advances in asthma management such as
those contained in the National Heart, Lung and Blood Institute Guidelines for asthma, including
attention to number of symptomatic days, use of inhaled corticosteroids, and use of a metered dose
inhaler with spacer for proper delivery of medication, which may be more effective than nebulization in
older infants and children.

References:

Brunnhuber K, Cummings KM, Feit S, Sherman S, Woodcock J.Putting evidence into practice: smoking
cessation. BMJ Clinical Evidence. 2007. Available for subscription at:
http://clinicalevidence.bmj.com/ceweb/resources/index.jsp

Kum-Nji P, Meloy L, Herrod HG. Environmental tobacco smoke exposure: prevalence and mechanisms
of causation of infections in children. Pediatrics. 2006;117:5:1745-1754. Available at:
http://pediatrics.aappublications.org/cgi/content/full/117/5/1745

Roseby R, Waters E, Polnay A, Campbell R, Webster P, Spencer N. Family and carer smoking control
programmes for reducing children's exposure to environmental tobacco smoke. Cochrane Database
Syst Rev. 2003;3:CD001746. Available at: http://www.cochrane.org/reviews/en/ab001746.html

Stein RT, Holberg CJ, Sherrill D, et al. Influence of parental smoking on respiratory symptoms during
the first decade of life: The Tucson Children's Respiratory Study. Am J Epidemiol. 1999;149:1030-1037.
Available at: http://aje.oxfordjournals.org/cgi/reprint/149/11/1030

page 68
2009 PREP SA on CD-ROM

U.S. Environmental Protection Agency. Health effects of wood smoke. Available at:
http://www.epa.gov/woodstoves/healtheffects.html

U.S. Environmental Protection Agency. National volatile organic compound emission standards for
consumer products. Available at:
http://www.epa.gov/fedrgstr/EPA-AIR/1998/September/Day-11/a22660.htm

page 69
2009 PREP SA on CD-ROM

Question: 21
You are evaluating a newborn 6 hours after his birth. Labor and delivery were uncomplicated, but
amniocentesis performed during the pregnancy revealed trisomy 21. Fetal echocardiography at 20
weeks’ gestation showed normal findings. The infant currently is sleeping and is well-perfused, with a
heart rate of 140 beats/min and no audible murmurs. His physical features are consistent with Down
syndrome.

Of the following, the MOST appropriate diagnostic study to perform is

A. barium swallow

B. cervical spine radiography

C. echocardiography

D. head ultrasonography

E. radiography of the abdomen

page 70
2009 PREP SA on CD-ROM

Critique: 21 Preferred Response: C


Congenital heart defects (CHDs) are the most common of the congenital anomalies, occurring with an
incidence of approximately 5 to 8 per 1,000 live births (0.5% to 0.8%). The incidence of CHD is greater
in stillbirths and there is an increased incidence of CHD in those who have aneuploidy. It is believed by
some that the abnormal chromosomal composition rather than the cardiac abnormality is responsible
for the fetal demise of those who have CHDs. The strong association between chromosomal
abnormality and CHD has been shown in a number of studies that focus on identification of fetal cardiac
abnormality. A number of well-defined chromosomal anomalies are associated with CHD (Item C21).
Early and accurate diagnosis of CHD is important in counseling parents of children in whom a
chromosomal abnormality or syndrome is suspected. Such diagnoses may have significant effects on
the health and well-being of the newborns because some CHDs may require ductal patency for
perfusion of either the systemic or pulmonary circulation. Therefore, echocardiography now is
considered an important component of the routine health supervision of infants who have Down
syndrome and other syndromes predisposing to structural heart defects.
Although results of fetal echocardiography for the infant in the vignette were interpreted as normal,
not all CHDs can be diagnosed routinely with fetal echocardiography because of the shunting pathways
of the fetal circulation. For example, persistent patency of the ductus arteriosus and secundum atrial
septal defects are diagnosed postnatally. Small ventricular septal defects frequently are not seen during
fetal echocardiography because the pressure in the right and left ventricles are equal due to the ductus
arteriosus, resulting in minimal flow across the defect prenatally.
Given the strong association between Down syndrome and CHD, echocardiography should be
performed in the newborn described in the vignette. Although children born with Down syndrome have
an increased incidence of duodenal atresia and other types of gastrointestinal obstruction, a barium
swallow is not an appropriate initial test for an asymptomatic newborn. Cervical radiography, beginning
at age 3 years, is important because of the risk of atlantoaxial (C1-C2) subluxation. There is no
indication for routine head ultrasonography or abdominal radiography in an otherwise asymptomatic
newborn who has Down syndrome.

References:

Committee on Genetics. Health supervision for children with Down syndrome. Pediatrics.
2001;107:442-449. Available at: http://pediatrics.aappublications.org/cgi/content/full/107/2/442

Silberbach M, Hannon D. Presentation of congenital heart disease in the neonate and young infant.
Pediatr Rev. 2007;28:123-131. Available at:
http://pedsinreview.aappublications.org/cgi/content/full/28/4/123

Tennstedt C, Chaoui R, Körner H, Dietel M. Spectrum of congenital heart defects and extracardiac
malformations associated with chromosomal abnormalities: results of a seven year necropsy study.
Heart. 1999;82:34-39. Abstract available at: http://www.ncbi.nlm.nih.gov/pubmed/10377306

page 71
2009 PREP SA on CD-ROM

Critique: 21

page 72
2009 PREP SA on CD-ROM

Question: 22
A 4-year-old boy presents with headache and difficulty walking. On physical examination, he is afebrile,
all growth parameters are within normal limits, and his mentation appears normal. The optic discs are
clearly visible and appear normal. He has normal eye position in primary gaze but cannot abduct his
right eye fully. He has normal tone, strength, and reflexes in his upper limbs, but has bilateral
hyperreflexia at the knees and ankle clonus. On gait examination, he toe-walks.

Of the following, the MOST important next step is to obtain

A. computed tomography scan of the head

B. electromyography/nerve conduction studies of the legs

C. lumbar puncture

D. magnetic resonance imaging of the thoracolumbar spine

E. visual evoked potentials

page 73
2009 PREP SA on CD-ROM

Critique: 22 Preferred Response: A


The boy described in the vignette presents with a very concerning constellation of symptoms and signs
requiring urgent evaluation of the central nervous system. The first step in the diagnostic process is to
localize the problem to the proper level of the nervous system: brain, brainstem/cerebellum, spinal cord,
nerve, junction, or muscle. Such a determination not only allows for urgent appropriate diagnosis and
treatment but also reduces unnecessary discomfort, risks, and costs of inappropriate diagnostic testing.
Consultation with a neurologist can be useful before ordering testing. Headache, gait disturbance
(toe-walking), and acquired ocular misalignment (right eye cannot move to the right) localizes the
problem to the central nervous system and raises concerns for both hydrocephalus and a brainstem
lesion.
Computed tomography (CT) scan of the head is the preferred neuroimaging technique for this
patient. Although brain magnetic resonance imaging (MRI) can document the brainstem/posterior fossa
better, it may not be readily available. Moreover, a head CT scan in the emergency department is
adequate to rule out hydrocephalus that requires emergent neurosurgical consultation. Unfortunately, a
common cause for hydrocephalus in a child of this age is a brainstem or cerebellar neoplasm such as
an astrocytoma, glioma, medulloblastoma, or ependymoma. When these tumors enlarge in the
posterior fossa, they can obstruct the flow of cerebrospinal fluid and cause acute hydrocephalus, which
is a neurosurgical emergency.
Primary headache disorders such as migraine do not often present before the age of 6 years and
are not accompanied by the neurologic findings described in the vignette. Toe-walking is a nonspecific
finding that can occur in the context of relatively benign delayed development, although if this
represents a change for a child, an upper motor neuron (brain/spinal cord) lesion must be ruled out. The
ankle clonus described for the boy in the vignette mandates this approach. Similarly, inability to abduct
one eye can be congenital, but if this is a new finding, a brain or brainstem lesion must be ruled out. In
this case, headache and the oculomotor findings make a brain or brainstem lesion likely.
Gait impairment and hyperreflexia only in the legs can result from hydrocephalus or can localize to
the mid- to lower spinal cord. A spinal cord lesion would not explain the headache, and bilateral motor
findings without sensory, bowel, or bladder involvement are uncommon for a spinal cord lesion.
Therefore, MRI of the spine is not needed. Similarly, the hyperreflexia is not evidence of a nerve or
muscle problem, obviating the need for electromyography and nerve conduction studies. Lumbar
puncture is therapeutic for headache due to pseudotumor cerebri, which could present with this
constellation of symptoms, but this degree of gait abnormality would be uncommon in pseudotumor,
and the presence of true hydrocephalus or an intracranial mass must be excluded with a head CT or
brain MRI prior to lumbar puncture. Visual evoked potentials occasionally are used to determine
whether the axons from eye to occipital cortex function normally, but such a test generally is performed
when demyelinating disorders are suspected.

References:

Avellino AM. Hydrocephalus. In: Singer HS, Kossoff EH, Hartman AL, Crawford TO, eds. Treatment of
Pediatric Neurologic Disorders. Boca Raton, Fla: Taylor & Francis; 2005:25-36

Garton HJ, Piatt JH Jr. Hydrocephalus. Pediatr Clin North Am. 2004;51:305-325. Abstract available at:
http://www.ncbi.nlm.nih.gov/pubmed/15062673

Kestle JR. Pediatric hydrocephalus: current management. Neurol Clin. 2003;21:883-895. Abstract
available at: http://www.ncbi.nlm.nih.gov/pubmed/14743654

page 74
2009 PREP SA on CD-ROM

Kuttesch J Jr, Ater JL. Brain tumors in childhood. In: Behrman RE, Kliegman RM, Jenson HB, Stanton
BF, eds. Nelson Textbook of Pediatrics. 18th ed. Philadelphia, Pa: Saunders Elsevier; 2007:2128-2136

Piatt JH Jr. Recognizing neurosurgical conditions in the pediatrician's office. Pediatr Clin North Am.
2004;51:237-270. Abstract available at: http://www.ncbi.nlm.nih.gov/pubmed/15062671

page 75
2009 PREP SA on CD-ROM

Question: 23
Parents who are new to your area bring in their 3-year-old daughter for evaluation because they are
concerned about her delayed speech. They say that she uses about 50 single words. The girl has had
tetralogy of Fallot repaired surgically and recurrent upper respiratory tract infections with otitis media,
for which tympanostomy tubes have been placed. Findings on physical examination include
microcephaly, underfolded pinnae, a broad nasal bridge, cleft uvula, and a small chin. In addition, the
child’s speech has a hypernasal quality. The family history is negative for birth defects and
developmental delays.

Of the following, the contiguous gene deletion syndrome that BEST fits this child’s features is

A. Angelman

B. Beckwith-Wiedemann

C. 4p-

D. Prader-Willi

E. 22q11

page 76
2009 PREP SA on CD-ROM

Critique: 23 Preferred Response: E


The contiguous gene deletion syndromes comprise a group of disorders defined by characteristic
dysmorphisms or behaviors in association with a submicroscopic chromosome deletion that is detected
using fluorescence in situ hybridization (FISH) or microarray technologies. In each case, the deleted
segment contains multiple genes that play varying roles in producing a particular phenotype. Angelman,
Beckwith-Wiedemann, Wolf-Hirschhorn (4p-), Prader-Willi, and 22q11 deletion syndromes are among
the best delineated contiguous gene deletion syndromes.
The child described in the vignette has features most consistent with 22q11 (the long arm of
chromosome 22 at band 11) deletion syndrome, which occurs in approximately 1 in 4,000 births,
making it the most common of the contiguous gene deletion syndromes. Confusion has arisen
regarding the relationship of 22q11 deletion syndrome to the DiGeorge and velo-cardio-facial
syndromes. It now is known that the condition described by DiGeorge in 1965, consisting of
hypoplastic-to-absent thymus and parathyroid glands together with conotruncal heart defects, is due to
22q11 deletion in most cases. Similarly, the condition described by Shprintzen and associates in 1978
that has come to be called velo-cardio-facial syndrome, which includes cleft palate, speech disorder,
broad nasal root, immune dysfunction, and learning difficulties, is also caused by 22q11 deletion. The
two disorders represent different manifestations of the same submicroscopic deletion.
22q11 deletion syndrome should be suspected in any child who has a conotruncal heart defect (eg,
tetralogy of Fallot, double-outlet right ventricle, truncus arteriosus), especially if accompanied by
developmental delay, dysmorphic features, or recurrent upper respiratory tract infections. Testing for
the condition involves sending blood for FISH (it must be specified that the region of interest is 22q11)
or for microarray analysis, assuming the array covers the 22q11 region.
Angelman and Prader-Willi syndromes (AS and PWS, respectively) are caused by aberrations
(usually contiguous gene deletions) on the long arm of chromosome 15 at bands 11 through 13. This
region of the human genome is "imprinted," such that when the paternal region is deleted, the result is
PWS, and when the maternal region is deleted, the result is AS. Children who have PWS typically
present in infancy with poor feeding, hypotonia, mild facial dysmorphisms, and undescended testes with
or without micropenis in boys. At the age of 2 to 3 years, affected children start eating excessively and
gain weight rapidly, becoming obese (Item C23A). They are mildly to moderately mentally retarded and
often exhibit excessive skin picking and a facility for jigsaw puzzles. Children who have AS typically
appear normal at birth, but by 1 to 2 years of age, they have acquired microcephaly and severe speech
delay (Item C23B). During this same period, most (~85%) develop generalized seizures. Children who
have AS have an ataxic gait and jerky arm movements, resulting in a "puppet-like" gait. They are
typically nonverbal and severely mentally retarded and have paroxysms of inappropriate laughter.
4p- (4p minus), previously called Wolf-Hirschhorn syndrome, is a multiple congenital anomaly
syndrome characterized by prenatal-onset growth deficiency, prominent glabella, microcephaly, and
cleft lip+/-cleft palate. Profound mental retardation is the rule. Affected individuals may have a
cytogenetically detectable deletion on the short arm of chromosome 4 or they may have a
submicroscopic deletion involving 4p16.3.
Beckwith-Wiedemann syndrome (BWS) is characterized by pre- and postnatal overgrowth,
hypoglycemia in the newborn period, large tongue (Item C23C), unusual earlobe creases, omphalocele
in some (Item C23D), and organomegaly. Affected children are at increased risk for certain neoplasias,
including hepatoblastoma and Wilms tumor, making it important to follow a tumor surveillance protocol
up to 7 to 10 years of age. Intelligence typically is normal. The genetics of BWS are complex; the
condition is caused by submicroscopic deletions/alterations in genes located at 11p15, which is a highly
imprinted region.
The management of each of the contiguous gene deletion syndromes involves referrals to multiple
specialists, depending on the systems involved, together with anticipatory guidance based on natural

page 77
2009 PREP SA on CD-ROM

history information.

References:

Battaglia A, Carey JC, Wright TJ. Wolf-Hirschhorn syndrome. GeneReviews. 2006. Available at:
http://www.geneclinics.org/servlet/access?db=geneclinics&site=gt&id=8888891&key=OvKiicpzcfvnc&gry
=&fcn=y&fw=vH7o&filename=/profiles/whs/index.html

Bishara N, Clericuzio CL. Common dysmorphic syndromes in the NICU. NeoReviews. 2008;9:e29-e38.
Available for subscription at: http://neoreviews.aappublications.org/cgi/content/full/9/1/e29

Cassidy SB, Schwartz S. Prader-Willi syndrome. GeneReviews. 2006. Available at:


http://www.geneclinics.org/servlet/access?db=geneclinics&site=gt&id=8888891&key=OvKiicpzcfvnc&gry
=&fcn=y&fw=vWzr&filename=/profiles/pws/index.html

Lin RJ, Cherry AM, Bangs CD, Hoyme HE. FISHing for answers: the use of molecular cytogenetic
techniques in neonatology. NeoReviews. 2003;4:e94-e98. Available for subscription at:
http://neoreviews.aappublications.org/cgi/content/full/4/4/e94

McDonald-McGinn DM, Emanuel BS, Zackai EH. 22q11.2 deletion syndrome. GeneReviews. 2005.
Available at: http://www.ncbi.nlm.nih.gov/bookshelf/br.fcgi?book=gene&part=gr_22q11deletion

Shprintzen RJ. Velo-cardio-facial syndrome. In: Cassidy SB, Allanson JE, eds. Management of Genetic
Syndromes. 2nd ed. Hoboken, NJ: Wiley-Liss; 2005:615-632

Shuman C, Amith AC, Weksberg R. Beckwith-Wiedemann syndrome. GeneReviews. 2005. Available


at:
http://www.geneclinics.org/servlet/access?db=geneclinics&site=gt&id=8888891&key=OvKiicpzcfvnc&gry
=&fcn=y&fw=0F2J&filename=/profiles/bws/index.html

Weksberg R, Shuman C. Beckwith-Wiedemann syndrome and hemihyperplasia. In: Cassidy SB,


Allanson JE, eds. Management of Genetic Syndromes. 2nd ed. Hoboken, NJ: Wiley-Liss; 2005:101-116

Williams CA. Angelman syndrome. In: Cassidy SB, Allanson JE, eds. Management of Genetic
Syndromes. 2nd ed. Hoboken, NJ: Wiley-Liss; 2005:53-62

Williams CA, Driscoll DJ. Angelman syndrome. GeneReviews. 2007. Available at:
http://www.geneclinics.org/servlet/access?db=geneclinics&site=gt&id=8888891&key=OvKiicpzcfvnc&gry
=&fcn=y&fw=tkPG&filename=/profiles/angelman/index.html

page 78
2009 PREP SA on CD-ROM

Critique: 23

Prader-Willi syndrome is characterized by short stature, obesity,


hypogonadism, and small hands and feet. (Courtesy of Y. Lacassie)

page 79
2009 PREP SA on CD-ROM

Critique: 23

Angelman syndrome: Note the prominent mandible and happy expression.


(Reprinted with permission from Jonas DM, Demmer LA. Genetic syndromes
determined by alterations in genomic imprinting pathways. NeoReviews.
2007;8:e120-e126.)

page 80
2009 PREP SA on CD-ROM

Critique: 23

Note the macroglossia, salmon patch on the forehead, prominent eyes, and
infraorbital creases in a 3-month-old infant who has Beckwith-Wiedemann
syndrome. (Reprinted with permission from Jonas DM, Demmer LA. Genetic
syndromes determined by alterations in genomic imprinting pathways.
NeoReviews. 2007;8:e120-e126.)

page 81
2009 PREP SA on CD-ROM

Critique: 23

An omphalocele is observed in some patients who have Beckwith-Wiedemann


syndrome. (Courtesy of the Media Lab at Doernbecher)

page 82
2009 PREP SA on CD-ROM

Question: 24
A 13-year-old girl presents with severe lower abdominal pain of 24 hours’ duration. She states that the
pain is sharp and constant and that she has had similar pain for several days approximately monthly
over the past 4 months. She has no vomiting or diarrhea with the pain, but she is constipated
frequently, having a bowel movement about every 3 to 4 days. She feels that her jeans are getting
tighter around the waist, although she remains active, playing soccer daily. She has never had a
menstrual period and denies ever being sexually active. On physical examination, she is afebrile, her
heart rate is 85 beats/min, and her blood pressure is 110/70 mm Hg. Her weight is at the 60th percentile
and her height at the 50th percentile for age. Her breasts and genitalia are at Sexual Maturity Rating 5.
Abdominal examination reveals a firm and tender midline mass that is inferior to the umbilicus.

Of the following, the MOST likely diagnosis is

A. bladder obstruction

B. endometriosis

C. hematocolpos

D. megacolon

E. ovarian cyst

page 83
2009 PREP SA on CD-ROM

Critique: 24 Preferred Response: C


The adolescent described in the vignette has a clinical history and physical examination findings
compatible with an imperforate hymen, which probably is the most common obstructive anomaly of the
female reproductive tract. An adolescent patient who has an imperforate hymen may be asymptomatic
or may have a history of cyclic abdominal pain that may occur for several years before the diagnosis is
made. A bluish, bulging hymen may be seen on genital inspection (Item C24), and a distended vagina
may be palpated on rectoabdominal or abdominal examination. If the vagina becomes substantially
enlarged with accumulated blood, the patient may experience back pain, pain with defecation that can
result in constipation, nausea and vomiting, or difficulty in urinating.
Bladder outlet obstruction occurs rarely, and although it produces a suprapubic mass, it does not
cause cyclic abdominal pain. Megacolon also is unlikely and does not cause cyclic pain, although
colonic irritation may develop from the pressure produced by the mass. An ovarian cyst typically causes
a right- or left-sided (not midline) mass, and endometriosis is an unlikely cause of a palpable mass,
although it can cause cyclic and acyclic pain in adolescents.

References:

Adams Hillard PJ, Deitch HF. Gynecologic disorders. In: Osborn LM, DeWitt TG, First LR, Zenel JA,
eds. Pediatrics. Philadelphia, Pa: Elsevier Mosby;2005:1461-1471

Laufer MR, Goldstein DP, Hendren WH. Structural abnormalities of the female reproductive tract. In:
Emans SJH, Laufer MR, Goldstein DP, eds. Pediatric and Adolescent Gynecology. 5th ed. Philadelphia,
Pa: Lippincott, Williams & Wilkins; 2005:334-416

page 84
2009 PREP SA on CD-ROM

Critique: 24

Bulging of an imperforate hymen. (Reprinted with permission from Torok K,


Bhende MS. Index of suspicion (case 2). Pediatr Rev. 2008;29:25-30.)

page 85
2009 PREP SA on CD-ROM

Question: 25
During teaching rounds, the pediatric ward resident reports on a 4-month-old circumcised male infant
who was admitted to the pediatric ward for fever that morning. The infant is now afebrile and has had
respiratory rates of 40 breaths/min while sleeping and greater than 60 breaths/min when awake. The
infant has a soft, flat fontanelle on physical examination and is not irritable. The only diagnostic studies
obtained on admission were a urinalysis and complete blood count, the results of which were normal,
except for a white blood cell count of 16.0x103/mcL (16.0x109/L).

Of the following, the MOST appropriate next step is

A. administration of 100 mL normal saline

B. chest radiography

C. lumbar puncture

D. reassurance of the resident that this represents normal respiratory variation

E. urine culture

page 86
2009 PREP SA on CD-ROM

Critique: 25 Preferred Response: B


Respiratory rates vary across a relatively wide range in pediatric patients, depending on factors such as
age and activity status. Therefore, strict definitions of tachypnea and bradypnea are difficult to
determine and always must be considered in association with other factors such as current clinical
status and individual history. Because tachypnea is a sensitive indicator of lower airway disease,
patients who have elevated respiratory rates deserve a clinical evaluation in the context of other
associated symptoms.
The tachypnea, history of fever, and elevated white blood cell count described for the boy in the
vignette warrant chest radiography. Although lumbar puncture and a urine culture often are indicated to
evaluate infants who have fever, the elevated respiratory rate combined with reassuring neurologic
examination results and normal urinalysis make pneumonia a more likely diagnosis. Administration of a
normal saline bolus would not be expected to improve the abnormal respiratory rate.

References:

Bloomfield D. In brief: tachypnea. Pediatr Rev. 2002;23:294-295. Available at:


http://pedsinreview.aappublications.org/cgi/content/full/23/8/294

Sectish TC, Prober CG. Pneumonia. In: Kliegman RM, Behrman RE, Jenson HB, Stanton BF, eds.
Nelson Textbook of Pediatrics. 18th ed. Philadelphia, Pa: Saunders Elsevier;2007:1795-1799

page 87
2009 PREP SA on CD-ROM

Question: 26
A 6-year-old boy who has severe vomiting and dehydration is admitted to the hospital. Initial laboratory
studies demonstrate a serum sodium concentration of 126.0 mEq/L (126.0 mmol/L), potassium of 5.3
mEq/L (5.3 mmol/L), and pH of 7.26. After 24 hours of rehydration with 0.9% saline, his serum sodium
concentration is 129.0 mEq/L (129.0 mmol/L) and potassium is 4.9 mEq/L (4.9 mmol/L). On physical
re-examination, you note that his knees, elbows, dorsal fingers, and tongue are somewhat pigmented
(Item Q26), and his skin is darker than that of other family members.

Of the following, the MOST useful diagnostic laboratory study at this time is measurement of serum

A. antidiuretic hormone (ADH) and alpha-melanocortin-stimulating hormone

B. cortisol and adrenocorticotropic hormone (ACTH)

C. cortisol and dehydroepiandrosterone

D. dehydroepiandrosterone and ACTH

E. insulin-like growth factor 1 and ADH

page 88
2009 PREP SA on CD-ROM

Question: 26

Hyperpigmentation (arrows), as exhibitied by the boy in the vignette.


(Courtesy of the Media Lab at Doernbecher)

page 89
2009 PREP SA on CD-ROM

Critique: 26 Preferred Response: B


Acute or chronic vomiting and dehydration associated with hyponatremia and elevated potassium
concentrations, as described for the boy in the vignette, suggest adrenocortical insufficiency. Primary
adrenal insufficiency is associated with skin pigmentation (Item C26) because of ACTH overproduction.
ACTH acts directly on the melanocortin receptors of skin to activate melanin production. Normally,
ACTH controls the amount of cortisol produced by the adrenal cortex by stimulating adrenal
steroidogenesis. Cortisol then feeds back both to the pituitary and hypothalamus to inhibit pituitary
ACTH release. Measurement of low serum cortisol and markedly elevated serum ACTH concentrations
at any time of the day usually confirms the diagnosis of primary adrenal insufficiency. The presence of
skin pigmentation in the boy described in the vignette suggests that identifying an elevated ACTH value
will be easy, but because there is diurnal variation in ACTH and cortisol, with highest concentrations
during the early morning hours and lowest in the late afternoon and evening, children who have less
severe adrenal insufficiency should have these hormones assessed in the early morning. In addition, an
ACTH stimulation test might be necessary for diagnostic confirmation. In this test, synthetic ACTH1-24
is administered intravenously, and the adrenal cortisol response is measured before injection and at 1
hour postinjection. An adequate cortisol response at 1 hour rules out adrenal insufficiency.
Although ADH concentrations might be elevated in primary adrenal insufficiency because of loss of
intravascular fluid volume, measurement of this hormone does not help in the diagnosis of a child who
has low serum sodium and somewhat elevated potassium values. Low serum sodium is associated with
inappropriate ADH release, but the potassium would not be elevated. ACTH and
melanocyte-stimulating hormone (a melanocortin) both are produced from enzymatic cleavage of a
larger molecule, proopiomelanocortin. Although melanocyte-stimulating hormone might be
overproduced to some extent in the presence of excess ACTH, elevated concentrations of this hormone
are not necessary for skin pigmentation with ACTH excess.
Dehydroepiandrosterone (DHEA) is a weak androgen precursor produced by the fetal adrenal
initially; production increases again in mid-childhood with the onset of adrenarche. DHEA values rise
slowly from age 4 to 6 years. Elevated values may be found with some adrenal tumors and some
relatively rare types of congenital adrenal hyperplasia, but these disorders are associated with some
degree of early puberty. DHEA-S, the sulfated product of DHEA, usually has stable serum values
without diurnal variation. Therefore, normal DHEA or DHEA-S values in an adolescent or adult confirm
the presence of a functioning adrenal gland. This test is less useful in a young child because onset of
adrenarche is variable. Cortisol is low in primary or secondary adrenal insufficiency but tends toward
low ranges in most people after the early morning hours.
Insulin-like growth factor 1 is a marker for growth hormone sufficiency, and normal concentrations
vary with age and sex. Because this child does not seem to have a growth problem, the result of this
assay should be normal for age.

References:

Auchus RJ, Rainey WE. Adrenarche-physiology, biochemistry and human disease. Clin Endocrinol.
2004;60:288-296. Available at:
http://www.blackwell-synergy.com/doi/full/10.1046/j.1365-2265.2003.01858.x

Coco G, Dal Pra XC, Presotto F, et al. Estimated risk for developing autoimmune Addison's disease in
patients with adrenal cortex antibodies. J Clin Endocrinol Metab. 2006;91:1637-1645. Available at:
http://jcem.endojournals.org/cgi/content/full/91/5/1637

Donohoue PA. Diagnosis of adrenal insufficiency in children. UpToDate Online 15.3. 2008. Available for

page 90
2009 PREP SA on CD-ROM

subscription at: http://www.uptodateonline.com/utd/content/topic.do?topicKey=pediendo/20697

Perry R, Kecha O, Paquette J, Huot C, van Vliet G, Deal C. Primary adrenal insufficiency in children:
twenty years experience at the Sainte-Justine Hospital, Montreal. J Clin Endocrinol Metab.
2005;90:3243-3250. Available at: http://jcem.endojournals.org/cgi/content/full/90/6/3243

Wilson TA, Speiser P. Adrenal insufficiency. eMedicine Specialties, Pediatrics: General Medicine,
Endocrinology. 2007. Available at: http://www.emedicine.com/ped/TOPIC47.HTM

page 91
2009 PREP SA on CD-ROM

Critique: 26

In Addison disease, excess adrenocorticotropic hormone acts on


melanocortin, causing increased melanin production and hyperpigmentation
of the mucous membranes and skin (arrows). (Courtesy of the Media Lab at
Doernbecher)

page 92
2009 PREP SA on CD-ROM

Question: 27
A mother of a 6-year-old boy in your practice is concerned that her son may have dyslexia. She has
brought a sample of his printing to the visit in which the boy wrote "ded" instead of "bed" and "dad"
instead of "bad." She wants your advice on what she should do to help her son learn how to write
properly.

Of the following, the MOST appropriate response is to

A. reassure the mother that letter reversal can be normal through 7 years of age

B. recommend a comprehensive psychoeducational evaluation for a learning disability

C. recommend neurologic evaluation

D. refer the child for an occupational therapy evaluation and services to improve his writing skills

E. refer the child for vision therapy

page 93
2009 PREP SA on CD-ROM

Critique: 27 Preferred Response: A


Letter reversal in writing can be normal in children through 7 years of age. Dyslexia, a word recognition
defect, is a specific learning disability that is neurobiologically based. It is characterized by problems
with the ability to recognize words accurately and poor spelling and decoding skills. Its prevalence is as
high as 17.4% of the school-age population. Affected children have problems attaching the correct
labels or names to letters and words. They may call a "b" a "d" or read "saw" as "was." Because the
problem is linguistic, not visual, affected children do not have problems copying letters.
Backward writing and letter reversal occur commonly in early development for all children whether or
not they have learning disabilities. All children should receive routine vision screening, but a visual
acuity problem would not be the cause of the letter reversal for the boy described in the vignette. There
is no scientific evidence that vision therapy (eye exercise) is effective in the remediation of
language-based learning disorders. Because letter reversal still can be considered in the normal range
of development at 6 years of age, psychoeducational evaluation, neurologic evaluation, and
occupational therapy are not indicated for this child.

References:

Committee on Children With Disabilities, American Academy of Pediatrics (AAP) and American
Academy of Ophthalmology (AAO), and American Association for Pediatric Ophthalmology and
Strabismus (APOS). Learning disabilities, dyslexia, and vision: a subject review. Pediatrics.
1998;102:1217-1219. Available at: http://pediatrics.aappublications.org/cgi/content/full/102/5/1217

Fletcher JM, Lyon GR, Fuchs LS, Barnes MA. Reading disabilities: word recognition. In: Learning
Disabilities: From Identification to Intervention. New York, NY: The Guilford Press: 2007:85-163

Shaywitz SE, Shawitz BA. Dyslexia (specific reading disability). Pediatr Rev. 2003;24:147-153.
Available at: http://pedsinreview.aappublications.org/cgi/content/full/24/5/147

page 94
2009 PREP SA on CD-ROM

Question: 28
A 14-year-old girl presents to the emergency department with a 2-day history of fever and a rash. The
rash has been progressive, and now her mouth and eyes hurt. Upon further questioning, she reports
that she was started on an antibiotic 7 days ago for some complaints of dysuria, but she does not
remember its name. Physical examination reveals a moderately toxic-appearing female whose
temperature is 102.6°F (39.2°C), respiratory rate is 25 breaths/min, heart rate is 105 beats/min, and
blood pressure is 105/70 mm Hg. Her bulbar conjunctivae are erythematous (Item Q28A), and she has
some early bullous lesions developing in her mouth. She has right upper quadrant tenderness and
multiple target lesions (Item Q28B) on her chest, abdomen, arm, back, upper thighs, buttocks, and face.

Of the following, the antimicrobial agent that is MOST likely to be associated with these clinical findings
is

A. amoxicillin

B. azithromycin

C. cefdinir

D. clindamycin

E. trimethoprim-sulfamethoxazole

page 95
2009 PREP SA on CD-ROM

Question: 28

Conjunctival erythema, as exhibited by the girl in the vignette. (Reprinted


with permission from Bullen LK, Zenel J. Visual diagnosis: a 15-year-old who
has cough, rash, and painful swallow. Pediatr Rev. 2005;26:176-181

page 96
2009 PREP SA on CD-ROM

Question: 28

(Courtesy of D. Krowchuk)

page 97
2009 PREP SA on CD-ROM

Critique: 28 Preferred Response: E


Stevens-Johnson syndrome, the condition described in the vignette, may be caused by infectious
agents (eg, Mycoplasma pneumoniae, herpes simplex virus) or medications such as nonsteroidal
anti-inflammatory agents (eg, ibuprofen, salicylates), anticonvulsants (eg, phenytoin, carbamazepine),
and other antimicrobial agents (eg, trimethoprim-sulfamethoxazole [TMP-SXT], cephalosporins).
Although generally well tolerated, TMP-SXT has several adverse effects. From 3% to 8% of patients
may experience mild gastrointestinal symptoms, including nausea, vomiting, anorexia, diarrhea,
glossitis, and stomatitis. Approximately 3% to 4% of patients who receive TMP-SXT develop skin
lesions. These drug eruptions include maculopapular rashes, urticaria (Item C28A), pruritus,
photodermatitis (Item C28B), exfoliative dermatitis, erythema multiforme, toxic epidermal necrolysis
(Item C28C), and Stevens-Johnson syndrome. Although any of the antimicrobial agents listed in the
vignette (eg, amoxicillin, azithromycin, cefdinir, clindamycin) can cause these skin reactions,
sulfa-containing agents such as TMP-SXT most commonly are responsible. Patients infected with the
human immunodeficiency virus and taking TMP-SXT for Pneumocystis prophylaxis also are at
increased risk for the development of such reactions. Bone marrow suppression can occur with
prolonged administration of TMP-SXT, resulting in pancytopenia, agranulocytosis, anemia, or
thrombocytopenia. TMP-SXT also competes with bilirubin for plasma protein binding sites and is not
recommended for infants younger than 2 months of age. TMP-SXT can potentiate the effects of
warfarin, phenytoin, methotrexate, and oral hypoglycemic agents, leading to bleeding, phenytoin
toxicity, severe pancytopenia, and hypoglycemia, respectively. The half-life of digoxin also is increased,
resulting in elevated concentrations. Oral contraception metabolism can be accelerated when these
medications are administered in conjunction with TMP-SXT, leading to ineffective contraception.
TMP-SXT is recommended as first-line therapy for acute, uncomplicated urinary tract infections;
urinary tract prophylaxis; selected bacterial gastrointestinal infections (eg, Shigella); and for treatment
and prophylaxis of Pneumocystis infections. The use of amoxicillin for the empiric treatment of urinary
tract infections is limited due to drug resistance among Escherichia coli. Although cefdinir has very
good activity against most community-acquired isolates of E coli and other gram-negative organisms,
its use is often cost prohibitive. Clindamycin should be used only in certain situations (eg, when the
isolated organism is Staphylococcus aureus), and azithromycin is not indicated for the treatment of
urinary tract infections.

References:

Libecco JA, Powell KR. Trimethoprim/sulfamethoxazole: clinical update. Pediatr Rev. 2004;25:375-380.
Available at: http://pedsinreview.aappublications.org/cgi/content/full/25/11/375

Mittmann N, Chan B, Knowles S, Cosentino L, Shear N. Intravenous immunoglobulin use in patients


with toxic epidermal necrolysis and Stevens-Johnson syndrome. Am J Clin Dermatol. 2006;7:359-368.
Abstract available at: http://www.ncbi.nlm.nih.gov/pubmed/17173470

Saidinejad M, Ewald MB, Shannon MW. Transient psychosis in an immune-competent patient after oral
trimethoprim-sulfamethoxazole administration. Pediatrics. 2005;115:e739-e741. Available at:
http://pediatrics.aappublications.org/cgi/content/full/115/6/e739

page 98
2009 PREP SA on CD-ROM

Critique: 28

Wheals with unusul shapes are characteristic of urticaria, a hypersensitivity


reaction that may be triggered by antibiotics such as
trimethoprim-sulfamethoxazole. (Courtesy of D. Krowchuk)

page 99
2009 PREP SA on CD-ROM

Critique: 28

A phototoxic eruption has the appearance of a severe sunburn, and bullae


may be present. The eruption may be caused by a number of drugs, including
tetracyclines, sulfonamides, and fluoroquinolones. In this patient, the eruption
was caused by doxycycline. (Courtesy of D. Krowchuk)

page 100
2009 PREP SA on CD-ROM

Critique: 28

In toxic epidermal necrolysis, erythematous patches appear (top), bullae form


(bottom arrow), and later rupture, leaving widespread erosions. (Courtesy of
D. Krowchuk)

page 101
2009 PREP SA on CD-ROM

Question: 29
A 14-year-old girl presents for evaluation after 4 days of a temperature to 103.0°F (39.5°C), nausea,
abdominal cramping, and profuse bloody diarrhea. She reports that she has not traveled anywhere, has
no pets, and has had no ill contacts or unusual food exposures. One week ago, she was diagnosed
with a methicillin-sensitive Staphylococcus aureus chronic osteomyelitis of her distal radius and has
been receiving intravenous cefazolin therapy via a peripherally inserted central catheter line. Physical
examination reveals an uncomfortable teenager who complains of severe abdominal pain and has a
temperature of 102.8°F (39.4°C) and moist mucous membranes. Her abdomen is diffusely tender, with
voluntary guarding but no rebound tenderness on palpation. Rectal examination demonstrates normal
sphincter tone with no fissures or other lesions. Laboratory findings include a peripheral white blood cell
count of 15.0x103/mcL (15.0x109/L); hemoglobin of 13.0 g/dL (130.0 g/L); platelet count of
300.0x103/mcL (300.0x109/L); and a differential count of 65% neutrophils, 25% lymphocytes, and 10%
monocytes. Her stool appears watery and grossly bloody.

Of the following, the MOST appropriate treatment for this patient’s condition is

A. ceftriaxone

B. clindamycin

C. metronidazole

D. trimethoprim-sulfamethoxazole

E. vancomycin

page 102
2009 PREP SA on CD-ROM

Critique: 29 Preferred Response: C


The patient described in the vignette has findings of antibiotic-associated Clostridium difficile disease
due to her cefazolin therapy. C difficile accounts for approximately 20% to 30% of cases of
antibiotic-associated diarrhea, 50% to 70% of antibiotic-associated colitis cases, and more than 90% of
antibiotic-associated pseudomembranous colitis cases. The incidence of antibiotic-associated diarrhea
and colitis varies greatly, depending on the antibiotic and its spectrum of activity and pharmacokinetic
properties. Almost all antibiotic classes have been associated with disease, but clindamycin, the
penicillins, and the cephalosporins are implicated most commonly.
The pathogenesis of C difficile-mediated diarrhea and colitis includes: 1) disruption and eradication
of normal colonic organisms by antibacterial agents; 2) colonization with toxigenic C difficile; and 3)
production of toxin A or toxin B by C difficile, both of which mediate cytoskeletal damage of target cells,
resulting in mucosal injury and inflammation.
Infection with toxigenic C difficile causes a spectrum of disease, ranging from asymptomatic carriage
to a fulminant, relapsing, and occasionally fatal colitis. Signs and symptoms may develop as early as
the first day of therapy to as late as 10 weeks after therapy has ended, with a typical range of 5 to 10
days after the initiation of antibiotic therapy. The gastrointestinal manifestations of C difficile are
variable, ranging from watery diarrhea to colitis with blood. Other findings include fever (seen in 30% to
50% of patients), leukocytosis (mean peripheral leukocyte count of more than 15.0 x 103/mcL [15.0 x
109/L), nausea, malaise, abdominal pain, and cramping. Oral metronidazole and oral vancomycin are
equally effective for the treatment of C difficile-associated diarrhea and colitis, but metronidazole is
considered the drug of choice because of cost and concerns regarding the emergence of
vancomycin-resistant enterococci. Ceftriaxone, clindamycin, and trimethoprim-sulfamethoxazole are not
effective in the treatment of this infection.

References:

American Academy of Pediatrics. Clostridium difficile. In: Pickering LK, Baker CJ, Long SS, McMillan
JA, eds. Red Book: 2006 Report of the Committee on Infectious Diseases. 27th ed. Elk Grove Village,
Ill: American Academy of Pediatrics; 2006:261-263

Benson L, Song X, Campos J, Singh N. Changing epidemiology of Clostridium difficile-associated


disease in children. Infect Control Hosp Epidemiol. 2007;28:1233-1235. Abstract available at:
http://www.ncbi.nlm.nih.gov/pubmed/17926272

Klein EJ, Boster DR, Stapp JR, et al. Diarrhea etiology in a children's hospital emergency department: a
prospective cohort study. Clin Infect Dis. 2006;43:807-813. Abstract available at:
http://www.ncbi.nlm.nih.gov/pubmed/16941358

Thielman NM, Wilson KH. Antibiotic-associated colitis. In: Mandell GL, Bennett JE, Dolin R, eds.
Mandell, Douglas and Bennett's Principles and Practice of Infectious Diseases. 6th ed. New York, NY:
Elsevier Churchill Livingstone; 2005:1249-1263

page 103
2009 PREP SA on CD-ROM

Question: 30
A 14-year-old girl who has a history of insulin-dependent diabetes mellitus (IDDM) presents with
vomiting, increased urination, and decreased energy. Physical examination reveals Kussmaul breathing
and delayed capillary refill. Laboratory findings include:
· Sodium, 136.0 mEq/L (136.0 mmol/L)
· Potassium, 5.2 mEq/L (5.2 mmol/L)
· Chloride, 100.0 mEq/L (100.0 mmol/L)
· Bicarbonate, 10.0 mEq/L (10.0 mmol/L)
· Blood urea nitrogen, 24.0 mg/dL (8.6 mmol/L)
· Creatinine, 0.9 mg/dL (79.6 mcmol/L)
· Glucose, 550.0 mg/dL (30.5 mmol/L)
The patient receives initial hydration with 20 mL/kg of normal saline.

Of the following, a TRUE statement regarding this patient is that the

A. initial maintenance fluid regimen should avoid supplemental potassium

B. serum potassium concentration reflects adequate intracellular potassium

C. serum potassium concentration should increase with correction of the acidosis

D. serum potassium concentration should increase with correction of the hyperglycemia

E. total body potassium content is depleted

page 104
2009 PREP SA on CD-ROM

Critique: 30 Preferred Response: E


Sodium and potassium are cations that often are accompanied by chloride as an anion when consumed
in the diet. Within the body, these cations play very different roles. The cell membrane is permeable to
both sodium and potassium, but due to Na+-K+-ATPase, they are contained primarily on opposite sides
of the cell membrane, with sodium distributed almost exclusively extracellularly and potassium
contained intracellularly. Sodium is the principal extracellular cation and acts as the primary osmole to
maintain extracellular volume-including plasma volume, which is important for tissue perfusion.
Sodium balance is regulated in the kidney. Excessive sodium intake results in reduced sodium
conservation within the nephron and increased excretion to keep balance with the increased intake.
One tissue in which sodium is contained is the bone, and growing children who have normal kidney
function require 2 to 3 mEq/kg per day of sodium for growth.
Potassium is important for maintaining the resting membrane potential of cells. The typical
intracellular concentration of potassium (140.0 mEq/L [140.0 mmol/L]) is in marked contrast to the 3.5 to
5.5 mEq/L (3.5 to 5.5 mmol/L) of potassium in the extracellular compartment. The ratio of
extracellular-to-intracellular potassium determines the resting membrane potential and subsequent
action potentials in tissues such as skeletal/cardiac muscle and neuronal tissues. Accordingly,
hypokalemia and hyperkalemia can result in muscle paralysis or cardiac dysrhythmias such as
ventricular tachycardia, fibrillation, and ultimately cardiac arrest. The growing child requires 1 to 2
mEq/kg per day of potassium for cellular growth and to avoid potassium deficiency.
Certain conditions alter the measured extracellular potassium concentrations, often providing
misleading information about intracellular stores. In the setting of acidosis, potassium shifts from
intracellular to extracellular locations, increasing measured extracellular potassium values. Conversely,
alkalosis or correction of metabolic acidosis results in a lowering of measured potassium values.
Therapies that result in intracellular shifting of potassium include: insulin, beta-2 agonists such as
albuterol or epinephrine, and sodium bicarbonate (due to quenching of acidosis). The patient described
in the vignette has severe metabolic acidosis with increased urine output, and the serum potassium
value of 5.2 mEq/L (5.2 mmol/L) is misleading. In fact, potassium should be added to the intravenous
fluids because this girl's total body potassium is depleted, which will become very apparent upon
correction of the acidosis and of the hyperglycemia (with insulin).

References:

Plotnick L. Insulin-dependent diabetes mellitus. Pediatr Rev. 1994;15:137-148. Available at:


http://pedsinreview.aappublications.org/cgi/reprint/15/4/137

Rose BD, Post TW. Potassium homeostasis. In: Clinical Physiology of Acid-base and Electrolyte
Disorders. 5th ed. New York, NY: McGraw-Hill Medical Publishing Division; 2001:372-375

Rose BD, Post TW. The total body water and the plasma sodium concentration. In: Clinical Physiology
of Acid-base and Electrolyte Disorders. 5th ed. New York, NY: McGraw-Hill Medical Publishing Division;
2001:241-243

page 105
2009 PREP SA on CD-ROM

Question: 31
You have just assisted in the delivery of a 38-week gestational age male infant who was born via
cesarean section to a 25-year-old woman. As you are completing the infant’s initial physical
examination, the father mentions that he and his wife have allergic rhinitis and asthma. He asks
whether his son is at increased risk for allergies and how they can reduce the boy’s chance for
developing such allergic disorders.

Of the following, the MOST appropriate next step is to

A. explain that because both parents have asthma, breastfeeding will not reduce the risk of eczema

B. explain that breastfeeding or formula choices do not matter now because the mother did not restrict
her diet during pregnancy

C. measure the cord blood immunoglobulin E concentration to help establish the newborn’s risk for
atopic disorders

D. recommend exclusive breastfeeding for 4 months with the addition of a hypoallergenic formula if
needed

E. start the newborn on a cow milk formula for the first month, then switch to strict breastfeeding if he
develops eczema

page 106
2009 PREP SA on CD-ROM

Critique: 31 Preferred Response: D


The incidence of atopy (allergic rhinitis, asthma, eczema) has increased significantly over the past few
decades. The ability to intervene and either delay or prevent atopic disease in infants born to atopic
parents has been the subject of numerous studies. Application of these studies to the population as a
whole is difficult because the specific interventions and endpoints for each study often differ. However,
one aspect that is agreed on is that atopy risk for infants increases significantly when both parents have
a history of atopy (30% to 60%) compared with a history for just one parent (20% to 40%) or neither
parent (10% to 15%).
Prior to delivery, two prevention strategies have been studied: maternal diet restriction and
supplementation with probiotics. Currently, no evidence supports maternal dietary restriction to common
allergenic foods. Some studies have supported administration of probiotics (eg, Lactobacillus
rhamnosus) to the mother 2 to 4 weeks before delivery and to the infant for 6 months after birth. One
study demonstrated a reduction in eczema at 2 years but no reduction in asthma, immunoglobulin (Ig) E
concentrations, or allergen sensitization. Further, the dose and type of probiotic has differed in various
investigations, making generalized recommendations difficult.
Even if both parents have atopy, as described in the vignette, breastfeeding or formula choices may
affect atopy outcomes for the infant. In "high-risk" newborns (ie, both parents have atopy or one parent
and one sibling have atopy), the American Academy of Pediatrics Committee on Nutrition recommends
exclusive breastfeeding for at least 4 months, with supplementation of a hypoallergenic formula if
needed. Although it is difficult to compare studies because the duration of breastfeeding and atopic
outcome (ie, eczema, allergic rhinitis, asthma) differ, breastfeeding for at least 3 months reduces the
risk for eczema. The protective benefit becomes more complex when controlling for the specific
maternal atopic condition. For "high-risk" infants born to women who choose not to breastfeed, most
studies and experts support starting an extensively hydrolyzed formula. Starting a cow or soy milk
formula, compared with an extensively hydrolyzed formula, increases the risk for early eczema. Or note,
interventions resulting in decreased atopy early in life may not predict later atopic outcomes.
Cord IgE concentrations can be used to assess a newborn's risk for atopy, but its measurement
currently is not recommended as a routine screening tool. Furthermore, because both parents in the
vignette have a history of atopy, the child already is considered "high risk." The ability to predict atopy
based on cord IgE concentrations also depends on the cutoff value used. In one study, 80% of
newborns whose cord IgE concentrations were greater than 0.9 kU/L subsequently developed atopy by
5 years of age, but the specific IgE value did not correlate with atopy severity.

References:

American Academy of Pediatrics Committee on Nutrition. Hypoallergenic infant formulas. Pediatrics.


2000;106: 346-349. Available at: http://pediatrics.aappublications.org/cgi/content/full/106/2/346

Greer FR, Sicherer SH, Burks AW, Committee on Nutrition and Section on Allergy and Immunology.
Effects of early nutritional interventions on the development of atopic disease in infants and children:
the role of maternal dietary restriction, breastfeeding, timing of introduction of complementary foods and
hydrolyzed formulas. Pediatrics. 2008;121:183-191. Available at:
http://pediatrics.aappublications.org/cgi/content/full/121/1/183

Mihrshahi S, Ampon R, Webb K, et al for the CAPS Team. The association between infant feeding
practices and subsequent atopy among children with a family history of asthma. Clin Exp Allergy.
2007;37:671-679. Abstract available at: http://www.ncbi.nlm.nih.gov/pubmed/17456214

page 107
2009 PREP SA on CD-ROM

Prescott SL, Björkstön B. Probiotics for the prevention or treatment of allergic disease. J Allergy Clin
Immunol. 2007;120:255-262. Abstract available at: http://www.ncbi.nlm.nih.gov/pubmed/17544096

Snijders BEP, Thijs C, Dagnelie PC, et al. Breast-feeding duration and infant atopic manifestations, by
maternal allergic status, in the first two years of life (KOALA study). J Pediatr. 2007;151:347-351.
Abstract available at: http://www.ncbi.nlm.nih.gov/pubmed/17889066

page 108
2009 PREP SA on CD-ROM

Question: 32
A 2-year-old girl who has a 4-day history of varicella presents to the office with agitation. Her mother
reports that she treated the fever, rash, and pruritus with acetaminophen and diphenhydramine
regularly, which provided some relief. This morning her daughter seemed more irritable, had a higher
fever than yesterday, and "seemed delirious." On physical examination, the agitated and inconsolable
child has a temperature of 104.2°F (40.1°C), heart rate of 160 beats/min, respiratory rate of 36
beats/min, and blood pressure of 135/87 mm Hg. Her pupils are dilated and sluggishly reactive.
Examination of the skin reveals numerous small, crusted erosions without surrounding erythema.
Neurologic examination demonstrates no focal findings, and the patient is not ataxic.

Of the following, the MOST likely explanation for these symptoms is

A. diphenhydramine overdose

B. hypoglycemia

C. intracranial hemorrhage

D. Reye syndrome

E. varicella cerebellitis

page 109
2009 PREP SA on CD-ROM

Critique: 32 Preferred Response: A


The patient described in the vignette is exhibiting symptoms of anticholinergic toxicity. The classic
mnemonic "hot as a hare, dry as a bone, blind as a bat, red as a beet, and mad as a hatter" aptly
describes a number of the commonly seen signs and symptoms, including hyperpyrexia, dry skin,
dilated pupils, flushing, and delirium. In addition, affected patients typically exhibit tachycardia and
hypertension. The clinical syndrome caused by anticholinergic toxicity is due to competitive inhibition of
acetylcholine binding to the postganglionic parasympathetic muscarinic receptors by the offending
agent. Many commonly used medications have anticholinergic properties, including antihistamines,
tricyclic antidepressants, antispasmodics, and mydriatics. For this patient, diphenhydramine is the most
likely culprit.
Treatment of anticholinergic poisoning is primarily supportive, with stabilization of vital functions (the
ABCs) of greatest importance, followed by appropriate decontamination (eg, activated charcoal for
ingestions, skin cleansing and removal of transdermal delivery devices for dermal absorption). Agitation
and seizures may be treated with benzodiazepines. Physostigmine, a cholinergic agent that
competitively inhibits acetylcholinesterase, may be more effective for the treatment of severe agitation
than benzodiazepines, but significant toxicity risks make its use controversial.
Hypoglycemia can cause altered mental status but typically produces bradycardia, hypotension, and
coma, rather than agitation. Intracranial hemorrhage can be seen in patients who have varicella-related
thrombocytopenia, although affected patients frequently present with hemorrhagic skin lesions (Item
C32). In addition, patients who have intracranial hemorrhage typically present with focal neurologic
findings and bradycardia with hypertension, if intracranial pressure is increased.
The classic presentation of Reye syndrome includes sudden onset of protracted vomiting without
fever in association with lethargy that progresses to delirium, seizures, stupor, and coma. Reye
syndrome typically occurs 5 to 7 days after the onset of a viral illness, following apparent improvement
in the child's initial symptoms. It results from mitochondrial dysfunction leading to liver failure and
cerebral edema. Ninety percent of cases have been reported to follow an upper respiratory tract
infection; varicella has been associated with 5% of cases.
Central nervous system complications of varicella are uncommon and may include cerebellitis,
transverse myelitis, peripheral neuritis, and optic neuritis. Cerebellitis is the most common presentation
of encephalitis and typically presents on the third to eighth day of illness with ataxia.
Meningoencephalitis presents with fever, meningeal signs, seizures, and altered mental status.

References:

Burns JJ Jr. Toxicity, anticholinergic. eMedicine Specialties, Emergency Medicine, Toxicology. 2006.
Available at: http://www.emedicine.com/EMERG/topic36.htm

Burns MJ, Linden CH, Graudins A, Brown RM, Fletcher KE. A comparison of physostigmine and
benzodiazepines for the treatment of anticholinergic poisoning. Ann Emerg Med. 2000;35:374-381.
Abstract available at: http://www.ncbi.nlm.nih.gov/pubmed/10736125

Carey RG, Balisteri WF. Mitochondrial hepatopathies. In: Kleigman RM, Behrman RE, Jenson HB,
Stanton BF, eds. Nelson Textbook of Pediatrics. 18th ed. Philadelphia, Pa: Saunders Elsevier;
2007:1696-1697

Gershon AA, LaRussa P. Varicella-zoster virus infections. In: Gershon AA, Hotez PJ, Katz SL, eds.
Krugman's Infectious Diseases of Children. 11th ed. Philadelphia, Pa: Mosby; 2004:785-816

page 110
2009 PREP SA on CD-ROM

Su M, Goldman M. Anticholinergic poisoning. UpToDate Online 15.3. 2008. Available for subscription
at:
http://www.utdol.com/utd/content/topic.do?topicKey=ad_tox/13958&selectedTitle=1~376&source=searc
h_result

page 111
2009 PREP SA on CD-ROM

Critique: 32

Purpura fulminans may follow varicella and is characterized by hemorrhagic


necrosis of the skin and disseminated intravascular coagulation. (Courtesy of
Bernard Cohen, MD, DermAtlas, www.dermatlas.org)

page 112
2009 PREP SA on CD-ROM

Question: 33
A 7-month-old child presents for a follow-up office visit after undergoing a Kasai procedure for biliary
atresia at 6 weeks of age. The mother states that the boy is irritable when his right arm is moved. On
physical examination, the infant is jaundiced. You detect tenderness in the anterior radial head.
Radiography of the affected region demonstrates metaphyseal fraying (Item Q33) and a fracture.

Of the following, the MOST appropriate laboratory studies to obtain next are

A. calcium and phosphorus measurement and bone densitometry (DEXA scan)

B. calcium and phosphorus measurement and urinary calcium-to-creatinine ratio

C. calcium, phosphorus, and 25-hydroxyvitamin D measurement

D. calcium, phosphorus, and magnesium measurement

E. magnesium, phosphorus, and parathyroid hormone measurement

page 113
2009 PREP SA on CD-ROM

Question: 33

Metaphyseal fraying, cupping, and widening, as described for the infant in the
vignette. (Couretsy of R. Schwartz)

page 114
2009 PREP SA on CD-ROM

Critique: 33 Preferred Response: C


Chronic cholestasis due to biliary atresia results in decreased bile flow into the intestine. The absence
of intraluminal bile acids, in turn, causes decreased digestion of lipids, leading to fat malabsorption. In
addition, absorption of fat-soluble vitamins (A, D, E, and K) is impaired, which may lead to clinical
sequelae of fat-soluble vitamin deficiency. Finally, steatorrhea may impair calcium absorption because
intraluminal free fatty acids may bind calcium.
The clinical presentation of the patient in the vignette strongly suggests the presence of rickets from
vitamin D deficiency. Therefore, the most helpful initial laboratory testing is determination of calcium,
phosphorus, and 25-hydroxyvitamin D concentrations. The 25-hydroxyvitamin D assay is the best
measure of hepatic stores of vitamin D and is a better marker of vitamin D status than either serum
vitamin D or 1,25-dihydroxyvitamin D. Although bone density testing, measurement of serum
magnesium and parathyroid hormone, and determination of the urinary calcium-to-creatinine ratio may
provide useful additional information, they will not help establish the diagnosis of vitamin D-deficient
rickets.
Rickets is a potentially preventable complication of biliary atresia, but requires monitoring of calcium,
phosphorus, and 25-hydroxyvitamin D concentrations two to four times a year. Infants who have biliary
atresia routinely receive supplementation with approximately 8,000 IU of ergocalciferol (vitamin D2)
daily. This dose of vitamin D is approximately 20 times the recommended dietary allowance for a
healthy toddler. If rickets develops or the vitamin D concentration cannot be maintained within the
normal range, the patient should receive either calcitriol (1,25-dihydroxyvitamin D3) or intramuscular
vitamin D.

References:

Campbell KM, Bezerra JA. Biliary atresia. In: Walker WA, Goulet O, Kleinman RE, Sherman PM,
Shneider BL, Sanderson IR, eds. Pediatric Gastrointestinal Disease. 4th ed. Hamilton, Ontario, Canada:
BC Decker; 2004:1122-1138

Suchy FJ. Neonatal cholestasis. Pediatr Rev. 2004;25:388-396. Available at:


http://pedsinreview.aappublications.org/cgi/content/full/25/11/388

page 115
2009 PREP SA on CD-ROM

Question: 34
You are examining a 3.5-kg term infant 48 hours after his birth. Results of the physical examination are
normal, and you are considering discharging him from the hospital. He is being fed formula from a
bottle, and the nurses report intakes of 30 mL every 3 hours. He has wet at least six diapers daily for
the past 2 days, but he has not passed any meconium or expressed any stool since birth.

Of the following, the MOST likely diagnosis is

A. ileal atresia

B. imperforate anus

C. meconium ileus

D. meconium plug syndrome

E. neonatal small left colon syndrome

page 116
2009 PREP SA on CD-ROM

Critique: 34 Preferred Response: D


Ninety-five percent of term infants express meconium or pass a stool in the first 24 hours of postnatal
life. The infant described in the vignette is term and appropriately grown and has been feeding and
voiding well, but he has failed to pass meconium or any stool in the first 2 days of postnatal life.
Although he does not have any abdominal distention, emesis, or systemic illness, the pediatric clinician
should be concerned about potential bowel, particularly colonic, obstruction.
The most likely explanation for the symptoms described for the infant in the vignette is meconium
plug syndrome, which typically is an isolated phenomenon that is not associated with anatomic
obstruction (eg, atresia). It occurs commonly in term and preterm infants and may be associated with
maternal magnesium sulfate treatment for pre-eclampsia/eclampsia. Meconium plug obstruction
generally is related to hypomotility. Clinically, there may be no abdominal findings or a gradual increase
in girth but no other signs of illness. Plain radiographs of the abdomen generally provide nonspecific
findings, but may show a paucity of gas in the rectosigmoid. A contrast enema characteristically
illuminates the plugs of meconium and facilitates their evacuation. On occasion, a firm, paraffin-like
formed plug may be expressed spontaneously by affected infants during the second postnatal day.
Although some infants who have retained meconium may exhibit a small left colon on contrast enema,
colonic motility usually is normal upon evacuation of the meconium plug(s).
Although meconium plug syndrome is the most common cause of delayed passage of stool, the
clinician also should consider Hirschsprung disease, a congenital absence of ganglion cells. A failure to
pass meconium in the first 24 hours of postnatal life characterizes 95% of affected infants. The area of
affected bowel typically is in the rectosigmoid, where a transition zone may be observed on contrast
enema, although this finding is less common in neonates. If Hirschsprung disease is considered,
diagnostic rectal biopsy should be performed.
The neonatal small left colon syndrome is seen in infants of diabetic mothers and is diagnosed using
a contrast enema. The enema may be both diagnostic and therapeutic, as seen in meconium plug
syndrome. Gradual feeding and monitoring of the stooling pattern generally results in resolution of the
condition over the early weeks of postnatal life.
Imperforate anus occurs in about 1 in 4,000 to 5,000 births, and typically is apparent on physical
examination. In some cases, a fistulous tract may exist, and the expression of meconium may occur
anywhere along the perineal-scrotal-urethral line. Imperforate anus may be an isolated finding or seen
in conjunction with other anomalies such as vertebral malformations, cardiac malformation,
tracheoesophageal fistula/esophageal atresia, renal anomalies, and limb malformation (VACTERL
association).
Ileal atresia (proximal or distal) occurs as part of a spectrum of jejunal-ileal bowel atresia that likely
reflects a mesenteric vascular defect or interruption in development. Its absolute frequency is not well
reported, although it commonly is diagnosed prenatally (dilated bowel, polyhydramnios) on obstetric
ultrasonography. With a distal obstruction, the newborn may take early feedings well, but becomes ill,
with bile-stained emesis and abdominal distention, in the first 24 to 48 postnatal hours. Plain
radiographic findings may include multiple stacked loops of air-filled bowel and air-fluid levels.
Meconium ileus is a condition of intestinal obstruction related to thickened, inspissated mucus mixed
with meconium that is characteristic of cystic fibrosis and is related to altered chloride and water
balance in mucus. The meconium may be beadlike, in small, dense pellets, and even visible on
prenatal obstetric ultrasonography. A microcolon may exist distal to the small bowel obstruction.
Affected infants may have visible and palpable loops of bowel on examination, in addition to abdominal
distention, bilious emesis, and failure to pass meconium in the first 24 to 48 hours of postnatal life. Plain
radiographs may reveal a soap-bubble appearance characteristic of meconium stool. A contrast enema
may reveal a microcolon and failure to see contrast reflux past the ileocecal valve.

page 117
2009 PREP SA on CD-ROM

References:

Albanese CT, Sylvester KG. Pediatric surgery. In: Doherty GM, Way LW, eds. Current Surgical
Diagnosis and Treatment. 12th ed. New York, NY: The McGraw-Hill Companies, Inc; 2006:chap 45

Burge D, Drewett M. Meconium plug obstruction. Pediatr Surg Int. 2004;20:108-110. Abstract available
at: http://www.ncbi.nlm.nih.gov/pubmed/14760494

Casaccia G, Trucchi A, Spirydakis I, et al. Congenital intestinal anomalies, neonatal short bowel
syndrome, and prenatal/neonatal counseling. J Pediatr Surg. 2006;41:804-807. Abstract available at:
http://www.ncbi.nlm.nih.gov/pubmed/16567197

Hajivassiliou CA. Intestinal obstruction in neonatal/pediatric surgery. Semin Pediatr Surg.


2003;12:241-253. Abstract available at: http://www.ncbi.nlm.nih.gov/pubmed/14655163

Magnuson DK, Parry RL, Chwals WJ. Selected abdominal gastrointestinal anomalies. In: Martin RJ,
Fanaroff AA, Walsh MC, eds: Fanaroff and Martin's Neonatal-Perinatal Medicine. 8th ed. Philadelphia,
Pa: Mosby Elsevier; 2006:1381-1402

Nurko S. Motility of the colon and anorectum. NeoReviews. 2006;7:e34-e48. Available for subscription
at: http://neoreviews.aappublications.org/cgi/content/full/7/1/e34

Sutton TL. Index of suspicion in the nursery. NeoReviews. 2006;7:e269-e271. Available for subscription
at: http://neoreviews.aappublications.org/cgi/content/full/7/5/e269

Thilo EH, Rosenberg AA. The newborn infant. In: Hay WW Jr, Levin M, Sondheimer JM, Deterding RR,
eds. Current Pediatric Diagnosis & Treatment. 18th ed. New York, NY: The McGraw-Hill Companies,
Inc; 2007:chap 1

page 118
2009 PREP SA on CD-ROM

Question: 35
A 4-year-old boy who recently emigrated from Central America is brought to your clinic because of 2
weeks of colicky abdominal pain that recently has worsened. His vital signs are normal, and he is
afebrile. Physical examination reveals mild diffuse tenderness, but there is no rebound or guarding.
After your examination, he has an episode of vomiting. Examination of the vomitus reveals long, slim
objects that resemble worms (Item Q35).

Of the following, the BEST treatment choice is

A. albendazole

B. iodoquinol

C. metronidazole

D. praziquantel

E. voriconazole

page 119
2009 PREP SA on CD-ROM

Question: 35

(Courtesy of M. Rimsza)

page 120
2009 PREP SA on CD-ROM

Critique: 35 Preferred Response: A


Ascariasis is caused by infestation with the roundworm Ascaris lumbricoides. It occurs most commonly
in tropical regions and areas that have poor sanitation, although many cases also occur in the United
States each year. Adult worms (Item C35A) live in the small intestine and produce eggs (Item C35B)
that are excreted in the stool into the soil. Infestation occurs when the eggs in the contaminated soil are
ingested. Larvae pass from the small intestine into the bloodstream, traveling to the liver and lungs.
They migrate from the lung to the pharynx, where they are swallowed. The worms mature in the small
intestine, where they produce their eggs, completing the cycle.
Many patients who have ascariasis are asymptomatic, but symptoms such as nonspecific
gastrointestinal complaints and abdominal pain can occur, as described for the boy in the vignette.
Intestinal obstruction and symptoms related to the migration of the larvae, such as obstructive jaundice
and peritonitis, also may be seen. Adult worms may pass from the rectum, nose, or mouth if there is
heavy worm burden. The diagnosis is made by seeing either ova on microscopic stool examination or
the adult worm itself, as described for the boy in the vignette.
Treatment of ascariasis that is not associated with intestinal obstruction consists of a single dose of
either albendazole or pyrantel pamoate or a 3-day course of mebendazole. Intestinal obstruction may
require treatment with piperazine citrate solution or, rarely, surgical intervention.
Iodoquinol is a luminal amebicide that is effective for the treatment of asymptomatic amebic cyst
excreters, and metronidazole is the drug of choice for patients who have symptomatic amebiasis as
well as giardiasis. Neither is appropriate for the treatment of ascariasis. Praziquantel is useful for the
treatment of liver fluke infestations, schistosomiasis, and tapeworm infestations. Voriconazole is an
intravenous or oral medication used in the treatment of fungal infections.

References:

American Academy of Pediatrics. Amebiasis. In: Pickering LK, Baker CJ, Long SS, McMillan JA, eds.
Red Book: 2006 Report of the Committee on Infectious Diseases. 27th ed. Elk Grove Village, Ill:
American Academy of Pediatrics; 2006:204-208

American Academy of Pediatrics. Ascaris lumbricoides infections. In: Pickering LK, Baker CJ, Long SS,
McMillan JA, eds. Red Book: 2006 Report of the Committee on Infectious Diseases. 27th ed. Elk Grove
Village, Ill: American Academy of Pediatrics; 2006:218-219

Dent AE, Kazura JW. Ascariasis (Ascaris lumbricoides). In: Kliegman RM, Behrman RE, Jenson HB,
Stanton BF, eds. Nelson Textbook of Pediatrics. 18th ed. Philadelphia, Pa: Saunders Elsevier;
2007:1495

page 121
2009 PREP SA on CD-ROM

Critique: 35

Ascaris sp causes the most common human roundworm infection. Adult


worms may reach 30 cm in length. (Courtesy of M. Rimsza)

page 122
2009 PREP SA on CD-ROM

Critique: 35

Fertilized Ascaris lumbricoides eggs are rounded and have a thick shell.
(Courtesy of the Centers for Disease Control and Prevention, Public Health
Image Library, M Melvin)

page 123
2009 PREP SA on CD-ROM

Question: 36
An 8-month-old boy who has Down syndrome and a large ventriculoseptal defect has had recurrent
otitis media and sinusitis during the respiratory virus season that required four separate courses of
antibiotics in 4 months. At today’s visit, his mother states that his rhinitis and otitis media symptoms
have resolved, but she is concerned about a recurrent diaper rash that is unresponsive to both barrier
creams and repeated use of the nystatin cream prescribed last month. Examination reveals white
plaques (Item Q36A) on the buccal mucosa just inside the lips and a diaper rash (Item Q36B).

Of the following, the BEST therapeutic option for this child is

A. oral fluconazole

B. oral griseofulvin

C. oral itraconazole

D. topical clotrimazole alone

E. topical clotrimazole combined with triamcinolone

page 124
2009 PREP SA on CD-ROM

Question: 36

White plaques, as described for the infant in the vignette. (Courtesy of D.


Krowchuk)

page 125
2009 PREP SA on CD-ROM

Question: 36

(Reprinted with permission from Krowchuk DP, Mancini AJ, eds. Pediatric
Dermatology. A Quick Reference Guide. Elk Grove Village, Ill: American
Academy of Pediatrics; 2007)

page 126
2009 PREP SA on CD-ROM

Critique: 36 Preferred Response: A


The child described in the vignette has recurrent Candida diaper dermatitis and thrush that is probably
due to his four courses of antibiotics. Candida infections are a common problem in infants, toddlers,
and children who have underlying medical conditions that require frequent administration of antibiotics.
The administration of antibiotics disrupts normal skin and mucous membrane microbiologic flora. The
boy described in the vignette has been prescribed topical antifungal preparations without sustained
improvement.
Prescribing alternative topical antifungal preparations such as clotrimazole for the diaper rash might
treat Candida sp that are resistant to nystatin, but it will not treat thrush. Once-daily oral fluconazole has
been proven superior to oral nystatin for resistant thrush and effective for Candida diaper dermatitis.
Recurrence rates are similar for both fluconazole and nystatin. Oral itraconazole is another effective
antifungal, but it may be associated with more gastrointestinal adverse effects, including hepatotoxicity,
and is dosed twice daily. Griseofulvin is not the drug of choice for Candida infection, although it is useful
in fungal hair and nail infections.
Combining antifungal agents with steroids may diminish inflammation temporarily, but steroids also
delay the resolution of fungal infection by decreasing T-cell response in the skin, resulting in worsening
of the dermatitis after a few days. Further, high systemic absorption of steroids may occur in areas of
thin and occluded skin, such as the diaper area.

References:

American Academy of Pediatrics. Candidiasis (moniliasis, thrush). In: Pickering LK, Baker CJ, Long SS,
McMillan JA, eds. Red Book: 2006 Report of the Committee on Infectious Diseases. 27th ed. Elk Grove
Village, Ill: American Academy of Pediatrics; 2006:242-246

American Academy of Pediatrics. Drugs for invasive and other serious fungal infections in children. In:
Pickering LK, Baker CJ, Long SS, McMillan JA, eds. Red Book: 2006 Report of the Committee on
Infectious Diseases. 27th ed. Elk Grove Village, Ill: American Academy of Pediatrics; 2006:780

American Academy of Pediatrics. Recommended doses of parenteral and oral antifungal drugs. In:
Pickering LK, Baker CJ, Long SS, McMillan JA, eds. Red Book: 2006 Report of the Committee on
Infectious Diseases. 27th ed. Elk Grove Village, Ill: American Academy of Pediatrics; 2006:777-779

American Academy of Pediatrics. Topical drugs for superficial fungal infections. In: Pickering LK, Baker
CJ, Long SS, McMillan JA, eds. Red Book: 2006 Report of the Committee on Infectious Diseases. 27th
ed. Elk Grove Village, Ill: American Academy of Pediatrics; 2006:781-784

Goins RA, Ascher D, Waecker N, Arnold J, Moorefield E. Comparison of fluconazole and nystatin oral
suspensions for treatment of oral candidiasis in infants. Pediatr Infect Dis J. 2002;21:1165-1167.
Abstract available at: http://www.ncbi.nlm.nih.gov/pubmed/12506950

Pankhurst CL. Antifungal treatment in immunocompetent or immunocompromised infants and children.


BMJ Clinical Evidence. 2007. Available for subscription at:
http://clinicalevidence.bmj.com/ceweb/conditions/orh/1304/1304_I4.jsp#REF22

Weisse ME, Aronoff SC. Candida. In: Kliegman RM, Behrman RE, Jenson HB, Stanton BF, eds.
Nelson's Textbook of Pediatrics. 18th ed. Philadelphia, Pa: Saunders Elsevier; 2007:1207-1310

page 127
2009 PREP SA on CD-ROM

Question: 37
You are called to the newborn nursery to evaluate a 2-hour-old male who was born at term. The
pregnancy was uncomplicated, but meconium staining was noted at delivery. The baby weighs 3.8 kg,
is afebrile, and has a heart rate of 165 beats/min and a respiratory rate of 70 breaths/min. You note
tachypnea and hyperpnea with clear breath sounds, no murmurs, and strong distal pulses. His oxygen
saturation in room air is 68%. You place a nonrebreather mask to deliver an Fio2 of 1.0. After 5
minutes, the oxygen saturation is 72%.

Of the following, the BEST explanation for the findings of the hyperoxia test is

A. meconium aspiration syndrome

B. persistent pulmonary hypertension of the newborn

C. pneumonia

D. retained fetal lung liquid syndrome

E. transposition of the great arteries

page 128
2009 PREP SA on CD-ROM

Critique: 37 Preferred Response: E


Most commonly, the practitioner is alerted to hypoxemia in the newborn by the finding of a low oxygen
saturation value. Among the various causes of abnormal oxygenation in the newborn are pulmonary
pathologies, congenital cardiovascular malformations, persistent pulmonary hypertension of the
newborn, and disturbances of the hematologic and metabolic systems. Right-to-left shunting can be
thought of as a diversion of desaturated blood away from the lungs and to the systemic circulation. This
can occur because blood does not perfuse the ventilated portions of the lung (intrapulmonary
right-to-left shunting). Intrapulmonary shunting resulting from infection such as pneumonia,
pneumothorax, retained fetal lung liquid, and pulmonary prematurity is the most frequently encountered
reason for desaturation in a newborn. Conversely, abnormal oxygenation can result from the situations
in which the desaturated blood does not perfuse the pulmonary artery from the heart (intracardiac
right-to-left shunting) or is diverted from the pulmonary circuit through the ductus arteriosus
(extracardiac right-to-left shunting). Examples of these pathologies include pulmonary atresia,
transposition of the great arteries, tricuspid atresia, and pulmonary hypertension.
Whether the cause of the desaturation is intrapulmonary, intracardiac, or extracardiac right-to-left
shunting, cyanosis (blue, maroon, or purple discoloration of the skin) is likely to be present. Typically,
clinicians discern cyanosis in patients who have oxygen saturations of less than 85%, although it may
be apparent to some when the saturation is 90% or less. If cyanosis is suspected in the newborn,
cyanotic heart disease must be considered. It is reasonable to place the patient in a high-oxygen
atmosphere (near FiO2 of 1.0) to determine if the high-dose oxygen can overcome the shunting. If the
degree of cyanosis improves and oxygen saturations become normal, the problem likely is
intrapulmonary shunting rather than cyanotic heart disease. If hyperoxia does not lead to increased
oxygen saturation and higher PaO2 (=150 torr), cyanotic congenital heart disease should be considered
and the infant should undergo further cardiac evaluation.
The newborn described in the vignette has cyanosis but no murmurs. His tachypnea and hyperpnea
(deep breathing) represent the physiologic response to hypoxemia. His saturation improves slightly with
the delivery of high-dose oxygen. Transposition of the great arteries is the best explanation for the
infant's hypoxemia (Item C37). No amount of oxygen delivered to the patient's alveoli can improve
oxygenation of the pulmonary blood flow because the pulmonary blood flow in transposition already is
well saturated. The neonate remains desaturated until oxygenated blood from the left atrium adequately
crosses the atrial septum to be delivered to the system through the aorta. In contrast, the oxygenation
defect associated with meconium aspiration syndrome, persistent pulmonary hypertension of the
newborn, pneumonia, and retained fetal lung liquid syndrome is improved with high-dose oxygen
delivery to the alveoli.

References:

Driscoll D, Allen HD, Atkins DL, et al. Guidelines for evaluation and management of common congenital
cardiac problems in infants, children, and adolescents. A statement for healthcare professionals from
the Committee on Congenital Cardiac Defects of the Council on Cardiovascular Disease in the Young,
American Heart Association. Circulation. 1994;90:2180-2188. Available at:
http://circ.ahajournals.org/cgi/reprint/90/4/2180

Ranjit MS. Common congenital cyanotic heart defects--diagnosis and management. J Indian Med
Assoc. 2003;101:71-72, 74. Abstract available at: http://www.ncbi.nlm.nih.gov/pubmed/12841486

Silberbach M, Hannon D. Presentation of congenital heart disease in the neonate and young infant.
Pediatr Rev. 2007;28:123-131. Available at:

page 129
2009 PREP SA on CD-ROM

http://pedsinreview.aappublications.org/cgi/content/full/28/4/123

page 130
2009 PREP SA on CD-ROM

Critique: 37

Transposition of the great arteries: Computed tomography angiography of the


heart with three-dimensional reconstruction shows the aorta (Ao) arising from
the right ventricle (RV) and the pulmonary artery (PA) arising from the left
ventricle (LV). (Courtesy of D. Mulvihill)

page 131
2009 PREP SA on CD-ROM

Question: 38
The mother of a 10-month-old child who has mild hypotonia brings him to the office after he has an
unprovoked seizure. On physical examination, you note several hypopigmented macules on the trunk
(Item Q38A). Magnetic resonance imaging of the brain reveals several thickened areas of cerebral
cortex (Item Q38B), with abnormal signal and abnormalities along the walls of the lateral ventricles
(Item Q38C).

Of the following, the MOST likely diagnosis is

A. incontinentia pigmenti

B. neurofibromatosis type 1

C. Sturge-Weber syndrome

D. tuberous sclerosis

E. von Hippel-Lindau syndrome

page 132
2009 PREP SA on CD-ROM

Question: 38

(Courtesy of D. Krowchuk)

page 133
2009 PREP SA on CD-ROM

Question: 38

T2-weighted axial magnetic resonance imaging reveals dysplastic cortical gyri


(arrows), as described for the infant in the vignette. (Courtesy of D. Krowchuk)

page 134
2009 PREP SA on CD-ROM

Question: 38

Axial magnetic resonance imaging demonstrates lesions along the walls of


the lateral ventricles (arrows), as described for the infant in the vignette.
(Courtesy of D. Krowchuk)

page 135
2009 PREP SA on CD-ROM

Critique: 38 Preferred Response: D


Low tone and seizures are relatively common neurologic problems. Low tone is a nonspecific finding
that may be due to disease in the central or peripheral nervous system, but the occurrence of a seizure
suggests a central cerebral cause. Neuroimaging with brain magnetic resonance imaging (MRI)
generally is recommended for any infant who has a seizure because congenital brain malformations are
more common at this age. The combination of such cerebral symptoms and pigmentary abnormalities
of the skin are an indication to obtain brain MRI to assess for a possible neurocutaneous disorder. The
seizures, hypotonia, hypopigmented macules, and MRI findings described for the child in the vignette
are most suggestive of tuberous sclerosis complex (TSC) (Item C38A).
Incontinentia pigmenti is characterized by spasticity rather than hypotonicity, and skin findings
include swirled hyperpigmentation following the line Blaschko (Item C38B). Skin findings associated
with neurofibromatosis type 1 include café au lait macules (Item C38C), axillary freckling, and iris Lisch
nodules (Item C38D). Although Sturge-Weber syndrome can be associated with seizures in the first
postnatal year, the primary skin finding is a port wine stain (Item C38E). Neither seizures nor skin
abnormalities are seen with von Hippel-Lindau syndrome.
Initial management of this patient centers on the chief complaint, which is the seizure. In most
cases, no treatment is recommended in a child after a first unprovoked seizure. However, the
recurrence risk is much greater in TSC.
Subsequently, diagnostic assessment should be directed toward confirming whether this child has
TSC. In many cases, this is a clinical diagnosis based on the characteristic findings of skin examination
and the cerebral complications. However, many other organs may be involved at presentation or during
the child's lifetime, including the eyes, kidneys, lungs, and heart. In the future, this child is at risk for
developmental learning difficulties, behavior problems that can include features of autistic spectrum
disorders, and malignancies. Given the autosomal dominant inheritance, proper management involves
assessment of the parents and genetic counseling. Commercial genetic testing is available and is
helpful in cases where clinical features, particularly early in the disease, do not confirm the diagnosis.
Genetic testing may have false-negative results due to mosaicism, ie, some organs may be affected
due to TSC1 or TSC2 mutations that are not present in blood. Given the complexity of this diagnosis,
many experts recommend that children who have TSC be cared for in multidisciplinary specialty clinics.

References:

Ferner RE. Neurofibromatosis 1 and neurofibromatosis 2: a twenty first century perspective. Lancet
Neurol. 2007;6:340-351. Abstract available at: http://www.ncbi.nlm.nih.gov/pubmed/17362838

Haslam RHA. Neurocutaneous syndromes. In: Kliegman RM, Behrman RE, Jenson HB, Stanton BF,
eds. Nelson Textbook of Pediatrics. 18th ed. Philadelphia, Pa: Saunders Elsevier; 2007:2483-2488

Kandt RS. Tuberous sclerosis complex. In: Singer HS, Kossoff EH, Hartman AL, Crawford TO, eds.
Treatment of Pediatric Neurologic Disorders. Boca Raton, Fla: Taylor & Francis; 2005:553-560

Northrup H, Au K-S. Tuberous sclerosis complex. GeneReviews. 2005. Available at:


http://www.ncbi.nlm.nih.gov/bookshelf/br.fcgi?book=gene&part=tuberous-sclerosis

page 136
2009 PREP SA on CD-ROM

Critique: 38

page 137
2009 PREP SA on CD-ROM

Critique: 38

Swirled hyperpigmentation following the lines of Blaschko is characteristic of


the third stage of incontinentia pigmenti. (Courtesy of D. Krowchuk)

page 138
2009 PREP SA on CD-ROM

Critique: 38

Café au lait macules are a feature of neurofibromatosis type 1. (Courtesy of D.


Krowchuk)

page 139
2009 PREP SA on CD-ROM

Critique: 38

Lisch nodules (iris hamartomas [arrow]) are one of the diagnostic criteria for
neurofibromatosis type 1. (Courtesy of Wake Forest University Eye Center)

page 140
2009 PREP SA on CD-ROM

Critique: 38

A port wine stain involving the distribution of the first and second branches
of the trigeminal nerve is observed in Sturge-Weber syndrome. (Courtesy of
M. Rimsza)

page 141
2009 PREP SA on CD-ROM

Question: 39
You are called to the emergency department to evaluate a 5-month-old boy who has new-onset
seizures. On physical examination, you note that he is thin and has marked hepatomegaly. The mother
tells you that he has been irritable the past several mornings when he awakened from a full night’s
sleep. This morning, she found him seizing in his crib and called 911. Laboratory tests performed on
specimens taken prior to starting intravenous fluids reveal hypoglycemia, lactic acidosis, hyperuricemia,
and hyperlipidemia. You suspect a diagnosis of glycogen storage disease.

Of the following, the MOST appropriate long-term management of this disorder includes

A. coenzyme Q10 administration

B. oral administration of cornstarch

C. oral carnitine supplementation

D. protein restriction

E. restriction of long-chain fats

page 142
2009 PREP SA on CD-ROM

Critique: 39 Preferred Response: B


The hepatomegaly, severe fasting hypoglycemia, lactic acidosis, hyperuricemia, hyperlipidemia, and
ketonuria described for the thin child in the vignette are most consistent with glycogen storage disease
type I (GSD I) (von Gierke disease). GSD I is an autosomal recessive disorder resulting from deficiency
of the enzyme glucose-6-phosphatase, and it is the most serious of all the hepatic glycogenoses.
The laboratory findings result from complete blockage of the release of glycogen. Affected children
typically have massive hepatomegaly without splenomegaly on physical examination, and they may
have a wasted appearance. Kidneys are enlarged and may be palpable on examination. Parents may
give a history of irritability and pallor, especially prior to feedings (after fasting). Some of the children
develop seizures.
The mainstays of treatment for GSD I are the avoidance of fasting and frequent administration of
free glucose. The approaches that have been most successful include continuous nocturnal nasogastric
or gastrostomy feedings or administration of uncooked cornstarch every 4 hours during sleep or other
times of fasting. Maintenance of euglycemia reverses clinical and biochemical abnormalities in most
patients.
Coenzyme Q often is administered to individuals who have mitochondrial disorders and is of unclear
benefit, but it plays no role in the management of GSD I. Similarly, carnitine supplementation and
protein and long-chain fat restrictions are of no benefit in GSD I.
The management of disorders of carbohydrate metabolism, regardless of their cause, is aimed at
ensuring the availability of energy for cellular metabolism without compromising necessary fat and
protein stores. This requires frequent delivery of carbohydrates, and gastrostomy tube placement or
venous access may be necessary to ensure success.

References:

Hoffmann GF, Nyhan WL, Zschocke J, Kahler SG, Mayatepek E. Approach to the patient with hepatic
disease. In: Inherited Metabolic Diseases. Philadelphia, Pa: Lippincott Williams & Wilkins; 2002:191-214

Nyhan WL, Barshop BA, Ozand PT. Disorders of carbohydrate metabolism. In: Atlas of Metabolic
Diseases. 2nd ed. London, England: Hodder Arnold; 2005:371-402

page 143
2009 PREP SA on CD-ROM

Question: 40
A 13-year-old girl comes to your office because her menstrual periods are irregular. She attained
menarche at 12 years of age and states that she has had only four menstrual periods over the past
year. The periods last for 5 to 7 days and require the use of four pads per day. She has never been
sexually active. She plays no sports, but she swims in the summer for fun. On physical examination,
her weight and height are at the 50th percentile for age. She has minimal facial acne and no hirsutism
or other skin lesions. Her breast and genital development is at Sexual Maturity Rating 5.

Of the following, the MOST appropriate management strategy for this patient is to

A. measure follicle-stimulating hormone, luteinizing hormone, and prolactin

B. measure free thyroxine and thyroid-stimulating hormone

C. monitor menses and reassure the girl

D. obtain a bone age by radiography

E. obtain pelvic ultrasonography

page 144
2009 PREP SA on CD-ROM

Critique: 40 Preferred Response: C


Young women of low gynecologic age (ie, a few years after the onset of menstruation) often have
anovulatory cycles due to immaturity of the hormonal feedback system of the
hypothalamic-pituitary-ovarian axis. During the adolescent years, when ovulation does not occur with
every cycle, both frequent and infrequent menstruation can result. By 2 years after menarche, 55% to
82% of cycles are ovulatory, and by 5 years after menarche, 80% to 90% are ovulatory, resulting in
more regular menses. The girl described in the vignette has no signs of androgen excess or evidence
of weight loss that may be associated with an eating disorder but does have scant irregular periods
without cramps or heavy prolonged menses. Because these findings suggest physiologic anovulatory
cycles, no further laboratory studies, such as measurement of follicle-stimulating hormone, luteinizing
hormone, and prolactin, are necessary.
Her continued normal linear growth obviates the need for a bone age measurement. The girl's
normal growth coupled with no evidence of fatigue or skin dryness make thyroid disease unlikely. Pelvic
ultrasonography is not indicated in the absence of signs of androgen excess or other symptoms
suggestive of ovarian pathology. The possibility of pregnancy always should be assessed in the
presence of amenorrhea.

References:

Emans SJ. Amenorrhea in the adolescent. In: Emans SJH, Laufer MR, Goldstein DP, eds. Pediatric and
Adolescent Gynecology. 5th ed. Philadelphia, Pa: Lippincott, Williams & Wilkins; 2005:214-269

Ohlemeyer CL. Menstrual disorders. In Osborn LM, DeWitt TG, First LR, Zenel JA eds. Pediatrics.
Philadelphia, Pa: Elsevier Mosby; 2005:1455-1460

page 145
2009 PREP SA on CD-ROM

Question: 41
You are evaluating a 20-month-old boy who has a rectal temperature of 106°F (41.1°C) and a history of
coughing. His mother reports that the child has had a decrease in activity and eating over the past 2
days. On physical examination, the boy appears moderately ill but is alert and easily interacts with you.
He occasionally grunts, has a heart rate of 140 beats/min, and has a respiratory rate of 55 breaths/min.
His neck is supple, he is circumcised, and he has no evidence of otitis media.

Of the following, the BEST initial test in the evaluation of this child is

A. chest radiography

B. C-reactive protein measurement

C. erythrocyte sedimentation rate

D. lumbar puncture

E. urinalysis

page 146
2009 PREP SA on CD-ROM

Critique: 41 Preferred Response: A


Physical examination, a detailed history, and direct observation can help the clinician recognize up to
90% of children who have a serious illness. Carefully selected laboratory and diagnostic tests can
enhance the detection of invasive bacterial infections in children who have fever. Infants younger than 3
months of age who present with a fever are at high risk of invasive bacterial infection due to their
still-developing immune system. Specific high-risk factors, including a history of prematurity, evidence
of an abnormally elevated or depressed white blood cell count, a focal source of infection such as otitis
media or soft-tissue infection, or abnormal chest findings on radiography are associated with serious
bacterial infection in almost 25% of cases. Infants younger than 3 months of age who do not have such
high-risk factors still have a nearly 3% incidence of a serious bacterial infection. Infants between the
ages of 3 months and 3 years remain at risk for serious bacterial infections, with reports of a 3%
incidence of bacteremia associated with a temperature of 102.2°F (39.0°C) and 7% with a temperature
of 104.0°F (40.0°C). A temperature higher than 105.8°F (41.0°C) frequently is associated with invasive
bacterial infection.
The child described in the vignette has both an elevated temperature and signs and symptoms
suggestive of a primary respiratory infection that make chest radiography the best initial diagnostic test.
A lumbar puncture can be deferred because of the patient's age and appropriate interaction with the
examiner. The patient's age, sex, and circumcised status make a urinary tract infection unlikely; as a
result, urinalysis also could be deferred pending results of chest radiography. Positive urinalysis can be
suggestive of a urinary tract infection, but a urine culture remains the definitive test when a urinary tract
infection is suspected. Elevated erythrocyte sedimentation rates and C-reactive protein measurements
are indicative of the presence of acute inflammatory processes, but they are nonspecific findings. Their
elevation in young children who have fever may be useful in guiding the clinician toward additional
studies in selected children.

References:

Brook I. Unexplained fever in young children: how to manage severe bacterial infection. BMJ.
2003;327:1094-1097. Available at: http://www.bmj.com/cgi/content/full/327/7423/1094

McCarthy PL. Evaluation of the sick child in the office and clinic. In: Kliegman RM, Behrman RE,
Jenson HB, Stanton BF, eds. Nelson Textbook of Pediatrics. 18th ed. Philadelphia, Pa: Saunders
Elsevier; 2007:363-365

page 147
2009 PREP SA on CD-ROM

Question: 42
You are called to the emergency department to see an 8-year-old girl in whom congenital adrenal
hyperplasia was diagnosed at birth. She is being treated with oral mineralocorticoid daily
(9-alpha-fludrocortisone 0.1 mg) and hydrocortisone 5 mg orally every 8 hours. She is febrile
(temperature of 102.0°F [38.9°C]) and has vomited twice. According to her mother, other family
members recently recovered from a gastrointestinal illness that started with fever and vomiting.

Of the following, the MOST appropriate treatment for this child is to

A. administer usual oral medication

B. administer parenteral hydrocortisone hemisuccinate

C. administer parenteral mineralocorticoid

D. double the usual oral dose of mineralocorticoid

E. triple the usual oral dose of hydrocortisone

page 148
2009 PREP SA on CD-ROM

Critique: 42 Preferred Response: B


The girl described in the vignette has adrenal insufficiency and is vomiting and febrile. She needs stress
doses of glucocorticoids but cannot keep down orally administered agents. No parenteral preparation of
mineralocorticoid is available. In contrast to synthetic glucocorticoids, hydrocortisone has about 1% of
the mineralocorticoid effect of aldosterone (the most important natural mineralocorticoid), and a large
dose of hydrocortisone can act as a mineralocorticoid. Accordingly, this child needs a rapid-acting
parenteral hydrocortisone preparation. Hydrocortisone hemisuccinate administered subcutaneously,
unless she has poor perfusion, which would necessitate intramuscular or intravenous administration,
can stabilize her course.
The girl's usual oral medication is not sufficient for the increased need of stress. Increasing the oral
medication may not be sufficient because she probably will not be able to keep it down or absorb it if
she is vomiting. Mineralocorticoid need generally does not increase during stress. If she is vomiting and
receiving parenteral hydrocortisone, she might require intravenous 0.9% saline with glucose to maintain
her sodium stores and her blood glucose in the normal range.
For children who have usual febrile illnesses or require surgery, administration of glucocorticoid in
doses that supply three to six times the usual cortisol secretion rate of 3 to 7 mg/M2 per day is
reasonable treatment. However, children who have adrenal insufficiency and develop varicella because
of failure or lack of immunization must be managed very carefully. There are no controlled studies, but
some anecdotal data suggest that infection with varicella virus can become devastating in
glucocorticoid-treated individuals. For such children, careful titration of hydrocortisone dose to only
twice the usual need may provide the small amount of needed additional glucocorticoid without
compromising the children's immune responses.

References:

Donohoue PA. Treatment of adrenal insufficiency in children. UptoDate Online 15.3. 2008. Available for
subscription at: http://www.uptodateonline.com/utd/content/topic.do?topicKey=pediendo/19876

Shulman DI, Palmert MR, Kemp SF, for the Lawson Wilkins Drug and Therapeutics Committee. Adrenal
insufficiency: still a cause of morbidity and death in childhood. Pediatrics. 2007;119:e484-e494.
Available at: http://pediatrics.aappublications.org/cgi/content/full/119/2/e484

Wilson TA, Speiser P. Adrenal insufficiency. eMedicine Specialties, Pediatrics: General Medicine,
Endocrinology. 2007. Available at:
http://www.emedicine.com/ped/TOPIC47.HTM

page 149
2009 PREP SA on CD-ROM

Question: 43
A 9-year-old girl has been evaluated by a learning consultant and found to have a slow reading rate,
weakness in short-term memory, and problems with reading comprehension. Her parents ask you what
subjects other than reading will be most challenging for her due to these learning difficulties.

Of the following, the subject that this child should find MOST challenging is

A. art

B. creative writing

C. mathematics

D. music

E. social studies

page 150
2009 PREP SA on CD-ROM

Critique: 43 Preferred Response: E


A student who has a slow reading rate, reduced reading comprehension, and impaired short-term
memory, such as the girl described in the vignette, will encounter problems in "content" classes, which
include subjects such as science, history, and social studies. When children read texts in these
subjects, they need to read factual information and use the material in the text to obtain knowledge
about the subject. This requires comprehension of the text, which involves identifying and
understanding the words.
To determine the meaning of a word, a reader first must decode and identify the word on the page. A
slow reader takes much longer to complete assignments and test questions. Further, individuals who
have weakness in understanding and remembering the text will have much more difficulty with
homework assignments and on examinations. The girl in the vignette will not have the same difficulty
with art, music, creative writing, or mathematics because these subjects do not demand the same
emphasis on reading a text and recalling facts.

References:

Fletcher JM, Lyon GR, Fuchs LS, Barnes MA. Reading disabilities: comprehension. In: Learning
Disabilities: From Identification to Intervention. New York, NY: The Guilford Press; 2007:184-206

Shaywitz SE, Shaywitz BA. Dyslexia (specific reading disability). Pediatr Rev. 2003;24:147-153.
Available at: http://pedsinreview.aappublications.org/cgi/content/full/24/5/147

page 151
2009 PREP SA on CD-ROM

Question: 44
As you are leaving the supermarket, the cashier tells you that she is worried because her child recently
had a positive tuberculin skin test. She had to take him to the health department for skin testing
because he had been in contact with her father, who recently was diagnosed with active pulmonary
tuberculosis. They told her that the boy’s skin test was positive at "25," but his chest radiograph was
normal. She is concerned because the doctor told her that the case is a little unusual because of the
type of tuberculosis her father has. She asked the physician at the health department to write it down,
and she hands you a piece of paper that says "INH resistant." The mother asks you what type of
medication her boy should receive.

Of the following, the MOST appropriate antituberculous agent to prescribe for this boy is

A. ciprofloxacin

B. ethambutol

C. isoniazid

D. pyrazinamide

E. rifampin

page 152
2009 PREP SA on CD-ROM

Critique: 44 Preferred Response: E


With a positive tuberculin skin test, a negative chest radiograph, and no indications of active disease,
the patient described in the vignette meets the classification of a latent tuberculosis infection (LTBI).
LTBI usually is treated with isoniazid (INH) once daily for 9 months, but when the source case is known
to have INH resistance, this agent should not be used. Instead, a 6-month course of rifampin is
recommended. An exception to this approach would be if the source case was known to be resistant to
both INH and rifampin, in which case a tuberculosis expert should be consulted to determine the best
course of treatment. Ciprofloxacin, ethambutol, and pyrazinamide are used in combination with other
antituberculous agents for the treatment of active tuberculous disease; they are not indicated for
monotherapy in treating LTBI.
Although usually very well tolerated, patients who receive rifampin should be aware of possible
adverse effects. Urine, tears, and saliva change to a reddish-orange color, which may stain clothes or
contact lenses. Rifampin therapy also may be associated with mild "flulike" symptoms (eg, myalgias)
that resolve with continued therapy. Rifampin also induces cytochrome P-450 activity and, therefore,
decreases the half-life of medications such as warfarin, digoxin, thyroxine, oral contraceptives, and
some antimicrobial agents (eg, chloramphenicol), making them less effective. When rifampin is used in
combination with INH, patients are twice as likely to develop hepatitis as are patients treated with
rifampin in combination with other antituberculous medications. Thrombocytopenia and leukopenia both
have been associated with rifampin therapy exceeding 1 month's duration.

References:

Alsayyed B, Adam HM. In brief: rifampin. Pediatr Rev. 2004;25:216-217. Available at:
http://pedsinreview.aappublications.org/cgi/content/full/25/6/216

American Academy of Pediatrics. Tuberculosis. In: Pickering LK, Baker CJ, Long SS, McMillan JA, eds.
Red Book: 2006 Report of the Committee on Infectious Diseases. 27th ed. Elk Grove Village, Ill:
American Academy of Pediatrics; 2006:678-698

Bliziotis IA, Ntziora F, Lawrence KR, Falagas ME. Rifampin as adjuvant treatment of Gram-positive
bacterial infections: a systemic review of comparative clinical trials. Eur J Clin Microbiol Infect Dis.
2007;26:849-856. Abstract available at: http://www.ncbi.nlm.nih.gov/pubmed/17712583

page 153
2009 PREP SA on CD-ROM

Question: 45
You are speaking to a group of medical students about different antibiotic classes that can be used in
the treatment of meningitis. One student asks you about chloramphenicol, a drug with which he is not
familiar.

Of the following, the MOST common adverse effect associated with chloramphenicol therapy is

A. dose-related bone marrow suppression

B. drug eruption

C. gray baby syndrome

D. idiosyncratic aplastic anemia

E. optic neuritis

page 154
2009 PREP SA on CD-ROM

Critique: 45 Preferred Response: A


Chloramphenicol is an antibiotic that inhibits protein synthesis by reversibly binding to the 50S subunit
of the 70S ribosome. Against most sensitive organisms, chloramphenicol produces a static effect by
blocking protein synthesis. However, it is bactericidal against some meningeal pathogens, such as
Haemophilus influenzae, Streptococcus pneumoniae, and Neisseria meningitidis. Chloramphenicol is
not bactericidal against group B streptococci or enteric gram-negative bacilli because drug
concentrations in the cerebrospinal fluid sufficient for effective eradication of these organisms cannot be
achieved at therapeutic doses. With the possible exception of typhoid fever in some developing areas
of the world where cost and availability make it the primary therapy, chloramphenicol no longer is the
drug of choice for any specific infection.
Chloramphenicol is extremely active against a variety of organisms, including bacteria, spirochetes,
rickettsiae, chlamydiae, and mycoplasmas. Most gram-positive and gram-negative aerobic bacteria are
inhibited by concentrations achievable in the serum, and chloramphenicol is one of the most effective
antibiotics against anaerobic bacteria, including Bacteroides strains. Salmonellae, including S typhi,
generally are susceptible, although chloramphenicol-resistant strains have emerged rapidly since the
late 1980s in India, Korea, Vietnam, Peru, Mexico, and Thailand.
Plasma concentrations of chloramphenicol vary, depending on the time and route of administration.
Oral chloramphenicol is well absorbed from the gastrointestinal tract and produces peak serum
concentrations of 12 mcg/mL of active antibiotic after a 1-g dose. The intravenous preparation of the
drug produces active chloramphenicol concentrations in the serum that are 70% of those obtained after
an oral dose. Intramuscular administration is well tolerated and produces peak serum values that are
similar to those seen with intravenous administration. However, for patients who have enteric fever,
peak serum concentrations of the drug after intramuscular administration are only one half to two thirds
of those obtained by intravenous administration due to delayed drug absorption from the injection site.
Metabolism and excretion of chloramphenicol vary widely in the pediatric population and are age-
and patient-dependent. Therefore, monitoring of serum concentrations is critical. Significant drug-drug
interactions occur between chloramphenicol and other agents. Chloramphenicol prolongs the half-life of
tolbutamide, chlorpropamide, phenytoin, cyclophosphamide, and warfarin by inhibiting hepatic
microsomal enzymes. Drugs such as phenytoin, rifampin, and phenobarbital decrease the serum
concentration and increase the total body clearance of chloramphenicol by inducing hepatic microsomal
enzymes. Chloramphenicol may delay the response of anemias to iron, folic acid, and vitamin B12, and
in vitro it antagonizes the bactericidal activity of the penicillins, cephalosporins, and aminoglycoside
antibiotics.
The most common adverse effect associated with chloramphenicol is reversible bone marrow
depression that is characterized by a combination of reticulocytopenia, anemia, leucopenia, and
thrombocytopenia. Other adverse effects include idiosyncratic aplastic anemia that is rare but may be
fatal; hemolytic anemia in patients who have the Mediterranean form of glucose-6-phosphate
dehydrogenase deficiency; gray baby syndrome in neonates, which is characterized by abdominal
distention, vomiting, flaccidity, cyanosis, circulatory collapse, and death; and optic neuritis that may
occur in patients receiving prolonged therapy. Hypersensitivity reactions (eg, rashes, drug fevers,
anaphylaxis) are rare.

References:

Kauffman RE, Miceti JN, Strebel L, Buckley JA, Done AK, Dajani AS. Pharmacokinetics of
chloramphenicol and chloramphenicol succinate in infants and children. J Pediatr. 1981;98:315-320.
Abstract available at: http://www.ncbi.nlm.nih.gov/pubmed/7463235

page 155
2009 PREP SA on CD-ROM

Myers B, Salvatore M. Tetracyclines and chloramphenicol. In: Mandell GL, Bennett JE, Dolin R, eds.
Mandell, Douglas and Bennett's Principles and Practice of Infectious Diseases. 6th ed. New York, NY:
Elsevier Churchill Livingstone, 2005:356-373

Pickering LK, Hoecker JL, Kramer WG, Kohl S, Cleary TG. Clinical pharmacology of two
chloramphenicol preparations in children: sodium succinate (IV) and palmitate (oral) esters. J Pediatr.
1980;96:757-761. Abstract available at: http://www.ncbi.nlm.nih.gov/pubmed/6987361

Rahal JJ Jr, Simberkoff MS. Bactericidal and bacteriostatic action of chloramphenicol against
meningeal pathogens. Antimicrob Agents Chemother. 1979;16:13-18. Available at:
http://aac.asm.org/cgi/reprint/16/1/13?view=long&pmid=38742

Yunis AA. Chloramphenicol-induced bone marrow suppression. Semin Hematol. 1973;10:225-234

page 156
2009 PREP SA on CD-ROM

Question: 46
A 2-year-old boy presents with fever and abdominal pain. Urinalysis reveals a specific gravity of 1.010,
pH of 5.5, 2+ protein, no blood, and negative leukocyte esterase and nitrite tests. Microscopy findings
are negative.

Of the following, the MOST appropriate diagnostic test to assess the severity of proteinuria in this child
is

A. a random urine sample measurement for quantitative protein and creatinine concentrations

B. measurement of serum albumin concentration and correlation with urine protein concentration
measured by dipstick

C. measurement of urine microalbumin excretion

D. sulfosalicylic acid treatment of a freshly voided urine sample

E. 24-hour urine collection for protein and creatinine concentrations

page 157
2009 PREP SA on CD-ROM

Critique: 46 Preferred Response: A


The 2+ protein measurement on a dilute urine sample (urine specific gravity <1.015) described for the
boy in the vignette is abnormal and requires quantitation of the proteinuria. Although traditional teaching
favored 24-hour urine testing for quantitative assessment of proteinuria in the past, this technique no
longer is necessary. Spot/random urine samples for proteinuria correlate closely with 24-hour urine
collections and are much easier to collect via a urine bag, especially in the child who is not yet toilet
trained. A normal spot urine protein-to-creatinine ratio is less than 0.2.
The urine dipstick is a good screening test for proteinuria, but because it is a qualitative test, it is a
suboptimal indicator of the severity of proteinuria. Similarly, sulfosalicylic acid treatment of freshly
voided urine is a qualitative, not quantitative test. The sulfosalicylic acid induces precipitation of
proteins. Urine microalbumin is an ultrasensitive technique (using a radioimmunoassay, enzyme-linked
immunoassay, or nephelometry) to detect albumin excretion in the urine that falls below that measured
on a urine dipstick. Assessment of microalbuminuria is used commonly in patients who have diabetes
as a screening test for renal disease before overt proteinuria occurs. Microalbuminuria screens also
have been used in those who have hypertension. Although this technique is quantitative, it has no role
when overt proteinuria is present, as measured on a urine dipstick. Measurement of serum albumin
should be deferred until the severity of the proteinuria can be quantified.
Although 24-hour urine testing for proteinuria no longer is necessary, the test remains useful for
evaluating children for risk factors for nephrolithiasis/urolithiasis or to screen for catecholamine
metabolites in children who have suspected neural crest tumors (neuroblastoma or
pheochromocytoma).

References:

Abitbol C, Zilleruelo G, Freundlich M, Strauss J. Quantitation of proteinuria with urinary


protein/creatinine ratios and random testing with dipsticks in nephrotic children. J Pediatr.
1990;116:243-247. Abstract available at: http://www.ncbi.nlm.nih.gov/pubmed/2299494

Chahar OP, Bundella B, Chahar CK, Purohit M. Quantitation of proteinuria by use of single random spot
urine collection. J Indian Med Assoc. 1993;91:86-87. Abstract available at:
http://www.ncbi.nlm.nih.gov/pubmed/8409488

Gregianin LJ, McGill AC, Pinheiro CM, Brunetto AL. Vanilmandelic acid and homovanillic acid levels in
patients with neural crest tumor: 24-hour urine collection versus random sample. Pediatr Hematol
Oncol. 1997;14:259-265. Abstract available at: http://www.ncbi.nlm.nih.gov/pubmed/9185210

Pontremoli R, Leoncini G, Ravera M, et al. Microalbuminuria, cardiovascular, and renal risk in primary
hypertension. J Am Soc Nephrol. 2002;13:S169-S172. Available at:
http://jasn.asnjournals.org/cgi/content/full/13/suppl_3/S169

page 158
2009 PREP SA on CD-ROM

Question: 47
A 12-month-old girl presents with a 3-month history of a pruritic rash that involves her cheeks, neck,
anterior trunk, and antecubital and popliteal areas. The rash improves after use of an over-the-counter
topical steroid cream but still is present most days, and the infant often wakes up at night scratching.
On physical examination, you observe a raised erythematous rash that has areas of lichenification (Item
Q47).

Of the following, the MOST helpful intervention is to

A. eliminate fruit and acidic juices from the diet

B. eliminate milk, eggs, soy, and wheat from the diet

C. perform aeroallergen allergy testing

D. perform food allergy testing

E. recommend a skin biopsy

page 159
2009 PREP SA on CD-ROM

Question: 47

(Courtesy of D. Krowchuk)

page 160
2009 PREP SA on CD-ROM

Critique: 47 Preferred Response: D


Some 30% to 40% of infants who have moderate-to-severe atopic dermatitis (AD), such as described
for the infant in the vignette, may have an underlying immunoglobulin (Ig) E-mediated food allergy
exacerbating the AD. For some infants, food ingestion may result in immediate worsening of AD
severity, although most infants do not demonstrate this immediate reaction. Many foods have been
implicated in AD, but five (milk, eggs, soy, wheat, and peanut) account for 90% of the causative
allergens.
Both allergy skin testing and measurement of serum IgE concentrations to these foods can help to
identify and eliminate likely triggers. Either a negative IgE blood test (<0.35 kU/L) or a negative skin test
for a specific food provides a high negative predictive value. On the other hand, the positive predictive
value for a skin or blood test may be only 50%.
Although the most commonly implicated foods often are eliminated from the diet, such an approach
does not improve symptoms in most (60% to 70%) children because they do not have IgE-mediated
AD. The unnecessary elimination of multiple foods can have an adverse effect on nutrition, and food
avoidance should be guided by the dietary history, eczema severity, and skin or blood testing.
Frequently, children experience perioral rashes after drinking fruit juice. Such rashes typically are
nonpruritic, limited to the area of contact, and resolve within a few hours. The mechanism of such
rashes is unknown, but children generally outgrow such reactions by age 4 years. In cases involving
more widespread cutaneous symptoms, such as described in the vignette, elimination of fruit or acidic
juices is unnecessary.
Parents often request testing for environmental allergies. House dust mites have been implicated in
some cases of AD, although they are less likely a cause for moderate-to-severe atopic dermatitis than
food allergies. Climate changes such as cold, dry air or hot, humid weather can worsen AD, but specific
seasonal allergens such as oak tree or ragweed are not associated with eczema in infants.
A skin biopsy can provide insight into the pathophysiology of chronic rashes or lesions. Generally,
skin biopsies neither are advised nor provide insight into the causes of typical AD manifestations in
infants, but atypical presentations or lack of expected improvement with appropriate therapy should
prompt consideration of a dermatology referral.

References:

Burks W. Skin manifestations of food allergy. Pediatrics. 2003;111:1617-1624. Available at:


http://pediatrics.aappublications.org/cgi/content/full/111/6/S2/1617

Greer FR, Sicherer SH, Burks AW, Committee on Nutrition and Section on Allergy and Immunology.
Effects of early nutritional interventions on the development of atopic disease in infants and children:
the role of maternal dietary restriction, breastfeeding, timing of introduction of complementary foods
and hydrolyzed formulas. Pediatrics. 2008;121:183-191. Available at:
http://pediatrics.aappublications.org/cgi/content/full/121/1/183

Sampson HA, Leung DYM. Adverse reactions to foods. In: Kleigman RM, Behrman RE, Jenson HB,
Stanton BF, eds. Nelson Textbook of Pediatrics. 18th ed. Philadelphia, Pa: Saunders Elsevier;
2007:986-989

page 161
2009 PREP SA on CD-ROM

Question: 48
The mother of a 2-year-old boy calls you because she found her son holding an open bottle of liquid
dishwasher detergent. He is crying, drooling profusely, and has vomited three times. In answer to your
questions, she reports that he is not sleepy and did not seem to get it in his eyes or on his skin. On
examination in your office, you note an ulcer on his lower lip and several ulcers on his tongue.

Of the following, the most appropriate next step in the evaluation and management of this boy is to

A. administer activated charcoal

B. administer ipecac syrup

C. reassure the mother that the product is nontoxic

D. refer the boy to a gastroenterologist for urgent endoscopy

E. refer the boy to the emergency department for gastric lavage

page 162
2009 PREP SA on CD-ROM

Critique: 48 Preferred Response: D


Dishwasher detergent is a highly caustic alkali that can cause significant oropharyngeal, upper airway,
and esophageal injury if ingested. Alkaline agents cause deep liquefaction necrosis of the affected
tissues, with ulceration and perforation likely complications. Common household products such as oven
and drain cleaners, dishwasher detergents, and cleaning products that contain ammonia pose a risk of
caustic injury. Substances that have a pH greater than 11 are likely to cause injury, even in small
amounts.
Children who have ingested caustic or corrosive agents typically present with drooling, dysphagia, or
emesis, as reported for the boy in the vignette, although the presence and severity of symptoms is not
predictive of esophageal injury. The presence or absence of oral lesions also is not predictive of
esophageal injury.
The initial evaluation and treatment of patients who have ingested caustic substances includes
stabilization of vital functions, pain management, and other supportive measures. Decontamination with
activated charcoal is not indicated because it will not adsorb acids or alkalis and is likely to impair
endoscopic evaluation. Use of ipecac syrup or gastric lavage is contraindicated because of the potential
to cause further injury to the esophagus. In the symptomatic patient, endoscopy should be undertaken
within 6 to 24 hours of ingestion to determine the presence of esophageal burns and the degree of
severity.
Evaluation of the asymptomatic patient is controversial. Because the available data suggest that
esophageal burns may be found in as many as 45% of those who do not have oral lesions, endoscopy
is recommended in clear cases of ingestion. If the history is questionable and the patient remains
asymptomatic, with normal swallowing, after several hours of observation, some authors have
suggested that it is safe to forego endoscopy.

References:

Cordero B, Savage RR, Cheng TL. In brief: corrosive ingestions. Pediatr Rev. 2006;27:154-155.
Available at: http://pedsinreview.aappublications.org/cgi/content/full/27/4/154

Ferry GD. Caustic esophageal injury in children. UpToDate. 2008. Available for subscription at:
http://www.utdol.com/utd/content/topic.do?topicKey=pedigast/11441&view=print

Gaudreault P, Parent M, McGuigan MA, Chicoine L, Lovejoy FH Jr. Predictability of esophageal injury
from signs and symptoms: a study of caustic ingestion in 378 children. Pediatrics. 1983;71:767-770.
Available at: http://pediatrics.aappublications.org/cgi/content/full/71/5/767

page 163
2009 PREP SA on CD-ROM

Question: 49
A 12-year-old boy has had cholestasis since infancy from Alagille syndrome. He has been lost to
medical follow-up for the last several years. He now presents to your office with pain in his right upper
thigh after a fall. His thigh is intensely tender, and ultrasonography demonstrates a large hematoma in
his quadriceps. The parents state that he has tended to bruise easily in the past few months.

Of the following, the condition MOST likely to account for this patient’s symptoms is

A. factor VIII deficiency

B. idiopathic thrombocytopenic purpura

C. vitamin C deficiency

D. vitamin K deficiency

E. von Willebrand disease

page 164
2009 PREP SA on CD-ROM

Critique: 49 Preferred Response: D


Alagille syndrome is characterized by cardiac disease (especially peripheral pulmonary stenosis),
vertebral anomalies, ocular anomalies (posterior embryotoxon), facial dysmorphism (triangular facies,
macrocephaly, large ears) (Item C49A), and paucity of the intrahepatic bile ducts. The hepatic
manifestations of this syndrome account for much of the medical morbidity. Specifically, impaired bile
flow results in chronic cholestasis, which leads to severe pruritus, jaundice, malabsorption of nutrients,
and malabsorption of fat-soluble vitamins (Item C49B). Although most affected children have their
jaundice improve as they grow older, a subset progresses to cirrhosis and requires liver transplantation.
Patients who have hepatic disease must have their nutritional status monitored carefully. Chronic
anorexia, recurrent illnesses, and fat malabsorption may result in caloric deficiency and growth failure.
Caloric supplementation by nasogastric tube or gastrostomy may be necessary to ensure adequate
caloric intake. In addition, patients who have cholestasis are at risk for fat-soluble vitamin deficiency.
Vitamin D deficiency typically causes osteopenia and rickets, vitamin E deficiency causes peripheral
neuropathy and ataxia, and vitamin A deficiency may cause night blindness or corneal lesions. The
bruising described for the patient in the vignette most likely is due to vitamin K deficiency. Vitamin K is a
cofactor essential in posttranscriptional carboxylation of the clotting factors II, VII, IX, and X. Thus,
vitamin K deficiency leads to prolonged prothrombin and partial thromboplastin time, which predisposes
to bruising. Although factor VIII deficiency, vitamin C deficiency, von Willebrand disease, and idiopathic
thrombocytopenia purpura also may cause bruising, the patient who has Alagille syndrome is not at
increased risk for developing these conditions.
In addition to supplementing patients who have chronic liver disease with fat-soluble vitamins, the
clinician caring for these patients also must supply adequate calories. Patients who have advanced
chronic liver disease may have both anorexia and increased caloric requirements. In addition, patients
who have portal hypertension and ascites may need to have total fluid intake restricted, which, in turn,
means that they may require a more concentrated and less palatable formula. For these reasons,
nasogastric or gastrostomy feedings sometimes are necessary to achieve optimal growth, especially
when preparing a patient for liver transplantation.

References:

Cranenburg ECM, Shurgers LJ, Vermeer C. Vitamin K: the coagulation vitamin that became
omnipotent. Thromb Haemost. 2007;98:120-125. Available at:
http://www.schattauer.de/index.php?id=1268&pii=th07070120&no_cache=1

Kamath BM, Piccoli DA. Heritable disorders of the bile ducts. Gastroenterol Clin North Am.
2003;32:857-875. Abstract available at: http://www.ncbi.nlm.nih.gov/pubmed/14562578

Sokol RJ. Fat-soluble vitamins and their importance in patients with cholestatic liver diseases.
Gastroenterol Clin North Am. 1994;23:673-705. Abstract available at:
http://www.ncbi.nlm.nih.gov/pubmed/7698827

page 165
2009 PREP SA on CD-ROM

Critique: 49

The broad forehead and pointed chin characteristic of Alagille syndrome.


(Courtesy of M. Rimsza)

page 166
2009 PREP SA on CD-ROM

Critique: 49

page 167
2009 PREP SA on CD-ROM

Question: 50
A term infant is delivered by emergency cesarean section following the acute onset of maternal vaginal
bleeding and profound fetal bradycardia on electronic fetal heart rate monitoring. The Apgar scores are
1, 2, and 3 at 1, 5, and 10 minutes, respectively. Resuscitation includes intubation and assisted
ventilation, chest compressions, and intravenous epinephrine. The infant is admitted to the neonatal
intensive care unit and has seizures at 6 hours of age.

Of the following, a TRUE statement about infants who have seizures following perinatal asphyxia is that
most

A. develop epilepsy

B. develop microcephaly

C. do not have severe long-term neurodevelopmental delay

D. experience hearing loss

E. require multiple anticonvulsant medications

page 168
2009 PREP SA on CD-ROM

Critique: 50 Preferred Response: C


The infant described in the vignette has concerning perinatal events, low Apgar scores through 10
minutes of postnatal age, the need for vigorous resuscitation, and subsequent seizure activity 6 hours
after birth. Seizures occurring in the first 24 hours after birth in newborns who have perinatal asphyxia
are considered evidence of hypoxic-ischemic encephalopathy. Accompanied by profound acidemia,
these seizures also may reflect significant cerebral cellular injury or cell death.
The causes of neonatal seizure broadly include hypoxia-ischemia, which may be global (as in
perinatal asphyxia) or focal (as in arterial or venous thrombosis); hemorrhagic injury; brain
malformation; transient metabolic disorders (such as hypoglycemia or hypocalcemia); infectious (viral,
bacterial, or fungal); inborn errors of metabolism; or unknown (Item C50A). As demonstrated in Item
C50B, most term infants who have neonatal seizures do not manifest long-term neurodevelopmental
sequelae. Further, even though motor abnormalities may be found on examination in a slight majority
(53%) of newborns having a seizure for any cause, few have cerebral palsy.
However, when seizures occur in newborns due to asphyxia (generalized hypoxic-ischemic injury),
the prevalence of abnormal neurodevelopmental outcomes is more concerning. Although severe
impairment in neurodevelopmental outcomes may occur in fewer than 50% of these infants, the
presence of mild-to-moderate neurodevelopmental impairment in cognitive and motor function as well
as some (~33%) children having long-term epilepsy brings the level of abnormal outcomes to
approximately 50% (Item C50B). Early predictions of outcome for such children may be determined by
evaluating the worst early electroencephalography (EEG) finding, the follow-up 1-week EEG, and the
findings evident on cranial magnetic resonance imaging.
Following perinatal asphyxia and resultant seizures, most newborns can be treated with a single
anticonvulsant drug. Longstanding epilepsy and microcephaly occur in fewer than 50% of patients.
Hearing loss, though more common in asphyxiated infants who have neurologic injury than in
uncomplicated term infants, still occurs in fewer than 10% of infants.

References:

Hill A. Neonatal seizures. Pediatr Rev. 2000;21:117-121. Available at:


http://pedsinreview.aappublications.org/cgi/content/full/21/4/117

Ronen GM, Buckley D, Penney S, Streiner DL. Long-term prognosis in children with neonatal seizures:
a population-based study. Neurology. 2007;69:1816-1822. Abstract available at:
http://www.ncbi.nlm.nih.gov/pubmed/17984448

Tekgul H, Gauvreau K, Soul J, et al. The current etiologic profile and neurodevelopmental outcome of
seizures in term newborn infants. Pediatrics. 2006;117:1270-1280. Available at:
http://pediatrics.aappublications.org/cgi/content/full/117/4/1270

page 169
2009 PREP SA on CD-ROM

Critique: 50

page 170
2009 PREP SA on CD-ROM

Critique: 50

page 171
2009 PREP SA on CD-ROM

Question: 51
A 16-year-old girl who is new to your practice complains of a nearly constant headache for the past
year. She describes the pain as a band around her head that often is throbbing and is worse during the
middle of the day. She denies nausea or vomiting but reports occasional fatigue. There is no family
history of headaches. She has missed more than 20 days of school this year because of the headache,
and she is struggling to maintain a C average. She admits to hating school and does not participate in
extracurricular activities because she "doesn’t like anything." Findings on her physical examination,
including complete neurologic and funduscopic evaluation, are normal.

Of the following, the BEST next step in the management of this girl’s headaches is to

A. advise her to keep a headache diary and return in 2 months

B. obtain a lumbar puncture

C. obtain computed tomography scan of the brain

D. prescribe oral sumatriptan

E. refer her for psychosocial evaluation and counseling

page 172
2009 PREP SA on CD-ROM

Critique: 51 Preferred Response: E


Chronic headache is a common complaint in children and adolescents. A careful history and complete
neurologic examination are indicated in the evaluation of headache. The most important initial
consideration is to identify characteristics of the headache that suggest serious diagnoses, such as
brain tumors or other diseases causing increased intracranial pressure. Such characteristics include
worsening pain at night or immediately upon awakening, association with vomiting, and worsening pain
with coughing or straining. Papilledema or focal neurologic findings may be found in patients who have
increased intracranial pressure or the examination results may be normal.
Migraine headaches are periodic, may be accompanied by an aura, and typically are relieved by
sleep. A family history of migraines usually can be elicited. The neurologic examination typically yields
normal results, although complicated migraines can be accompanied by focal neurologic deficits such
as hemiparesis, cranial nerve palsies, and visual disturbances.
Pain from stress-related or tension headache generally is diffuse and may be described as
"bandlike" or throbbing, as reported for the girl in the vignette. Pain usually occurs on most days, and
school absence is frequent. The neurologic examination yields normal results.
It is also very important for the clinician to obtain further information regarding any emotional, social,
or academic difficulties the patient may be experiencing. Such data can help to determine both an
underlying cause for the headache and the effect the headache is having on the child's quality of life.
Family stressors and depression are known causes of headache, and questions regarding sleep
patterns, anhedonia, school performance, and relationships with family and friends can help to screen
for these conditions. Studies have shown that children and adolescents who have frequent or severe
headaches have greater impairment in academic and social functioning than those who do not have
headaches. A thorough psychosocial evaluation can aid in evaluating patients for depression and
assessing the effect of headaches on their daily functioning. Nonpharmacologic therapies, such as rest,
relaxation techniques, and removal of stress from the environment, can be effective once the stress is
identified.
Advising the girl in the vignette to keep a diary for 2 months likely would prolong her difficulties and
not address the underlying cause of her headaches. Lumbar puncture and computed tomography scan
of the brain may play some role in the evaluation of an acute headache, especially if meningitis or a
mass lesion of the brain is suspected, but these diagnoses are very unlikely in a patient who has had a
headache for a year and normal results on neurologic examination. Oral sumatriptan is used in the
treatment of migraine, but this girl's history points to depression or another psychological cause for her
headache, and prescribing medications without a psychosocial evaluation probably would not be
effective.

References:

Forsyth R, Farrell K. Headache in childhood. Pediatr Rev. 1999;20:39-45. Available at:


http://pedsinreview.aappublications.org/cgi/content/full/20/2/39

Silver N. Headache (chronic, tension type). BMJ Clinical Evidence. 2007. Available for subscription at:
http://clinicalevidence.bmj.com/ceweb/conditions/nud/1205/1205_guidelines.jsp

Strine TW, Okoro CA, McGuire LC, Balluz LS. The associations among childhood headaches,
emotional and behavioral difficulties, and health care use. Pediatrics. 2006;117: 1728-1735. Available
at: http://pediatrics.aappublications.org/cgi/content/full/117/5/1728

page 173
2009 PREP SA on CD-ROM

Question: 52
When a 14-year-old girl had frequent complaints of shoulder pain made worse by pitching softball a few
months ago, you diagnosed overuse injury. Nonsteroidal anti-inflammatory drugs and rest have
provided some relief. She presents today with complaints of recurrent upper arm pain that is unrelated
to exercise and sometimes awakens her from sleep. Physical examination reveals a slightly larger
circumference of the left proximal humerus compared with the right. There is minimal tenderness on
palpation over the area, although the girl reports a constant ache. She has full range of motion of the
arm at the shoulder and elbow. You obtain a shoulder radiograph (Item Q52).

Of the following, the MOST likely diagnosis is

A. acromioclavicular separation

B. acute osteomyelitis

C. chronic osteomyelitis

D. osteosarcoma

E. supracondylar fracture of the humerus

page 174
2009 PREP SA on CD-ROM

Question: 52

(Courtesy of the Media Lab at Doernbecher)

page 175
2009 PREP SA on CD-ROM

Critique: 52 Preferred Response: D


The girl described in the vignette has radiographic findings and clinical history that are most suggestive
of osteogenic sarcoma of the humerus. The most common presenting symptom of osteosarcoma is
pain, particularly with activity, as described for the girl. The affected patient may have a history of
swelling, depending on the size of the lesion and its location. Patients or their parents may complain of
a "sprain," "arthritis," or so-called "growing pains." Symptoms may be present for weeks, months, or
occasionally longer before osteosarcoma is diagnosed. The child often has a history of trauma, as in
this patient, because trauma occurs frequently in teenagers. Pathologic fractures are not particularly
common with this tumor compared with leukemia and other malignancies. Systemic symptoms, such as
fever and night sweats, are rare.
Osteosarcoma is the third most common cancer in adolescence, after lymphomas and brain tumors.
Peak age at diagnosis is 10 to 25 years. The incidence is 400 cases per year in the United States (4.8
cases per 1 million persons younger than 20 years) and is slightly higher in African Americans than in
whites. An increased incidence during adolescence corresponds with the growth spurt.
Osteosarcoma occurs in the long bones near metaphyseal growth plates (Item C52A). Most are
high-grade intramedullary osteosarcomas, with only about 5% being low-grade lesions. The most
common sites are the femur (42%, with 75% of femoral tumors occurring distally), tibia (19%, with 80%
in the proximal tibia), and humerus (10%, with 90% in the proximal humerus). The overall 5-year
survival rate for patients whose condition was diagnosed between 1974 and 1994 was 63% (59% for
males, 70% for females). The current 5-year survival rate is estimated to be 65%. The mainstay of
therapy is excision of the lesion. Chemotherapy is required to treat micrometastatic disease, which is
present but not detectable in most patients at diagnosis. Metastatic spread to the lungs only rarely
results in respiratory symptoms; such symptoms usually indicate extensive lung involvement.
Metastasis to other sites is extremely rare.
The exact cause of osteosarcoma is unknown, but it is believed to be a tumor of osteoprogenitor
cells, which are multipotential, hormone-responsive stromal cells in the periosteum and marrow that are
capable of differentiating into many lineages, depending on their environment. Among the known risk
factors are rapid bone growth, exposure to radiation, and potentially a genetic predisposition. Children
who have a prior personal or family history of retinoblastoma and those who have received radiation
therapy for a previous malignancy are at higher risk of developing osteosarcoma.
Initial evaluation of an adolescent or older child presenting with bone pain and swelling, especially
with a palpable mass, should include:
o Plain radiographs (two views) of the suspected lesions, although no single feature on radiographs
is diagnostic. Osteosarcomatous lesions can be purely osteolytic (about 30% of patients), purely
osteoblastic (about 45% of patients), or a mixture of both. Elevation of the periosteum may appear as
the characteristic Codman triangle. Extension of tumor through the periosteum may result in a so-called
"sunburst appearance" (about 60% of patients).
o Both magnetic resonance imaging of the primary lesion and computed tomography scan of the
chest are necessary to confirm the diagnosis and for staging purposes. Such scans frequently are
performed at the tertiary center using their protocols.
The differential diagnosis of bone pain and swelling includes stress fracture, hematoma, bone cysts,
and other bony tumors such as Ewing sarcoma.
Although acute osteomyelitis is common in children, it is relatively less common in teenagers;
approximately 50% of cases occur in preschool-age children. Infection frequently is characterized by
overlying erythema, warmth, and more acute systemic signs such as fever and malaise. The white
blood cell count is elevated in 50% of patients, and the erythrocyte sedimentation rate or C-reactive
protein value is increased. Plain radiographs may be read as normal early in the disease course.
Chronic osteomyelitis also may cause pain. However, results of blood tests often are normal. The

page 176
2009 PREP SA on CD-ROM

radiograph for the patient in the vignette, however, shows a classic sunburst pattern, which is not
consistent with chronic osteomyelitis.
Acromioclavicular (AC) separation typically results in sudden pain and limited range of motion. The
diagnosis can be made by an anteroposterior radiograph, which can demonstrate excessive separation
of the AC joint (Item C52B). Supracondylar fracture of the humerus usually is caused by falling onto the
extremity (often outstretched) and may be associated with acute pain, swelling, and deformity near the
elbow (Item C52C).

References:

Arndt CAS. Neoplasms of bone. In: Kliegman RM, Behrman RE, Jenson HB, Stanton BF, eds. Nelson's
Textbook of Pediatrics. 18th ed. Philadelphia, Pa: Saunders Elsevier; 2007:2146-2150

Cripe TP. Osteosarcoma. eMedicine Specialties, Pediatrics, Oncology. 2006. Available at:
http://www.emedicine.com/ped/topic1684.htm

Gorlick R, Anderson P, Andrulis I, et al. Biology of childhood osteogenic sarcoma and potential targets
for therapeutic development. Clin Cancer Res. 2003;9:5442-5453. Available at:
http://clincancerres.aacrjournals.org/cgi/content/full/9/15/5442

Gurney JG, Swensen AR, Bulterys M. Malignant bone tumors. In: Ries LAG, Smith MA, Gurney JG, et
al, eds. Cancer Incidence and Survival Among Children and Adolescents: United States SEER Program
1975-1995. Bethesda, Md: National Cancer Institute, SEER Program; 1999:88-110. Available at:
http://seer.cancer.gov/publications/childhood/bone.pdf

Kalyoussef S, Tolan RW Jr. Osteomyelitis. eMedicine Specialties, Pediatrics, Infectious Diseases.


2006. Available at: http://www.emedicine.com/PED/topic1677.htm

Seade LE, Bryan, WJ, Bartz RL, Josey R. Acromioclavicular joint injury. eMedicine Specialties, Sports
Medicine, Shoulder. 2006. Available at: http://www.emedicine.com/sports/TOPIC3.HTM

page 177
2009 PREP SA on CD-ROM

Critique: 52

Intense periosteal new bone formation extending into soft tissue is


characteristic of osteogenic sarcoma. (Courtesy of the Media Lab at
Doernbecher)

page 178
2009 PREP SA on CD-ROM

Critique: 52

Anteroposterior radiograph of a normal shoulder (top): A line drawn on the


underside of the clavicle intersects the achromion. In acromioclavicular
separation (bottom), this line does not intersect the achromion, indicating that
the distal end of the clavicle is no longer in contact with the achromion. There
is an incidental finding of separation of the VP shunt tubing in the neck
(arrow). (Courtesy of D. Mulvihill)

page 179
2009 PREP SA on CD-ROM

Critique: 52

Lateral radiograph of the elbow in supracondylar fracture of the humerus: the


posterior fat pad (darker area noted by arrow) is displaced from the
supracondylar fossa by hemorrhage into the joint. There is posterior
displacement of the capitellum (the anterior humeral line passes along the
anterior edge of the capitellum rather than through its center). (Courtesy of D.
Mulvihill)

page 180
2009 PREP SA on CD-ROM

Question: 53
A 7-month-old female has undergone the second stage of surgical palliation (Glenn operation) for
hypoplastic left heart syndrome. She was discharged from the hospital 1 week ago, and her mother
brings her to the office because of irritability that began this morning. On physical examination, the
infant is awake and irritable, with a heart rate of 150 beats/min and a respiratory rate of 50 breaths/min.
She has cyanosis of the face and mucosal surfaces and swelling of the arms and head.

Of the following, the BEST explanation for this patient’s clinical presentation is

A. polycythemia

B. postpericardiotomy syndrome

C. protein-losing enteropathy

D. superior vena cava syndrome

E. thoracic duct injury

page 181
2009 PREP SA on CD-ROM

Critique: 53 Preferred Response: D


The child described in the vignette has had surgery involving her superior vena cava, which has been
sewn by an end-to-side anastomosis to her right pulmonary artery (Glenn operation). She is at risk for
stenosis at the surgical site, thrombosis within the superior vena cava, and altered hemodynamics if the
pulmonary vascular resistance (and, thus, the pulmonary artery pressure) increases, which raises the
pressure against which the venous drainage must occur. Her symptoms and physical findings are
consistent with superior vena cava syndrome (Item C53), and she should undergo an aggressive
evaluation and rapid institution of treatment.
Obstruction of the systemic venous return may result from one of three primary causes: extrinsic
compression of either the superior or inferior vena cava, intrinsic obstruction of systemic return, or
abnormal hemodynamics with significantly elevated right atrial pressures. The systemic veins are
thin-walled vascular structures that typically drain at low pressure into the superior and inferior vena
cavae. Normally, these large veins drain into the right atrium at low pressure. Flow in any system
moves from high to low pressure, and the cardiovascular system is no exception. The right atrium in the
healthy individual typically has a pressure of less than 10 mm Hg, often less than 5 mm Hg.
Mechanisms that facilitate venous drainage to the right atrium include gravity for the vessels of the
upper body and muscular contraction of the lower extremities, which serves to "push blood up" the
valved veins of the caudal portion of the body. As long as the pressure in the right atrium
("downstream") remains lower than the pressures in the veins, forward flow ensues. Any process that
increases the pressure in the right atrium raises the pressure needed to ensure forward drainage of the
systemic veins. Similarly, any obstruction of the superior or inferior vena cava raises the pressure
"upstream" and may limit normal venous drainage.
Extrinsic compression can result from a mediastinal mass or tumor that physically compresses the
vena cava, thereby raising the pressure needed to pass blood through the narrowing and into the right
atrium. Intrinsic obstructions can result from surgical anastomotic sites, baffle stenosis, thrombosis (eg,
from an indwelling catheter), or cardiac tumors that physically obstruct blood return through the vena
cava.
When blood return from the superior vena cava into the right atrium is obstructed significantly,
patients may demonstrate signs of superficial venous distention, venous congestion, and facial and
upper body edema, as described for the infant in the vignette. As the venous pressure increases
proximal to the obstruction, the venous drainage of the brain may become engorged, leading to
discomfort, irritability, and seizure and brain injury.
Polycythemia (elevated hemoglobin concentrations) might lead to sluggish blood flow through the
small capillaries, but would not cause the findings described in this child. Postpericardiotomy syndrome,
which can occur in children who have had cardiac surgery, generally presents with fever and systemic
symptoms. Protein-losing enteropathy is a serious complication that can occur in patients who have
increased pressure in the venous drainage of the gut, including those who undergo single ventricle
palliation. Such patients typically present with diarrhea and edema of the entire body, not localized to
the upper compartments, as in this patient. Thoracic duct injury can occur in any patient undergoing
cardiac surgery and often leads to a chylothorax.

References:

Moore P. Obstructive lesions. In: Rudolph CD, Rudolph AM, eds. Rudolph's Pediatrics. 21st ed. New
York, NY: McGraw-Hill Medical Publishing Division; 2003:1800-1813

Silberbach M, Hannon D. Presentation of congenital heart disease in the neonate and young infant.
Pediatr Rev. 2007;28:123-131. Available at:

page 182
2009 PREP SA on CD-ROM

http://pedsinreview.aappublications.org/cgi/content/full/28/4/123

page 183
2009 PREP SA on CD-ROM

Critique: 53

Superior vena cava syndrome is characterized by swelling of the head and


neck and engorgement of veins on the chest wall. (Courtesy of M. Rimsza)

page 184
2009 PREP SA on CD-ROM

Question: 54
A 6-year-old boy presents with a sudden-onset loss of awareness characterized by staring, drooling,
and chewing movements for more than 15 minutes, followed by confusion, then deep sleep. On
physical examination in the emergency department, the child is afebrile and appears to be returning to
normal. Vital signs and general examination findings are normal, and there are no focal findings. Head
computed tomography scan shows a large, contrast-enhancing cerebral mass (Item Q54) without
edema or midline shift.

Of the following, the MOST likely diagnosis is

A. arteriovenous malformation

B. brain damage from a seizure

C. ependymoma

D. glioblastoma multiforme

E. herpes encephalitis

page 185
2009 PREP SA on CD-ROM

Question: 54

(Courtesy of D. Gilbert)

page 186
2009 PREP SA on CD-ROM

Critique: 54 Preferred Response: A


The most common cause of hemorrhagic stroke in children is vascular malformation, of which there are
two types: arteriovenous malformations (AVMs) and cavernous malformations. These can present in
childhood with hemorrhage that leads to headache and seizures, as described for the boy in the
vignette. Severe, "worst-ever" headache; seizure; and mental status change are indications for
emergency evaluation and imaging of the central nervous system. The computed tomography scan
obtained for this boy shows a contrast-enhancing tortuous vascular mass without surrounding edema or
midline shift (Item C54A).
Seizures are not believed to cause brain damage in children, except in rare cases. A large
hemorrhage can cause seizures, but seizures cannot cause a large hemorrhage. Ependymomas are
one of the more common brain tumors in children. They emerge from the ependymal lining of ventricles,
do not hemorrhage, and typically occur in the posterior fossa of young children (Item C54B). Therefore,
they present more indolently, with headache and cranial nerve findings. Glioblastoma multiforme, the
highest grade astrocytoma, is uncommon in young children. Although glioblastoma multiforme can
hemorrhage, it does not have a tortuous vascular appearance on imaging and has surrounding edema.
Herpes encephalitis causes hemorrhagic necrosis (Item C54C) and can present with seizures, but
usually the patient is febrile and the characteristic imaging appearance does not involve a tortuous
vascular-appearing mass.

References:

Huang J, Gailloud PH, Tamargo RJ. Vascular malformations. In: Singer HS, Kossoff EH, Hartman AL,
Crawford TO, eds. Treatment of Pediatric Neurologic Disorders. Boca Raton, Fla: Taylor & Francis;
2005:409-414

Klimo P Jr, Rao G, Brockmeyer D. Pediatric arteriovenous malformations: a 15-year experience with an
emphasis on residual and recurrent lesions. Childs Nerv Syst. 2007;23:31-37. Abstract available at:
http://www.ncbi.nlm.nih.gov/pubmed/17053936

Thai Q, Moriarty JL, Tamargo RJ. Central nervous system vascular malformations in pediatric patients.
In: Maria BL, ed. Current Management in Child Neurology. 3rd ed. Hamilton, Ontario, Canada: BC
Decker Inc; 2005:595-605

page 187
2009 PREP SA on CD-ROM

Critique: 54

Computed tomography scan with contrast demonstrates a tortuous vascular


mass without midline shift or edema. (Courtesy of D. Gilbert)

page 188
2009 PREP SA on CD-ROM

Critique: 54

Ependymoma: Sagittal T1-weighted magnetic resonance imaging with


gadolinium contrast demonstrates a contrast-enhancing cerebellar mass
(arrow) causing brainstem compression and obstructive hydrocephalus.
(Courtesy of D. Gilbert)

page 189
2009 PREP SA on CD-ROM

Critique: 54

Axial computed tomography scan 2 weeks after the onset of symptoms of


herpes simplex virus encephalitis shows necrosis in the right temporal lobe.
(Courtesy of D. Gilbert)

page 190
2009 PREP SA on CD-ROM

Question: 55
While examining a newborn, you note a persistent curve in the spine regardless of the baby’s position.
You order spine radiographs, which reveal multiple vertebral malformations and segmentation defects
(Item Q55).

Of the following, the MOST appropriate studies to guide further management are

A. chromosome analysis and renal ultrasonography

B. echocardiography and chromosome analysis

C. echocardiography and renal ultrasonography

D. head ultrasonography and ophthalmology consultation

E. renal and head ultrasonography

page 191
2009 PREP SA on CD-ROM

Question: 55

Vertebral malformations (arrows), as described for the newborn in the


vignette. (Courtesy of T. Jewett)

page 192
2009 PREP SA on CD-ROM

Critique: 55 Preferred Response: C


Congenital scoliosis, as described for the infant in the vignette, is caused by errors in vertebral
development during embryogenesis, resulting in abnormal segmentation or formation (Item C55). It is
associated with other malformations in more than 50% of affected individuals, most commonly involving
the heart, kidneys, and spinal cord. Most cases of congenital scoliosis are not hereditary, although
numerous syndromes include spinal malformations.
The child who has congenital scoliosis should receive serial posteroanterior and lateral spine
radiographs beginning, ideally, at birth. After 1 year of age, spine radiographs should be taken with the
child standing and the pelvis maintained in a level position. Computed tomography scan with thin slices
and three-dimensional reconstruction may be helpful, especially if malformations are multiple or
complex. Magnetic resonance imaging (MRI) with cartilage sequences is recommended to determine
growth potential. Significant progression of scoliosis usually warrants surgery. Bracing typically is not
helpful as a primary treatment modality but may be recommended after surgery.
Every child who has congenital scoliosis and vertebral anomalies should be evaluated clinically for
associated malformations. Up to 20% of affected individuals have abnormalities of the urinary tract, and
up to 25% have cardiac anomalies. Therefore, children should receive screening renal ultrasonography
and echocardiography. Up to 40% of affected children may have abnormalities of the spinal cord such
as syrinx and tethered cord, warranting spinal MRI. Chromosome analysis, ophthalmology evaluation,
and head ultrasonography typically are not helpful in directing management but must be considered on
a case-by-case basis.

References:

Arlet V, Odent T, Aebi M. Congenital scoliosis. Eur Spine J. 2003;12:456-463. Abstract available at:
http://www.ncbi.nlm.nih.gov/pubmed/14618384

Ferguson RL. Medical and congenital comorbidities associated with spinal deformities in the immature
spine. J Bone Joint Surg Am. 2007;89:34-41

Hedequist D, Emans J. Congenital scoliosis. J Am Acad Orthop Surg. 2004;12:266-275. Abstract


available at: http://www.ncbi.nlm.nih.gov/pubmed/15473678

page 193
2009 PREP SA on CD-ROM

Critique: 55

Congenital scoliosis: Left thoracic scoliosis is associated with multiple


vertebral anomalies (red arrows) and rib abnormalities (eg, rib fusion, yellow
arrow). (Courtesy of T. Jewett)

page 194
2009 PREP SA on CD-ROM

Question: 56
A community group asks you to speak at a forum on teenage pregnancy. The number of pregnancies
among young adolescents at the local middle school has increased this year, and several community
members want more information about adolescent pregnancy and its long-term effects.

Of the following, the MOST appropriate statement to include in your talk about pregnant and parenting
adolescents in the United States is that

A. adolescent fathers do not have increased school drop-out rates

B. adolescents who become pregnant have the same vocational opportunities as their nonpregnant
female peers

C. most adolescent pregnancies occur in 14- to 16-year-old young women

D. poverty is correlated significantly with adolescent pregnancy

E. the adolescent pregnancy rate is increasing in the United States

page 195
2009 PREP SA on CD-ROM

Critique: 56 Preferred Response: D


Although the proportion of teenagers reporting sexual activity has dropped and adolescent pregnancy
rates have declined in recent years, the United States still has the highest adolescent birth rate among
comparable developed countries. Most adolescent pregnancies occur among older adolescents (ie, 18
and 19 year olds), and young women in this age group have a lower risk of medical complications
involving the mother or child than those younger than 17 years of age. The adverse consequences of
adolescent pregnancy are myriad, including school interruption, persistent poverty, limited vocational
opportunities, separation from the father of the baby, divorce, and repeat pregnancy. Poverty is an
important factor correlated with adolescent pregnancy, with as many as 83% of adolescents who gave
birth and 61% of adolescents who had abortions coming from poor or low-income homes compared
with 38% of nonpregnant adolescent females. Adolescent fathers have been shown to have similar
outcomes to adolescent mothers, with a higher likelihood of having poor academic performance, higher
school drop-out rates, and decreased income potential compared with their peers who are not fathers.
In an attempt to decrease adolescent child-bearing rates as well as unsafe sexual practices, every
pediatrician should integrate sexuality education into clinical practice with children from early childhood
through adolescence. Pediatricians also should consider participating in development and
implementation of sexuality education curricula for schools or public efforts.

References:

American Academy of Pediatrics Committee on Psychosocial Aspects of Child and Family Health and
Committee on Adolescence. Sexuality education for children and adolescents. Pediatrics.
2001;108:498-502. Available at: http://pediatrics.aappublications.org/cgi/content/full/108/2/498

Klein JD, and the AAP Committee on Adolescence. Adolescent pregnancy: current trends and issues.
Pediatrics. 2005;116:281-286. Available at:
http://pediatrics.aappublications.org/cgi/content/full/116/1/281

page 196
2009 PREP SA on CD-ROM

Question: 57
You are treating a 14-year-old boy in the pediatric intensive care unit who suffered a traumatic brain
injury in a motor vehicle crash earlier today and underwent surgery to drain a right-sided epidural
hematoma. He is currently receiving mechanical ventilation and is sedated. The nurse calls you to the
bedside because the intraventricular catheter is clotted and no intracranial pressure waveform is seen
on the monitor. On physical examination, you note that his right pupil is dilated and unresponsive to
light, which differs from findings on your examination immediately after surgery.

Of the following, the MOST appropriate immediate next step is

A. administration of fentanyl

B. administration of mannitol

C. cerebral angiography

D. replacement of the intraventricular catheter

E. ophthalmology consultation

page 197
2009 PREP SA on CD-ROM

Critique: 57 Preferred Response: B


Increased intracranial pressure can cause herniation of the cerebral contents, which is a neurologic
emergency. Transtentorial herniation results from downward or upward displacement of the brain
through the tentorium at the level of the incisura. Temporal lobe (uncal) herniations are a subcategory
of transtentorial herniations that are characterized by dilation of a unilateral pupil due to compression of
the oculomotor nerve, as described for the boy in the vignette (Item C57). Uncal herniations usually
occur with rapid expansion of the contents of the temporal lobe fossa, such as seen with epidural
hematomas, focal injury, or infection.
Findings for the boy in the vignette suggest rebleeding of the epidural hematoma, which requires
prompt action to reduce the associated swelling and reverse the herniation. Osmotic agents such as
mannitol or hypertonic saline should be administered while calling the neurosurgeon and planning for
emergent radiologic evaluation.
Pain or anxiety may produce bilateral, not unilateral, pupillary dilation. Therefore, treatment with
fentanyl would not address this patient's neurologic emergency. Ischemic stroke can produce pupil
dilation but is less likely than uncal herniation in this scenario. Cerebral angiography to evaluate blood
vessel characteristics usually is performed after stroke is diagnosed by computed tomography scan or
magnetic radiographic imaging. Replacement of the clotted intraventricular catheter is indicated for
overall management, but acute measures to reverse the uncal herniation are of much higher priority.
Ophthalmology consultation for anisocoria is not indicated at this time because the most likely cause of
the patient's signs and symptoms is acute herniation.

References:

Avner JR. Altered states of consciousness. Pediatr Rev. 2006:27:331-338. Available at:
http://pedsinreview.aappublications.org/cgi/content/full/27/9/331

Frankel LR. Neurological emergencies and stabilization. In: Kliegman RM, Behrman RE, Jenson HB,
Stanton BF, eds. Nelson Textbook of Pediatrics. 18th ed. Philadelphia, Pa: Saunders Elsevier;
2007:405-410

page 198
2009 PREP SA on CD-ROM

Critique: 57

Uncal herniation may result in ipsilateral pupillary dilation, diminished level of


consciousness, and contralateral hemiparesis. (Courtesy of the Media Lab at
Doernbecher)

page 199
2009 PREP SA on CD-ROM

Question: 58
The mother of a 10-year-old boy, whom you have been following since he was 3 years old, complains
that he is always hungry and is gaining weight. The mother, who is overweight, reports that the boy
refuses to exercise, and she cannot control his diet. She just read an article in a magazine about weight
gain from Cushing syndrome and wonders if he could have this condition.

Of the following, the growth chart shown in Item 58 that suggests Cushing syndrome is

A. Growth chart A

B. Growth chart B

C. Growth chart C

D. Growth chart D

E. Growth chart E

page 200
2009 PREP SA on CD-ROM

Question: 58

(Courtesy of L. Levitsky)

page 201
2009 PREP SA on CD-ROM

Critique: 58 Preferred Response: C


Weight gain from exogenous obesity can be confused with Cushing syndrome, but glucocorticoid
excess, as seen in Cushing syndrome, almost always is associated with attenuation of normal growth,
as documented with Growth Chart C (Item C58A). The other growth charts are more typical for
exogenous obesity, with height either enhanced or unchanged in the presence of weight gain. Other
signs and symptoms of Cushing syndrome include hypertension, violaceous skin striae (Item C58B),
"buffalo hump" and muscle weakness because of loss of muscle mass, centripetal obesity, cushingoid
facies, easy bruisability, hirsutism, failure of pubertal progression or amenorrhea in women, loss of
libido in men, headache, depression, and dysphoria. Comparison of school photographs from past
years can be a useful exercise, although the clinical diagnosis of pituitary Cushing syndrome (Cushing
disease) can be difficult.
Documentation of several elevated 24-hour urine free cortisol measurements as well as elevated
overnight dexamethasone-suppressed serum cortisol, evening salivary cortisol, or midnight serum
cortisol values aid in diagnosis. However, children who are very obese, stressed, or depressed may
have inappropriate elevations in serum or urine cortisol values, and children who have mild Cushing
disease may have normal values on one or more occasions.

References:

Batista DL, Riar J, Keil M, Stratakis CA. Diagnostic tests for children who are referred for the
investigation of Cushing syndrome. Pediatrics. 2007;120:e575-e586. Available at:
http://pediatrics.aappublications.org/cgi/content/full/120/3/e575

Greening JE, Storr HL, McKensie SA, et al. Linear growth and body mass index in pediatric patients
with Cushing's disease or simple obesity. J Endocrinol Invest. 2006;29:885-887. Abstract available at:
http://www.ncbi.nlm.nih.gov/pubmed/17185896

Klish WJ. Clinical evaluation of the obese child and adolescent. UpToDate Online 15.3. 2008. Available
for subscription at: http://www.uptodateonline.com/utd/content/topic.do?topicKey=pedigast/11089

Magiakou MA, Mastorakas G, Oldfield EH, et al. Cushing's syndrome in children and adolescents.
Presentation, diagnosis, and therapy. N Engl J Med. 1994;331:629-636. Abstract available at:
http://www.ncbi.nlm.nih.gov/pubmed/8052272

Nieman L, Ilias I. Evaluation and treatment of Cushing's syndrome. Am J Med. 2005;118:1340-1346.


Abstract available at: http://www.ncbi.nlm.nih.gov/pubmed/16378774

page 202
2009 PREP SA on CD-ROM

Critique: 58

Attenuation of linear growth is observed in most patients who have Cushing


syndrome. (Courtesy of L. Levitsky)

page 203
2009 PREP SA on CD-ROM

Critique: 58

Violaceous striae may be seen in patients who have Cushing syndrome.


(Courtesy of M. Rimsza)

page 204
2009 PREP SA on CD-ROM

Question: 59
The parents of an 8-year-old boy are concerned because he recently has begun to struggle in school. In
the past, he always had been an attentive and motivated student. His current teacher reports that at
times when he is speaking in class, he stops speaking abruptly, stares with glassy eyes, then resumes
speaking. At home, his parents note that he "spaces out" when eating dinner. His parents ask your
input and the best approach to treat his issues.

Of the following, the MOST appropriate initial evaluation is

A. computed tomography scan of the brain

B. educational evaluation

C. electroencephalography

D. formal audiologic evaluation

E. magnetic resonance imaging of the brain

page 205
2009 PREP SA on CD-ROM

Critique: 59 Preferred Response: C


The child described in the vignette appears to be experiencing absence seizures. These present as
multiple daily episodes of abrupt arrest of consciousness and voluntary movement that last a few
seconds (Item C59), followed by immediate return to normal consciousness. Decline in school function
is common at the onset of this disorder. Electroencephalography (EEG) confirms the diagnosis,
showing a pattern of spike and wave electrical complexes that have a frequency of 3 Hz (3/second).
Computed tomography scan and magnetic resonance imaging are not indicated in the evaluation of
children who have uncomplicated absence seizures.
An audiologic evaluation would be indicated if the child was suspected of not hearing properly.
Decisions regarding an educational evaluation should be deferred until the diagnosis of epilepsy is
clarified. Occasionally, inattention associated with attention-deficit/hyperactivity disorder (ADHD) may
mimic absence epilepsy. However, children who have ADHD do not have a sudden cessation of motor
activity, blank facial expression, or flickering of the eyelids characteristic of absence seizures.
Computed tomography scan, magnetic resonance imaging, and EEG generally are not indicated for the
diagnosis of either ADHD or a learning disability. Children who have learning disabilities are identified
by results from standardized psychoeducational testing. ADHD is diagnosed if the child meets
American Psychiatric Association Diagnostic and Statistical Manual of Mental Disorders, fourth edition,
text revision (DSM-IV-TR) criteria and has symptoms of inattention and hyperactivity/impulsivity that
have lasted for more than 6 months and were observed prior to age 7 years. The behaviors cause
"clinically significant impairment in social, academic or occupational functioning" and present in more
that one setting. Symptoms do not include the type of "staring spells" described for the boy in the
vignette.

References:

American Academy of Pediatrics. Committee on Quality Improvement, Subcommittee on


Attention-Deficit/Hyperactivity Disorder. Clinical practice guideline: diagnosis and evaluation of the child
with attention-deficit/hyperactivity disorder. Pediatrics. 2000;105:1158-1170. Available at:
http://pediatrics.aappublications.org/cgi/content/full/105/5/1158

American Psychiatric Association. Diagnostic criteria for ADHD. In: Diagnostic and Statistical Manual of
Mental Disorders. 4th ed. Text revision. Arlington, Va: American Psychiatric Association; 2000:85-94

American Psychiatric Association. Diagnostic criteria for learning disability. In: Diagnostic and Statistical
Manual of Mental Disorders. 4th ed. Text revision. Arlington, Va: American Psychiatric Association;
2000:49-56

Posner E. Absence seizures in children. BMJ Clinical Evidence. 2007. Available for subscription at:
http://clinicalevidence.bmj.com/ceweb/conditions/chd/0317/0317_background.jsp

Pritchard D. Attention deficit hyperactivity disorder in children. BMJ Clinical Evidence. 2006. Available
for subscription at: http://clinicalevidence.bmj.com/ceweb/conditions/chd/0312/0312_background.jsp

page 206
2009 PREP SA on CD-ROM

Question: 60
You are seeing a young girl for a health supervision visit. Her older brother recently underwent a bone
marrow transplant, and you inquire about his health. The mother is tearful as she tells you it has been
difficult, explaining that he has had fever for about 10 days, his "counts are still down," and they are
planning to start amphotericin B just in case he has a fungal infection. She is concerned because she
was told about potential adverse effects of the medication and how they need to watch the "electrolytes
in his blood" very closely. She doesn’t know what "electrolytes" are, but asks what parameter in his
blood might be affected.

Of the following, the MOST clinically important parameter to monitor during the initiation of amphotericin
B therapy is

A. bicarbonate

B. creatinine

C. glucose

D. potassium

E. sodium

page 207
2009 PREP SA on CD-ROM

Critique: 60 Preferred Response: D


The major metabolic abnormality that occurs early in therapy with amphotericin B administration is
hypokalemia, which can be difficult to correct, especially if there is concurrent hypomagnesemia.
Amphotericin B augments potassium and magnesium excretion by the kidney. Therefore, potassium
concentrations cannot be corrected in most patients until the hypomagnesemia also is corrected. The
hypokalemia can occur whether amphotericin B deoxycholate or the lipid formulations of amphotericin B
are used. Problems with sodium, glucose, and bicarbonate usually do not occur with amphotericin B
therapy. Elevation of the blood urea nitrogen or creatinine values associated with amphotericin B
therapy occurs slowly and is due to the cumulative renal effects of the medication. Once the medication
is stopped, renal function returns to normal in most patients.
Other common adverse reactions associated with amphotericin B therapy include fever, chills,
rigors, malaise, nausea, vomiting, and headaches. Younger children and infants generally do not have
these reactions. Older children and adolescents may require premedication with ibuprofen or
acetaminophen to lessen such reactions. In addition, lengthening of infusion times to between 4 and 6
hours can blunt such symptoms. In severe cases, patients may require antihistamines, antiemetics,
meperidine, or the addition of hydrocortisone to the infusion before administration of amphotericin B.

References:

American Academy of Pediatrics. Antifungal drugs for systemic fungal infections. In: Pickering LK,
Baker CJ, Long SS, McMillan JA, eds. Red Book: 2006 Report of the Committee on Infectious
Diseases. 27th ed. Elk Grove Village, Ill: American Academy of Pediatrics; 2006:774-776

Zaoutis TE, Benjamin DK, Steinbach WJ. Antifungal treatment in pediatric patients. Drug Resist Update.
2005;8:235-245. Abstract available at: http://www.ncbi.nlm.nih.gov/pubmed/16054422

page 208
2009 PREP SA on CD-ROM

Question: 61
You are evaluating an 8-year-old boy who has acute lymphoblastic leukemia and is in septic shock
caused by Klebsiella pneumoniae. The antibiotic susceptibilities for the organism reveal that it is
resistant to ampicillin, cefazolin, ceftriaxone, and gentamicin.

Of the following, the MOST appropriate antibiotic to use in this patient is

A. cefuroxime

B. clindamycin

C. meropenem

D. penicillin G

E. piperacillin

page 209
2009 PREP SA on CD-ROM

Critique: 61 Preferred Response: C


Imipenem and meropenem are carbapenems, a type of beta-lactam antibiotic that has a very broad
antibacterial spectrum of activity. These agents are beta-lactamase-stable and bind with high affinity to
the penicillin-binding proteins of gram-positive and gram-negative bacteria. Imipenem is slightly more
active against gram-positive bacteria than meropenem; meropenem is slightly more active against
gram-negative aerobic organisms. Both have excellent activity against aerobic hemolytic streptococci,
Streptococcus pneumoniae (including resistant strains), methicillin-susceptible S aureus and S
epidermidis, Listeria sp, and Bacillus sp. Penicillin-susceptible strains of Enterococcus faecalis are
susceptible to imipenem (bacteriostatic), but resistant to meropenem. As a class, carbapenems also are
highly active against Neisseria gonorrhoeae and N meningitidis, Haemophilus influenzae and other
Haemophilus sp, Morganella sp, Proteus sp, most Enterobacteriaceae, Citrobacter sp, Enterobacter sp,
Providencia sp, Pseudomonas sp, and most anaerobic species, including anaerobic gram-positive
cocci. They also are active against Nocardia sp and Actinomyces sp.
Carbapenems are useful for the treatment of a wide variety of infections. Imipenem and meropenem
are most appropriate for treatment of infections caused by cephalosporin-resistant Enterobacteriaceae,
especially Serratia marcescens, Providencia sp, C freundii, and Enterobacter sp; as empiric therapy for
serious infections in patients previously treated with multiple antibiotics; as single agents in the
treatment of febrile neutropenia; and as treatment for polymicrobial and nosocomial infections. Thus,
meropenem is appropriate therapy for the boy described in the vignette, who is infected with a multiple
drug-resistant strain of Klebsiella pneumoniae (a gram-negative organism) that is resistant to the
cephalosporins (including the third-generation cephalosporins).
An infrequent adverse effect of imipenem (<1%) is seizures. This effect has been reported in
patients receiving very high doses who have underlying central nervous system pathology and in
patients who have decreased renal function in whom the dose has not been adjusted. Seizures are not
a major problem with meropenem, even in patients being treated for meningitis.
Cefuroxime is a second-generation cephalosporin to which the organism infecting the patient in the
vignette is resistant. Clindamycin only has activity against gram-positive organisms. Because the K
pneumoniae strain is resistant to ampicillin, it also is resistant to penicillin G. The antibiotic resistance
pattern of the identified pathogen indicates that one of its major resistance mechanisms is the
production of extended-spectrum beta-lactamases, which are enzymes that hydrolyze many different
penicillins and cephalosporins. Because of the production of these enzymes, piperacillin is not an
effective treatment option.

References:

Balfour JA, Bryson HM, Brogden RN. Imipenem/cilastatin: an update of its antibacterial activity,
pharmacokinetics and therapeutic efficacy in the treatment of serious infections. Drugs.
1996;51:99-136. Abstract available at: http://www.ncbi.nlm.nih.gov/pubmed/8741235

Nicolau DP. Carbapenems: a potent class of antibiotics. Expert Opin Pharmacother. 2008;9:23-37.
Abstract available at: http://www.ncbi.nlm.nih.gov/pubmed/18076336

Norrby SR. Carbapenems. Med Clin North Am. 1995;79:745-759. Abstract available at:
http://www.ncbi.nlm.nih.gov/pubmed/7791421

Wiseman LR, Wagstaff AJ, Brogden RN, Bryson HM. Meropenem: a review of its antibacterial activity,
pharmacokinetic properties and clinical efficacy. Drugs. 1995;50:73-101. Abstract available at:
http://www.ncbi.nlm.nih.gov/pubmed/7588092

page 210
2009 PREP SA on CD-ROM

Zhanel GG, Wiebe R, Diley L, et al. Comparative review of the carbapenems. Drugs.
2007;67:1027-1052. Abstract available at: http://www.ncbi.nlm.nih.gov/pubmed/17488146

page 211
2009 PREP SA on CD-ROM

Question: 62
A 10-year-old boy has marked fluid intake, frequent urination, and decreased visual acuity. On physical
examination, the boy is short (<5th percentile), neurologic evaluation findings are normal, and no
edema is present. His electrolyte values are normal. Other laboratory results include:
· Blood urea nitrogen, 36.0 mg/dL (12.9 mmol/L)
· Creatinine, 2.0 mg/dL (176.8 mcmol/L)
· Hemoglobin, 6.5 g/dL (65.0 g/L)
· Urine specific gravity, 1.005
· Urine pH, 6
· Urine protein, 1+

Ophthalmologic evaluation reveals features of retinitis pigmentosa.

Of the following, the MOST likely diagnosis is

A. Alport syndrome

B. diabetic nephropathy

C. juvenile nephronophthisis

D. Lowe syndrome

E. nephropathic cystinosis

page 212
2009 PREP SA on CD-ROM

Critique: 62 Preferred Response: C


The symptoms of polyuria and polydipsia; clinical finding of short stature; and laboratory findings of
azotemia, anemia, and proteinuria described for the patient in the vignette point to the diagnosis of
juvenile nephronophthisis (NPH). This autosomal recessive inherited disorder is characterized by
chronic tubulointerstitial disease. NPH begins with tubular dysfunction, which progresses to azotemia
and ultimately, end-stage renal disease (ESRD). The clinical presentation includes symptoms
consistent with a urinary concentrating defect (polyuria and polydipsia) at 2 to 3 years of age that
progresses to more overt symptoms in the next few years. Tubular wasting of sodium can lead to
hyponatremia and salt craving. Signs of acidosis, anemia, and azotemia appear next. The median age
of onset of ESRD is 13 years. NPH is estimated to account for 5% to 10% of all cases of ESRD in
children. The diagnosis is based primarily on clinical symptoms; renal ultrasonography usually is not
helpful because the finding of mild increased echogenicity of the kidneys is nonspecific. Medullary
cysts, which characterize the disease, may be missed on ultrasonography but usually are seen on
computed tomography scan. Renal biopsy, which is not always performed, demonstrates tubular
damage, with interstitial fibrosis and normal glomeruli. Cysts may be seen in advanced disease.
NPH may be associated with other disorders, such as the Senior-Loken syndrome, which is seen in
15% of cases. This syndrome involves the eye, with tapetoretinal degeneration, retinitis pigmentosa
(Item C62A), and the development of blindness. Other findings include nystagmus, coloboma (Item
C62B), and cataracts (Item C62C). NPH also is associated with Joubert syndrome (NPH with aplasia of
the cerebellar vermis causing ataxia and retinal coloboma/retinitis pigmentosa), hepatic fibrosis, and
skeletal defects (cone-shaped epiphyses).
Four genes have been identified in association with NPH. Most cases of juvenile NPH are related to
the NPHP1 gene, which is located on chromosome 2q12.3 and encodes for nephrocystin 1. NPHP2
and NPHP3 encode for proteins for the infantile and adolescent forms of NPH, respectively.
Alport syndrome is an X-linked disorder associated with a defect of the alpha 5 chain of type IV
collagen (Col4A5). It manifests with hematuria, high-frequency sensorineural hearing loss, and anterior
lenticonus (and occasionally cataracts). Diabetic nephropathy can result in retinal and renal disease,
but the patient in the vignette lacks a history of diabetes and has no glycosuria. Lowe syndrome is an
X-linked disorder characterized by Fanconi syndrome (glycosuria, metabolic acidosis, aminoaciduria,
and hypophosphatemia), mental retardation, and congenital cataracts, but it does not result in
azotemia, anemia, or retinal abnormalities. Nephropathic cystinosis is a lysosomal storage disorder that
has an autosomal recessive inheritance pattern and involves the Fanconi syndrome, failure to thrive,
and cystine crystal deposition within the cornea (Item C62D), resulting in severe photophobia.

References:

Hildebrandt F. Nephronophthisis-medullary cystic kidney disease. In: Avner ED, Harmon WE, Niaudet
P, eds. Pediatric Nephrology. 5th ed. Philadelphia, Pa: Lippincott Williams & Wilkins; 2004:665-673

Niaudet P. Inherited nephropathies. In: Kher KK, Schnaper HW, Makker SP, eds. Clinical Pediatric
Nephrology. 2nd ed. London, England: Informa Healthcare; 2007:195-212.

Saunier S, Calado J, Benessy F, et al. Characterization of the NPHP1 locus: mutational mechanism
involved in deletions in familial juvenile nephronophthisis. Am J Hum Genet. 2000;66:778-789.
Available at: http://www.pubmedcentral.nih.gov/articlerender.fcgi?tool=pubmed&pubmedid=10712196

page 213
2009 PREP SA on CD-ROM

Critique: 62

Retinitis pigmentosa is observed in a number of inherited disorders.


Progressive atrophy of retinal pigment epithelium results in poor night vision
and tunnel vision. Abnormalities observed on direct ophthalmoscopy include
peripheral "bone spicule" hyperpigmentation (white arrows), optic atrophy,
and attenuation of retinal vessels (black arrow).

page 214
2009 PREP SA on CD-ROM

Critique: 62

Keyhole-shaped pupil representing an iris coloboma. (Courtesy of M. Rimsza)

page 215
2009 PREP SA on CD-ROM

Critique: 62

A cataract may result in leukocoria. (Courtesy of M. Rimsza)

page 216
2009 PREP SA on CD-ROM

Critique: 62

Slitlamp examination of the cornea in a boy who has nephropathic cystinosis


reveals fine crystals packing the full thickness of the cornea. This produces
the speckled appearance of the cornea. (Courtesy of National Eye Institute,
National Institute of Health)

page 217
2009 PREP SA on CD-ROM

Question: 63
A mother brings in her 11-month-old son after he broke out in "hives" today during breakfast. The infant
had stayed home from child care with a low-grade fever, and the mother had let him eat eggs for the
first time. Immediately after breakfast, the mother noted a diffuse erythematous, pruritic rash covering
the boy’s trunk and extremities. She is concerned that her son may have an egg allergy.

Of the following, the BEST statement regarding immunoglobulin E-mediated egg food allergy is that

A. cooking the egg eliminates its allergic potential

B. egg is the most common food allergy in the first postnatal year

C. egg white is more allergenic than egg yolk

D. most children do not outgrow their egg allergy

E. the measles-mumps-rubella vaccine is contraindicated in children who have egg allergy

page 218
2009 PREP SA on CD-ROM

Critique: 63 Preferred Response: C


Immunoglobulin (Ig) E-mediated egg allergy is one of the more common childhood food allergies,
affecting approximately 1% to 2% of children. As described in the vignette, cutaneous features are
common, including atopic dermatitis, urticaria, and pruritus. Once the diagnosis of egg allergy is
determined, patients generally are advised to avoid all egg food products with the hope that most
children will outgrow their egg allergy within 3 to 5 years (Item C63).
The primary allergenic egg protein is ovomucoid, a protein predominantly in the egg white.
Approximately 50% of children may be able to tolerate small amounts of egg protein that has been
heated extensively (eg, baked goods). Prolonged heating at high temperatures can denature proteins
from a conformational form to a linear form. Some children who are allergic to eggs do not recognize
the linear protein form as an allergen and, therefore, do not experience a reaction. Of note, the brief
cooking used to make scrambled eggs will not denature heat-stable proteins.
The relationship between egg allergy and vaccination is a common question. The
measles-mumps-rubella vaccine is safe for children who have egg allergy and should be administered
without special precautions. The trivalent influenza and live attenuated influenza vaccines contain small
amounts of egg protein and are contraindicated for patients who have egg allergy. However, studies
have supported a two-dose protocol for the administration of the influenza vaccine in egg-allergic
patients. The two-dose protocol involves administering one tenth of the vaccine, observing the recipient
for a period of time, and administering the rest of the vaccine, followed by a similar observation period.
In westernized countries, milk generally is regarded as the most common food allergen in infants,
with an incidence of 2.5%, compared with an incidence of 1.5% for egg allergy.

References:

Boyano-Martínez, T, García-Ara C, Díaz-Pena JM, and Martín-Esteban M. Prediction of tolerance on


the basis of quantification of egg white-specific IgE antibodies in children with egg allergy. J Allergy Clin
Immunol. 2002;110:304-309. Abstract available at: http://www.ncbi.nlm.nih.gov/pubmed/12170273

Piquer-Gibert M, Plaza-Martín A, Martorell-Aragonés A, et al. Recommendations for administering the


triple viral vaccine and antiinfluenza vaccine in patients with egg allergy. Allergol Immunopathol (Madr).
2007;35:209-212. Available at:
http://db.doyma.es/cgi-bin/wdbcgi.exe/doyma/mrevista.pubmed_full?inctrl=05ZI0102&rev=105&vol=35&
num=5&pag=209

Sampson HA, Leung DYM. Adverse reactions to foods. In: Kleigman RM, Behrman RE, Jenson HB,
Stanton BF, eds. Nelson Textbook of Pediatrics. 18th ed. Philadelphia, Pa: Saunders Elsevier;
2007:986-989

page 219
2009 PREP SA on CD-ROM

Critique: 63

page 220
2009 PREP SA on CD-ROM

Question: 64
A 12-year-old boy is brought to the emergency department by emergency medical services after
sustaining a lower leg injury sliding into home plate during a baseball game. He tells you that he thinks
his leg twisted when he slid. He reports that he had immediate pain in his right ankle and has been
unable to walk since the injury occurred. Prior to transport, the paramedics splinted his right lower leg.
On physical examination, he has significant swelling and ecchymosis around his distal tibia and fibula.
Following the administration of analgesia, radiographs are obtained (Item Q64).

Of the following, the MOST likely complication of this injury is

A. avascular necrosis of the distal tibia

B. osteochondritis desiccans

C. osteomyelitis

D. tibial growth arrest

E. unicameral bone cyst

page 221
2009 PREP SA on CD-ROM

Question: 64

Radiograph findings as exhibited by the boy in the vignette. (Courtesy of D.


Mulvihill)

page 222
2009 PREP SA on CD-ROM

Critique: 64 Preferred Response: D


Pediatric fracture patterns differ from those in adults for a variety of biomechanical, physiologic, and
anatomic reasons. Most importantly, children are at risk of orthopedic injuries involving the growth
plates (physes) that may lead to adverse sequelae if not diagnosed acutely and managed appropriately.
Physeal injuries occur most commonly in school-age children and account for 15% to 30% of all
pediatric fractures.
The physis, or growth plate, of the pediatric bone is located between the metaphysis and the
epiphysis. This region of the pediatric skeleton is composed predominantly of cartilage cells that
multiply rapidly, calcify along the metaphyses, and lead to longitudinal bone growth. Although the
growth activity at the physes is responsible for rapid fracture healing, damage to this area can impair
healing and future growth. This region of the bone is particularly vulnerable to injury because the
weaker physeal cartilage is less resistant to traumatic forces than the mineralized bone and ligaments.
Epiphyseal fractures typically are categorized using the five-tier Salter-Harris classification, a scale
based on the degree of physeal, bone, and joint involvement (Item C64A and Item C64B). This
categorization largely determines fracture management and prognosis. The radiograph of the boy
described in the vignette indicates that he has suffered a type IV tibial fracture (Item C64C). His fracture
is unique in that it occurs through three anatomic planes: coronally through the distal tibial metaphysis,
transversely through the physis, and sagittally through the epiphysis (ie, a triplane fracture). These
fractures occur because the physis has not fused completely. An associated fibular fracture is a
common finding.
Osteochondritis desiccans most commonly affects the medial femoral condyle of the knee. Caused
by repetitive stress, it is characterized by bone fragment development and separation at the
posterolateral aspect of the distal femur (Item C64D).
Unicameral, or simple, bone cysts usually are found in the metaphyses of long bones of children
between 4 and 12 years (Item C64E). They may lead to pathologic fractures but most often are
asymptomatic and stop expanding after skeletal maturity is reached.
Unless the fracture is open, osteomyelitis is not a likely complication. Distal tibial avascular necrosis
has been reported as an extremely rare complication of severe fractures in adults whose physes
already are closed.

References:

Dinolfo EA, Adam HM. In brief: fractures. Pediatr Rev. 2004;25:218-219. Available at:
http://pedsinreview.aappublications.org/cgi/content/full/25/6/218

Gholve PA, Hosalkar HS, Wells L. Common fractures. In: Kleigman RM, Behrman RE, Jenson HB,
Stanton BF, eds. Nelson Textbook of Pediatrics. 18th ed. Philadelphia, Pa: Saunders Elsevier;
2007:2834-2841

Perron AD, Miller MD, Brady WJ. Orthopedic pitfalls in the ED: pediatric growth plate injuries. Am J
Emerg Med. 2002;20:50-54. Abstract available at: http://www.ncbi.nlm.nih.gov/pubmed/11781914

page 223
2009 PREP SA on CD-ROM

Critique: 64

Salter-Harris classification system for fractures of the growth plate. See Item
C64B for description of fractures, treatment, and prognosis. M= metaphysis,
E=epiphysis (Reprinted with permission from Metzl JD, Sports Medicine in the
Pediatric Office. Elk Grove Village, Ill: American Academy of Pediatrics; 2008.

page 224
2009 PREP SA on CD-ROM

Critique: 64

page 225
2009 PREP SA on CD-ROM

Critique: 64

Oblique view of the ankle reveals a Salter-Harris type IV fracture of the tibia
that passes through the metaphysis, growth plate, and epiphysis (yellow
arrows); there is also a frature of the distal fibula (red arrow). This unique
fracture occurs in adolescence before there is complete closure of the growth
plate. (Courtesy of D. Mulvihill)

page 226
2009 PREP SA on CD-ROM

Critique: 64

Anteroposterior radiograph of the knee in osteochondritis dessicans


(dissecans) shows a well-demarcated fragment of subchondral bone
surrounded by a lucent line (arrows) that separates it from the lateral condyle
of the femur. (Courtesy of D. Rosenbaum)

page 227
2009 PREP SA on CD-ROM

Critique: 64

Unicameral bone cyst: Anteroposterior radiograph of the knee shows a


metaphyseal bony defect in the tibia. There is minimal expansion of the bony
contours and few septations. (Courtesy of D. Mulvihill)

page 228
2009 PREP SA on CD-ROM

Question: 65
A 4-month-old male infant presents for his initial examination. The family recently immigrated to the
United States from southeast Asia. They describe progressive abdominal distention (Item Q65A) in the
infant over the past 2 months. Physical examination demonstrates a firm liver edge 2 cm below the right
costal margin and a spleen tip palpable 3 cm below the costal margin. Abdominal ultrasonography
demonstrates a fluid-filled abdomen (Item Q65B).

Of the following complications from his underlying disorder, this child is MOST at risk for

A. acute intussusception

B. emphysematous chronic lung disease

C. gastrointestinal bleeding

D. pneumococcal meningitis

E. renal failure

page 229
2009 PREP SA on CD-ROM

Question: 65

Abdominal distention, as described for the infant in the vignette. (Courtesy of


M. Rimsza)

page 230
2009 PREP SA on CD-ROM

Question: 65

Abdominal ultrasonography of the abdomen demonstrates anechoic fluid


surrounding the bladder. (Courtesy of A. Bousvaros)

page 231
2009 PREP SA on CD-ROM

Critique: 65 Preferred Response: C


The firm liver, enlarged spleen, and fluid in the abdomen seen on ultrasonography described for the
infant in the vignette is consistent with a number of clinical scenarios, including infection and
malignancy. However, the findings strongly suggest chronic liver disease, with secondary portal
hypertension resulting in ascites and splenomegaly. Accordingly, he is at risk for esophageal variceal
bleeding. Acute intussusception, emphysematous chronic lung disease, pneumococcal meningitis, and
renal failure are not associated directly with portal hypertension.
Portal hypertension may occur from either hepatic venous thrombosis (Budd-Chiari syndrome),
portal venous thrombosis ("cavernous transformation of the portal vein"), or hepatic cirrhosis. Hepatic
cirrhosis is the most common cause in both adults and children. In adults, alcoholic cirrhosis is the most
common cause. In children, cirrhosis may occur from a number of congenital, hereditary, and infectious
conditions. These include biliary atresia, Alagille syndrome, alpha-1-antitrypsin deficiency, cystic
fibrosis, hepatitis C, congenital hepatic fibrosis, and congestive heart failure.
The portal vein drains the intestinal blood and lymph that subsequently enter the liver, where blood
and nutrients are filtered through the hepatic sinusoids. The cirrhotic liver has increased resistance to
the flow of portal blood, resulting in increased portal venous pressure. This increase in pressure is
transmitted to the splenic vein (resulting in splenomegaly) and the esophageal and hemorrhoidal
venous systems (resulting in esophageal varices and hemorrhoids). The changes in oncotic pressure
also may result in hypoalbuminemia and accumulation of ascitic fluid in the abdominal cavity, as seen in
this infant.
The earliest clinical sign that suggests portal hypertension usually is an enlarged spleen. As liver
disease progresses, ascites, prominent abdominal veins, and esophageal or gastric varices may
develop. Many patients who have portal hypertension exhibit laboratory signs of chronic liver disease,
including transaminase elevation and jaundice. However, patients who have some conditions
associated with portal hypertension, including congenital hepatic fibrosis and alpha-1-antitrypsin
deficiency, may not have jaundice. In such patients, portal hypertension should be suspected if the liver
or spleen is chronically enlarged.

References:

Molleston JP. Variceal bleeding in children. J Pediatr Gastroenterol Nutr. 2003;37:538-545

Shepherd RW, Ramm GA. Liver function and dysfunction: fibrogenesis and cirrhosis. In: Walker WA,
Goulet O, Kleinman RE, Sherman PM, Shneider BL, Sanderson IR. Pediatric Gastrointestinal Disease.
4th ed. Hamilton, Ontario, Canada: BC Decker; 2004:80-88

page 232
2009 PREP SA on CD-ROM

Question: 66
A 2.1-kg, 34-week gestation infant is delivered to a mother who has chorioamnionitis and had a positive
group B streptococcal urinary tract infection at 30 weeks of gestation. Four hours after birth, the infant
requires admission to the intensive care nursery because of respiratory distress. Physical examination
reveals a temperature of 96.8°F (36.0°C), heart rate of 160 beats/min, respiratory rate of 80
breaths/min, blood pressure of 60/30 mm Hg, mean arterial pressure of 40 mm Hg, and pulse oximetry
of 82% on room air. The infant audibly grunts, has flaring of the ala nasi and intercostal and subcostal
chest wall retractions, and is poorly perfused, with a capillary refill time of 4 seconds and mild
acrocyanosis. There is no heart murmur.

Of the following, the MOST likely radiographic findings expected for this infant are

A. air bronchograms, diffusely hazy lung fields, and low lung volume

B. cardiomegaly, hazy lung fields, and pulmonary vascular engorgement

C. fluid density in the horizontal fissure, hazy lung fields with central vascular prominence, and normal
lung volume

D. gas-filled loops of bowel in the left hemithorax and opacification of the right lung field

E. patchy areas of diffuse atelectasis, focal areas of air-trapping, and increased lung volumes

page 233
2009 PREP SA on CD-ROM

Critique: 66 Preferred Response: A


Infants delivered at any gestational age are at risk for respiratory distress from many causes.
Respiratory distress syndrome (RDS) is due to surfactant deficiency and characteristically affects
newborns delivered at less than 32 weeks' gestation. In the term or late preterm infant, such as the
infant described in the vignette, additional or alternative causes of respiratory distress should be
considered, including infection, aspiration, cardiac disease, or congenital anomalies (eg,
tracheoesophageal fistula, diaphragmatic hernia). Infants born to mothers who have a history of
prolonged rupture of membranes, chorioamnionitis, or known maternal group B streptococcal carriage
have a high risk for infection, including congenital pneumonia. Respiratory distress may be seen
anytime after birth but characteristically is present in the first 24 hours. Tachypnea, tachycardia, and
increased work of breathing with a supplemental oxygen requirement are the most common clinical
signs. Temperature instability, irritability, lethargy, and poor feeding also may be present. The chest
radiograph for a newborn who has congenital pneumonia may resemble that of RDS with hyaline
membrane disease (air bronchograms, diffusely hazy lung fields, and low lung volumes) (Item C66A).
Rarely, a lobar infiltrate may be seen.
The most likely infectious agents in congenital pneumonia include those that ascend from the genital
tract before or during labor: group B streptococci, Escherichia coli, and Klebsiella sp. No specific clinical
sign or laboratory finding in the sick newborn confirms the presence or absence of pneumonia.
Therefore, all newborns who exhibit respiratory distress should have blood cultures obtained and be
started on broad-spectrum antibiotic therapy (eg, ampicillin and gentamicin) until a diagnosis of
bacterial infection can be excluded. Obtaining a Gram stain of tracheal aspirates from newborns in
whom pneumonia is suspected and who have required tracheal intubation and assisted ventilation may
be helpful.
The chest radiograph depicting cardiomegaly, hazy lung fields, and pulmonary vascular
engorgement is characteristic of congenital heart disease with pulmonary overcirculation (eg, critical
aortic stenosis, total anomalous pulmonary venous return with obstruction, truncus arteriosus,
transposition of the great arteries) (Item C66B). Fluid densities in the horizontal fissure, hazy lung fields
with central vascular prominence, and normal lung volumes are characteristic of retained fetal lung
fluid, referred to clinically as transient tachypnea of the newborn (Item C66C). Gas-filled loops of bowel
in either hemithorax are suspicious for a diaphragmatic hernia, more commonly seen on the left, with a
mediastinal shift to the right and compression atelectasis and opacification of the right lung field (Item
C66D). Patchy atelectasis with focal areas of air-trapping and increased lung volumes are characteristic
of aspiration syndromes, most notably meconium (Item C66E).

References:

Aly H. Respiratory disorders in the newborn: identification and diagnosis. Pediatr Rev.
2004;25:201-208. Available at: http://pedsinreview.aappublications.org/cgi/content/full/25/6/201

Herting E, Gefeller O, Land M, van Sonderen L, Harms K, Robertson B, and Members of the
Collaborative European Multicenter Study Group. Surfactant treatment of neonates with respiratory
failure and group B streptococcal infection. Pediatrics. 2000;106:957-964. Available at:
http://pediatrics.aappublications.org/cgi/content/full/106/5/957

Sivit CJ. Diagnostic imaging. In: Martin RJ, Fanaroff AA, Walsh MC, eds. Fanaroff and Martin's
Neonatal-Perinatal Medicine. 8th ed. Philadelphia, Pa: Mosby Elsevier; 2006:713-732

Thilo EH, Rosenberg AA. The newborn infant. In: Hay WW Jr, Levin M, Sondheimer JM, Deterding RR,

page 234
2009 PREP SA on CD-ROM

eds. Current Pediatric Diagnosis & Treatment. 18th ed. New York, NY: The McGraw-Hill Companies,
Inc; 2007:chap 1

page 235
2009 PREP SA on CD-ROM

Critique: 66

The chest radiograph in neonatal pneumonia may show features identical to


those of respiratory distress syndrome, including underinflation, a "ground
glass" appearance, and air bronchograms (arrows). (Courtesy of B. Carter)

page 236
2009 PREP SA on CD-ROM

Critique: 66

Chest radiograph of an infant who has transposition of the great vessels


reveals prominent pulmonary vessels (arrows), suggesting pulmonary
overcirculation. (Reprinted with permission from Aly H. Respiratory disorders
in the newborn: identification and diagnosis. Pediatr Rev. 2004;25:201-208.)

page 237
2009 PREP SA on CD-ROM

Critique: 66

Chest radiograph of transient tachypnea of the newborn shows increased


pulmonary interstitial markings and fluid in the interlobar fissures (arrows).
(Reprinted with permission from Aly H. Respiratory disorders in the newborn:
identification and diagnosis. Pediatr Rev. 2004;25:201-208.)

page 238
2009 PREP SA on CD-ROM

Critique: 66

Plain radiograph of the chest and abdomen in a patient who has congenital
diaphragmatic hernia shows bowel in the left chest, with displacement of the
heart to the right. (Courtesy of B. Carter)

page 239
2009 PREP SA on CD-ROM

Critique: 66

Meconium aspiration is characterized by areas of atelectasis (arrows) and


hyperinflation (seen best at the bases). (Reprinted with permission from Aly
H. Respiratory disorders in the newborn: identification and diagnosis. Pediatr
Rev. 2004;25:201-208.)

page 240
2009 PREP SA on CD-ROM

Question: 67
A 16-month-old boy is brought to your clinic because his mother says he is "walking funny" today. She
states that he has been walking for 4 months and is very active, but she is unaware of any trauma or
falls. She denies fever or other symptoms. He appears well and has normal vital signs. Physical
examination reveals mild tenderness to palpation over the medial aspect of the lower leg just above the
ankle. There is no overlying bruising, erythema, or edema, and you can elicit full range of motion in the
hips, knees, and ankles.

Of the following, the MOST likely diagnosis is

A. aneurysmal bone cyst

B. ankle sprain

C. fracture

D. osteomyelitis

E. transient synovitis

page 241
2009 PREP SA on CD-ROM

Critique: 67 Preferred Response: C


The differential diagnosis of the child who has an acute limp includes infectious causes such as
osteomyelitis and septic arthritis, postinfectious entities such as transient synovitis and rheumatic fever,
and injuries related to accidental or nonaccidental trauma. The presence of fever or other signs and
symptoms, including erythema and limitation of joint motion, should lead the clinician to investigate
infectious or postinfectious causes. However, in the absence of constitutional symptoms and typical
external signs of infection, such as described for the boy in the vignette, an occult fracture should be
considered, even if no history of trauma is elicited.
Careful questioning regarding any falls or injuries, observation of the parent-child interaction, and a
complete physical examination that includes the skin are important because many children who have
fractures from inflicted trauma come to medical attention because of vague limb pain or limp. However,
the presence of a fracture in a toddler does not necessarily indicate inflicted injury (eg, child abuse)
because children who are just learning to walk and run can fall and sustain injury without the parent's
awareness of the event.
Anteroposterior and lateral radiographs of the lower extremities are indicated if a fracture is
suspected, and the type of fracture seen can provide some clues as to the mechanism of injury. One of
the most common fractures in children is a nondisplaced spiral fracture of the tibia, otherwise as known
as a "toddler's fracture" (Item C67A). The mechanism of injury is a twisting movement of the lower leg
that occurs as the toddler learns to move around his or her environment. Examination often reveals only
a limp or tenderness over the affected area. Initial radiographs can appear normal, with a periosteal
reaction appearing in follow-up films. Torus (buckle) and greenstick (Item C67B) fractures also can be
seen with relatively minor trauma because children's bones are more porous and more likely to "buckle"
or bend with trauma than adult bones. Fractures of the growth plate are common with trauma and are
particularly concerning because of the risk of growth retardation at the site of the fracture. The
Salter-Harris system is used to classify types of growth plate fractures and identify patients who are at
high risk of growth failure. Finally, some types of fractures should alert the clinician immediately to the
possibility of child abuse, including multiple fractures in different stages of healing, metaphyseal "chip"
fractures (Item C67C), femoral fracture in children younger than 1 year of age (Item C67D), and
scapular fracture.
The minimal external signs and no constitutional symptoms such as fever reported for the child in
the vignette make the diagnosis of osteomyelitis unlikely. Transient synovitis occurs after a viral illness
and generally affects the knees and hips. Some limitation of motion at the joint is expected, but external
signs such as erythema typically are absent. Aneurysmal bone cysts are metaphyseal lesions (Item
C67E) that usually occur in adolescence and cause pain with or without swelling. The ligaments and
tendons in children generally are stronger than the growing bones underneath them, in contrast to
adults, whose bones are stronger. The amount of injuring force required to cause a sprain in an adult is
more likely to cause a fracture in a child.

References:

Dinolfo EA. Fractures. Pediatr Rev. 2004;25:218-219. Available at:


http://pedsinreview.aappublications.org/cgi/content/full/25/6/218

Eiff MP, Hatch RL. Boning up on common pediatric fractures. Contemp Pediatr. 2003;20:30-59

Kellogg ND and the Committee on Child Abuse and Neglect. Evaluation of suspected child physical
abuse. Pediatrics. 2007;119:1232-1241. Available at:
http://pediatrics.aappublications.org/cgi/content/full/119/6/1232

page 242
2009 PREP SA on CD-ROM

Critique: 67

Oblique (left) and anteroposterior (right) views of the distal tibia reveal a
nondisplaced spiral (toddler's) fracture. (Courtesy of D. Mulvihill)

page 243
2009 PREP SA on CD-ROM

Critique: 67

A greenstick fracture is characterized by a fracture in one cortex (yellow


arrow) and buckling of the opposite side. This phenomenon occurs because
of the elasticity of children's bones. (Courtesy of D. Mulvihill)

page 244
2009 PREP SA on CD-ROM

Critique: 67

Metaphyseal corner (chip) fractures may be observed in children who are the
victims of nonaccidental trauma. (Courtesy of D. Krowchuk)

page 245
2009 PREP SA on CD-ROM

Critique: 67

Spiral fracture of the femur in a 6-week-old infant who had been physically
abused. (Courtesy of D. Krowchuk)

page 246
2009 PREP SA on CD-ROM

Critique: 67

Aneurysmal bone cyst: Lateral radiograph of the humerus shows a lytic,


expansile lesion with thin internal strands and a thin continuous rim of bone.
(Courtesy of D. Mulvihill)

page 247
2009 PREP SA on CD-ROM

Question: 68
An 11-year-old girl presents 2 weeks after an office visit for a presumed viral illness characterized by
fever, malaise, and flushing of the cheeks. Today, her mother notes that she no longer has a fever, but
she complains of pain in her knees and elbows. On physical examination, the left knee is slightly
swollen and warm but not erythematous. The girl reports pain on movement of both elbows, but there
are no physical findings on examination of the elbows or other joints. The remainder of the physical
examination findings are normal, except for an oral temperature of 100.6°F (38.1°C). Results of
laboratory studies include a white blood cell count of 8.9x103/mcL (8.9x109/L) with 40%
polymorphonuclear leukocytes, 45% lymphocytes, and 15% monocytes; hemoglobin of 11.0 g/dL (110.0
g/L); platelet count of 472.0x103/mcL (472.0x109/L); and erythrocyte sedimentation rate of 20 mm/hr.

Of the following, the MOST likely pathogen to cause this child’s joint complaints is

A. Borrelia burgdorferi

B. Coxsackievirus

C. group A beta-hemolytic streptococci

D. influenza A virus

E. parvovirus B19

page 248
2009 PREP SA on CD-ROM

Critique: 68 Preferred Response: E


The girl described in the vignette has developed swelling of the knee and arthralgias following a recent
febrile illness, which is typical of a postinfectious arthritis. The flushing of her cheeks suggests that her
febrile illness was due to human parvovirus B19 infection, commonly known as erythema infectiosum
(EI) (Item C68). Parvovirus B19 infection is a common cause of postinfectious arthritis. Multiple viruses
can cause postinfectious arthritis, including influenza, hepatitis B, rubella, and Epstein-Barr.
Arthralgias may occur in 10% of children who have clinical or laboratory evidence of EI. Older
children, particularly girls and young women, frequently experience involvement of the knees, although
involvement of both large and small joints has been reported. Parvovirus B19 has been detected in
synovial fluid and serum samples of such patients. Some clinical features of parvovirus B19 infection
are similar to those of autoimmune connective tissue diseases, and some children who have EI may
develop positive antinuclear antibody serum test results or rheumatoid factor-positive serology.
Lyme disease results from infection with the spirochete Borrelia burgdorferi, which is transmitted by
deer tick bites. In North America, Lyme disease is most prevalent in the northeastern, midwestern, and
southern and western coastal areas of the United States as well as in Ontario, Canada. School-age
children are affected most commonly, with boys and girls affected equally. Arthritis is the second most
frequent presentation of Lyme disease, following the cutaneous signs of erythema migrans. Arthralgias
usually develop in the early phase, but the onset of arthritis may occur months to years after the original
infection. Initially, the arthritis is episodic, but it may evolve to a recurrent and prolonged condition. Two
thirds of affected children present with monoarthritis of the knee, but oligoarticular involvement of the
large joints and, rarely, a polyarthritis of the small joints also can occur.
The diagnosis of Lyme arthritis is based on history and physical examination findings as well as
laboratory tests to document infection with B burgdorferi. The immunoglobulin G titers to B burgdorferi
can remain positive for years and, therefore, cannot be used to monitor treatment response or failure. In
contrast to adults, the prognosis for Lyme arthritis in children generally is good, and symptoms resolve
over time without permanent damage to joints.
Group A beta-hemolytic streptococcal infection can be associated with arthritis that may occur during
the acute illness or after the acute illness has resolved (poststreptococcal reactive arthritis). Other
bacterial causes of postinfectious arthritis include Neisseria gonorrhoeae, Staphylococcus aureus, and
other streptococcal species. Coxsackieviruses are not often associated with arthritis. Enterovirus,
hepatitis B, rubella, and mumps infections may cause transient arthritis. Many other infections result in
malaise and myalgias associated with a prodrome, but they do not cause true arthritis.

References:

American Academy of Pediatrics. Enterovirus (nonpoliovirus) infections (group A and B


Coxsackieviruses, echoviruses, numbered enteroviruses. In: Pickering LK, Baker CJ, Long SS,
McMillan JA, Eds. Red Book: 2006 Report of the Committee on Infectious Diseases. 27th ed. Elk Grove
Village, Ill: American Academy of Pediatrics; 2006:284-285

American Academy of Pediatrics. Group A streptococcal infections. In: Pickering LK, Baker CJ, Long
SS, McMillan JA, Eds. Red Book: 2006 Report of the Committee on Infectious Diseases. 27th ed. Elk
Grove Village, Ill: American Academy of Pediatrics; 2006:610-620

American Academy of Pediatrics. Influenza. In: Pickering LK, Baker CJ, Long SS, McMillan JA, Eds.
Red Book: 2006 Report of the Committee on Infectious Diseases. 27th ed. Elk Grove Village, Ill:
American Academy of Pediatrics; 2006:401-411

page 249
2009 PREP SA on CD-ROM

American Academy of Pediatrics. Parvovirus B19/erythema infectiosum, fifth disease). In: Pickering LK,
Baker CJ, Long SS, McMillan JA, Eds. Red Book: 2006 Report of the Committee on Infectious
Diseases. 27th ed. Elk Grove Village, Ill: American Academy of Pediatrics; 2006:484-487

Khouqeer R, Cohen M. Viral arthritis. eMedicine Specialties, Rheumatology, Infectious Arthritis. 2006.
Available at: http://www.emedicine.com/med/topic3414.htm

Koch WC. Parvovirus B19. In: Kliegman RM, Behrman RE, Jenson HB, Stanton BF, eds. Nelson's
Textbook of Pediatrics. 18th ed. Philadelphia, Pa: Saunders Elsevier; 2007:1357-1359

Lehman HW, Knöll, A, Küster RM, Modrow S. Frequent infection with a viral pathogen, parvovirus B19,
in rheumatic diseases of childhood. Arthritis Rheum. 2003;48:1631-1638. Available at:
http://www3.interscience.wiley.com/cgi-bin/fulltext/104536478/HTMLSTART

Siegel DM. In brief: antinuclear antibody (ANA) testing. Pediatr Rev. 2003;24:320-321. Available at:
http://pedsinreview.aappublications.org/cgi/content/full/24/9/320

Tse SML, Laxer RM. Approach to acute limb pain in childhood. Pediatr Rev. 2006;27:170-180.
Available at: http://pedsinreview.aappublications.org/cgi/content/full/27/5/170

page 250
2009 PREP SA on CD-ROM

Critique: 68

A feature of erythema infectiosum is flushed cheeks. (Courtesy of D.


Krowchuk)

page 251
2009 PREP SA on CD-ROM

Question: 69
You are prescribing atenolol for a 15-year-old boy in whom you diagnosed hypertrophic
cardiomyopathy. There is a family history of asthma. He is concerned about the potential adverse
effects of medicines.

Of the following, a TRUE statement about treatment with this drug is that

A. he cannot use atenolol because of his family history of asthma

B. he has an increased risk for developing diabetes mellitus

C. he may develop gynecomastia

D. his school performance may be adversely affected

E. tachyphylaxis may develop over time

page 252
2009 PREP SA on CD-ROM

Critique: 69 Preferred Response: D


Beta-blocking drugs have an antagonistic effect on the beta-adrenergic receptors. This diverse and
widely used group of medications can have multiple and various effects on the heart and other organ
systems. In the heart, they typically exhibit some degree of negative chronotropic (slowing of the heart
rate), negative dromotropic (slowing of the conduction through the atrioventricular node), and negative
inotropic (decrease in the ventricular force of contraction) effects. As a result, this class of medications
is used in the treatment of many pediatric conditions, including hypertension, arrhythmias, hypertrophic
cardiomyopathy, dilated cardiomyopathy, Marfan syndrome, and migraine prophylaxis.
The adverse effects of beta-blocker drugs have been well-documented and may be divided into
three broad categories based on end-organ effects (Item C69).
The patient described in the vignette has been prescribed a beta blocker that has relative cardiac
selectivity. Some children and adolescents experience a decrease in school performance while being
treated with beta blockers, most likely related to the central nervous penetration, and it may be difficult
to differentiate this effect from the known fatigue or depression that also can occur.
Although bronchospasm or asthma is a contraindication to the use of some beta blockers (eg,
propranolol), a family history alone is not a contraindication for their use. Some beta blockers can cause
a decreased reaction to hypoglycemia, leading to a relative contraindication of their use in those who
have diabetes, but their use does not increase the risk of developing diabetes mellitus. Similarly, beta
blocker use typically does not lead to gynecomastia. Although reflex tachycardia may be noted when
chronic beta blocker use is discontinued, these drugs are not associated with tachyphylaxis.

References:

Feld LG, Corey H. Hypertension in childhood. Pediatr Rev. 2007;28:283-298. Available at:
http://pedsinreview.aappublications.org/cgi/content/full/28/8/283

Opie LH, Sonnenblick EH, Frishman WH, Thadani U. Beta-blocking agents. In: Opie LH, ed. Drugs for
the Heart. 4th ed. Philadelphia, Pa: W.B. Saunders Co; 1995:1-30

page 253
2009 PREP SA on CD-ROM

Critique: 69

page 254
2009 PREP SA on CD-ROM

Question: 70
A 14-year-old girl is brought to the emergency department because she has back pain and a sudden
inability to walk. Neurologic examination shows normal upper limb strength. However, her legs are
flaccid, relatively symmetrically weak, areflexic, and numb to pinprick. Vibratory and position sense in
the legs persists. A sensory deficit exists below the sixth thoracic dermatome. Rectal examination
shows low rectal tone. The remainder of her physical examination findings, including vital signs, are
normal.

Of the following, the MOST appropriate next diagnostic study is emergent

A. brain magnetic resonance imaging

B. lumbar puncture

C. nerve conduction/electromyography of the legs

D. somatosensory evoked potentials

E. spine magnetic resonance imaging

page 255
2009 PREP SA on CD-ROM

Critique: 70 Preferred Response: E


The child described in the vignette has findings consistent with an acute spinal cord lesion, which is a
neurologic emergency. In some cases, emergency neurosurgery is needed for lesions causing acute
spinal cord compression, and the initial diagnostic emphasis should be directed toward identifying such
lesions. The first step is to localize the problem to the proper level of the nervous system: brain,
brainstem/cerebellum, spinal cord, nerve, junction, or muscle. Such localization not only allows for
urgent appropriate diagnosis and treatment but can reduce discomfort, risks, and costs of inappropriate
diagnostic testing. Urgent phone or in-person consultation with a neurologist or neurosurgeon can be
useful before ordering testing.
The back pain, acute bilateral flaccid weakness, and sensory loss below the level of the lesion
described for the girl in the vignette localize this problem to the spinal cord. In this case, the rapid onset
of symptoms and the preservation of vibratory and proprioceptive sensation point to the anterior cord,
bilaterally, which is consistent with an anterior spinal artery stroke. Spinal cord strokes in children are
uncommon but can occur after aortic surgery or as a consequence of thrombotic disorders, infection,
inflammatory diseases, or trauma.
Based on the history and examination findings, magnetic resonance imaging (MRI) of the spine is
the test of choice. This should aid in determining the specific location of the lesion and whether
emergency neurosurgery to decompress the spinal cord is needed.
Brain MRI is not the initial modality of choice because of the urgency of the presentation and the
localization of the lesion suggested by clinical findings. If a spinal cord lesion is radiologically consistent
with the diagnosis of transverse myelitis, a subsequent brain MRI with gadolinium should be obtained to
look for evidence of demyelinating lesions in the brain, which is supportive of a diagnosis of multiple
sclerosis.
A subsequent lumbar puncture also may help identify the cause of the patient's symptoms, but it
may not be necessary and should not delay obtaining the spine MRI. Lumbar puncture is the
appropriate test for suspected Guillain-Barré syndrome, an acute inflammatory demyelinating
polyneuropathy, which can present with weakness beginning in the proximal legs but not with the
sensory findings seen in this child. Lumbar puncture is also helpful for identifying inflammatory causes
of spinal cord disease. Somatosensory evoked potentials are neurophysiologic tests that document the
integrity of sensory pathways from peripheral nerves through the spinal cord to the brain. They are not
part of standard emergency management in this clinical setting. Nerve conduction studies and
electromyography are not needed to assess the nerves because the problem localizes to spinal cord.

References:

Hakimi KN, Massagli TL. Anterior spinal artery syndrome in two children with genetic thrombotic
disorders. J Spinal Cord Med. 2005;28:69-73. Abstract available at:
http://www.ncbi.nlm.nih.gov/pubmed/15832907

Haslam RHA. Spinal cord disorders. In: Kliegman RM, Behrman RE, Jenson HB, Stanton BF, eds.
Nelson Textbook of Pediatrics. 18th ed. Philadelphia, Pa: Saunders Elsevier; 2007:2526-2530

Menkes JH, Ellenbogen RC. Traumatic brain and spinal cord injuries in children. In: Maria BL, ed.
Current Management in Child Neurology. 3rd ed. Hamilton, Ontario, Canada: BC Decker Inc;
2005:515-527

Nance JR, Golomb MR. Ischemic spinal cord infarction in children without vertebral fracture. Pediatr
Neurol. 2007;36:209-216. Abstract available at: http://www.ncbi.nlm.nih.gov/pubmed/17437902

page 256
2009 PREP SA on CD-ROM

Question: 71
An infant in the newborn nursery is normally grown and normally formed, except for a preauricular pit
(preauricular sinus) bilaterally (Item Q71). He has passed his newborn hearing screening. When you
meet the baby’s mother, you learn that she has progressive, bilateral sensorineural hearing loss for
which she uses hearing aids.

Of the following, the MOST helpful test to aid in diagnosis and management of this baby’s condition is

A. chromosome analysis

B. head ultrasonography

C. ophthalmology consultation

D. radiographic skeletal survey

E. renal ultrasonography

page 257
2009 PREP SA on CD-ROM

Question: 71

(Courtesy of P. Sagerman)

page 258
2009 PREP SA on CD-ROM

Critique: 71 Preferred Response: E


For many years, the association between ear anomalies and other malformations, especially involving
the kidneys, have been noted in the medical literature. Based on numerous studies, the documented
frequency of defects in other organ systems is 5% to 40% in children who have ear anomalies.
Largely due to the association between crumpled, deformed ears and renal agenesis (as seen in
Potter sequence), it has become customary in many centers to evaluate the kidneys in the newborn,
even if only a preauricular pit/sinus (PPS) is noted (Item C71). However, recent studies have shown
that PPS alone is not associated with an increased risk for renal anomalies, and renal ultrasonography
is not warranted in these cases.
It is critical to evaluate newborns and children who have auricular anomalies carefully for
dysmorphic features and evidence of organ malformations. It is important to know if the mother has
diabetes because diabetic embryopathy is associated with ear anomalies. In addition, a family history
should be obtained, with special attention to any individuals who have ear anomalies, deafness, and
renal malformations. Should such a history be elicited, as for the mother in the vignette, renal
ultrasonography is warranted, and further evaluations may be in order. Chromosome analysis, head
ultrasonography, ophthalmology consultation, and radiographic skeletal survey are not necessary initial
tests for a child who has auricular anomalies.

References:

Adam M, Hudgins L. The importance of minor anomalies in the evaluation of the newborn. NeoReviews.
2003;4:e99-e104. Available for subscription at:
http://neoreviews.aappublications.org/cgi/content/full/4/4/e99

Arora RS, Pryce R. Is ultrasonography required to rule out renal malformations in babies with isolated
preauricular tags? Arch Dis Child. 2004;89:492-493

Huang XY, Tay GS, Wansaicheong GK-L, Low WK. Preauricular sinus: clinical course and
associations. Arch Otolaryngol Head Neck Surg. 2007;133:65-68. Available at:
http://archotol.ama-assn.org/cgi/content/full/133/1/65

Wang RY, Earl DL, Ruder RO, Graham JM Jr. Syndromic ear anomalies and renal ultrasounds.
Pediatrics. 2001;108:e32. Available at: http://pediatrics.aappublications.org/cgi/content/full/108/2/e32

page 259
2009 PREP SA on CD-ROM

Critique: 71

A preauricular pit. (Courtesy of P. Sagerman)

page 260
2009 PREP SA on CD-ROM

Question: 72
A 16-year-old girl comes to your office with complaints of a thick white vaginal discharge. She is
sexually active with one partner with whom she always uses condoms. She has no complaints of fever
or abdominal pain, but she reports external "burning" of the vaginal area when she urinates. On
physical examination, she is afebrile. Pelvic examination reveals fiery red labia majora and minora and
an adherent white discharge on the vaginal walls, with a moderate amount of white discharge in the
vaginal vault. The speculum examination is uncomfortable for her, but there is no cervical motion,
uterine, or adnexal tenderness, and the cervix shows no friability or discharge.

Of the following, the MOST likely pathogen responsible for this patient’s symptoms is

A. Candida albicans

B. Chlamydia trachomatis

C. group A Streptococcus

D. Neisseria gonorrhoeae

E. Trichomonas vaginalis

page 261
2009 PREP SA on CD-ROM

Critique: 72 Preferred Response: A


Candidal vulvovaginitis is a common problem for young women and usually is caused by Candida
albicans, although other candidal species also may be involved. Typical symptoms of vulvovaginal
candidiasis include a thick, white, creamy vaginal discharge; pruritus; vaginal discomfort; dyspareunia;
and external dysuria. The diagnosis is suggested clinically by the previously noted symptoms and the
presence of vulvar swelling, erythema, and fissures or erosions, as described for the girl in the vignette.
The diagnosis may be confirmed by a wet preparation or Gram stain showing pseudohyphae or yeasts
(Item C72A) or by culture.
Chlamydia trachomatis and Neisseria gonorrhoeae, both of which may present with abnormal
vaginal discharge, produce cervical, not vaginal infections. C trachomatis and N gonorrhoeae infections
can be asymptomatic, but they often present with a yellowish purulent or mucopurulent endocervical
discharge, friability of the endocervix, and cervical motion tenderness; vulvar inflammation does not
occur. Group A Streptococcus (S pyogenes) is a respiratory pathogen that can cause vaginitis in
prepubertal girls, but rarely causes vaginal discharge in the adolescent. Infection with Trichomonas
vaginalis usually causes a diffuse, malodorous, yellow-green vaginal discharge (Item C72B) with vulvar
irritation, although some affected women can have minimal or no symptoms.

References:

American Academy of Pediatrics. Chlamydia trachomatis. In: Pickering LK, Baker CJ, Long SS,
McMillan JA, eds. Red Book: 2006 Report on the Committee on Infectious Diseases. 27th ed. Elk
Grove Village, Ill: American Academy of Pediatrics; 2006:252-257

American Academy of Pediatrics. Gonococcal infections. In: Pickering LK, Baker CJ, Long SS,
McMillan JA, eds. Red Book: 2006 Report on the Committee on Infectious Diseases. 27th ed. Elk
Grove Village, Ill: American Academy of Pediatrics; 2006:301-309

American Academy of Pediatrics. Group A streptococcal infections. In: Pickering LK, Baker CJ, Long
SS, McMillan JA, eds. Red Book: 2006 Report on the Committee on Infectious Diseases. 27th ed. Elk
Grove Village, Ill: American Academy of Pediatrics; 2006:610-620

Emans SJ. Office evaluation of the child and adolescent. In: Emans SJH, Laufer MR, Goldstein DP,
eds. Pediatric and Adolescent Gynecology. 5th ed. Philadelphia, Pa: Lippincott, Williams & Wilkins;
2005:1-50

Workowski KA, Berman SM, Centers for Disease Control and Prevention. Sexually transmitted
diseases treatment guidelines, 2006. MMWR Recomm Rep. 2006;55(RR11):1-94. Available at:
http://www.cdc.gov/mmwr/preview/mmwrhtml/rr5511a1.htm

page 262
2009 PREP SA on CD-ROM

Critique: 72

Numerous pseudohyphae (arrows) observed on a normal saline preparation of


vaginal secretions from a patient who has vaginal candidiasis. (Courtesy of
D. Krowchuk)

page 263
2009 PREP SA on CD-ROM

Critique: 72

Vaginal discharge caused by trichomoniasis often is yellow-green and frothy.

page 264
2009 PREP SA on CD-ROM

Question: 73
An 18-month-old boy fell into a swimming pool 12 hours ago. He had no heart rate when he was pulled
from the pool, and cardiopulmonary resuscitation (CPR) was initiated at the scene. The CPR was
continued for 30 minutes until spontaneous circulation was restored in the emergency department. He
is now in the pediatric intensive care unit, receiving mechanical ventilation with maximal intensive care
support. Over the past several hours, his blood pressure has increased, he has developed persistent
bradycardia, and he exhibits no movement in response to stimulation. He has not received any
neuromuscular blockers or sedation. In addition, his pupils are dilated bilaterally and do not respond to
light. Bedside electroencephalography demonstrates generalized burst suppression with loss of
reactivity to external stimuli.

In discussion with his parents, you inform them that these recent changes are MOST likely a result of

A. agitation

B. increasing intracranial pressure

C. myocardial failure

D. ongoing seizure activity

E. physiologic response to the ventilator

page 265
2009 PREP SA on CD-ROM

Critique: 73 Preferred Response: B


Hypoxic-ischemic injury, which occurs after periods of impaired perfusion, such as with
cardiopulmonary arrest, produces a broad spectrum of organ failure. Unfortunately, the brain is one of
the organs most vulnerable to decreased oxygen and nutrient delivery, and hypoxic-ischemic central
nervous system (CNS) injury is common following asphyxia. Irreversible CNS injury may occur after as
little as 3 to 5 minutes of interrupted blood flow or oxygen delivery. Both ischemia and hypoxia trigger
numerous pathophysiologic processes that result in cellular injury, cell death, and subsequent
development of cerebral edema that compromises blood flow to adjacent areas of the brain. These
areas are initially either potentially recoverable or uninjured but at risk due to impaired perfusion.
The boy described in the vignette is exhibiting evidence of increased intracranial pressure due to
significant global hypoxic-ischemic injury. His hypertension and bradycardia represent the Cushing
reflex (the raising of systemic arterial pressure to increase cerebral perfusion with associated
stimulation of the carotid bodies and subsequent bradycardia). Cushing triad (systemic arterial
hypertension, bradycardia, and depressed or irregular respirations) is a late sign of increased
intracranial pressure that often develops just prior to cerebral herniation. The lack of spontaneous
movements and dilated, unresponsive pupils described for the boy are consistent with severe CNS
injury and a poor outcome.
Agitation or pain due to underlying injuries, ventilator asynchrony, or anxiety can produce
hypertension, but this typically is associated with tachycardia and increased motor movements.
Myocardial hypoxic-ischemic injury normally is manifested by decreased ventricular function and
hypotension. Although patients who have hypoxic-ischemic injuries are at risk for the development of
seizures, the electroencephalographic findings in this patient are consistent with a severe
hypoxic-ischemic encephalopathy and portend a poor prognosis. Abnormal motor movements or vital
signs in combination with neurologic examination results that are not consistent with a known cause
should prompt the consideration of seizure and subsequent evaluation and treatment.

References:

Doherty DR, Hutchison JS. Hypoxic ischemic encephalopathy after cardiorespiratory arrest. In: Wheeler
DS, Wong HR, Shanley T, eds. Pediatric Critical Care Medicine: Basic Science and Clinical Evidence.
New York, NY: Springer-Verlag; 2007:935-946

Kallas HJ. Drowning and submersion injury. In: Kliegman RM, Behrman RE, Jenson HB, Stanton BF,
eds. Nelson Textbook of Pediatrics. 18th ed. Philadelphia, Pa: Saunders Elsevier; 2007:438-449

Meyer RJ, Theodorou AA, Berg RA. Childhood drowning. Pediatr Rev. 2006;27:163-169. Available at:
http://pedsinreview.aappublications.org/cgi/content/full/27/5/163

page 266
2009 PREP SA on CD-ROM

Question: 74
On the initial health supervision visit of a 7-year-old boy who is new to your practice, you note that his
height is 43 inches, which is at the 50th percentile for a 5-year-old, and that his weight is appropriate for
his age. His parents say that he has been wearing the same size clothes for at least the past year. The
boy also has dry skin. You suspect he has hypothyroidism and decide to measure thyroid-stimulating
hormone concentrations.

Of the following, the MOST appropriate additional study needed to evaluate this child for
hypothyroidism is

A. bone age radiography

B. measurement of free thyroxine

C. measurement of total thyroxine

D. measurement of tri-iodothyronine

E. thyroid ultrasonography

page 267
2009 PREP SA on CD-ROM

Critique: 74 Preferred Response: B


The initial evaluation of the boy described in the vignette should be directed at determining whether he
has hypothyroidism and defining its severity. The assay of free thyroxine (FT4) generally provides the
best estimate of circulating active concentrations of thyroxine. Total thyroxine (TT4) concentrations are
dependent on circulating thyroxine and on circulating binding proteins, including thyroxine-binding
globulin (TBG) and thyroxine-binding prealbumin (TBPA). In general, TT4 is an accurate measure of
thyroid hormone sufficiency, but about 1 in 8,000 individuals has familial sex-linked deficiency or excess
of TBG and, therefore, has a TT4 value that cannot be interpreted properly. Although hypothyroidism in
children usually is caused by primary thyroid gland failure, a substantial proportion of affected
individuals could have a hypothalamic or pituitary disease, in which case the FT4 value is low and the
thyroid-stimulating hormone value is normal or low.
Tri-iodothyronine (T3) is the active form of intracellular thyroid hormone. Only about 15% of T3 is
produced by the thyroid gland; the rest is produced by the enzymatic deiodination of T4 in most cells of
the body, including the hepatocyte. Because the process of enzymatic deiodination is nutritionally
controlled, individuals who are particularly well-fed or obese or small children who eat frequently have
elevated T3 values compared with adult normal values. In contrast, sick people or people who are not
eating well have very low values. Therefore, T3 cannot be used to diagnose hypothyroidism.
Children who have hypothyroidism often have delayed bone age, which may be used as an indicator
of how long the hypothyroidism has been present. Although a bone age radiograph might be part of the
extended evaluation of hypothyroidism, the diagnosis first must be established by measuring FT4.
Most children who have acquired primary hypothyroidism have chronic lymphocytic thyroiditis, with
measurable serum antibodies directed against the thyroperoxidase enzyme, the thyroid microsomes
that contain these antibodies, or thyroglobulin. In addition, they often have palpable thyroid glands. If
they do not have positive antithyroid antibodies, thyroid ultrasonography can aid in assessing the shape
and position of the thyroid gland. Some affected children could have congenitally abnormal thyroid
glands with maldescent but enough thyroid hormone production not to become hypothyroid until early
childhood. Assessment using technetium or radioactive iodine imaging also might be appropriate.

References:

Hunter I, Greene SA, MacDonald TM, Morris AD. Prevalence and aetiology of hypothyroidism in the
young. Arch Dis Child. 2000;83:207-210. Abstract available at:
http://www.ncbi.nlm.nih.gov/pubmed/10952634

LaFranchi S. Acquired hypothyroidism in childhood and adolescence. UpToDate Online 15.3. 2008.
Available for subscription at:
http://www.uptodateonline.com/utd/content/topic.do?topicKey=pediendo/4633

Ferry RJ Jr, Bauer AJ. Hypothyroidism. eMedicine Specialties, Pediatrics, Endocrinology. 2006.
Available at: http://www.emedicine.com/ped/TOPIC1141.HTM

page 268
2009 PREP SA on CD-ROM

Question: 75
An 8-year-old boy has difficulty in academics and a short attention span. His father states that he had
the same problems when he was a child. Physical examination reveals macrocephaly, multiple café au
lait macules (Item Q75A) and axillary freckles (Item Q75B). Upon questioning, the father explains that
he has similar skin findings.

Of the following, the MOST likely diagnosis is

A. fragile X syndrome

B. hypomelanosis of Ito

C. neurofibromatosis type 1

D. tuberous sclerosis

E. velocardiofacial syndrome

page 269
2009 PREP SA on CD-ROM

Question: 75

Café au lait macules, as described for the child in the vignette. (Courtesy of P.
Fisher)

page 270
2009 PREP SA on CD-ROM

Question: 75

Axillary freckling and a café au lait macule, as described for the child in the
vignette. (Courtesy of D. Krowchuk)

page 271
2009 PREP SA on CD-ROM

Critique: 75 Preferred Response: C


The frequency of learning difficulties is increased in individuals who have a positive family history for
learning disorders or genetic syndromes such as neurofibromatosis type 1 (NF-1), fragile X syndrome,
or velocardiofacial syndrome (VCFS). It is important to obtain a family history when evaluating children
who have suspected learning disorders to ascertain if there is a possible genetic cause. A thorough
family history also may identify speech or language problems in up to 30% of first-degree relatives of
children who have language delays, a risk factor for learning difficulties.
The multiple café au lait macules and axillary freckling described for the boy in the vignette meet
diagnostic criteria for NF-1. The offspring of an affected individual has a 50% risk of inheriting the
altered NF1 gene, which can be detected by molecular genetic testing, although this rarely is necessary
for diagnosis. Learning disabilities occur with increased frequency in those who have a family history of
NF-1, ranging from 40% to 60%.
Fragile X syndrome, the most common genetic cause of intellectual disabilities, occurs in 1 in 1,200
males and 1 in 2,500 females. A mutation in the FMR1 gene (Xq27.3) leads to expansion of the CGG
trinucleotide repeat. Women who have alleles in the range of 29 to 200 CGG repeats are at risk for
giving birth to affected children. Males who have fragile X syndrome have large heads, long faces, large
ears, and macro-orchidism but no skin findings. Males who have the full mutation have mean
intelligence quotient (IQ) scores of 40; affected females have mean IQ scores in the low-average to
borderline range.
Velocardiofacial syndrome affects 1 in 400 individuals, and 10% of cases are inherited in an
autosomal dominant pattern. The condition is caused by a microdeletion on chromosome 22q11.2 and
may be associated with cardiac and facial anomalies, but not cutaneous abnormalities. Learning
difficulties are reported in 82% to 100% of affected children.
Hypomelanosis of Ito is a congenital skin disorder that is characterized by linear or whorled
hypopigmentation that follows the lines of Blaschko, the paths of embryonic neural crest cell migration
(Item C75A). There is no evidence for genetic transmission. Associated intellectual disability has been
reported in as many as 70% of cases, seizures in 40%, and microcephaly in 25%. However, these high
percentages may be the result of selection bias, and the actual frequency of associated abnormalities
may be lower.
Tuberous sclerosis complex (TSC) is inherited in an autosomal dominant pattern, but as many as
two thirds of cases represent new mutations. TSC may present during infancy with infantile spasms and
a hypsarrhythmic electroencephalographic pattern. Autism is seen 1% of affected individuals. The most
common skin finding is hypopigmented macules that have been likened to an ash leaf; they are seen in
more than 90% of affected individuals (Item C75B).

References:

Haslam RHA. Neurocutaneous syndromes. In: Kliegman RM, Behrman RE, Jenson HB, Stanton BF,
eds. Nelson Textbook of Pediatrics. 18th ed. Philadelphia, Pa: Saunders Elsevier; 2007:2483-2488

Kates WR, Antshel KM, Femont W, Roizen, NJ, Shprintzen RJ. Velocardiofacial syndrome. In: Accardo
PJ, ed. Capute & Accardo's Neurodevelopmental Disabilities in Infancy and Childhood. Volume II: The
Spectrum of Neurodevelomental Disabilities. 3rd ed. Baltimore, Md: Paul H. Brookes Publishing Co;
2008:363-373

Lyon GR, Shaywitz SE, Shaywitz BA. Dyslexia. In: Kliegman RM, Behrman RE, Jenson HB, Stanton
BF, eds. Nelson Textbook of Pediatrics. 18th ed. Philadelphia, Pa: Saunders Elsevier; 2007:150-151

page 272
2009 PREP SA on CD-ROM

Morelli JG. Hyperpigmented lesions. In: Kliegman RM, Behrman RE, Jenson HB, Stanton BF, eds.
Nelson Textbook of Pediatrics. 18th ed. Philadelphia, Pa: Saunders Elsevier; 2007:2682-2685

National Fragile X Foundation Web site. Available at: http://www.fragilex.org/

Nowicki ST, Hansen RL, Hagerman RJ. X-linked intellectual disabilities In: Accardo PJ, ed. Capute &
Accardo's Neurodevelopmental Disabilities in Infancy and Childhood. Volume II: The Spectrum of
Neurodevelomental Disabilities. 3rd ed. Baltimore, Md: Paul H Brookes Publishing Co; 2008:331-351

page 273
2009 PREP SA on CD-ROM

Critique: 75

Hypomelanosis of Ito is characterized by swirled, hypopigmented patches that


follow the lines of Blaschko, the paths of embryonic neural crest cell
migration. (Courtesy of D. Krowchuk)

page 274
2009 PREP SA on CD-ROM

Critique: 75

Ash leaf macules are a feature of tuberous sclerosis. (Courtesy of D.


Krowchuk)

page 275
2009 PREP SA on CD-ROM

Question: 76
A 6-year-old girl presents with a history of swelling on her jaw of 1 month’s duration. The mother has
been to a "couple of emergency rooms," but nobody can tell her what is wrong with the girl. The child’s
father died about 3 years ago from pneumonia, and the mother reports that she has "no energy," but
she has not sought medical care. The mother states that her daughter has been fairly healthy except for
frequent ear infections. On physical examination, the girl is afebrile; her weight is 16 kg (3rd percentile);
her height is 105 cm (3rd percentile); and she has scarred tympanic membranes, bilateral parotid
swelling, mild clubbing, and some fine crackles on lung examination.

Of the following, the MOST likely diagnosis is

A. bacterial parotitis

B. common variable immunodeficiency

C. human immunodeficiency virus infection

D. lymphoma

E. mumps

page 276
2009 PREP SA on CD-ROM

Critique: 76 Preferred Response: C


The chronic parotid swelling, clubbing of the digits, history of chronic otitis media, and paternal and
maternal illness described for the girl in the vignette are most characteristic of human immunodeficiency
virus (HIV) infection. Among HIV-infected children, digital clubbing and wheezing are due to a chronic
lung disease known as lymphocytic interstitial pneumonitis.
Patients who have bacterial parotitis usually are febrile and have purulent material coming from
Stensen's duct (Item C76A). Although patients who have HIV infection may have lymphoma, the
presentation usually is not bilateral and is much less common than chronic parotitis. Sinopulmonary
infections can occur with common variable immunodeficiency (CVID), but HIV is more common and
also is more likely to affect both parents, unlike CVID.
Mumps can present with parotid swelling (Item C76B) but is an uncommon infection in developed
countries, and the duration of the swelling is not long. The parotid swelling associated with mumps
usually peaks in 3 days and resolves over the next 7 days. If mumps is being considered, the diagnosis
can be confirmed by isolation of the virus in cell culture from throat washings, saliva, urine, or
cerebrospinal fluid. In addition, a positive mumps immunoglobulin M titer, mumps polymerase chain
reaction test, or a significant increase between acute and convalescent serum mumps titers can help
establish the diagnosis.

References:

American Academy of Pediatrics. Mumps. In: Pickering LK, Baker CJ, Long SS, McMillan JA, eds. Red
Book: 2006 Report of the Committee on Infectious Diseases. 27th ed. Elk Grove Village, Ill: American
Academy of Pediatrics; 2006:464-468

Burchett SK, Pizzo PA. HIV infection in infants, children, and adolescents. Pediatr Rev.
2003;24:186-194. Available at: http://pedsinreview.aappublications.org/cgi/content/full/24/6/186

page 277
2009 PREP SA on CD-ROM

Critique: 76

Purulent material emanating from Stensen's duct in an infant who has


staphylococcal parotitis. (Courtesy of the Red Book® Online.)

page 278
2009 PREP SA on CD-ROM

Critique: 76

Mumps may present with unilateral (as shown here) or bilateral parotid
swelling. (Courtesy of M. Rimsza)

page 279
2009 PREP SA on CD-ROM

Question: 77
You are evaluating a 2-year-girl who recently was adopted from Russia for a 4-day history of
temperature to 102.5°F (39.2°C), rash, coryza, malaise, conjunctivitis, and cough that have worsened
over the last 24 hours. She had nasal congestion and rhinorrhea for 5 days prior to developing the
fever, rash, and cough. The girl has been in the United States for 7 days. She was adopted from a rural
orphanage, where she was exposed to farm animals, but information regarding her past medical history
and immunizations is unavailable. Physical examination shows a tired-appearing, irritable toddler who is
clinging to her adopted mother. She has a temperature of 103.0°F (39.5°C), bilateral conjunctival
injection, profuse clear rhinorrhea, an erythematous buccal mucosa with scattered whitish specks (Item
Q77A) on the left side, and an erythematous posterior pharynx with no tonsillar exudates. There is a
confluent erythematous maculopapular rash on her face, trunk, and abdomen (Item Q77B), with
scattered patches on her legs.

Of the following, the test MOST likely to confirm the diagnosis for this child is

A. blood culture

B. C-reactive protein measurement

C. serology

D. throat culture

E. urine culture

page 280
2009 PREP SA on CD-ROM

Question: 77

White papules on the buccal mucosa (arrow), as described for the girl in the
vignette. (Courtesy of W.W. Tunnessen, Jr)

page 281
2009 PREP SA on CD-ROM

Question: 77

(Courtesy of the Centers for Disease Control and Prevention, Public Health
Image Library, Dr. Heinz F. Eichenwald)

page 282
2009 PREP SA on CD-ROM

Critique: 77 Preferred Response: C


The patient described in the vignette has history and physical examination findings that are classic for
measles. Measles is caused by the measles (rubeola) virus, which is an RNA virus in the
Paramyxoviridae family. The simplest and preferred method for establishing the diagnosis is by
demonstrating a positive serologic test for measles immunoglobulin M (IgM) antibody on a single serum
specimen obtained during the first encounter with a person suspected of having the disease. Measles
IgM antibody is detectable for at least 1 month after the onset of the rash. The measles vaccine alone
does not give an IgM response. Other methods of diagnosis include detection of measles virus antigen
in respiratory epithelial cells or tissue by immunofluorescent methods and detection of viral genome by
polymerase chain reaction. Isolation of the measles virus from blood, throat, and urine cultures is
possible but is very difficult and is not readily available. C-reactive protein is a nonspecific
measurement of inflammation that is not specific for the diagnosis of measles.

References:

American Academy of Pediatrics. Measles. In: Pickering LK, Baker CJ, Long SS, McMillan JA, eds. Red
Book: 2006 Report of the Committee on Infectious Diseases. 27th ed. Elk Grove Village, Ill: American
Academy of Pediatrics; 2006:441-452

Centers for Disease Control and Prevention. Measles, mumps, and rubella - vaccine use and strategies
for elimination of measles, rubella, and congenital rubella syndrome and control of mumps:
recommendations of the Advisory Committee on Immunization Practices (ACIP). MMWR Recomm Rep.
1998;47(RR-8):1-57. Available at: http://www.cdc.gov/mmwr/preview/mmwrhtml/00053391.htm

Maldonado YA. Rubeola virus (measles and subacute sclerosing panencephalitis). In: Long SS,
Pickering LK, Prober CG, eds. Principles and Practice of Pediatric Infectious Diseases. 2nd ed.
Philadelphia, Pa: Churchill Livingstone; 2003:1148-1155

page 283
2009 PREP SA on CD-ROM

Question: 78
You employ voiding cystourethrography (VCUG) to evaluate a 4-year-old girl who had a febrile urinary
tract infection 1 month ago. The study reveals a smooth-walled bladder, absence of vesicoureteral
reflux, and a mildly narrowed urethra.

Of the following, the MOST appropriate management in this situation is

A. administration of prophylactic antibiotics until 6 years of age

B. no treatment

C. placement of the child on a clean intermittent catheterization program

D. repeat VCUG in 6 months

E. urologic consultation for cystoscopic evaluation

page 284
2009 PREP SA on CD-ROM

Critique: 78 Preferred Response: B


The patient described in the vignette has had a febrile urinary tract infection (UTI), but voiding
cystourethrography (VCUG) failed to demonstrate vesicoureteral reflux. It did, however, demonstrate a
narrow urethra with an otherwise normal bladder. The question that arises is whether the "narrow
urethra" is associated with potential voiding dysfunction and development of the UTI.
The recognition of a distal urethral ring in some females who had UTIs in the 1960s led to the
hypothesis that a narrowed urethra or "urethral stenosis" was responsible for UTIs. However, it is now
recognized that the previously termed "narrow urethra" in females is a normal variant. Indeed,
dilatations led to more invasive surgeries of the internal urethra to prevent "restenosis." A recent survey
of Fellows in the Section of Urology of the American Academy of Pediatrics found that 61% of
respondents never use dilatation under any circumstance, and only 2% perform this procedure
regularly. Therefore, no treatment is needed for the child in the vignette.
Children who have recurrent UTIs may have vesicoureteral reflux or dysfunctional voiding with
incomplete bladder emptying. More recently, greater emphasis has been placed on the effects of pelvic
floor muscle dysfunction in the child who has dysfunctional voiding and symptoms of enuresis, UTI, and
constipation. It is theorized that the pelvic floor muscles responsible for continence in such children are
hyperactive, resulting in abnormal bladder emptying and high postvoid residual volumes. Biofeedback is
being used to teach children how to improve relaxation of pelvic floor muscles and reduce urinary
retention and UTI risk.
There is no indication for prophylactic antibiotics for a child who has a single UTI and no
vesicoureteral reflux. Only children who have significant urinary retention, such as seen with neurogenic
bladder (myelomeningocele) or detrusor sphincter dyssynergia (Hinman syndrome), warrant intermittent
catheterization. Follow-up VCUG is not needed for the patient in the vignette. Cystoscopy is not
indicated in a child who has a smooth wall bladder and no urinary symptoms.

References:

Brock WA, Kaplan GW. Abnormalities of the lower urinary tract. In: Edelmann CM Jr, Bernstein J,
Meadow SR, Spitzer A, Travis LB, eds. Pediatric Kidney Disease. 2nd ed. Boston, Ma: Little, Brown,
and Company; 1992:2037-2076

McKenna PH, Herndon CD, Connery S, Ferrer FA. Pelvic floor muscle retraining for pediatric voiding
dysfunction using interactive computer games. J Urol. 1999;162:1056-1063. Abstract available at:
http://www.ncbi.nlm.nih.gov/pubmed/10458431

Metwalli AR, Cheng EY, Kropp BP, Pope JC 4th. The practice of urethral dilation for voiding dysfunction
among fellows of the Section on Urology of the American Academy of Pediatrics. J Urol.
2002;168:1764-1767. Abstract available at: http://www.ncbi.nlm.nih.gov/pubmed/12352355

page 285
2009 PREP SA on CD-ROM

Question: 79
An 18-year-old girl is admitted to the hospital for intravenous therapy for a complicated urinary tract
infection that failed to respond to outpatient therapy with a sulfa-based antibiotic. Her urine culture
shows more than 100,000 colony-forming units/mL of Pseudomonas aeruginosa that is sensitive to
aztreonam and imipenem. As you take her medical history, she mentions she is "highly allergic" to
penicillin.

Of the following, a TRUE statement regarding penicillin drug reactions is that

A. a nonpruritic maculopapular rash that occurs in patients who receive amoxicillin during
mononucleosis is a contraindication for future penicillin therapy

B. aztreonam can be administered safely to patients who have a history of immunoglobulin E


(IgE)-mediated penicillin allergy

C. desensitization can be used to administer penicillin safely to patients who have experienced
Stevens-Johnson reactions to penicillin

D. skin testing to major and minor determinants of penicillin can exclude IgE-mediated and
non-IgE-mediated reactions

E. a patient who can only recall a childhood history of penicillin allergy but does not remember the
details is very likely to react to future penicillin courses

page 286
2009 PREP SA on CD-ROM

Critique: 79 Preferred Response: B


Penicillin is the most common cause of drug-induced anaphylaxis. Because the likelihood of an
immunoglobulin (Ig) E-mediated penicillin allergy is less than 20% based on history alone, patients
often are referred to an allergist-immunologist for evaluation and skin testing. The negative predictive
value of penicillin skin testing for IgE-mediated reactions is 97% to 99%. However, standardized skin
testing materials such as benzylpenicilloyl polylysine or minor determinants either no longer are
available or are not approved by the United States Food and Drug Administration, respectively. In
addition, non-IgE-mediated penicillin reactions, such as Stevens-Johnson syndrome (Item C79A), result
in negative IgE skin tests, even though drug administration causes adverse effects.
Because appropriate penicillin skin testing material is not available in most allergy clinics,
administration of a non-beta-lactam antibiotic is advised for the patient who has a history of suspected
IgE-mediated penicillin allergy. If a penicillin, cephalosporin, or carbapenem drug is required,
consultation with an allergist-immunologist should be considered. A detailed discussion regarding
avoidance, graded challenge, or drug desensitization usually ensues from the consultation.
Carbapenems have a high cross-reactivity with penicillin and should be avoided, but monobactams
such as aztreonam do not cross-react and may be administered safely.
Desensitization is a process that allows the gradual administration of a drug to a patient who has an
IgE-mediated drug allergy. Thus, desensitization cannot be successful for a patient who experienced a
non-IgE-mediated reaction such as Stevens-Johnson syndrome. Also, once the desensitization is
complete and the patient finishes the appropriate antibiotic therapy, he or she still is considered
"allergic" to that drug and must undergo repeat desensitizations to receive subsequent courses of that
antibiotic.
One common non-IgE-mediated reaction to penicillin occurs when an antibiotic such as amoxicillin is
administered to a patient who has an Epstein-Barr virus infection (eg, infectious mononucleosis).
Typical symptoms include a nonpruritic, maculopapular rash that occurs a few days into therapy (Item
C79B). Clinically, it may be difficult to distinguish this reaction from a true drug allergy, but this rash is
not due to an allergic reaction, and future drug use need not be restricted.

References:

Boguniewicz M, Leung DYM. Adverse reactions to drugs. In: Kleigman RM, Behrman RE, Jenson HB,
Stanton BF, eds. Nelson Textbook of Pediatrics. 18th ed. Philadelphia, Pa: Saunders Elsevier;
2007:990-994

Wolf R, Orion E, Marcos B, Matz H. Life-threatening acute adverse cutaneous drug reactions. Clin
Dermatol. 2005;23:171-181. Abstract available at: http://www.ncbi.nlm.nih.gov/pubmed/15802211

page 287
2009 PREP SA on CD-ROM

Critique: 79

Stevens-Johnson syndrome is characterized by erythematous macules and


patches that develop bullae or erosions. Target lesions are absent or few in
number. Involvement of multiple mucosal sites (eg, mouth and conjunctiva) is
typical. (Courtesy of M. Rimsza)

page 288
2009 PREP SA on CD-ROM

Critique: 79

Morbilliform eruption that occurred in an adolescent who had infectious


mononucleosis and received amoxicillin. (Courtesy of D. Krowchuk)

page 289
2009 PREP SA on CD-ROM

Question: 80
A 5-year-old boy is brought to the emergency department because of a nose bleed that has lasted 1}
hours. His mother reports that he has had nose bleeds in the past that usually stopped when she
pinched his nose, but this time he continued to bleed. She says that he does pick his nose and that he
has had cold symptoms for the past 3 days. There is no family history of bleeding disorders, and he had
no excessive bleeding after circumcision. On physical examination, the awake, alert, and anxious
patient is holding a bloody washcloth to his nose. His heart rate is 140 beats/min, respiratory rate is 24
breaths/min, blood pressure is 100/60 mm Hg, and oxygen saturation is 98%. There is active bleeding
from his right naris, but an active anterior bleeding site is not visible. Bleeding is controlled with
phenylephrine instillation and packing.

Of the following, the MOST appropriate further evaluation is

A. chest radiograph

B. computed tomography scan of the sinuses

C. magnetic resonance arteriography of the sinuses

D. nasopharyngoscopy

E. no further evaluation

page 290
2009 PREP SA on CD-ROM

Critique: 80 Preferred Response: D


Nosebleeds are common in children, most often associated with trauma (eg, nose-picking), mucosal
friability due to upper respiratory tract infections, and mucosal drying related to environmental
conditions. Most epistaxis episodes are self-limited and require only bleeding control instructions and
comfort care. Recurrent nosebleeds are seen in fewer than 10% of patients and only rarely are related
to an underlying anatomic or hematologic abnormality. Evidence suggests that antibacterial creams
instilled in the affected naris may decrease the incidence of recurrent idiopathic epistaxis as the child
"outgrows" the predisposition.
Patients who have nosebleeds that recur frequently or are difficult to control or localize, have a
family history suggestive of a bleeding disorder, or have signs and symptoms suggestive of a blood
dyscrasia (eg, petechiae) may require further evaluation for an underlying cause. Evaluation should be
directed at likely causes, including vascular anomalies, foreign bodies, benign and malignant masses,
and primary or secondary bleeding diatheses such as thrombocytopenia, von Willebrand disease,
platelet dysfunction, hemophilias, leukemia, liver disease, or those related to medications.
The history and physical examination should guide further evaluation. The inability to localize the
bleeding to the anterior vestibule for the boy described in the vignette suggests bleeding from the
posterior nasopharynx. Ninety percent of epistaxis events result from injury to the Kiesselbach vascular
plexus in the anterior nasal septum, and the bleeding site should be visible on nasal inspection.
Bleeding from the anterior or posterior ethmoidal or sphenopalatine arteries in the posterior
nasopharynx is more difficult to see and control. Therefore, nasopharyngoscopy is indicated to identify
the site and to aid in directing treatment. If a mass or vascular anomaly is identified, computed
tomography scan or magnetic resonance angiography/magnetic resonance venography should be
undertaken. A chest radiograph is not likely to provide information that would guide evaluation or
treatment of epistaxis.
When a bleeding disorder is suspected, the initial hematologic evaluation should include complete
blood and platelet counts, with evaluation of the blood smear, prothrombin time, and partial
thromboplastin time and typing and cross-matching if transfusion is anticipated. If abnormalities are
identified, consultation with a hematologist can help to guide further evaluation, which may include a
closure test to evaluate for platelet dysfunction and von Willebrand factor or ristocetin cofactor assays
to assess for von Willebrand disease. Of note, in one study of patients who had recurrent epistaxis and
were referred to a hematologist, one third ultimately were diagnosed with bleeding disorders, and 50%
of these patients had von Willebrand disease. Family history was the only predictor that identified
patients who had primary bleeding diatheses.

References:

Haddad J Jr. Acquired disorders of the nose. In: Kleigman RM, Behrman RE, Jenson HB, Stanton BF,
eds. Nelson Textbook of Pediatrics. 18th ed. Philadelphia, Pa: Saunders Elsevier; 2007:1744-1745

McGarry G. Nosebleeds in children. BMJ Clinical Evidence. 2006. Available for subscription at:
http://clinicalevidence.bmj.com/ceweb/conditions/chd/0311/0311.jsp

Messner AH. Epidemiology and etiology of epistaxis in children. UpToDate Online 15.3. 2008. Available
for subscription at: http://www.utdol.com/utd/content/topic.do?topicKey=ped_lryn/5986

Messner AH. Evaluation of epistaxis in children. UpToDate Online 15.3. 2008. Available for subscription
at:
http://www.utdol.com/utd/content/topic.do?topicKey=ped_lryn/6248&selectedTitle=4~150&source=searc

page 291
2009 PREP SA on CD-ROM

h_result

Sandoval C, Dong S, Visintainer P, Ozkaynak MF, Jayabose S. Clinical and laboratory features of 178
children with recurrent epistaxis. J Pediatr Hematol Oncol. 2002;24:47-49. Abstract available at:
http://www.ncbi.nlm.nih.gov/pubmed/11902740

page 292
2009 PREP SA on CD-ROM

Question: 81
A 4-month-old infant who has gastroschisis underwent surgical repair on the first day after birth, but
continues to require support with parenteral nutrition and lipids. He now has developed poor feeding,
irritability, and progressive diarrhea. Radiography demonstrates metaphyseal fraying, but calcium,
phosphorus, and 25-hydroxyvitamin D concentrations are normal. When you review his prior laboratory
studies, you note he has had neutropenia for the past 4 weeks.

Of the following, this child’s symptoms are MOST consistent with

A. copper deficiency

B. magnesium deficiency

C. vitamin A deficiency

D. vitamin B6 deficiency

E. zinc deficiency

page 293
2009 PREP SA on CD-ROM

Critique: 81 Preferred Response: A


The poor feeding, irritability, metaphyseal irregularity, normal vitamin D value, and neutropenia
described for the infant in the vignette are consistent with copper deficiency, a rare disorder that
develops when children receive parenteral nutrition without trace element supplementation. Copper is a
trace element that has many important biochemical functions. It is an important component of
respiratory chain enzymes (eg, cytochrome C) and lysyl oxidase (an enzyme critical to collagen
production and bone formation). Menkes disease, a rare syndrome involving a defect in copper
transport and profoundly low serum copper concentrations, is characterized by hypotonia,
developmental delay, seizures, and "steely hair." Copper deficiency also may be seen in preterm infants
who do not receive appropriate trace element supplementation. The clinical presentation of copper
deficiency in the preterm infant is more subtle, but can include pallor, poor feeding, hypochromic
anemia, neutropenia, and skeletal changes (including metaphyseal fraying and osteoporosis). The
patient's symptoms are more consistent with copper deficiency than with magnesium, vitamin B6, zinc,
or vitamin A deficiency. Specifically, magnesium deficiency can cause hypotonia and apnea, vitamin B6
deficiency can cause hypotonia and seizures, zinc deficiency can cause diarrhea and skin rashes, and
vitamin A deficiency can lead to corneal lesions and impaired vision.
To prevent copper deficiency, most parenteral nutrition is supplemented with 200 mcg/L of copper.
Infants who have cholestasis may have impaired copper excretion into the bile, necessitating a
decrease in the concentration of copper in the parenteral nutrition to prevent copper overload.

References:

Collier S, Gura KM, Richardson D, Duggan C. Parenteral nutrition. In: Hendricks KM, Duggan C.
Manual of Pediatric Nutrition. 4th ed. Hamilton Ontario, Canada: BC Decker; 2005:317-375

Giles E, Doyle LW. Copper in extremely low-birthweight or very preterm infants. NeoReviews.
2007;8:e159-e164. Available for subscription at:
http://neoreviews.aappublications.org/cgi/content/full/8/4/e159

page 294
2009 PREP SA on CD-ROM

Question: 82
You are counseling a 23-year-old woman who has diabetes mellitus and has been your patient for the
past 18 years. She recently found out that she is pregnant and asks you about potential complications
for her unborn child.

Of the following, the MOST likely complications to expect for this woman’s child are

A. hyperacusis, hypercalcemia, hydronephrosis

B. hyperbilirubinemia, hypercalcemia, polydactyly

C. hyperglycemia, hypocalcemia, polysplenia

D. hypoglycemia, hypocalcemia, polycythemia

E. hypogonadism, hypocalcemia, polyuria

page 295
2009 PREP SA on CD-ROM

Critique: 82 Preferred Response: D


Disorders of glucose regulation such as diabetes mellitus may complicate as many as 5% of
pregnancies. This includes women who develop diabetes (insulin resistance) during pregnancy, known
as gestational diabetes; those who have pre-existing insulin resistance (type 2 diabetes); and those
who have pre-existing insulin-dependent diabetes mellitus (IDDM) (type 1 diabetes). The pregnant
woman described in the vignette has IDDM. Risks posed to her developing fetus and newborn are
numerous (Item C82).
Postnatal hypoglycemia frequently is encountered in the first 4 to 12 hours in infants of diabetic
mothers (IDMs) and is related to macrosomia, an increased metabolic rate, and fetal hyperinsulinemia
that takes a few days after birth to diminish. Postnatal hypocalcemia results from the effects of poor
late-trimester transfer of calcium across the placenta in pregnancies affected by diabetes, a delay in
normal postnatal parathyroid hormone elevation, and poor fetal and neonatal bone mineralization (ie,
poor calcium stores). If hypomagnesemia is found in the IDM, it must be corrected to allow normal
parathyroid function to resume. Polycythemia (hematocrit >65% [0.65]) represents the fetal response to
its increased metabolic rate and a relative fetal hypoxemia in utero when pregnancy is complicated by
diabetes and fetal macrosomia. Resultant hyperbilirubinemia needs to be anticipated.
Hyperacusis is not seen in IDMs, whose risk for hearing impairment is similar to that of other
newborns requiring intensive care. Hypercalcemia is seen in Williams syndrome, but not in IDMs.
Hydronephrosis, seen on prenatal ultrasonography, may be present in a number of high-risk
pregnancies, but not IDMs. Polydactyly may be seen in trisomies and some other congenital syndromes
but does not occur with greater frequency in IDMs. Polysplenia is seen in defects of left-right asymmetry
but does not have an increased incidence in IDMs. Hypogonadism is characteristic of congenital
adrenal hyperplasia, Prader-Willi syndrome, and Turner syndrome; it is not more common in IDMs.

References:

Cowett RM. The infant of the diabetic mother. NeoReviews. 2002;3:e173-e189. Available for
subscription at: http://neoreviews.aappublications.org/cgi/content/full/3/9/e173

Sivit CJ. Diagnostic imaging. In: Martin RJ, Fanaroff AA, Walsh MC, eds. Fanaroff and Martin's
Neonatal-Perinatal Medicine. 8th ed. Philadelphia, Pa: Mosby Elsevier; 2006:713-732

page 296
2009 PREP SA on CD-ROM

Critique: 82

page 297
2009 PREP SA on CD-ROM

Question: 83
A 17-year-old girl complains of an itchy rash all over her back and trunk for 2 weeks. Topical
hydrocortisone has not relieved the rash or itching. She denies fever or other symptoms, and her vital
signs are normal. Examination of the skin reveals multiple 5- to 8-mm salmon-colored thin scaling
plaques over her trunk (Item Q83). There is one similar lesion on her abdomen that measures 2x3 cm.
There are no other lesions, and the remaining findings of her physical examination are normal.

Of the following, the BEST approach to managing this girl’s rash is to

A. administer intramuscular penicillin

B. administer topical coal tar lotion

C. administer topical selenium sulfide shampoo

D. prescribe oral corticosteroids

E. reassure her that the rash will resolve in several weeks

page 298
2009 PREP SA on CD-ROM

Question: 83

(Courtesy of D. Krowchuk)

page 299
2009 PREP SA on CD-ROM

Critique: 83 Preferred Response: E


The rash described for the girl in the vignette is most consistent with the diagnosis of pityriasis rosea, a
self-limited condition believed to have a viral etiology, although this remains unproven. A nonspecific
prodrome of fever and malaise is seen occasionally. Typical features include a salmon-colored patch
surrounded by a darker rim and a fine scale, known as a "herald patch" (Item C83A). Several days after
the appearance of the herald patch, multiple similar but smaller lesions develop over the trunk and back
along lines of skin cleavage, giving the impression of a "Christmas tree" pattern (Item C83B). The palms
and soles typically are spared. Some pruritus occurs in approximately 25% of patients. The rash is
generally present for 2 to 12 weeks and resolves spontaneously. Exposure to sunlight may hasten
resolution of the lesions.
Numerous therapies have been prescribed for the treatment of pityriasis rosea, including oral
antibiotics, oral antihistamines, and oral steroids, but most have not proven to be effective. Therefore,
reassurance is all that is necessary. Topical antipruritic agents may relieve itching in some cases. Coal
tar preparations are useful in the treatment of psoriasis and other chronic dermatoses. Topical selenium
sulfide shampoo may be helpful for tinea versicolor and seborrheic dermatitis, but these conditions are
not suggested by the appearance of the girl's rash.
One of the most important diagnoses to consider in the differential diagnosis of pityriasis rosea is
secondary syphilis. The skin lesions can look similar to those of pityriasis rosea and may follow the
same lines of distribution, but the palms and soles generally are affected (Item C83C). Patients also
may complain of vague constitutional symptoms, such as headaches, fatigue, and lymph node swelling.
It is critical to obtain an accurate sexual history in any adolescent who presents with this type of rash.
Intramuscular penicillin is the drug of choice for secondary syphilis.

References:

Chuh AAT, Dofitas BL, Comisel GG, et al. Interventions for pityriasis rosea. Cochrane Database Syst
Rev. 2007;2:CD005068. Available at:
http://www.mrw.interscience.wiley.com/cochrane/clsysrev/articles/CD005068/frame.html

Morelli JG. Diseases of the epidermis. In: Kliegman RM, Behrman RE, Jenson HB, Stanton BF, eds.
Nelson Textbook of Pediatrics. 18th ed. Philadelphia, Pa: Saunders Elsevier; 2007:2702-2707

Wolfrey JD, Billica WH, Gulbranson SH, et al. Pediatric exanthems. Clin Fam Pract. 2003;5:557-588

page 300
2009 PREP SA on CD-ROM

Critique: 83

The herald patch of pityriasis rosea is a round or oval erythematous scaling


patch that may be mistaken for tinea corporis. (Courtesy of D. Krowchuk)

page 301
2009 PREP SA on CD-ROM

Critique: 83

In pityriasis rosea, the long axes of lesions are aligned parallel to lines of skin
stress. As a result, the distribution of lesions may have the appearance of the
branches of a fir tree. (Courtesy of D. Krowchuk)

page 302
2009 PREP SA on CD-ROM

Critique: 83

The eruption of secondary syphilis often involves the palms and soles.
(Courtesy of C. Haverstock)

page 303
2009 PREP SA on CD-ROM

Question: 84
A 4-year-old boy who has had mild eczema in the past that was treated successfully with emollients
presents with the worst exacerbation he ever has had. He has multiple lichenified lesions, especially in
the antecubital fossae (Item Q84) and popliteal fossa, which is usual for him, but he also has nummular
lesions on the trunk. His mother reports no changes in detergents or personal hygiene products. The
boy has been going to a summer day camp at the local community center for the first time.

Of the following, the factor MOST likely involved in his eczema exacerbation is

A. activities in the air-conditioned gymnasium

B. craft activities using water-soluble paint and glue

C. daily snacks of popcorn and fruit juice

D. outdoor soccer practice every afternoon

E. wearing a cotton t-shirt

page 304
2009 PREP SA on CD-ROM

Question: 84

Lichenified and crusted plaques, as described for the boy in the vignette.
(Courtesy of D. Krowchuk)

page 305
2009 PREP SA on CD-ROM

Critique: 84 Preferred Response: D


A number of factors may worsen atopic dermatitis by causing itching and scratching. Exposure to heat
(as may occur during warm weather months or because of overdressing during cold weather months),
sunlight, chemicals (including swimming pool chemicals such as chlorine), and sweat retention cause
exacerbations. Cold weather and low humidity may cause skin to become dry, leading to increased
pruritus. Wool or synthetic materials, fragrances, harsh soaps or detergents, and some fabric softeners
also may cause itching.
For the child described in the vignette, overheating during outdoor play is the most likely activity that
is exacerbating his eczema. Activities in an air-conditioned gymnasium would be beneficial. Although
the use of glues and craft materials may cause irritation, they would be anticipated to produce a rash on
the hands at sites of exposure. The snacks the child is receiving are unlikely to worsen atopic
dermatitis. Food allergy as an exacerbating factor is observed most often in infants, not older children.
The allergens most commonly implicated are milk, eggs, soy, wheat, and peanuts.

References:

Ashcroft DM, Chen L-C, Garside R, Stein K, Williams HC. Topical pimecrolimus for eczema. Cochrane
Database Syst Rev. 2007;4:CD005500. Available at:
http://www.mrw.interscience.wiley.com/cochrane/clsysrev/articles/CD005500/frame.html

Bath-Hextall F, Williams H. Eczema (atopic). BMJ Clinical Evidence. 2006. Available for subscription at:
http://clinicalevidence.bmj.com/ceweb/conditions/skd/1716/1716_I15.jsp

Beattie PE, Lewis-Jones MS. A comparative study of impairment of quality of life in children with skin
disease and children with other chronic childhood diseases. Br J Dermatol. 2006;155:145-151. Abstract
available at: http://www.ncbi.nlm.nih.gov/pubmed/16792766

Byremo G, Rød G, Carlsen KH. Effect of climatic change in children with atopic eczema. Allergy.
2006;61:1403-1410. Abstract available at: http://www.ncbi.nlm.nih.gov/pubmed/17073869

Ersser SJ, Latter S, Sibley A, Satherley PA, Welbourne S. Psychological and educational interventions
for atopic eczema in children. Cochrane Database Syst Rev. 2007;3:CD004054. Available at:
http://www.mrw.interscience.wiley.com/cochrane/clsysrev/articles/CD004054/frame.html

Kramer MS, Kakuma R. Maternal dietary antigen avoidance during pregnancy or lactation, or both, for
preventing or treating atopic disease in the child. Cochrane Database Syst Rev. 2006;3:CD000133.
Available at: http://www.mrw.interscience.wiley.com/cochrane/clsysrev/articles/CD000133/frame.html

Osborn DA, Sinn J. Formulas containing hydrolysed protein for prevention of allergy and food
intolerance in infants. Cochrane Database Syst Rev. 2003;4:CD003664. Available at:
http://www.mrw.interscience.wiley.com/cochrane/clsysrev/articles/CD003664/frame.html

Osborn DA, Sinn JK. Probiotics in infants for prevention of allergic disease and food hypersensitivity.
Cochrane Database Syst Rev. 2007;4:CD006475. Available at:
http://www.mrw.interscience.wiley.com/cochrane/clsysrev/articles/CD006475/frame.html

Osborn DA, Sinn J. Soy formula for prevention of allergy and food intolerance in infants. Cochrane
Database Syst Rev. 2006;4:CD003741. Available at:

page 306
2009 PREP SA on CD-ROM

http://www.mrw.interscience.wiley.com/cochrane/clsysrev/articles/CD003741/frame.html

page 307
2009 PREP SA on CD-ROM

Question: 85
You are called by the mother of 3-year-old girl because the child appears confused and is pale and
sweating. The mother thinks the child may have taken some of her grandmother’s imipramine. You
advise her to contact emergency medical services for immediate transport to the emergency
department, where you plan to meet them.

Of the following, the MOST appropriate action to take in the emergency department is

A. chest radiography to evaluate for pulmonary edema

B. continuous cardiac monitoring for dysrhythmias

C. echocardiography to assess cardiac function

D. electroencephalography to identify a seizure focus

E. measurement of the serum concentration of imipramine

page 308
2009 PREP SA on CD-ROM

Critique: 85 Preferred Response: B


The child described in the vignette has symptoms suggestive of tricyclic antidepressant (TCA)
ingestion. This class of antidepressants is used less frequently today for treatment of depression
because of the availability of selective serotonin reuptake inhibitors (SSRIs). TCAs are not the drugs of
choice for treatment of depression in children and adolescents but are used occasionally in the
treatment of other disorders (eg, enuresis, narcolepsy). SSRIs are much safer to use than TCAs
because they have fewer adverse effects and are unlikely to result in death when overdose occurs.
When TCAs are ingested in toxic amounts, they primarily affect the central nervous and
cardiovascular systems. Central nervous system signs and symptoms of TCA toxicity include irritability,
euphoria, seizures, and unresponsiveness. Autonomic nervous system symptoms such as mydriasis,
dry skin, dry mouth, urinary retention, and tachycardia also may be evident. Among the direct effects on
the cardiac system are a delay in signal conduction through the bundle of His, depression of myocardial
contractile function, and prolongation of the QRS and the QT intervals. These latter cardiac effects may
potentiate arrhythmia formation.
Continuous cardiac monitoring for the possible occurrence of arrhythmia is essential in patients who
are suspected of toxic TCA ingestion. Electrocardiography is the best tool for assessing the function of
the conduction system. The voltage intervals should be measured, with particular attention to the QRS
duration and the QT interval. A QRS duration of greater than 100 msec is associated with the
development of seizures; a QRS duration of more than 160 msec is associated with ventricular
dysrhythmias that may be particularly difficult to treat.
Echocardiography is an excellent tool to evaluate cardiac structure and function but is not
necessarily indicated as part of the emergency department evaluation of a child in whom toxic TCA
ingestion is suspected and does not obviate the need for continuous cardiac monitoring. Similarly, chest
radiography does not play a role in the acute management of such a patient. Electroencephalography
has no place in the initial evaluation and management of children in whom a TCA toxic ingestion is
suspected. Seizures, when they occur, usually do so early in the course and often resolve by the time
anticonvulsants are administered. Measuring serum drug concentrations of TCAs is not helpful for the
prognosis or management of TCA ingestion. Serum electrolyte concentrations may be measured, but
they have no predictive value in the management of TCA toxicity.

References:

Boehnert MT, Lovejoy FH Jr. Value of the QRS duration versus the serum drug level in predicting
seizures and ventricular arrhythmias after an acute overdose of tricyclic antidepressants. N Engl J Med.
1985;313:474-479. Abstract available at: http://www.ncbi.nlm.nih.gov/pubmed/4022081

Hatcher-Kay C, King CA. Depression and suicide. Pediatr Rev. 2003;24:363-371. Available at:
http://pedsinreview.aappublications.org/cgi/content/full/24/11/363

Liebelt EL, Francis PD, Woolf AD. ECG lead aVR versus QRS interval in predicting seizures and
arrhythmias in acute tricyclic antidepressant toxicity. Ann Emerg Med. 1995;26:195-201. Abstract
available at: http://www.ncbi.nlm.nih.gov/pubmed/7618783

McGuigan ME. Poisoning potpourri. Pediatr Rev. 2001;22:295-302. Available at:


http://pedsinreview.aappublications.org/cgi/content/full/22/9/295

Pérez-Fontán J, Lister G. The acutely ill infant and child. In: Rudolph CD, Rudolph AM, eds. Rudolph's
Pediatrics. 21st ed. New York, NY: McGraw-Hill Medical Publishing Division; 2003:364-365

page 309
2009 PREP SA on CD-ROM

Question: 86
A 4-year-old boy who has neuroblastoma presents with back pain and an inability to urinate. He is alert,
with normal general examination findings and normal mental status. Strength and tone in the arms are
normal, but tone is low in the legs, and patellar reflexes are diminished.

Of the following, the MOST appropriate next step for diagnosis is

A. lumbar puncture

B. magnetic resonance imaging with contrast of the spine

C. postvoid bladder residual measurement

D. radiograph of the spine

E. voiding cystourethrography

page 310
2009 PREP SA on CD-ROM

Critique: 86 Preferred Response: B


Back pain in a young child is unusual and requires prompt evaluation. When there are additional signs
that can localize to the spinal cord, such as inability to urinate, low tone, and diminished reflexes, as
described for the boy in the vignette, immediate evaluation in the emergency department is needed.
Lesions within or near the spinal cord that are causing symptoms can progress to irreversible damage
that may be avoided with emergency neurosurgery.
The child described in the vignette has a history of neuroblastoma, an aggressive tumor that is prone
to metastasis, and emerging flaccid paralysis. Therefore, magnetic resonance imaging (MRI) of the
spine with gadolinium contrast is indicated to assess the nature of the problem and determine the
location of the lesion more exactly (Item C86). Hypervascularity or vascular wall breakdown in a
metastasis may result in contrast enhancement, improving visualization of the lesion.
Lumbar puncture may be needed subsequently to evaluate for other causes or for tumor cytology,
but this procedure should be deferred until after imaging the spine and, if necessary, emergent
neurosurgical consultation. Similarly, spine imaging takes precedence over urologic dynamic studies
such as postvoid bladder residual measurement or voiding cystourethrography. A plain radiograph may
provide some information, but if MRI is available, it is preferred because it can assist in emergency
surgical planning.
Management of spinal cord compression due to a tumor includes immediate neurosurgical
consultation. Intravenous dexamethasone typically is used, along with a proton pump inhibitor or
histamine2 blocker. Aggressive pain management may be needed.

References:

Haslam RHA. Spinal cord disorders. In: Kliegman RM, Behrman RE, Jenson HB, Stanton BF, eds.
Nelson Textbook of Pediatrics. 18th ed. Philadelphia, Pa: Saunders Elsevier; 2007:2526-2530

Kim S, Chung DH. Pediatric solid malignancies: neuroblastoma and Wilms' tumor. Surg Clin North Am.
2006;86:469-487

page 311
2009 PREP SA on CD-ROM

Critique: 86

T1-weighted sagittal magnetic resonance imaging of the spine demonstrating


spinal cord compression by an extramedullary mass (arrow). (Courtesy of P.
Fisher)

page 312
2009 PREP SA on CD-ROM

Question: 87
The mother of a boy in your practice is contemplating another pregnancy and asks for your advice. The
woman is tall and thin and works as a model part-time. She had previously reported to you a history of
bulimia. She is extremely concerned about any "extra" weight she may gain during the pregnancy, and
she confides that she sometimes smokes cigarettes to avoid eating. Additionally, she occasionally has
taken her son’s methylphenidate to suppress her appetite. When asked about alcohol use, she
describes herself as a "social drinker."

Of the following, the MOST accurate statement to make in counseling this woman is that

A. bulimia during pregnancy is not associated with birth defects

B. cigarette smoking increases the risk of sudden infant death syndrome in the exposed infant

C. one or two alcoholic beverages per day will do no harm to the embryo/fetus

D. prenatal methylphenidate exposure is associated with craniofacial malformations

E. vitamin supplements reduce the risk of defects associated with prenatal alcohol exposure

page 313
2009 PREP SA on CD-ROM

Critique: 87 Preferred Response: B


The extent of the impact that environmental exposures have on pregnancy outcome is still largely
unknown. However, some large, well-designed epidemiologic studies offer insight into complications
that can arise from fetal exposure to maternal eating disorders, tobacco smoking, and substances of
abuse. Undoubtedly, genetic and other factors modify outcomes.
The impact of cigarette smoking on fetal development and pregnancy has been, and continues to
be, an active area of study. Intrauterine growth restriction is the most consistent negative effect of
maternal smoking on the fetus, with an average 200-g reduction in the birthweights of term infants born
to mothers who smoke during pregnancy. The more a woman smokes, the greater the reduction in fetal
weight.
Cigarette smoking during pregnancy also has been associated with increased health risks to
exposed children as they grow. Among the reported complications are abnormal pulmonary function,
increased cancer risk, and lower sperm counts. Numerous studies have found an increased risk for
sudden infant death in prenatally exposed infants.
Studies of the effects of eating disorders on pregnancy outcome are limited. However, both
prospective, controlled studies and literature reviews indicate that pregnant women who have past or
active eating disorders, including bulimia, are at increased risk for delivering babies of significantly
lower-than-average birthweight and head circumference (including microcephaly).
Prenatal exposure to alcohol may result in varied outcomes collectively termed fetal alcohol
spectrum disorder. To date, no safe quantity of alcohol has been established for consumption during
pregnancy. Therefore, the strong recommendation is that women abstain from drinking alcoholic
beverages from at least the time of conception and throughout pregnancy.
The effects of methylphenidate on pregnancy outcome have been studied in experimental animals
and a limited number of humans. To date, there does not appear to be an increase in congenital
anomalies associated with prenatal exposure to methylphenidate.
The question of the potential benefits of vitamin and micronutrient supplementation for women who
smoke cigarettes and drink alcohol during pregnancy and their babies is under investigation. At present,
it is not clear whether supplementation can mitigate poor outcomes, and no specific recommendations
exist. However, it is clear that good nutrition and vitamin supplementation during pregnancy reduce the
risk for pregnancy complications.

References:

Cigarette smoking, methamphetamine. Reprotox. Available for subscription at: http://www.reprotox.org

Cigarette smoking, methamphetamine. Teris. Available for subscription at


http://depts.washington.edu/terisweb/teris/

Cogswell ME, Weisberg P, Spong C. Cigarette smoking, alcohol use and adverse pregnancy outcomes:
implications for micronutrient supplementation. J Nutr. 2003;133:1722S-1731S. Available at:
http://jn.nutrition.org/cgi/content/full/133/5/1722S

Kouba S, Hällström T, Londholm C, Lindén Hirschbe A. Pregnancy and neonatal outcomes in women
with eating disorders. Obstet Gynecol. 2005;105:255-260. Available at:
http://www.greenjournal.org/cgi/content/full/105/2/255

Kunz LH, King JC. Impact of maternal nutrition and metabolism on health of the offspring. Semin Fetal
Neonatal Med. 2007;12;71-77. Abstract available at: http://www.ncbi.nlm.nih.gov/pubmed/17200031

page 314
2009 PREP SA on CD-ROM

Micali N, Simonoff E, Treasure J. Risk of major adverse outcomes in women with eating disorders. Br J
Psychiatry. 2007;190:255-259. Abstract available at: http://www.ncbi.nlm.nih.gov/pubmed/17329747

page 315
2009 PREP SA on CD-ROM

Question: 88
An 18-year-old young man comes to your office with complaints of burning pain with urination over the
past 24 hours. He has seen a small amount of yellowish discharge from his penis during this time. He
also complains of some lower back pain over the past 48 hours. He denies fever or rashes, but his eyes
are a little irritated. He is sexually active and uses condoms "most of the time." On physical
examination, he is afebrile, his palpebral and bulbar conjunctivae are mildly injected (Item Q88), and his
back is tender at the lower lumbar area, but there is no costovertebral angle tenderness. Genital
examination reveals no scrotal tenderness and scant yellow discharge at the urethral orifice.

Of the following, the MOST likely cause of this patient’s symptoms is

A. Chlamydia trachomatis

B. Gardnerella vaginalis

C. Neisseria gonorrhoeae

D. Treponema pallidum

E. Trichomonas vaginalis

page 316
2009 PREP SA on CD-ROM

Question: 88

(Courtesy of P. Sagerman)

page 317
2009 PREP SA on CD-ROM

Critique: 88 Preferred Response: A


Urethritis can have infectious and noninfectious causes. Symptoms include mucopurulent or purulent
discharge, dysuria, and urethral pruritus. Several organisms, including Neisseria gonorrhoeae and
Chlamydia trachomatis, cause urethritis. Ureaplasma urealyticum, Mycoplasma genitalium, Gardnerella
vaginalis, herpes simplex virus, adenovirus, and Trichomonas vaginalis are implicated in
nonchlamydial, nongonococcal urethritis (NGU), but they are more difficult to detect than N
gonorrhoeae and C trachomatis.
The constellation of conjunctivitis, urethritis, and arthritis reported for the young man in the vignette
represents the classic symptoms of a form of reactive arthritis once called Reiter syndrome. The term
reactive arthritis refers to rheumatic disorders that appear after an infection, but in which the
responsible pathogen is not detected in the affected joint. C trachomatis is the only genital pathogen
commonly accepted to be a cause of reactive arthritis. N gonorrhoeae can cause septic arthritis or
disseminated gonococcal infection (ie, a rash and tenosynovitis) but does not produce reactive arthritis.
Although T vaginalis and G vaginalis may cause urethritis, they do not produce the other symptoms
exhibited by the boy described in the vignette. Syphilis, caused by infection with Treponema pallidum,
may affect bones congenitally (osteochondritis) or in late stages of the disease (with gummas,
granulomatous lesions that involve bones as well as soft tissue or viscera) but does not produce
urethral discharge or conjunctivitis.
Reactive arthritis caused by C trachomatis is treated with a single 1-g oral dose of azithromycin or
with 100 mg doxycycline orally twice a day for 7 days, after testing for both N gonorrhoeae and C
trachomatis is completed. This is also the recommended regimen for all NGUs. First-line treatment of
uncomplicated gonococcal urethritis is accomplished with ceftriaxone 125 mg intramuscularly or
cefixime 400 mg orally, both in a single dose.

References:

Centers for Disease Control and Prevention. Update to CDC's sexually transmitted diseases treatment
guidelines, 2006: fluoroquinolones no longer recommended for treatment of gonococcal infections.
MMWR Morbid Mortal Wkly Rep. 2007;56:332-336. Available at:
http://www.cdc.gov/mmwr/preview/mmwrhtml/mm5614a3.htm

Fortenberry JD, Neinstein LS. Syphilis. In: Neinstein LS, ed. Adolescent Health Care: A Practical Guide.
5th ed. Philadelphia, Pa: Lippincott Williams & Wilkins; 2008:825-833

Workowski KA, Berman SM, Centers for Disease Control and Prevention. Sexually transmitted
diseases treatment guidelines, 2006. MMWR Recomm Rep. 2006;55(RR11):1-94. Available at:
http://www.cdc.gov/mmwr/preview/mmwrhtml/rr5511a1.htm

Yu DT. Reactive arthritis (formerly Reiter syndrome): definition, diagnosis, and management. UpToDate
Online 15.3. 2008. Available for subscription at:
http://www.utdol.com/utd/content/topic.do?topicKey=spondylo/7349

page 318
2009 PREP SA on CD-ROM

Question: 89
You are evaluating an 18-month-old girl for vomiting. She has a history of febrile seizures and recurrent
ear infections. She is receiving no medications. Over the past several weeks, her parents have noticed
that she has been "increasingly clumsy." She has vomited each of the last three mornings but has had
no diarrhea or fever. Physical examination findings are normal except for an ataxic gait and
hyperreflexia.

Of the following, the MOST appropriate next step is

A. administration of an antiemetic

B. computed tomography scan of the head

C. electroencephalography

D. lumbar puncture

E. reassurance and re-evaluation in 3 to 5 days

page 319
2009 PREP SA on CD-ROM

Critique: 89 Preferred Response: B


Initial symptoms of increased intracranial pressure often consist of headaches and confusion that may
be accompanied by lethargy. The child described in the vignette exhibits signs of a progressive
increase in pressure, such as vomiting (especially in the morning) and changes in motor tone. Physical
examination findings include a full or bulging fontanelle and widened sutures. These signs, as well as
pupillary changes and papilledema, should prompt rapid evaluation to prevent permanent neurologic
injury and progression of the increased pressure with potential neurologic catastrophe.
Intracranial pressure is maintained by the balance of the contents of the cranial vault, which includes
brain, blood, and cerebrospinal fluid. Increases in intracranial pressure can occur with a wide variety of
disease processes, such as brain tumors, hydrocephalus, infections, head trauma, and
hypoxic-ischemic injury. Intracranial pressure increases when the compensatory mechanisms of the
cranial vault are exceeded and produce numerous symptoms, depending on the age of the patient and
the underlying pathology.
Computed tomography scan or magnetic resonance imaging of the head is the first priority in
evaluating suspected increased intracranial pressure. Meningitis is unlikely in this patient due to the
chronicity of symptoms and absence of fever. Accordingly, lumbar puncture is not indicated at this time.
Electroencephalography might be indicated if atypical migraines or seizures were suspected, but the
initial priority is to evaluate the patient for potential life-threatening disease processes. The child has no
evidence of viral infection, and reassurance or administration of antiemetics is not appropriate.

References:

Frankel LR. Neurological emergencies and stabilization. In: Kliegman RM, Behrman RE, Jenson HB,
Stanton BF, eds. Nelson Textbook of Pediatrics. 18th ed. Philadelphia, Pa: Saunders Elsevier;
2007:405-412

Larsen GY, Goldstein B. Consultation with the specialist: increased intracranial pressure. Pediatr Rev.
1999;20:234-239. Available at: http://pedsinreview.aappublications.org/cgi/content/full/20/7/234

page 320
2009 PREP SA on CD-ROM

Question: 90
A 16-year-old girl comes to your office complaining that her menstrual periods have been irregular and
scanty. Her last period was 3 months ago and lasted for only 2 days. Among the findings on physical
examination are fine, moist skin; firm, palpable thyroid gland (Item Q90); and finger tremor. Results of
laboratory studies include a thyroid-stimulating hormone value of less than 0.05 mIU/L (normal, 0.5 to
5.0 mIU/L) and free thyroxine value of 1.9 ng/dL (24.5 pmol/L) (normal, 0.6 to 1.3 ng/dL [7.7 to 16.8
pmol/L]).

Of the following, the additional physical examination finding that BEST supports the diagnosis of
hyperthyroidism is

A. abdominal obesity

B. atrophy of lingual papillae

C. hepatomegaly

D. hirsutism

E. muscle weakness

page 321
2009 PREP SA on CD-ROM

Question: 90

(Courtesy of M. Rimsza)

page 322
2009 PREP SA on CD-ROM

Critique: 90 Preferred Response: E


The clinical findings of hyperthyroidism in children usually are obvious, but the diagnosis can be subtle
in mild disease. Classic clinical findings include weight loss, increased appetite, decreased strength and
sports performance, hyperactivity, tremors, sweating, nocturnal sleeplessness (sometimes with daytime
somnolence), irritability, decreased school performance, pruritus, and nocturia. Menarcheal girls may
have scant, infrequent menses, as described for the girl in the vignette. On physical examination, most
children have a palpable, firm thyroid gland (Item C90A) that has an audible bruit. Some children may
have fine, moist skin; a visible tremor; slight skin darkening; fine scalp hair, with some hair loss at the
temples; muscle weakness; and some loss of muscle mass that can be identified by examining the
thenar and hypothenar eminences. A "thyrotoxic stare" accompanies hyperthyroidism. Exophthalmos
(Item C90B) may be found in thyrotoxicosis due to Graves disease. Hirsutism, hepatomegaly,
abdominal obesity, and changes in the lingual papillae are not findings of hyperthyroidism.
Other laboratory studies that are of use in caring for the girl in the vignette include a measurement of
triiodothyronine, which often is substantially elevated in hyperthyroidism, and measures of
thyroid-stimulating immunoglobulins, which are elevated in Graves disease. Radioactive iodine or
technetium uptake imaging can distinguish between subacute thyroiditis (low uptake) and Graves
disease (high uptake). This distinction is important because treatments for Graves disease, including
use of antithyroid drugs, radioactive iodine, and surgery, are not appropriate for transient subacute
thyroiditis.

References:

Fenton CL, Gold JG. Hyperthyroidism. eMedicine Specialties, Pediatrics, Endocrinology. 2006.
Available at: http://www.emedicine.com/ped/topic1099.htm

Ferry RJ Jr, Levitsky LL. Graves disease. eMedicine Specialties, Pediatrics, Endocrinology. 2006.
Available at: http://www.emedicine.com/ped/topic899.htm

LaFranchi S. Clinical manifestations and diagnosis of hyperthyroidism in children and adolescents.


UpToDate Online 15.3. 2008. Available for subscription at:
http://www.uptodateonline.com/utd/content/topic.do?topicKey=pediendo/5570

page 323
2009 PREP SA on CD-ROM

Critique: 90

Most children who have hyperthyroidism have a goiter. (Courtesy of M.


Rimsza)

page 324
2009 PREP SA on CD-ROM

Critique: 90

Exophthalmos in an adolescent who has thyrotoxicosis. (Courtesy of M.


Rimsza)

page 325
2009 PREP SA on CD-ROM

Question: 91
An infant in the newborn nursery does not appear to respond to visual or auditory input. On physical
examination, he shows evidence of intrauterine growth restriction (IUGR), absent red reflexes, and
numerous bluish papules (Item Q91). The mother, who immigrated to the United States during her third
trimester, did not receive prenatal care. She denies use of alcohol, drugs, or tobacco products during
pregnancy. She reports that she had a low-grade fever and rash during the second month of the
pregnancy.

Of the following, the MOST likely infectious cause of the findings in this infant is

A. cytomegalovirus

B. human immunodeficiency virus

C. rubella virus

D. Toxoplasma gondii

E. varicella-zoster virus

page 326
2009 PREP SA on CD-ROM

Question: 91

Blue papules, as exhibited by the infant in the vignette. (Courtesy of M.


Rimsza)

page 327
2009 PREP SA on CD-ROM

Critique: 91 Preferred Response: C


Congenital infections may lead to developmental sequelae in infancy and childhood, including visual
impairment, hearing loss, and intellectual disabilities (Item C91A). The infant described in the vignette
has clinical findings most consistent with congenital rubella syndrome (CRS). These findings include
intrauterine growth restriction (IUGR), absent red reflexes (due to cataracts), and bluish papules (Item
C91B). Other clinical findings associated with CRS include nerve deafness, microphthalmia, cardiac
defects, meningoencephalitis, hepatomegaly, and microcephaly.
Maternal cytomegalovirus (CMV) infection is common, but 90% of infants who have CMV infection
are asymptomatic at birth. Maternal symptoms include a flulike illness that may involve fever, but a rash
is not seen, as reported by the mother in the vignette. Among the clinical findings of congenital CMV
infection are hepatomegaly, splenomegaly, jaundice, petechiae, chorioretinitis, IUGR, purpura, and
microcephaly. CMV infection is the leading nongenetic reason for sensorineural hearing loss and the
most common congenital infection to cause intellectual disability.
Congenital varicella infection may result in zigzag scarring of the skin, limb deformities, or cataracts.
Occasionally, severely affected infants may have central nervous system involvement with necrotizing
cerebral lesions or microcephaly.
Congenital toxoplasmosis presents with IUGR, anemia, jaundice, hepatosplenomegaly, intracranial
calcifications, hydrocephalus, microcephaly, and chorioretinitis, but not skin lesions.
A newborn infected perinatally with human immunodeficiency virus exhibits no symptoms or signs.
Later, the infant may develop subtle clinical findings, such as lymphadenopathy and
hepatosplenomegaly, or nonspecific symptoms, such as failure to thrive, chronic or recurrent diarrhea,
interstitial pneumonia, or oral thrush. Central nervous system involvement is variable, ranging from mild
learning disabilities to severe mental retardation.

References:

Adler SP, Marshall B. Cytomegalovirus infections. Pediatr Rev. 2007;28:92-100. Available at:
http://pedsinreview.aappublications.org/cgi/content/full/28/3/92

Mason W. Rubella. In: Kliegman RM, Behrman RE, Jenson HB, Stanton, BF, eds. Nelson Textbook of
Pediatrics. 18th ed. Philadelphia, Pa: Saunders Elsevier; 2007:1337-1341

McLeod R, Remington JS. Toxoplasmosis (Toxoplasma gondii). In: Kliegman RM, Behrman RE,
Jenson HB, Stanton BF, eds. Nelson Textbook of Pediatrics. 18th ed. Philadelphia, Pa: Saunders
Elsevier; 2007:1486-1495

Myers MG, Seward J, La Russa P. Varicella-zoster virus. In: Kliegman RM, Behrman RE, Jenson HB,
Stanton BF, eds. Nelson Textbook of Pediatrics. 18th ed. Philadelphia, Pa: Saunders Elsevier;
2007:1366-1372

Stagno S. Cytomegalovirus. In: Kliegman RM, Behrman RE, Jenson HB, Stanton BF, eds. Nelson
Textbook of Pediatrics. 18th ed. Philadelphia, Pa: Saunders Elsevier; 2007:1377-1379

Yogev R, Gould Chadwick E. Acquired immunodeficiency syndrome (human immunodeficiency virus).


In: Kliegman RM, Behrman RE, Jenson HB, Stanton BF, eds. Nelson Textbook of Pediatrics. 18th ed.
Philadelphia, Pa: Saunders Elsevier; 2007:1427-1442

page 328
2009 PREP SA on CD-ROM

Critique: 91

page 329
2009 PREP SA on CD-ROM

Critique: 91

Purpuric macules and papules may be present in congenital rubella, creating


a "blueberry muffin" appearance. (Courtesy of M. Rimsza)

page 330
2009 PREP SA on CD-ROM

Question: 92
A 6-month-old boy presents to the emergency department with a 2-day history of fever and a 1-day
history of left cheek swelling. You discover that his parents do not believe in providing their children with
immunizations. Despite this, the boy has never been ill. He has two older siblings, and nobody is sick at
home. The mother denies any recent bug bites or trauma to the area on his cheek. Physical
examination reveals a mildly toxic-appearing child who has a temperature of 103.0°F (39.4°C), heart
rate of 145 beats/min, respiratory rate of 26 breaths/min, and blood pressure of 80/45 mm Hg. His
anterior fontanelle is slightly bulging, his tympanic membranes are erythematous, his left cheek is
indurated and appears erythematous to slightly violaceous (Item Q92), and he is irritable.

Of the following, the MOST likely organism to cause this child’s illness is

A. Haemophilus influenzae type b

B. Neisseria meningitidis

C. Staphylococcus aureus

D. Streptococcus pneumoniae

E. Streptococcus pyogenes

page 331
2009 PREP SA on CD-ROM

Question: 92

(Courtesy of D. Krowchuk)

page 332
2009 PREP SA on CD-ROM

Critique: 92 Preferred Response: A


Children who have not received the Haemophilus influenzae type b (Hib) vaccine are at risk for
illnesses commonly caused by this organism, including buccal and periorbital cellulitis (as described for
the boy in the vignette) (Item C92A), pyogenic arthritis, epiglottitis (Item C92B), and bacterial meningitis.
Neisseria meningitidis usually does not cause a facial cellulitis, and Staphylococcus aureus and
Streptococcus pyogenes are less likely pathogens in the absence of a history of a break in the skin. S
pneumoniae can be the cause of a nontraumatic facial cellulitis in Hib-vaccinated children, but in an
unvaccinated child, Hib would be the most likely pathogen.
Hib disease can be verified by recovery of the organism from a sterile site (eg, blood, cerebrospinal
fluid, joint fluid) or by urine antigen testing. Once the organism is isolated, antimicrobial susceptibility
testing is important because approximately 30% to 40% of Hib isolates produce beta-lactamase,
making these organisms resistant to ampicillin.

References:

American Academy of Pediatrics. Haemophilus influenzae infections. In: Pickering LK, Baker CJ, Long
SS, McMillan JA, eds. Red Book: 2006 Report of the Committee on Infectious Diseases. 27th ed. Elk
Grove Village, Ill: American Academy of Pediatrics; 2006:310-318

Tzanakaki G, Mastrantonio P. Aetiology of bacterial meningitis and resistance to antibiotics of causative


pathogens in Europe and in the Mediterranean region. Int J Antimicrob Agents. 2007;29:621-629.
Abstract available at: http://www.ncbi.nlm.nih.gov/pubmed/17368858

page 333
2009 PREP SA on CD-ROM

Critique: 92

In buccal cellulitis caused by Haemophilus influenzae type b infection, the


affected area is indurated and has an erythematous-to-violaceous color.
(Courtesy of D. Krowchuk)

page 334
2009 PREP SA on CD-ROM

Critique: 92

Epiglottitis: Swelling and erythema of the epiglottis caused by Haemophilus


influenzae type b infection. (Courtesy of K. Woodin)

page 335
2009 PREP SA on CD-ROM

Question: 93
You are seeing a 5-year-old boy who has developed diplopia, dysphagia, dry mouth, diarrhea,
weakness in his arms, and shortness of breath over the past 18 hours. According to his records, he
received his diphtheria, tetanus, acellular pertussis (DTaP), poliovirus inactivated (IPV),
measle-mumps-rubella (MMR), and varicella booster immunizations about 1 month ago. He attended a
class picnic 3 weeks ago that was held in a state park. He has no history of unusual exposures or ill
contacts, and except for falling off his bike 5 days ago and scraping his arm, he has had no other
trauma. Physical examination reveals an awake and alert boy who complains of "seeing double" and of
pain with swallowing. His pupils are 3 mm bilaterally and sluggish, and his mucous membranes are dry.
He takes shallow breaths, but his lungs are clear, and his abdomen is mildly distended. His left arm has
a 4x4-cm abrasion that is mildly swollen, erythematous, and tender, with some serosanguineous
drainage. His left arm has 2/5 strength and decreased tone. He has 1+ reflexes in the upper and lower
extremities.

Of the following, the MOST likely cause of this patient’s condition is

A. botulism

B. cerebral vascular accident

C. Guillain-Barré syndrome

D. tetanus

E. tick paralysis

page 336
2009 PREP SA on CD-ROM

Critique: 93 Preferred Response: A


Clostridium botulinum spores are found worldwide in soil and marine sediments. Botulism results from
the absorption of botulinum toxins, most commonly produced by C botulinum (a large, gram-positive,
anaerobic bacillus that has a subterminal spore), into the circulation from a wound or mucosal surface.
Of the seven antigenic toxin types that the organism produces, human botulism is caused by
neurotoxins A, B, E, and occasionally F. The clinical forms of botulism include: foodborne, infant,
wound, and that of undetermined cause, which is rare. Almost all cases of infant botulism and wound
botulism are caused by types A and B toxin. The onset of symptoms may be abrupt (within hours) or
evolve gradually over several days. Foodborne botulism occurs with the ingestion of food that is
contaminated with spores of C botulinum and has been stored improperly under anaerobic conditions,
allowing for toxin production. In wound botulism, the spores are introduced into the wound at the time of
trauma, where they germinate and produce toxin. Infant botulism results from ingestion of C botulinum
spores that germinate, multiply, and produce toxin in the intestine. Most cases of infant disease occur in
breastfed infants at the time of introduction of nonhuman milk substances, and the source of the spores
usually is not identified.
The classic presentation of botulism is acute, bilateral cranial nerve palsies associated with
symmetric descending weakness. Fever is absent, sensory deficits do not occur, and the patient
remains responsive, with a normal level of consciousness. Foodborne disease usually develops
between 12 and 36 hours after toxin ingestion. The initial symptoms include nausea, dry mouth, and
diarrhea. Disease progresses to cranial nerve dysfunction, most commonly starting with the eyes
(diplopia, blurry vision), and descending to include dysphagia, upper extremity weakness, respiratory
dysfunction, and lower extremity weakness. The symptoms of wound botulism are similar to that of
food-borne disease, although the incubation period is 4 to 14 days from the time of injury until the onset
of symptoms. Also, in many cases, the wound may not appear to be healing, as described for the boy in
the vignette. Infant botulism develops 3 to 30 days from the time of exposure to the spore-containing
material. Infants present with constipation, which is followed by feeding difficulties, hypotonia, increased
drooling, a weak cry, diminished gag reflex, truncal weakness, cranial nerve palsies, and generalized
weakness with ventilatory failure. Progression occurs for 1 to 2 weeks, followed by stabilization for
another 2 to 3 weeks before recovery starts.
An important element in the treatment of all forms of botulism is meticulous supportive care. In
addition, patients who have suspected foodborne and wound botulism should be treated with equine
trivalent antitoxin (types A, B, and E) available from the Centers for Disease Control and Prevention
through state health departments. Immediate administration of antitoxin is critical for successful therapy
because it arrests the progression of paralysis but does not reverse it. Patients who have wound
botulism also should undergo wound debridement, even if the wound appears to be healing. The role of
antibiotic therapy is unknown, but penicillin G or metronidazole frequently is recommended.
Human-derived botulinum antitoxin (botulism immune globulin intravenous) (BIGIV) is used in the
treatment of infant botulism caused by type A or type B C botulinum toxin. BIGIV therapy should be
initiated as early in the illness as possible. Antibiotic therapy is not recommended for infant botulism.
Patients who have cerebral vascular accidents usually do not present with bilateral symmetric
weakness or the other symptoms demonstrated by the patient in the vignette. Patients who have
tetanus have persistent, painful tonic spasms of the muscles of the neck, jaw, and trunk and are very
rigid. The paralysis of tick paralysis is ascending, beginning in the lower extremities and ascending
symmetrically to involve the trunk, upper extremities, and head within a few hours. Patients who have
Guillain-Barré syndrome typically present with numbness and paresthesias of the hands and feet,
followed by progressive weakness involving all four extremities. Motor impairment begins in the lower
extremities and progresses in an ascending pattern to involve the upper extremities, trunk, and cranial
nerves.

page 337
2009 PREP SA on CD-ROM

References:

American Academy of Pediatrics. Botulism and infant botulism (Clostridium botulinum). In: Pickering
LK, Baker CJ, Long SS, McMillan JA, eds. Red Book: 2006 Report of the Committee on Infectious
Diseases. 27th ed. Elk Grove Village, Ill: American Academy of Pediatrics; 2006:257-260

American Academy of Pediatrics. Tetanus (lockjaw). In: Pickering LK, Baker CJ, Long SS, McMillan JA,
eds. Red Book: 2006 Report of the Committee on Infectious Diseases. 27th ed. Elk Grove Village, Ill:
American Academy of Pediatrics; 2006:648-653

Bleck TP. Clostridium botulinum (botulism). In: Mandell GL, Bennett JE, Dolin R, eds. Mandell,
Douglas, and Bennett's Principles and Practice of Infectious Diseases. 6th ed. Philadelphia, Pa:
Elsevier Churchill Livingstone; 2005:2822-2828

Mathieu ME, Wilson BB. Ticks (including tick paralysis). In: Mandell GL, Bennett JE, Dolin R, eds.
Mandell, Douglas, and Bennett's Principles and Practice of Infectious Diseases. 6th ed. Philadelphia,
Pa: Churchill Livingstone; 2005:3312-3315

Parke JT. Peripheral neuropathies. In: McMillan JA, Feigin RD, DeAngelis CD, Jones MD Jr, eds.
Oski's Pediatrics Principles and Practice. 4th ed. Philadelphia, Pa: Lippincott Williams & Wilkins;
2006:2310-2316

Schlagger B, Kornberg AJ, Prensky AL. Cerebrovascular disease in childhood. In: McMillan JA, Feigin
RD, DeAngelis CD, Jones MD Jr, eds. Oski's Pediatrics Principles and Practice. 4th ed. Philadelphia,
Pa: Lippincott Williams & Wilkins; 2006:2270-2279

page 338
2009 PREP SA on CD-ROM

Question: 94
An 8-month-old girl who has a history of cardiomyopathy following viral myocarditis presents with poor
weight gain. She is receiving a 20-kcal/oz milk-based formula and has no history of vomiting or
diarrhea. Her only medication is furosemide. Physical examination findings include a heart rate of 130
beats/min, respiratory rate of 60 breaths/min, and blood pressure of 88/44 mm Hg.

Of the following, the MOST appropriate initial strategy to increase weight gain for this girl is to

A. change to a 24-kcal/oz formula

B. discontinue furosemide therapy

C. increase the volume of 20-kcal/oz formula

D. place a gastrostomy feeding tube

E. start parenteral nutrition

page 339
2009 PREP SA on CD-ROM

Critique: 94 Preferred Response: A


Young infants who have cardiac dysfunction, such as the girl described in the vignette, often have
difficulty ingesting sufficient calories for growth because they frequently have increased caloric
expenditure with feeding. Increased caloric intake may place the infants at risk of fluid overload,
necessitating the use of diuretics. Because an increased volume of feedings with a 20-kcal/oz formula
may place the infant at risk of fluid overload, use of a more concentrated formula is required.
Accordingly, the infant described in the vignette should be changed to a 24-kcal/oz formula. Increasing
the caloric density of feedings can meet the goals of increased calories and "relative" fluid restriction.
Discontinuation of furosemide likely would lead to weight gain from fluid retention, not a true weight
gain. A gastrostomy tube provides a conduit for feeding and probably plays a role in a child incapable of
taking in sufficient calories (eg, chronic renal failure or severe developmental delay), but the child in the
vignette deserves a trial of high-calorie feedings before subjecting her to this surgical procedure.
Finally, parenteral nutrition is not a suitable option because this child has a functional gastrointestinal
tract and can tolerate enteral nutrition.

References:

Kelleher DK, Laussen P, Teixeira-Pinto A, Duggan C. Growth and correlates of nutritional status among
infants with hypoplastic left heart syndrome (HLHS) after stage 1 Norwood procedure. Nutrition. 2006;
22:237-244. Abstract available at: http://www.ncbi.nlm.nih.gov/pubmed/16500550

Pillo-Blocka F, Adatia I, Sharieff W, McCrindle BW, Zlotkin S. Rapid advancement to more concentrated
formula in infants after surgery for congenital heart disease reduces duration of hospital stay: a
randomized clinical trial. J Pediatr. 2004;145:761-766. Abstract available at:
http://www.ncbi.nlm.nih.gov/pubmed/15580197

Yahav J, Avigad S, Frand M, et al. Assessment of intestinal and cardiorespiratory function in children
with congenital heart disease on high-caloric formulas. J Pediatr Gastroenterol Nutr. 1985;4:778-785.
Abstract available at: http://www.ncbi.nlm.nih.gov/pubmed/4045636

page 340
2009 PREP SA on CD-ROM

Question: 95
An 18-month-old girl has been having an intermittent nonproductive cough for the past 6 months. Her
parents state that the cough awakens the toddler at night a few times a month and occurs when playing
vigorously. During a recent upper respiratory tract illness, her cough worsened and occurred daily for 3
weeks. On physical examination, there is no nasal discharge, and the toddler appears healthy.

Of the following, the MOST likely diagnosis is

A. asthma

B. atypical pneumonia

C. gastroesophageal reflux

D. sinusitis

E. upper airway cough syndrome

page 341
2009 PREP SA on CD-ROM

Critique: 95 Preferred Response: A


The chronic cough that is exacerbated during the night, with activity, and during an upper respiratory
tract infection described for the child in the vignette most likely represents asthma. Chronic cough
typically is defined as one that persists for more than 8 weeks. When the patient's chest radiograph
appears normal, three causes account for 95% of chronic coughs: asthma, gastroesophageal reflux
(GER), and upper airway cough syndrome (UACS) (previously termed postnasal drip syndrome) (Item
C95).
Asthma usually develops in early childhood, with 80% of patients reporting symptoms prior to age 6
years. Symptoms may include cough, wheezing, shortness of breath, and chest tightness. The most
common trigger for infants and toddlers is a viral upper respiratory tract infection (URI). Fortunately,
URI-induced wheezing resolves in most infants by age 6 years (so-called "transient wheezers"). Those
who continue to have asthma symptoms after age 6 are at greater risk for persistent asthma.
UACS encompasses allergic rhinitis, nonallergic rhinitis, and sinusitis. Allergic rhinitis typically
occurs in children older than 3 years of age and is associated with other ocular and nasal symptoms,
such as pruritus, sneezing, and rhinorrhea. Sinusitis also is characterized by rhinorrhea and postnasal
symptoms.
Atypical pneumonia caused by Mycoplasma pneumoniae and Chlamydophila pneumoniae
(previously termed Chlamydia pneumoniae ) may present at any age, although it is unusual prior to age
3 years. Characteristic constitutional symptoms include fever, malaise, and headache.
Cough can represent the sole manifestation of GER, but GER usually becomes symptomatic during
the first few postnatal months, improving by 12 months of age. GER may worsen at night during supine
positioning, but exercise and URIs are uncommon precipitating factors for GER symptoms.

References:

Liu AH, Covar RA, Spahn JD, Leung DYM. Childhood asthma. In: Kleigman RM, Behrman RE, Jenson
HB, Stanton BF, eds. Nelson Textbook of Pediatrics. 18th ed. Philadelphia, Pa: Saunders Elsevier;
2007:953-969

Weinberger M, Abu-Hasan M. Pseudo-asthma: when cough, wheezing, and dyspnea are not asthma.
Pediatrics. 2007;120:855-864. Available at:
http://pediatrics.aappublications.org/cgi/content/full/120/4/855

page 342
2009 PREP SA on CD-ROM

Critique: 95

page 343
2009 PREP SA on CD-ROM

Question: 96
A 10-year-old boy comes to the office 2 days after falling off of his bicycle and injuring his forehead. He
denies vomiting or headache but complains of a runny nose. Physical examination reveals a
well-appearing boy who has a large ecchymotic swelling over the central portion of his forehead with an
overlying abrasion. The area is diffusely tender to palpation, and there is a depression over the right
lateral aspect of the swelling. Erythema around the abrasion is minimal, and no purulent drainage is
present. Clear fluid is draining from his right naris. The remainder of his physical examination findings
are normal. You order a computed tomography scan (Item Q96).

Of the following, the MOST appropriate treatment of this boy’s injury should include

A. decongestants

B. nasal packing

C. no specific treatment

D. prophylactic antibiotics

E. surgical repair

page 344
2009 PREP SA on CD-ROM

Question: 96

(Courtesy of D. Mulvihill)

page 345
2009 PREP SA on CD-ROM

Critique: 96 Preferred Response: E


The history of trauma and the physical findings of forehead ecchymosis with clear, persistent rhinorrhea
described for the boy in the vignette should raise concern for a cerebrospinal fluid (CSF) leak. Prompt
radiologic imaging can aid in determining the cause. In most cases, a frontal sinus fracture is the cause,
as exhibited by the boy in the vignette, and head computed tomography (CT) scan usually is adequate
to identify the injury (Item C96). If an obvious fracture is not seen on CT scan, further imaging with
magnetic resonance imaging or radionucleotide cisternography may be necessary to localize the
source. Testing of the fluid for glucose or protein is not diagnostically sensitive and should not be used
to determine further evaluation and treatment. The incidence of CSF leak following frontal sinus fracture
is nearly 20% in children, twice the rate seen in adults. The primary concern in patients who have CSF
rhinorrhea is the development of central nervous system (CNS) infections such as meningitis or brain
abscess. Such complications are seen most commonly after trauma and in patients whose leaks do not
resolve spontaneously within 7 days.
Anterior wall frontal sinus fractures are repaired primarily for cosmetic reasons, but those involving
the posterior wall (which occurs in 70% of these injuries in children) require open reconstruction to
ensure that the communication between the sinus and brain is eliminated.
Decongestants and nasal packing are not indicated in the treatment of CSF rhinorrhea. The use of
prophylactic antibiotics for the prevention of CNS infection is controversial, although most studies
suggest that the risk of developing antibiotic resistance is greater than the potential prevention of
meningitis.

References:

Kellman RM. Maxillofacial trauma. In: Cummings CW, Flint PW, Haughey BH, Robbins KT, Thomas JR
eds. Cummings Otolaryngology: Head & Neck Surgery. 4th ed. Philadelphia, Pa: Mosby Elsevier;
2005:chap 26

Kerr JT, Chu FW, Bayles SW. Cerebrospinal fluid rhinorrhea: diagnosis and management. Otolaryngol
Clin North Am. 2005;38:597-611. Abstract available at: http://www.ncbi.nlm.nih.gov/pubmed/16005720

Kravitz PR, Koltai PJ. Pediatric facial fractures. In: Cummings CW, Flint PW, Haughey BH, Robbins KT,
Thomas JR eds. Cummings Otolaryngology: Head & Neck Surgery. 4th ed. Philadelphia, Pa: Mosby
Elsevier; 2005:chap 202

page 346
2009 PREP SA on CD-ROM

Critique: 96

Axial computed tomography scan of the head, as described for the child in the
vignette, shows fractures through the anterior and posterior walls of the right
frontal sinus and air within the cranium. (Courtesy of D. Mulvihill)

page 347
2009 PREP SA on CD-ROM

Question: 97
A 16-year-old boy in your practice has cystic fibrosis. As a complication of his illness, he has developed
cirrhosis and cholestasis. He now complains of shaky hands. Neurologic examination demonstrates
hyporeflexia and tremor with hands outstretched.

Of the following, the patient’s symptoms are MOST consistent with deficiency of

A. vitamin A

B. vitamin B1 (thiamine)

C. vitamin C

D. vitamin D

E. vitamin E

page 348
2009 PREP SA on CD-ROM

Critique: 97 Preferred Response: E


Because the young man described in the vignette has chronic cholestasis, he is at risk for developing
deficiency of any of the fat-soluble vitamins, including vitamins A, D, E, and K. His neurologic symptoms
of tremor and hyporeflexia most strongly suggest vitamin E deficiency.
Vitamin E (tocopherol) is an important factor in stabilizing the lipid membrane of the red blood cell
and the lipids in the myelin sheath of neurons. Therefore, the most common presenting features of
hypovitaminosis E are hemolysis (primarily reported in preterm infants) and peripheral neuropathy
(identified in infants and children who have chronic cholestasis, pancreatic insufficiency, or
malabsorption).
Supplementation of formulas and parenteral nutrition with vitamin E has reduced substantially the
incidence of hemolysis in the vitamin E-deficient preterm infant. However, patients who have cystic
fibrosis or cholestatic liver disease require both monitoring of vitamin E concentrations and
supplementation with vitamin E. Because vitamin E is a fat-soluble vitamin, those who have cholestasis
may have difficulty absorbing alpha-tocopherol, the form of vitamin E available in most dietary
supplements. For this reason, d-alpha-tocopheryl polyethylene glycol 1,000 succinate, a water-soluble
form of vitamin E, should be given to patients who have significant cholestatic liver disease. The
recommended dose for a patient who has cholestatic liver disease is 15 to 25 IU/kg per day.
Deficiency of vitamin A, B1, C, or D would not be expected to cause such a clinical presentation.
Vitamin A deficiency causes impaired vision ("night blindness") and corneal ulcers; vitamin B1
deficiency can cause myopathy and heart failure ("beriberi"); vitamin C deficiency causes irritability,
bone lesions, and bruising (scurvy); and vitamin D deficiency causes osteopenia or rickets.

References:

Harmatz P, Burensky E, Lubin B. Nutritional anemias. In: Walker WA, Watkins JB, Duggan C, eds.
Nutrition in Pediatrics. 3rd ed. Hamilton, Ontario, Canada: BC Decker; 2003:830-847

Spinozzi NS. Hepatobiliary diseases. In: Hendricks KM, Duggan C. Manual of Pediatric Nutrition. 4th
ed. Hamilton, Ontario, Canada: BC Decker; 2005:586-592

page 349
2009 PREP SA on CD-ROM

Question: 98
You are making rounds with medical students in the neonatal intensive care unit and examining a 2-kg,
34 weeks’ gestation newborn whose mother had gestational diabetes mellitus. The infant has no
respiratory distress. A medical student asks how to test for fetal lung maturity to predict the risk of
neonatal respiratory distress syndrome in the offspring of a pregnant woman who has diabetes mellitus.

Of the following, the MOST appropriate test is

A. disaturated lecithin presence

B. lecithin:sphingomyelin ratio

C. phosphatidylglycerol presence

D. phosphatidylinositol presence

E. total surfactant activity

page 350
2009 PREP SA on CD-ROM

Critique: 98 Preferred Response: E


The human fetus continues to develop lung maturity up until a term gestation, but the lungs generally
are sufficiently mature to maintain extrauterine respiration by 36 weeks' gestation. Infants delivered
prior to this time have variable degrees of pulmonary maturity as surfactant composition, synthesis, and
storage change with advancing fetal development. Tests for determining fetal lung maturation depend
on amniotic fluid analysis for surfactant presence and composition because fetal surfactant leaves the
lung as an effluent that ebbs and flows with fetal breathing movement and is balanced by fetal
swallowing.
In general, in the healthy fetus, growth, size, and maturation typically are linked with gestational age.
Fetal lung maturity has been assessed using the lecithin-to-sphingomyelin ratio (L:S) for more than 30
years. This test is time-consuming and requires thin-layer chromatography. It depends on fetal lung fluid
flowing into the amniotic fluid and altering the amniotic fluid phospholipid composition, with the results
expressed as the ratio of a lecithin (phosphatidylcholine) to sphingomyelin per milliliter of amniotic fluid.
The sphingomyelin content of amniotic fluid decreases after 32 weeks' gestational age, while the
lecithin content, including that portion that is disaturated, increases. The L:S ratio is designed to
account for varying amniotic fluid volumes that generally cannot be measured precisely. An L:S ratio of
2.0 typically is achieved by 35 weeks' gestation. Clinically, respiratory distress syndrome (RDS) due to
surfactant deficiency is very unlikely if the L:S ratio is 2.0 or greater, it is indeterminate if the L:S ratio is
1.5 to 2.0, and the incidence of RDS is high if the L:S ratio is less than 1.0. Of note, the L:S ratio can
increase over a period of several days or can be induced by the administration of glucocorticoids to the
mother.
Phosphatidylinositol (PI) is a pulmonary phospholipid that increases throughout gestation, as does
disaturated lecithin. PI concentrations decrease after 35 weeks' gestation. The mere presence of PI or
disaturated lecithin, therefore, does not indicate lung maturity. Phosphatidylglycerol (PG) generally can
be detected in the amniotic fluid at 36 weeks' gestation. Due to its appearance relatively late in
gestation, it has been used as an indicator of pulmonary surfactant maturity.
In pregnancies complicated by maternal diabetes, a mature L:S ratio (>2.0) or the presence of PG
may not indicate pulmonary surfactant maturity. In this setting, fetal lung maturity is determined best
using a quantitative analysis of total surfactant activity. This rapidly determined, automated
fluorescence polarization assay measures all surfactant phospholipids in amniotic fluid referenced as
milligrams of surfactant per gram of albumin present. Recent data suggest that a total surfactant activity
of greater than 45 mg surfactant phospholipids per gram of albumin is as good as, or better than, an L:S
ratio of 2.0 in predicting fetal lung maturity, especially in the presence of maternal diabetes.

References:

Jobe AH. Lung development and maturation. In: Martin RJ, Fanaroff AA, Walsh MC, eds. Fanaroff and
Martin's Neonatal-Perinatal Medicine. 8th ed. Philadelphia, Pa: Mosby Elsevier; 2006:1069-1086

Grenache DG, Gronowski AM. Fetal lung maturity. Clin Biochem. 2006;39:1-10. Abstract available at:
http://www.ncbi.nlm.nih.gov/pubmed/16303123

Winn-McMillan T, Karon BS. Comparison of the TDx-FLM II and lecithin to sphingomyelin ratio assays
in predicting fetal lung maturity. Am J Obstet Gynecol. 2005;193:778-782. Abstract available at:
http://www.ncbi.nlm.nih.gov/pubmed/16150274

page 351
2009 PREP SA on CD-ROM

Question: 99
At the end of the summer, you notice an increase in the number of preparticipation sports examinations
you are performing. You are pleased at the number of your patients who are involved in sports activities
but are reminded that many medical conditions preclude sports participation and must be screened for
during the preparticipation visit.

Of the following, the medical condition that is considered a CONTRAINDICATION for sports
participation is

A. a boy who has chronic leukemia and splenomegaly wishing to play golf

B. a boy who has insulin-dependent diabetes wishing to play tennis

C. a boy who has well-controlled seizures wishing to participate in basketball

D. a girl who has Marfan syndrome wishing to participate in gymnastics

E. a girl who has one ovary wishing to participate in softball

page 352
2009 PREP SA on CD-ROM

Critique: 99 Preferred Response: D


Sports participation can play an important role in the social development of children and adolescents by
teaching leadership and team-building skills and encouraging physical fitness. Most children can be
cleared easily for such participation, but not all sports are safe for children and adolescents, and some
medical conditions warrant special consideration with regard to participation in some sports. In all
cases, the clinician should perform a complete preparticipation sports physical examination and discuss
the implications of the sports the patient plans to pursue.
Sports can be classified by contact level (Item C99A). Children who have splenomegaly, acute
hepatomegaly, and contagious skin lesions should avoid contact sports but can participate in
noncontact sports. Children who have a single kidney should be advised to avoid high-contact sports,
but those who have other single organs, such as ovaries and testes, can be cleared because the risk of
injury is low or protective gear can be worn. Protective eyewear can reduce the risk of injury in the case
of a single functional eye. It is prudent for the clinician to advise such patients of the risk to the
remaining organ during sports participation, and some advocate having the parents and the athlete sign
a document that indicates their understanding and acceptance of such risk.
Sports also may be classified by intensity (dynamic and static demand) (Item C99B). This
classification is especially important for children and adolescents who have cardiovascular diseases.
For example, children who have ventricular dysfunction should avoid high dynamic sports; those who
have significant essential hypertension, left heart obstructive disease, or an increased risk of aortic
dissection should avoid high static sports. Accordingly, a girl who has Marfan syndrome should avoid
gymnastics, a sport that has potentially high static intensity. Several cardiac diseases, such as acute
pericarditis or myocarditis, cardiomyopathy, severe pulmonary hypertension, and right-to-left shunting,
are absolute contraindications to participation in sports. A full listing of these conditions is outlined in the
36th Bethesda Conference guidelines.
Patients who have well-controlled epilepsy can be cleared to participate in most sports, but those
whose seizures are poorly controlled should be advised to avoid sports in which they could sustain
significant injury should a seizure occur during participation, such as swimming, diving, and riflery.
Patients who have diabetes should be encouraged to participate in sports, but they should pay attention
to hydration and insulin therapy, especially with sports that last longer than 30 minutes.

References:

36th Bethesda conference: eligibility recommendations for competitive athletes with cardiovascular
abnormalities. J Am Coll Cardiol. 2005;45:1312-1375. Available at:
http://content.onlinejacc.org/cgi/reprint/45/8/1312

American Academy of Pediatrics Committee on Sports Medicine and Fitness. Medical conditions
affecting sports participation. Pediatrics. 2001;107:1205-1209. Available at:
http://pediatrics.aappublications.org/cgi/content/full/107/5/1205

Metzl JD. Preparticipation examination of the adolescent athlete: part 1. Pediatr Rev. 2001;22:199-204.
Available at: http://pedsinreview.aappublications.org/cgi/content/full/22/6/199

Metzl JD. Preparticipation examination of the adolescent athlete: part 2. Pediatr Rev. 2001;22:227-239.
Available at: http://pedsinreview.aappublications.org/cgi/content/full/22/7/227

Singh A, Silberbach M. Consultation with the specialist: cardiovascular preparticipation sports


screening. Pediatr Rev. 2006;27:418-424. Available at:

page 353
2009 PREP SA on CD-ROM

http://pedsinreview.aappublications.org/cgi/content/full/27/11/418

page 354
2009 PREP SA on CD-ROM

Critique: 99

page 355
2009 PREP SA on CD-ROM

Critique: 99

page 356
2009 PREP SA on CD-ROM

Question: 100
A child presents to a clinic associated with a disaster relief shelter after a hurricane destroys the
community. The mother states that they have run out of the creams prescribed for her daughter’s
eczema before the storm, and the child is itchy. She is concerned because there are some blisters and
crusting in the antecubital fossae and popliteal fossa where the itching is worst. Physical examination
reveals erosions (Item Q100) and erythema surrounding areas of lichenification, with a few vesicles
both in clusters and scattered.

Of the following, the MOST likely pathogen involved in this pattern of infection is

A. herpes simplex virus

B. human papillomavirus

C. Sporothrix sp

D. varicella-zoster virus

E. viridans streptococci

page 357
2009 PREP SA on CD-ROM

Question: 100

Erosions that have a "punched-out" appearance, as described for the child in


the vignette. (Reprinted with permission from Stricker T, Lips U, Sennhauser
FH. Visual diagnosis: an 8-month-old who has an erupting rash. Pediatr Rev.
2007;28:231-234.)

page 358
2009 PREP SA on CD-ROM

Critique: 100 Preferred Response: A


Children who have atopic dermatitis are prone to recurrent skin infections, particularly with
Staphylococcus aureus and herpes simplex virus (HSV), for several reasons. Exacerbations of eczema
disrupt the skin's protective barrier. The failure to produce endogenous antimicrobial peptides has been
offered as a reason for an increase risk for infection with S aureus. Dissemination of cutaneous viral
infections (eg, HSV and molluscum contagiosum) is believed to be due to defects in cytokine production
and T-cell function.
Secondary infection with S aureus may result in "weepy" or crusted erosions (Item C100A). Use of
topical and systemic antistaphylococcal antibiotics, with attention to the emergence of
community-acquired antibiotic-resistant strains, is indicated. Use of topical immune modulators (eg,
tacrolimus) should be interrupted during the infection because of the potential for inhibiting local
immune response and the development of irritation, including burning and stinging.
Eczema herpeticum results when areas of active dermatitis are infected by HSV. Initially, there may
be an increase in complaints of itching and scratching, with a noticeable increase in erythema; fever
may be present. As described for the child in the vignette, clustered, often umbilicated vesicles appear;
these rupture, forming ulcers that have a "punched-out" appearance (Item C100B). Infection may
become disseminated and sometimes is fatal. For those who have extensive involvement,
recommended treatment is parenteral acyclovir.
Sporotrichosis is a dimorphic fungus that can be isolated from soil and plants such as roses; it also
may be found in hay, straw, and decaying vegetation. It may cause infection at a site of minor trauma
that appears as a solitary violaceous nodule (Item C100C). Although infections caused by streptococci
(viridans and enterococci) may be observed in immunocompromised hosts and in neonates, they do not
cause the type of disease described for the child in the vignette. Papules, not vesicles, are
characteristic of human papillomavirus infection of the skin. Primary infection with varicella-zoster virus
results in varicella, with individual, not grouped, vesicles that rupture, leaving shallow erosions, not
"punched-out" ulcers.

References:

American Academy of Pediatrics. Non-group A or B streptococcal and enterococcal infections. In:


Pickering LK, Baker CJ, Long SS, McMillan JA, eds. Red Book: 2006 Report of the Committee on
Infectious Diseases. 27th ed. Elk Grove Village, Ill: American Academy of Pediatrics; 2006:627-629

American Academy of Pediatrics. Sporotrichosis. In: Pickering LK, Baker CJ, Long SS, McMillan JA,
eds. Red Book: 2006 Report of the Committee on Infectious Diseases. 27th ed. Elk Grove Village, Ill:
American Academy of Pediatrics; 2006:595-597

Bunikowski R, Mielke M, Skarabis H, et al. Prevalence and role of serum IgE antibodies to the
Staphylococcus aureus-derived superantigens SEA and SEB in children with atopic dermatitis. J Allergy
Clin Immunol. 1999;103:119-124. Abstract available at: http://www.ncbi.nlm.nih.gov/pubmed/9893195

Horii KA, Simon SD, Liu DY, Sharma V. Atopic dermatitis in children in the United States, 1997-2004:
visit trends, patient and provider characteristics, and prescribing patterns. Pediatrics.
2007;120:e527-e534. Available at: http://pediatrics.aappublications.org/cgi/content/full/120/3/e527

Knoell KA, Greer KE. Atopic dermatitis. Pediatr Rev. 1999;20:46-52. Available at:
http://pedsinreview.aappublications.org/cgi/content/full/20/2/46

page 359
2009 PREP SA on CD-ROM

Stanbury LR. Herpes simplex virus. In: Kliegman RM, Behrman RE, Jenson HB, Stanton BF, eds.
Nelson's Textbook of Pediatrics. Philadelphia, Pa: Saunders Elsevier; 2007:1360-1365

Waggoner-Fountain LA, Grossman LB. Herpes simplex virus. Pediatr Rev. 2004;25:86-93. Available at:
http://pedsinreview.aappublications.org/cgi/content/full/25/3/86

page 360
2009 PREP SA on CD-ROM

Critique: 100

When infected with S aureus, lesions of atopic dermatitis become moist and
crusted. (Courtesy of D. Krowchuk)

page 361
2009 PREP SA on CD-ROM

Critique: 100

In eczema herpeticum, erosions may have a "punched-out" appearance.


(Reprinted with permission from Stricker T, Lips U, Sennhauser FH. Visual
diagnosis: an 8-month-old who has an erupting rash. Pediatr Rev.
2007;28:231-234.)

page 362
2009 PREP SA on CD-ROM

Critique: 100

Sporotrichosis produces erythematous or violaceous nodules at the site(s) of


inoculation. (Courtesy of Peter Somers, MD, PhD, DermAtlas;
www.dermatlas.org)

page 363
2009 PREP SA on CD-ROM

Question: 101
A 4-week-old infant who was born at term without any complications ate well and gained weight for the
first 3 weeks after birth. Over the last week, however, his mother reports that he appears hungry but
fatigues with feeding and now takes twice as long to complete his feeding as he did 1 week ago. He
also breathes fast during his feedings and stops frequently to "catch his breath."

Of the following, the MOST likely explanation for the symptoms in this infant is

A. aspiration syndrome

B. congestive heart failure

C. gastroesophageal reflux disease

D. inborn error of metabolism

E. pneumonia

page 364
2009 PREP SA on CD-ROM

Critique: 101 Preferred Response: B


The symptoms described for the child in the vignette are typical of progressive congestive heart failure
(CHF). The "hunger" described by the mother suggests that the problem results from the infant's
inability to take enough calories for satiation and growth. Such so-called poor feeding is due to his
inability to generate a prolonged suck while maintaining nasal breathing because of the tachypnea that
is caused by pulmonary congestion. Thus, the infant seems to stop sucking and "catch his breath."
Decreased caloric intake coupled with increased caloric expenditure caused by tachypnea and
tachycardia makes it difficult for the infant to gain weight. Indeed, weight loss is common in infants who
have CHF.
CHF is a clinical syndrome that reflects the inability of the myocardium to meet the metabolic
requirements of the body, including those for growth. Congenital heart defects are the most common
reason for pediatric heart failure, and the failure develops most frequently during early infancy. CHF
results from excessive workload imposed on the cardiac muscle that usually is caused by the structural
defects. The cardiac defects may impose an excessive volume load on the left ventricle (eg, large
ventricular septal defect, atrioventricular septal defect), an excessive pressure load on the ventricle (eg,
aortic stenosis), or a combination of volume and pressure load (eg, ventricular septal defect with
coarctation). Less commonly, CHF may be caused by an intrinsic alteration in myocardial performance,
which could result from an inflammatory or an infectious process that directly affects the myocardium
and depresses its contractile function.
Aspiration syndrome typically is an acute event resulting from the passage of gastrointestinal
contents (including food) into the lungs. The symptoms usually are acute and not associated with poor
feeding in spite of hunger. Similarly, gastrointestinal reflux presents more acutely with signs of
discomfort that might include arching during feedings. Pneumonia is associated with other signs of
infection, including tachypnea when at rest, fever, or change in behavior. Inborn errors of metabolism,
although less common, are important to consider in the neonate and infant who is not feeding well.
Most commonly, they are associated with symptoms that are present both at rest and during feeding as
well as other systemic signs.

References:

Balfour I. Management of chronic congestive heart failure in children. Curr Treat Options Cardiovasc
Med. 2004;6:407-416. Abstract available at: http://www.ncbi.nlm.nih.gov/pubmed/15324616

Dreyer WJ, Fisher DJ. Clinical recognition and management of chronic congestive cardiac failure. In:
Garson A Jr, Bricker JT, Fisher DJ, Neish SR, eds. The Science and Practice of Pediatric Cardiology.
2nd ed. Baltimore, Md: Williams & Wilkins, 1998:2309-2325

Silberbach M, Hannon D. Presentation of congenital heart disease in the neonate and young infant.
Pediatr Rev. 2007;28:123-131. Available at:
http://pedsinreview.aappublications.org/cgi/content/full/28/4/123

Talner NS, McGovern JJ, Carboni MP. Congestive heart failure. In: Moller JH, Hoffman JIE, eds.
Pediatric Cardiovascular Medicine. Philadelphia, Pa: Churchill Livingstone; 2000:817-829

page 365
2009 PREP SA on CD-ROM

Question: 102
A 10-year-old boy has double vision and drooping eyelids. On physical examination, he is afebrile and
has normal mentation. Pupillary responses are normal, but he has bilateral ptosis. He cannot fully
adduct his right eye. You note that his ptosis increases with sustained upward gaze (Item Q102).
Bedside forced vital capacity is normal.

Of the following, the test MOST likely to confirm the diagnosis is

A. brainstem auditory evoked potentials

B. cold caloric testing

C. edrophonium test

D. lumbar puncture

E. visual evoked potentials

page 366
2009 PREP SA on CD-ROM

Critique: 102 Preferred Response: C


The child described in the vignette presents with progressive fatigable weakness affecting his ocular
muscles and bilateral eyelids. Fatigable weakness means that at the onset of a voluntary movement, or
after a period of rest, muscle strength is normal. However, after a period of sustained use, muscle
strength diminishes. Such a pattern is characteristic of weakness due to pathology at the
neuromuscular junction.
New-onset weakness mandates urgent evaluation and consideration of problems involving airway
protection and respiratory insufficiency, particularly if there is evidence on physical examination of
generalized weakness. If the pharyngeal muscles are affected, speech sounds hyponasal. Negative
inspiratory force or forced vital capacity should be documented promptly because these findings
determine whether the child should be transferred to an intensive care unit for ventilatory monitoring
and support.
The first step in the diagnostic process is to localize the problem to the proper level of the nervous
system: brain, brainstem/cerebellum, spinal cord, nerve, junction, or muscle. Such a determination not
only allows for urgent appropriate diagnosis and treatment but can reduce unnecessary discomfort,
risks, and costs of inappropriate diagnostic testing. Consultation with a neurologist can be useful before
ordering testing. In this case, bilateral weakness in the eyelids and normal mentation make a cerebral
or midbrain cause unlikely. Lateral gaze palsy from a pontine lesion also should affect the full face (as
occurs in Bell palsy), which is not present. A multifocal cranial neuropathy is rare and unlikely. The
ocular fatigability suggests a neuromuscular junction problem such as myasthenia gravis.
The diagnosis of myasthenia gravis can be confirmed with the edrophonium test ("Tensilon® test"), a
specialized assessment using the acetylcholinesterase inhibitor edrophonium. Because the test can
pose a risk of life-threatening bradycardia, it should be undertaken by an experienced physician with
atropine at the bedside. Neurologic examination before and after administration ideally should be
documented on video (Item C102).
Lumbar puncture is the appropriate test for suspected Guillain-Barré syndrome (GBS), an acute
inflammatory demyelinating polyneuropathy. Typically, affected children present with weakness
beginning in the proximal legs, pain, and absent reflexes. A variant of GBS can involve predominantly
the brainstem and cerebellum, but usually weakness is more extensive than described for this child,
and the weakness is not fatigable. Further, at this early stage, the characteristic cerebrospinal fluid
findings in GBS, normal cells and high protein, often have not emerged. Therefore, lumbar puncture is
not the preferred study in this setting. Cold caloric testing evaluates vestibular function. This patient has
no nystagmus and does not require this uncomfortable test. Neurophysiologic testing with auditory or
visual evoked potentials is not needed when no clinical evidence suggests that these systems are
affected.
Treatment of myasthenia gravis is complex and is managed best by an experienced neurologist.
Treatment may include immunomodulation (eg, intravenous immunoglobulin or plasmapheresis) and
acetylcholinesterase inhibitors. Imaging to assess for the presence of a thymoma also is standard
practice in a child. The patient and family also should be educated about disease management and
early recognition of impending myasthenic crisis.

References:

Andrews PI. Autoimmune myasthenia gravis in childhood. Semin Neurol. 2004;24:101-110. Abstract
available at: http://www.ncbi.nlm.nih.gov/pubmed/15229797

Mehta S. Neuromuscular disease causing acute respiratory failure. Respir Care. 2006;51:1016-1023.
Available at: http://www.rcjournal.com/contents/09.06/09.06.1016.pdf

page 367
2009 PREP SA on CD-ROM

Parr JR, Jayawant S. Childhood myasthenia: clinical subtypes and practical management. Dev Med
Child Neurol. 2007;49:629-635. Available at:
http://www.blackwell-synergy.com/doi/abs/10.1111/j.1469-8749.2007.00629.x

Sarnat HB. Disorders of neuromuscular transmission and of motor neurons. In: Kliegman RM, Behrman
RE, Jenson HB, Stanton BF, eds. Nelson Textbook of Pediatrics. 18th ed. Philadelphia, Pa: Saunders
Elsevier; 2007:2554-2558

page 368
2009 PREP SA on CD-ROM

Question: 103
You are called to the newborn nursery to evaluate an infant who has a limb anomaly. The infant is
normally grown and vigorous. On physical examination, you note a terminal transverse limb defect at
the distal aspect of the right forearm, resulting in absence of the hand on that side (Item Q103).

Of the following, these findings are MOST likely related to prenatal exposure to

A. alcohol

B. cocaine

C. marijuana

D. methamphetamine

E. tobacco

page 369
2009 PREP SA on CD-ROM

Question: 103

Terminal transverse limb defect, as exhibited by the infant described in the


vignette. (Courtesy of V. Shashi)

page 370
2009 PREP SA on CD-ROM

Critique: 103 Preferred Response: B


Despite extensive medical literature on the teratogenic effects of cocaine, the degree to which maternal
cocaine use affects the developing embryo/fetus remains largely uncertain. Many studies on this
subject are fraught with design flaws, and a bias toward publishing adverse outcomes has been
suggested. Adding to this confusion is the fact that women who abuse cocaine during pregnancy
typically do not use the drug in isolation; they often drink alcohol, smoke cigarettes, and use other
recreational drugs as well. Even so, prenatal cocaine exposure generally is believed to be associated
with vascular disruptive events in the embryo/fetus due to its potent vasoconstrictive and hypertensive
effects. Such events include cerebral infarction, urogenital anomalies, and limb reduction defects, as
described for the infant in the vignette (Item C103A). In addition, the incidence of placental abruption
appears to be increased in cocaine-exposed pregnancies. Although neurobehavioral alterations have
been reported in neonates who were exposed to cocaine prenatally, it is not clear that these are due
solely to cocaine exposure.
Prenatal alcohol exposure is associated with a spectrum of abnormalities termed fetal alcohol
spectrum disorder. Individuals who have fetal alcohol syndrome, which is the most severe end of the
spectrum, typically exhibit intrauterine growth restriction, microcephaly, neurodevelopmental
impairment, and a characteristic pattern of dysmorphic features (Item C103B). Although they may have
hypoplasia of the distal fingers and toes, they typically do not have more severe limb deficiencies.
The effects of cigarette smoking on pregnancy outcome have been, and continue to be, studied
extensively. Cigarette smoking is associated with an increased risk for miscarriage, reduced fetal
growth, and abnormal placentation. Generally, cigarette smoking is not associated with major
congenital anomalies, although there may be an increase in facial clefting. An association with
congenital limb deficiencies has been reported in some studies, but it has not been borne out in others.
No increased incidence of birth defects appears to be associated with marijuana or
methamphetamine use during pregnancy.

References:

Cigarette smoking, methamphetamine, alcohol, cocaine, marijuana. Reprotox. Available for subscription
at: http://www.reprotox.org

Cigarette smoking, methamphetamine, alcohol, cocaine, marijuana. Teris. Available for subscription at
http://depts.washington.edu/terisweb/teris/

Hoyme HE, May PA, Kalberg WO, et al. A practical clinical approach to diagnosis of fetal alcohol
spectrum disorders: clarification of the 1996 Institute of Medicine criteria. Pediatrics. 2005;115:39-47.
Available at: http://pediatrics.aappublications.org/cgi/content/full/115/1/39

page 371
2009 PREP SA on CD-ROM

Critique: 103

Limb reduction defects may result from vascular disruptive events. (Courtesy
of V. Shashi)

page 372
2009 PREP SA on CD-ROM

Critique: 103

Characteristics of fetal alcohol syndrome include midface hypoplasia, a


relatively smooth philtrum, and a narrow upper lip with a poorly defined
"cupid's bow." (Courtesy of M. Rimsza)

page 373
2009 PREP SA on CD-ROM

Question: 104
A 16-year-old girl who attends boarding school in your community comes to your office because she is
feeling depressed. You see her alone for the visit, and she relates that she feels suicidal at this time
and has a plan to kill herself.

Of the following, the BEST description of your obligation to alert her parents to her situation is that

A. no parental notification is necessary because she is a mature minor

B. no parental notification is necessary because she is an emancipated minor

C. parental notification is necessary due to billing issues

D. parental notification is necessary due to her serious threats of self-harm

E. parental notification is prohibited by the Health Insurance Portability and Accountability Act

page 374
2009 PREP SA on CD-ROM

Critique: 104 Preferred Response: D


Confidentiality and consent issues related to adolescents are often complex and problematic.
Confidentiality is defined as an agreement between the patient and clinician that medical information
will not be shared without the patient's explicit permission. For the girl described in the vignette,
confidentiality must be breached with parental notification due to the serious nature of her threat to
herself. Such breaching of confidentiality also would be necessary if she posed a threat to others.
Parental notification is not necessary for billing purposes. However, confidentiality may be breached
inadvertently by billing procedures, such as parents receiving an explanation of benefits document. The
physician should explain such possible breaches and work with the adolescent to avoid them.
Informed consent has the vital components of being given voluntarily, the patient having all of the
information necessary to make an informed decision, and the individual being capable of making a
decision based on the information provided. All states require parental consent for medical care for
minors, with some notable exceptions, including emergency care; care for the "emancipated minor";
and care related to certain conditions such as pregnancy, sexually transmitted infections, contraception,
substance abuse, and mental illness. The exceptions vary by state. An emancipated minor is a legal
status existing for minors who are living apart from parents and are financially independent, but this
designation varies among states, and some states do not have this legal designation. Marriage,
parenthood, military service, and living independently are common criteria for the emancipated status.
Generally, this status allows a minor to consent for all health care. A "mature minor" is one who is
deemed by a physician as capable of giving the same degree of consent for treatment as an adult,
provided the treatment is low-risk. The mature minor rule may be used in states where no state law
regarding minor consent exists. The girl described in the vignette is not financially independent from her
parents and, as a result, cannot be considered emancipated. Because her condition and treatment are
not low-risk, the mature minor principle would not apply.
The Health Insurance Portability and Accountability Act (HIPAA) of 1996 protects the confidentiality
of adolescents who are considered minors under some circumstances. Parents and guardians have
control over health information access for nonemancipated minors, except in situations in which minors
are able to consent to their own health care. HIPAA defers to state laws that allow or prohibit disclosure
of confidential information to parents. HIPAA allows a physician to disclose information to a parent if a
state law requires the physician to do so. If the state law permits, but does not require, disclosure to a
parent, HIPAA allows the physician discretion to disclose. If state law prohibits the disclosure of
information to a parent, disclosure must not be made without the minor's permission. If there is no state
law in place, the physician has discretion to disclose or not to a parent. For the girl described in the
vignette, disclosure is necessary due to the health threat, not by any HIPAA regulation.
The Center for Adolescent Health and the Law has published a compendium of state laws that
addresses confidentiality and consent and allows clinicians to determine the specifics of the law in their
own state of practice.

References:

English A, Kenney KE. State Minor Consent Laws: A Summary. 2nd ed. Chapel Hill, NC: Center for
Adolescent Health & the Law; 2003

Joffe A. Legal and ethical issues in adolescent health care. In: Osborn LM, DeWitt TG, First LR, Zenel
JA eds. Pediatrics. Philadelphia, Pa: Elsevier Mosby; 2005:1428-1430.

Weddle M, Kokotailo P. Adolescent substance abuse: confidentiality and consent. Pediatr Clin North
Am. 2002;49:301-315. Abstract available at: http://www.ncbi.nlm.nih.gov/pubmed/11993284

page 375
2009 PREP SA on CD-ROM

Weddle M, Kokotailo PK. Confidentiality and consent in adolescent substance abuse: an update. Virtual
Mentor: American Medical Association Journal of Ethics. 2005;7(3). Available at:
http://virtualmentor.ama-assn.org/2005/03/pfor1-0503.html

page 376
2009 PREP SA on CD-ROM

Question: 105
You are assisting a pediatric resident in evaluating a 12-year-old girl who has type 1 diabetes and has
been vomiting for the past 12 hours. Initial laboratory results include:
· Blood glucose, 630.0 mg/dL (35.0 mmol/L)
· Serum sodium, 150.0 mEg/L (150.0 mmol/L)
· Serum potassium, 6.0 mEq/L (6.0 mmol/L)
· Serum chloride, 90.0 mEq/L (90.0 mmol/L)
· Serum bicarbonate, 10.0 mEq/L (10.0 mmol/L)
The anion gap for this child is CLOSEST to

A. 4

B. 10

C. 50

D. 80

E. 323

page 377
2009 PREP SA on CD-ROM

Critique: 105 Preferred Response: C


The anion gap is the difference between routinely measured cations and anions in the blood. Clinically,
this is estimated by using the following equation: [Na+]-[Cl-]-[HCO3-]. Thus, the anion gap for the child
described in the vignette is 150-90-10=50. A normal anion gap is 8 to 16 and reflects unmeasured
anions such as proteins, sulfates, phosphates, and organic acids. Calculation of the anion gap assists
the clinician in both classifying metabolic acidosis as normal anion gap and elevated anion gap and
directing further evaluation and treatment.
Metabolic acidosis with an increased anion gap occurs when bicarbonate is lost and replaced by
unmeasured anions such as ketones, lactate, phosphate, or urate. Common causes of increased anion
gap acidosis in children include ketoacidosis, as seen in diabetes mellitus or starvation; lactic acidosis,
as seen in shock or with inborn errors of metabolism; and renal failure. Increased anion gap acidosis
also is seen with the accumulation of toxic metabolites from ingestions such as methanol, ethylene
glycol, and salicylate.
Normal anion gap metabolic acidosis is characterized by loss of bicarbonate with an equal increase
in chloride that preserves the normal anion difference. Common causes of normal anion gap acidosis in
children include diarrhea, renal tubular acidosis, administration of large volumes of normal saline, and
hypoaldosteronism.

References:

Greenbaum LA. Electrolyte and acid-base disorders. In; Kliegman RM, Behrman RE, Jenson HB,
Stanton BF, eds. Nelson Textbook of Pediatrics. 18th ed. Philadelphia, Pa: Saunders Elsevier;
2007:267-308

Schwaderer AL, Schwartz GJ. Back to basics: acidosis and alkalosis. Pediatr Rev. 2004;25:350-357.
Available at: http://pedsinreview.aappublications.org/cgi/content/full/25/10/350

page 378
2009 PREP SA on CD-ROM

Question: 106
The parents of a 12-year-old girl in whom you recently diagnosed type 1 diabetes mellitus ask you
about potential long-term complications. In your discussion, you stress the importance of blood glucose
control to prevent complications and review risk factors for diabetes complications, including
hyperglycemia and tobacco smoking.

Of the following, the MOST important additional risk factor for diabetes complications is

A. celiac disease

B. hypertension

C. hypothyroidism

D. lack of regular exercise

E. undernutrition

page 379
2009 PREP SA on CD-ROM

Critique: 106 Preferred Response: B


The Diabetes Control and Complications Trial (DCCT) results, published in 1994, demonstrated
unambiguously that glycemic control directly correlates with the long-term prevention of complications
of diabetes mellitus type 1 (DM1). However, additional risk factors, such as hypertension and cigarette
smoking, are almost as important as hyperglycemia in the development of diabetes complications.
Celiac disease occurs in approximately 6% of individuals who have DM1 in North America and may
hamper diabetes control because of malabsorption of nutrients. It is also a risk factor for poor bone
mineralization in individuals who have DM1. However, it does not alter the risk of long-term
complications directly. Hypothyroidism due to chronic lymphocytic thyroiditis may develop in 5% or
more of people who have DM1, but it is not an independent risk factor for cardiovascular or other DM1
complications unless it is chronically untreated and affects lipid metabolism. Lack of regular exercise
has been associated with an increase in cardiovascular risk factors for children who have DM1, but it
has not yet been correlated directly with the development of long-term complications. Undernutrition,
unless it is the result of poor diabetes control, has not been correlated with long-term complications of
diabetes.

References:

Freemark M, Levitsky LL. Screening for celiac disease in children with type 1 diabetes: two views of the
controversy. Diabetes Care. 2003;26:1932-1939. Available at:
http://care.diabetesjournals.org/cgi/content/full/26/6/1932

Gallego PH, Wiltshire E, Donaghue KC. Identifying children at particular risk of long-term diabetes
complications. Pediatr Diabetes. 2007;8(suppl 6):40-48. Abstract available at:
http://www.ncbi.nlm.nih.gov/pubmed/17727384

Glastras SJ, Mohsin F, Donaghue KC. Complications of diabetes mellitus in childhood. Pediatr Clin
North Am. 2005;52:1735-1753. Abstract available at: http://www.ncbi.nlm.nih.gov/pubmed/16301091

Herbst A, Kordonouri O, Schwab KO, Schmidt F, Holl RW, on behalf of the DPV Initiative of the German
Working Group for Pediatric Diabetology Germany. Impact of physical activity on cardiovascular risk
factors in children with type 1 diabetes. Diabetes Care. 2007;30:2098-2100. Available at:
http://care.diabetesjournals.org/cgi/content/full/30/8/2098

Levitsky LL, Misra M. Complications and screening in children and adolescents with type 1 diabetes
mellitus. UpToDate Online 15.3. 2008. Available for subscription at:
http://www.uptodateonline.com/utd/content/topic.do?topicKey=pediendo/17677

Raile K, Galler A, Hofer S, et al. Diabetic nephropathy in 27,805 children, adolescents, and adults with
type 1 diabetes: effect of diabetes duration, A1C, hypertension, dyslipidemia, diabetes onset, and sex.
Diabetes Care. 2007;30:2523-2528. Available at:
http://care.diabetesjournals.org/cgi/content/full/30/10/2523

page 380
2009 PREP SA on CD-ROM

Question: 107
A 12-year-old boy recently took a standardized achievement test at school. His score dropped from 105
on last year’s achievement test to 95 on the most recent test. Last season the boy played hockey and
fell down, hitting his head, although he did not lose consciousness. He had no previous head injury. He
was evaluated in the emergency department and had normal findings on computed tomography scan.
The mother asks whether the boy had suffered brain injury due to his fall that caused him to lose
academic skills.

Of the following, the MOST appropriate response is to

A. explain that it is normal to have a small variation between scores

B. recommend neurorehabilitation due to loss of his academic skills

C. restrict the child’s contact sports activity for the next season

D. send the child for comprehensive neuropsychological testing

E. send the child for head magnetic resonance imaging

page 381
2009 PREP SA on CD-ROM

Critique: 107 Preferred Response: A


The small difference in scores on standardized achievement tests described for the boy in the vignette
is normal. To be significant, the change would need to be greater than 1 standard deviation (15 points).
Because the values reported for the boy are within that expected for test-retest variation, brain magnetic
resonance imaging, contact sports restriction, neuropsychological testing, and neurorehabilitation are
not needed.
Standardized scores measure performance for a group of students, such as all the students in one
school, at a certain grade, or in a single school district. Standardized scores employ a procedure in
which scores are reported in terms of standard deviation units from the mean. When scores are
standardized to the normal bell curve, two thirds of the students in the sample fall between -1 and +1
standard deviations from the mean. Scores also may be expressed as percentiles or, in some cases,
grade equivalents. Standardized scores are less useful for evaluating change in an individual student. A
single student may be learning more every day and growing in ability or skill, but still be at the same
relative rank compared with his or her peers. Achievement tests measure what a child has learned in
specific subject areas such as reading or mathematics. Criterion-based testing evaluates how much a
student knows or can do using a specific set of standards, not by comparison with other students, which
probably makes it more useful for identifying whether a student is improving or meeting objectives. A
child who has a significant discrepancy (determined by each state) between an expected achievement
score based on an intelligence quotient test and the actual score may have a specific learning disability
and be eligible for specialized educational services.

References:

Ewen JB, Shapiro BK. Specific learning disabilities. In: Accardo PJ. Capute & Accardo's
Neurodevelopmental Disabilities in Infancy and Childhood. Volume II: The Spectrum of
Neurodevelomental Disabilities. 3rd ed. Baltimore, Md: Paul H. Brookes Publishing Co; 2008:553-577

Mahone EM. Psychological assessment. In: Accardo PJ, ed. Capute & Accardo's Neurodevelopmental
Disabilities in Infancy and Childhood. Volume II: The Spectrum of Neurodevelopmental Disabilities. 3rd
ed. Baltimore, MD: Paul H. Brookes Publishing Co; 2008:261-281

page 382
2009 PREP SA on CD-ROM

Question: 108
The microbiology laboratory called your junior partner today to tell her that the blood culture from a
patient she admitted 2 days ago is growing Haemophilus influenzae type b. Because she has never
treated an infection caused by this organism, she wants to know what antimicrobial agent would be best
to use for her patient.

Of the following, the MOST appropriate antimicrobial agent to treat this infection is

A. ampicillin

B. cefotaxime

C. clindamyin

D. gentamicin

E. vancomycin

page 383
2009 PREP SA on CD-ROM

Critique: 108 Preferred Response: B


Haemophilus influenzae type b (Hib) is a pleomorphic gram-negative coccobacillus. These types of
organisms are not susceptible to antimicrobial agents used against gram-positive pathogens, such as
clindamycin or vancomycin, or some agents that are effective against other gram-negative organisms,
such as the aminoglycosides (eg, gentamicin). Because approximately 30% to 40% of Hib isolates
produce beta-lactamase, ampicillin should not be used empirically until susceptibility testing supports its
potential efficacy. For patients who have potentially life-threatening illness (eg, bacteremia) from Hib,
cefotaxime or ceftriaxone has become the antimicrobial agent of choice, with meropenem or
chloramphenicol serving as alternative agents.

References:

American Academy of Pediatrics. Haemophilus influenzae infections. In: Pickering LK, Baker CJ, Long
SS, McMillan JA, eds. Red Book: 2006 Report of the Committee on Infectious Diseases. 27th ed. Elk
Grove Village, Ill: American Academy of Pediatrics; 2006:310-318

Tzanakaki G, Mastrantonio P. Aetiology of bacterial meningitis and resistance to antibiotics of causative


pathogens in Europe and in the Mediterranean region. Int J Antimicrob Agents. 2007;29:621-629.
Abstract available at: http://www.ncbi.nlm.nih.gov/pubmed/17368858

page 384
2009 PREP SA on CD-ROM

Question: 109
A 9-year-old previously healthy boy presents for evaluation of a progressively worsening cellulitis of his
left leg. Two days ago, he sustained an abrasion to his shin after falling off his bicycle onto a gravel
road. Over the last 12 hours, he has developed a temperature of 102.0°F (38.9°C), and the wound has
become very erythematous, swollen, and tender, with some red streaking. On physical examination, the
boy has a temperature of 101.5°F (38.6°C) and a 5x6-cm abrasion of the anterior lateral surface of his
left shin that is draining a serosanguineous discharge. The abrasion is surrounded by an 8-cm area of
erythema, swelling, and induration, with a red streak extending up toward his knee. The area is tender
to palpation, and he limps when walking. There is some shotty left inguinal adenopathy. A complete
blood count shows a peripheral white blood cell count of 16.0x103/mcL (16.0x109/L) with a differential
count of 65% neutrophils, 5% band forms, 25% lymphocytes, and 5% monocytes.

Of the following, the MOST likely pathogen causing this patient’s condition is

A. Staphylococcus aureus (methicillin-resistant)

B. Staphylococcus aureus (methicillin-sensitive)

C. Staphylococcus epidermidis

D. Streptococcus pyogenes

E. Streptococcus pneumoniae

page 385
2009 PREP SA on CD-ROM

Critique: 109 Preferred Response: D


Streptococcus pyogenes (group A Streptococcus) is a ubiquitous organism that is the most common
bacterial cause of pharyngitis and can be the source of a variety of other cutaneous and systemic
infections. Streptococcal cellulitis is an acute, spreading inflammation of the skin and subcutaneous
tissues. Clinical findings include local pain, tenderness, swelling, and erythema that progresses rapidly
and may involve large areas of skin. Systemic manifestations include fever, chills, and malaise with
associated lymphangitis (red, tender, linear streaks directed toward enlarged, tender regional lymph
nodes) or bacteremia. The patient described in the vignette has the classic clinical picture for a cellulitis
caused by group A Streptococcus.
Streptococcal pharyngitis primarily occurs among children 5 to 15 years of age, with the peak
incidence seen during the first few years of school. However, all age groups are susceptible to infection.
Symptoms consist of the abrupt onset of severe sore throat, malaise, fever, headache, nausea,
vomiting, and abdominal pain. Coryza, cough, and conjunctivitis are absent. Prominent physical
findings include: erythema and edema of the posterior pharynx, with enlarged hyperemic tonsils; patchy
discrete tonsillopharyngeal exudates (Item C109A); and tender, enlarged, submandibular lymph nodes.
The cutaneous clinical manifestations of a group A streptococcal infection include scarlet fever,
impetigo, erysipelas, streptoccocal cellulitis, and necrotizing fasciitis. Scarlet fever results from infection
with a streptococcal strain that produces streptococcal pyrogenic exotoxins. This disease usually is
associated with pharyngeal infections but also may follow streptococcal infections at other sites. Clinical
findings include a diffuse erythematous blush that blanches with pressure and spares the face, palms,
and soles. Skin folds in the neck, axillae, groin, elbows, and knees appear as lines of deeper red
(Pastia lines) (Item C109B). The skin has a sandpaper texture (Item C109C), and there is usually
circumoral pallor.
Impetigo (pyoderma) consists of discrete purulent lesions representing primary infection of the skin.
The lesion begins as a papule that rapidly evolves into a vesicle surrounded by an area of erythema.
The vesicle gives rise to pustules that enlarge and subsequently break down to form thick crusts (Item
C109D). The lesions heal slowly and may leave areas of hypopigmentation.
Erysipelas is a superficial cutaneous process that has prominent lymphatic involvement. Its
distinguishing features are lesions that are raised and have a clear line of demarcation between
involved and uninvolved tissue. The cutaneous lesion begins as a localized area of erythema and
swelling and subsequently spreads rapidly, with advancing red margins that are raised and
well-demarcated from adjacent normal tissue. There is marked edema, often with bleb formation (Item
C109E).
Necrotizing fasciitis is an infection of the deeper subcutaneous tissues and fascia characterized by
extensive and rapidly spreading necrosis and gangrene of the skin and underlying tissue. Typically,
necrotizing fasciitis begins at a site of minor or inapparent trauma. The initial lesion is an area of mild
erythema that rapidly evolves to severe extensive inflammation; the skin becomes dusky and purplish,
and bullae containing yellow or hemorrhagic fluid appear (Item C109F). Systemically, patients are ill
and have a high temperature and toxic appearance.
The systemic clinical manifestations of group A streptococcal infection include toxic shock syndrome
(TSS), pneumonia, septic arthritis and osteomyelitis, and meningitis. TSS is defined as any
streptococcal infection associated with the sudden onset of shock and organ failure. The first phase of
TSS begins with an influenza-like prodrome characterized by fever, chills, myalgias, nausea, vomiting,
and diarrhea that precedes the development of hypotension by 24 to 48 hours. A defined or superficial
portal of entry or evidence of infection also may be present during this stage. Phase 2 of TSS is
characterized by tachycardia, tachypnea, persistent fever, and if present, increasingly severe pain at a
site of infection. Phase 3 of TSS is characterized by the previously noted symptoms accompanied by
the sudden onset of shock and organ failure. Pneumonia due to group A Streptococcus frequently is

page 386
2009 PREP SA on CD-ROM

associated with a preceding viral infection. The onset of disease usually is abrupt and is characterized
by fever, dyspnea, cough, and pleuritic chest pain. Chest radiography shows a consolidative lobar
pneumonia that has moderate-to-large pleural effusion or empyema. Meningitis due to group A
Streptococcus is relatively rare and typically follows an upper respiratory tract infection (otitis and
sinusitis) or neurosurgical conditions.
Staphylococcus epidermidis does not cause cellulitis. S pneumoniae is a cause of preseptal cellulitis
but is not associated with other forms of cellulitis. The cellulitis caused by S aureus (methicillin-sensitive
or -resistant) is not associated with lymphangitis, and progression is not as rapid as with group A
streptococcal cellulitis.

References:

Bisno AL, Stevens DL. Streptococcal infections of skin and soft tissues. N Engl J Med.
1996;334:240-246. Extract available at: http://content.nejm.org/cgi/content/extract/334/4/240

Bisno AL, Stevens DL. Streptococcus pyogenes. In: Mandell GL, Bennett JE, Dolin R, eds. Mandell,
Douglas, and Bennett's Principles and Practice of Infectious Diseases. 6th ed. Philadelphia, Pa:
Elsevier Churchill Livingstone; 2005:2362-2379

Sellers BJ, Woods ML, Morris SE, Saffle JR. Necrotizing group A streptococcal infections associated
with streptococcal toxic shock syndrome. Am J Surg. 1996;172:523-528. Abstract available at:
http://www.ncbi.nlm.nih.gov/pubmed/8942557

Stevens DL. Dilemmas in the treatment of invasive Streptococcus pyogenes infections. Clin Infect Dis.
2003;37:341-343. Available at: http://www.journals.uchicago.edu/doi/full/10.1086/376652

Stevens DL. Streptococcal toxic-shock syndrome: spectrum of disease, pathogenesis, and new
concepts in treatment. Emerg Infect Dis. 1995;1:69-78. Abstract available at:
http://www.ncbi.nlm.nih.gov/pubmed/8903167

page 387
2009 PREP SA on CD-ROM

Critique: 109

Pharyngitis caused by Streptococcus pyogenes is characterized by enlarged


tonsils and exudates. (Courtesy of M. Rimsza)

page 388
2009 PREP SA on CD-ROM

Critique: 109

In patients who have scarlet fever, erythema or petechiae may be


concentrated in skin folds in the antecubital fossae (ie, Pastia lines).
(Courtesy of D. Krowchuk)

page 389
2009 PREP SA on CD-ROM

Critique: 109

The rash of scarlet fever is comprised of fine erythematous papules that have
a rough or "sandpaper" feel. (Courtesy of D. Krowchuk)

page 390
2009 PREP SA on CD-ROM

Critique: 109

In nonbullous impetigo, vesicles rupture and serous fluid dries, leaving a


yellow or "honey-colored" crust. (Courtesy of D. Krowchuk)

page 391
2009 PREP SA on CD-ROM

Critique: 109

Erysipelas represents a form of cellulititis characterized by a well-defined and


often elevated border and vesicle formation. (Courtesy of Red Book® Online)

page 392
2009 PREP SA on CD-ROM

Critique: 109

In necrotizing fasciitis, affected areas of the skin become dusky or violaceous


and bullae appear. (Courtesy of Bernard Cohen, MD, DermAtlas;
www.dermatlas.org)

page 393
2009 PREP SA on CD-ROM

Question: 110
A 6-year-old boy presents to an urgent care center with the complaint of bright red blood and clots in
the urine. There is no history of trauma, and the boy has no dysuria, frequency, urgency, abdominal
pain, or back pain. On physical examination, his temperature is 98.6°F (37°C), heart rate is 76
beats/min, respiratory rate is 14 breaths/min, and blood pressure is 110/68 mm Hg. He has no
abdominal tenderness, flank tenderness, or edema. Urinalysis reveals a specific gravity of 1.025, pH of
6.5, 3+ blood, trace protein, and negative leukocyte esterase and nitrite. Microscopy shows more than
100 red blood cells/high-power field (HPF), less than 5 white blood cells/HPF, and no casts. Electrolyte
values are normal, blood urea nitrogen is 14.0 mg/dL (5.0 mmol/L), and creatinine is 0.5 mg/dL (44.2
mcmol/L).

Of the following, the MOST appropriate next test is

A. antinuclear antibody titer

B. complement component 3 (C3) measurement

C. computed tomography scan of the abdomen

D. renal biopsy

E. renal/bladder ultrasonography

page 394
2009 PREP SA on CD-ROM

Critique: 110 Preferred Response: E


A good first step in the evaluation of gross hematuria in a child who is otherwise well is to characterize
the urine color to aid in localizing the lesion to a particular anatomic region. Patients who have cola- or
tea-colored urine often have painless hematuria without clots, as is seen classically in
glomerulonephritis. Bright red urine (often with clots), as described for the boy in the vignette, may have
an underlying nonglomerular renal cause, a lower urinary tract cause (ureter or bladder), or rarely be
due to a hematologic disorder.
Patients who have bright red urine with clots need to be evaluated for possible structural causes for
hematuria. Such nonglomerular renal causes include a ruptured cyst in cases of cystic kidney disease
(as in autosomal dominant and recessive forms of polycystic kidney disease), a renal mass (Wilms
tumor) (Item C110), or a renal stone (nephrolithiasis or urolithiasis). Additional causes include renal
vein thrombosis, papillary necrosis, or hypercalciuria. Hematologic disorders such as sickle trait/disease
or a bleeding disorder (von Willebrand disease) can result in a similar clinical picture and should be
considered in the investigation. Lower urinary tract causes include hemorrhagic cystitis, bladder calculi,
or rarely, a tumor of the lower urinary tract (hemangioma of the bladder or rhabdomyosarcoma of the
bladder). The evaluation for possible cysts, stones, or tumors in all children who have gross hematuria
involves renal/bladder ultrasonography to look for a structural abnormality. In rare cases, additional
testing is required, such as an abdominal computed tomography (CT) scan to look for renal calculi if the
clinical suspicion is high and the ultrasonography result is negative. However, routine use of the CT
scan is strongly discouraged due to long-term concerns about the ionizing radiation exposure
(estimated to be equivalent to the radiation dose of 250 two-view chest radiographs).
Patients who have cola-colored urine require monitoring of blood pressure and renal function while
looking for an underlying cause for the suspected glomerulonephritis. The best single diagnostic test in
the evaluation of acute glomerulonephritis is measurement of complement component 3 (C3).
Additional testing usually performed at the outset includes measurement of C4, antinuclear antibody,
antidouble-stranded DNA antibody, and serum albumin. These tests help the clinician to categorize the
glomerulonephritis into hypocomplementemic or normocomplementemic types and narrow the
differential diagnosis accordingly. A renal biopsy usually is considered in patients who have acute
glomerulonephritis accompanied by a rising serum creatinine value or in cases of
normocomplementemic glomerulonephritis or suspected lupus nephritis. Because the patient in the
vignette is unlikely to have glomerulonephritis, he does not require testing for C3 or ANA or a renal
biopsy.

References:

Brody AS, Frush DP, Huda W, Brent RL, and the Section on Radiology. Radiation risk to children from
computed tomography. Pediatrics. 2007;120:677-682. Available at:
http://pediatrics.aappublications.org/cgi/content/full/120/3/677

Moxey-Mims M. Hematuria and proteinuria. In: Kher KK, Schnaper HW, Makker SP, eds. Clinical
Pediatric Nephrology. 2nd ed. London, England: Informa Healthcare; 2007:129-141

Pan CG. Evaluation of gross hematuria. Pediatr Clin North Am. 2006;53:401-412. Abstract available at:
http://www.ncbi.nlm.nih.gov/pubmed/16716787

page 395
2009 PREP SA on CD-ROM

Critique: 110

Wilms tumor: A transverse image from a computed tomography scan


demonstrates a normal right kidney and a very large tumor originating from
the displaced left kidney. (Courtesy of D. Mulvihill)

page 396
2009 PREP SA on CD-ROM

Question: 111
A 10-year-old boy presents with a 2-month history of chronic cough. His parents are unsure of a specific
preceding trigger. They are concerned because the school nurse has called on multiple occasions
requesting that the boy be taken home due to his persistent cough. The boy denies any chest pain,
dyspnea, or syncope. Use of a sedating antihistamine and over-the-counter cold and cough liquid has
not alleviated his symptoms. On physical examination, the boy has vital signs within the normal range
and appears healthy. A thorough examination reveals no abnormalities. During the encounter, the boy
repeatedly exhibits a harsh, "barky" cough that resolves when you leave the examination room, only to
recur when you return. You suspect he has a psychogenic cough.

Of the following, the MOST accurate statement regarding psychogenic cough is that

A. most cases are associated with underlying psychological illness

B. most cases are preceded by an upper respiratory tract infection

C. resolution of the cough often is followed by recurrent wheezing

D. symptoms persist during the day and while the child is asleep

E. the cough noise often is dramatically different from the postnasal drip syndrome cough

page 397
2009 PREP SA on CD-ROM

Critique: 111 Preferred Response: B


Psychogenic cough, sometimes called habit-cough syndrome, is an uncommon cause of chronic cough
in children and adolescents. Many experts discourage using the label "psychogenic" because
underlying psychological or psychosomatic concerns are uncommon in affected children. Most cases of
psychogenic cough begin with an uncomplicated upper respiratory tract viral illness, but the cough
lingers for months to years. The classic triad of symptoms includes a repetitive cough, cessation during
sleep, and the characteristic hacking, barky cough, as described for the boy in the vignette. Although
most affected children undergo multiple therapeutic interventions and diagnostic tests, the characteristic
presentation may allow clinicians to avoid expensive or invasive tests to rule out other causes.
Successful therapies for habit-cough include behavioral therapy, suggestion therapy, and
self-hypnosis. Once resolved, most cases of habit-cough do not recur. Unlike some cases of viral upper
respiratory tract infections, habit-cough resolution is not followed by recurrent wheezing.

References:

Linz AJ. The relationship between psychogenic cough and the diagnosis and misdiagnosis of asthma: a
review. J Asthma. 2007;44:347-355. Abstract available at:
http://www.ncbi.nlm.nih.gov/pubmed/17613629

Weinberger M, Abu-Hasan M. Pseudo-asthma: when cough, wheezing, and dyspnea are not asthma.
Pediatrics. 2007;120:855-864. Available at:
http://pediatrics.aappublications.org/cgi/content/full/120/4/855

page 398
2009 PREP SA on CD-ROM

Question: 112
A 6-year-old girl fell onto her outstretched right arm while roller skating yesterday. She continued to
skate, but on returning home, she noticed that her right forearm was swollen and painful. Her mother
applied ice and gave her ibuprofen, but the swelling is worse today. On physical examination, the girl
has moderate swelling over the right distal radius with minimal pain on palpation. She has full range of
motion of her wrist and hand. Radiographs are obtained (Item Q112).

Of the following, a TRUE statement regarding this patient’s fracture is that

A. median nerve injury often is seen in association with this fracture

B. reduction of the fracture is not necessary if there is less than 15 degrees of angulation

C. the fracture frequently is associated with growth arrest

D. the fracture is likely to produce permanent deformity

E. the patient should be immobilized in a cast for 8 to 10 weeks

page 399
2009 PREP SA on CD-ROM

Question: 112

Radiograph of the forearm, as described for the child in the vignette.


(Courtesy of E. Anthony)

page 400
2009 PREP SA on CD-ROM

Critique: 112 Preferred Response: B


Pediatric fracture patterns differ from those in adults for a variety of biomechanical, physiologic, and
anatomic reasons. Compared to adult bone, pediatric bone is highly porous; contains lower amounts of
mineral and higher amounts of cartilage; has thicker, stronger, and more metabolically active
periosteum; and has relatively stronger ligaments and open growth plates. These differences lead to
fracture types such as greenstick fractures, plastic deformation, torus fractures, and physeal or growth
plate fractures that are unique to children.
Greenstick fractures are incomplete fractures characterized by a cortical break on one side of the
bone with intact periosteum on the opposing side (Item C112A), as seen in the radiograph obtained for
the girl in the vignette. In some cases, the bone may bend without breaking (Item C112B). The strong
periosteum and decreased resistance to bending forces of pediatric bones are responsible for this
phenomenon. Torus fractures result from compressive forces on the bone that lead to a periosteal
"buckle" (Item C112C). Children who have open growth plates are at risk for physeal fractures that
involve the relatively weak, cartilage-rich growth plates.
Treatment for the greenstick fracture with less than 15 degrees of angulation, as described for the
child in the vignette, involves cast immobilization and close orthopedic follow-up, but no reduction of the
fracture is necessary. In older children who have fractures with greater than 15 degrees of angulation,
reduction often is necessary to decrease the risk of permanent deformity. Greenstick fractures typically
are not associated with nerve entrapment or growth arrest (if the physis is not involved) and heal
without residual sequelae following cast immobilization for 4 to 6 weeks. Patients should be cautioned
to avoid reinjury following cast removal. Refracture is seen in 7% to 17% of forearm fractures and
occurs most commonly following greenstick fractures.

References:

Carson S, Woolridge DP, Colletti J, Kilgore K. Pediatric upper extremity injuries. Pediatr Clin North Am.
2006;53:41-67. Abstract available at: http://www.ncbi.nlm.nih.gov/pubmed/16487784

Gholve PA, Hosalkar HS, Wells L. Common fractures. In: Kleigman RM, Behrman RE, Jenson HB,
Stanton BF, eds. Nelson Textbook of Pediatrics. 18th ed. Philadelphia, Pa: Saunders Elsevier;
2007:2834-2841

Greenstick fracture of forearm. In: Wheeless' Textbook of Orthopaedics. Available at:


http://wheelessonline.com/ortho/green_stick_frx_of_forearm

page 401
2009 PREP SA on CD-ROM

Critique: 112

Greenstick fractures are incomplete fractures characterized by a cortical


break on one side of the bone (area of lucency, yellow arrow) and intact
periosteum on the opposite side. This patient also has a greenstick fracture of
the ulna (red arrow). (Courtesy of E. Anthony)

page 402
2009 PREP SA on CD-ROM

Critique: 112

In some greenstick fractures, the bone may bend but not break (arrow).
(Courtesy of D. Krowchuk)

page 403
2009 PREP SA on CD-ROM

Critique: 112

Torus (buckle) fracture of the left distal radius (yellow arrows) and the radial
side of the ulna (red arrow). The term "torus" is used because of the similarity
of appearance of the fracture with the convex molding often used at the base
of columns. (Courtesy of E. Anthony)

page 404
2009 PREP SA on CD-ROM

Question: 113
A 5-year-old child is admitted to the hospital with epigastric pain and vomiting. On physical examination,
she has a tender epigastrium, but no peritoneal signs. Her amylase is 400 U/L and lipase is 670 U/L.
Abdominal ultrasonography demonstrates a prominent pancreatic head, but no gallstones or biliary tract
dilation. Review of her chart demonstrates two prior hospitalizations over the past 3 years due to
pancreatitis. She has no other significant findings in her medical history and no history of trauma
preceding any of these episodes.

Of the following, the condition that BEST explains the patient’s history is

A. alpha-1-antitrypsin deficiency

B. colipase deficiency

C. hereditary pancreatitis

D. primary sclerosing cholangitis

E. systemic lupus erythematosus

page 405
2009 PREP SA on CD-ROM

Critique: 113 Preferred Response: C


The child described in the vignette has a history of at least three episodes of pancreatitis before age 5
years, and gallstones have been excluded by ultrasonography. Children who have hereditary
pancreatitis (HP) can present with recurrent episodes of pancreatitis and often are well between
episodes. In adulthood, affected individuals are at risk for developing pancreatic insufficiency and
cancer.
Systemic lupus erythematosus (SLE) may be associated with pancreatitis in the context of severe
vasculitis, but SLE is rare before 5 years of age, and the child has no other signs of the condition.
Alpha-1-antitrypsin deficiency and primary sclerosing cholangitis may cause chronic liver disease, but
they rarely affect the pancreas. Colipase deficiency may result in pancreatic insufficiency and fat
malabsorption, but it usually does not cause pancreatic inflammation.
Pancreatitis generally is less severe in children than in adults. A child who has acute pancreatitis
typically presents with epigastric pain and vomiting, and epigastric tenderness can be appreciated on
palpation. More severe pancreatitis can cause intravascular volume depletion, third spacing, oliguria,
pulmonary edema, and pleural effusions. The most common laboratory abnormalities in mild
pancreatitis are elevated amylase and lipase values. Lipase is a more specific enzyme for pancreatitis;
hyperamylasemia also can be seen with salivary gland inflammation, ovarian tumors, renal
insufficiency, diabetic ketoacidosis, and ruptured ectopic pregnancy. Laboratory abnormalities
associated with more severe pancreatitis include hyperglycemia, hypocalcemia, high blood urea
nitrogen and creatinine, low hematocrit, and coagulopathy. In severe cases of pancreatitis, abdominal
computed tomography scan can identify the extent and severity of pancreatic injury.
Some children experience recurrent episodes of pancreatitis without an obvious cause (ie, no history
of medications that can cause pancreatitis, gallstones, trauma, or systemic vasculitis). Anatomic and
genetic causes should be considered in such children. Anatomic causes such as choledochocele,
pancreas divisum, or a long common channel connecting the pancreas and bile ducts can be identified
by either magnetic resonance imaging or endoscopic retrograde cholangiopancreatography.
The most common genetic causes of recurrent pancreatitis are cystic fibrosis (CF) and HP. Patients
who have CF and present with pancreatitis often have minimal pulmonary disease and no clinical signs
of pancreatic insufficiency or malabsorption. In some cases, a single mutation in the CFTR gene may
cause a heterozygous carrier for CF to present with recurrent pancreatitis. HP is caused most
commonly by a mutation in the pancreatic cationic trypsinogen gene and is inherited in an autosomal
dominant pattern. Although there often is a family history, some patients have no family history because
a de novo gene mutation has developed.

References:

Pietzak MM, Thomas DW. Pancreatitis in childhood. Pediatr Rev. 2000;21: 406-412. Available at:
http://pedsinreview.aappublications.org/cgi/content/full/21/12/406

The National Pancreas Foundation web site. Available at: www.pancreasfoundation.org

page 406
2009 PREP SA on CD-ROM

Question: 114
You admit a term newborn to the neonatal intensive care unit because of noisy breathing. Findings on
physical examination include mild micrognathia, an intact palate, and inspiratory stridor with
suprasternal retractions when the infant is in the supine position that diminish but do not disappear
when the infant is prone. Stridor becomes more audible when the infant cries. When the infant is asleep
and prone, the breath sounds are clear and equal bilaterally, with no stridor or wheezing. There is no
heart murmur. Pulse oximetry is 94% on room air.

Of the following, the MOST likely cause of this infant’s stridor is

A. cleft lip

B. laryngomalacia

C. tracheal hemangioma

D. tracheomalacia

E. vocal cord polyp

page 407
2009 PREP SA on CD-ROM

Critique: 114 Preferred Response: B


The infant described in the vignette has congenital stridor (noisy inspiratory breathing) and exhibits only
mild increased work of breathing when supine and when distressed. Although no immediate
intervention is required to maintain airway patency, the clinician examining the patient needs to
determine the cause of the stridor. Some have reported a characteristic coarseness of the stridor when
due to a supraglottic (laryngeal) disorder, a musical quality associated with glottic (vocal cord)
disorders, and more of an expiratory noise or prolonged expiratory phase with a wheeze in subglottic
disorders of airway obstruction (Item C114A).
The most common cause of congenital stridor is laryngomalacia, a congenital weakness, or
floppiness, in airway laryngeal cartilages (epiglottis, arytenoid cartilages, larynx) (Item C114B). A cleft
lip does not cause stridor. A tracheal hemangioma (Item C114C) may cause airway obstruction, but
usually this is associated with expiratory wheezing. Tracheomalacia may complicate prolonged
intubation and positive pressure ventilation in preterm infants and typically is associated with expiratory
wheezing or inspiratory cessation of airflow with airway collapse unless positive distending pressure is
applied (Item C114D). A vocal cord polyp is a rare congenital condition that may be associated with
human papillomavirus infection that can cause stridor, which would not be expected to change with
position.

References:

Aly H. Respiratory disorders in the newborn: identification and diagnosis. Pediatr Rev.
2004;25:201-208. Available at: http://pedsinreview.aappublications.org/cgi/content/full/25/6/201

Brodsky L. Consultation with the specialist: congenital stridor. Pediatr Rev. 1996;17:408-411. Available
at: http://pedsinreview.aappublications.org/cgi/reprint/17/11/408

Sprecher RC, Arnold JE. Upper airway lesions. In: Martin RJ, Fanaroff AA, Walsh MC, eds. Fanaroff
and Martin's Neonatal-Perinatal Medicine. 8th ed. Philadelphia, Pa: Mosby Elsevier; 2006:1146-1154

Vicencio AG, Parikh S, Adam HM. In brief: laryngomalacia and tracheomalacia: common dynamic
airway lesions. Pediatr Rev. 2006;27:e33-e35. Available at:
http://pedsinreview.aappublications.org/cgi/content/full/27/4/e33

page 408
2009 PREP SA on CD-ROM

Critique: 114

page 409
2009 PREP SA on CD-ROM

Question: 115
A 13-year-old boy who plays baseball comes to your office for a preparticipation sports physical
examination. He always has been an average player and is interested in a preseason conditioning
program to improve his strength and agility because he wants to play on his school team. His mother is
concerned about the program because it involves weight training, and she asks for your advice.

Of the following, a TRUE statement about conditioning programs for young athletes is that these
programs

A. have been shown to decrease ultimate linear growth if begun before puberty

B. prevent future catastrophic sports-related injuries

C. should begin with low-resistance exercise, with weight added in small increments as tolerated

D. should involve high-weight, low-repetition activities

E. should involve weight training only without aerobic activities

page 410
2009 PREP SA on CD-ROM

Critique: 115 Preferred Response: C


Preventive conditioning is an important part of sports programs for preadolescents and adolescents
because participation can help to improve strength and overall performance. Programs should include
both aerobic conditioning and strength training. The most significant risks to the athlete involve
strength, or resistance, training, but such risks can be minimized by adhering to a few guidelines.
Weight training programs that consist of high-repetition activities with low resistance are safe for
both preadolescents and adolescents. Training should begin by teaching proper technique with no
weight, adding small increments of weight as strength increases. Sessions lasting 20 to 30 minutes
undertaken 2 to 3 days a week are sufficient to improve strength. Power lifting programs, which involve
low-repetition, high-resistance exercises, should not be undertaken by preadolescents because of the
risk of injury. In general, such programs should be avoided until skeletal maturity is reached. Injuries
reported in studies evaluating strength training programs include muscle strain, epiphyseal injuries in
the wrist, and apophyseal injuries, but most of these can be avoided by using proper technique.
Participation in these programs does not decrease linear growth.
Although preventive conditioning has been shown to improve strength and performance, there is no
evidence that participation in such programs can prevent catastrophic injury. However, athletes who
have particular recurrent injuries, such as to the shoulder or ankle, can reduce their risk of further injury
by participating in specific strengthening exercises focusing on those areas.

References:

American Academy of Pediatrics Committee on Sports Medicine and Fitness. Strength training by
children and adolescents. Pediatrics. 2001;107:1470-1472. Available at:
http://pediatrics.aappublications.org/cgi/content/full/107/6/1470

Metzl JD. Preparticipation examination of the adolescent athlete: part 2. Pediatr Rev. 2001;22:227-239.
Available at: http://pedsinreview.aappublications.org/cgi/content/full/22/7/227

page 411
2009 PREP SA on CD-ROM

Question: 116
A 10-year-old boy who recently emigrated from Central America is referred by the school nurse for
evaluation of obesity.Physical examination reveals an obese but generally healthy boy who has
acanthosis nigricans (Item Q116).He has had limited access to medical care in the past.

Of the following, the physical finding MOST likely to suggest an underlying cause for the child's obesity
is

A. a normal blood pressure

B. abdominal striae

C. penile length at 1 standard deviation below the mean

D. small hands and feet

E. stature greater than the 95th percentile

page 412
2009 PREP SA on CD-ROM

Question: 116

(Courtesy of M. Rimsza)

page 413
2009 PREP SA on CD-ROM

Critique: 116 Preferred Response: D


The incidence of obesity continues to increase among children in industrialized nations. In general, the
causes in most children relate to increased caloric intake and decreased physical activity. However, the
pediatrician must exclude genetic, metabolic, or other underlying causes of obesity in children for two
reasons: 1) some underlying causes may require treatment and 2) the focus on changing the child's
lifestyle must involve parents who are reassured that hormonal or other abnormalities are not the
cause. A child who exhibits normal linear growth, has normal developmental milestones, and has
normal findings on physical examination is unlikely to have an underlying cause for his or her obesity.
The most common metabolic cause of obesity is hypothyroidism, but routine testing for thyroid
function in the overweight child who has normal linear growth velocity for age and no clinical signs of
hypothyroidism is unwarranted. A stature greater than the 95th percentile should be interpreted with
caution because one measurement is not sufficient to determine growth velocity. However, this finding
is reassuring because normal to above-average height is unlikely in a child who has hypothyroidism
and common in children who have exogenous obesity.
Children who have small hands and feet, hypogonadism, learning disabilities, or mental retardation
should be evaluated for Prader- Willi and Bardet-Biedl syndromes. These syndromes often present in
infancy or early childhood with hypotonia and developmental delay. Small hands and feet are common
and strongly suggest an underlying anomaly for the child described in the vignette (Item C116A).
Genetic testing for these conditions is available.
The normal blood pressure reported for the boy in the vignette is reassuring because hypertension
often complicates obesity. Penile length is normal at 1 standard deviation below the mean, but it is
important to remember that measurement of penile length can be a challenge in obese children due to
interference from the pubic fat pad.
Striae may be seen in many overweight children simply due to rapid weight gain. Although this
finding also occurs with cortisol excess (Item C116B), conditions associated with hypercortisolism
usually are associated with other signs and symptoms. Thus, the finding of striae in an obese child is
not an indication that additional testing is necessary.
Treatment of childhood obesity is difficult, although there is some evidence for a variety of
interventions that may prove successful in some populations. Numerous diets, including
low-carbohydrate diets, appear to have some impact on obesity in children. For morbidly obese
adolescents who have comorbidities unresponsive to diet and exercise, some centers are moving
toward bariatric surgery.
Barriers to healthy lifestyles (lack of availability in some communities of safe outdoor play areas,
specific exercise programs geared to very obese children, and school lunch programs) remain topics of
scientific and political investigation.

References:

Arterburn DE. Obesity in children. BMJ Clinical Evidence. 2007. Available for subscription at:
http://clinicalevidence.bmj.com/ceweb/conditions/chd/0325/0325.jsp

Rodearmel SJ Wyatt HR, Stroebele N, Smith SM, Ogden LG, Hill JO. Small changes in dietary sugar
and physical activity as an approach to preventing excessive weight gain: the America on the Move
Family Study. Pediatrics. 2007;120:e869-e879 Available at:
http://pediatrics.aappublications.org/cgi/content/full/120/4/e869

Schneider MB, Brill SR. Obesity in children and adolescents. Pediatr Rev. 2005;26:155-162. Available
at: http://pedsinreview.aappublications.org/cgi/content/full/26/5/155

page 414
2009 PREP SA on CD-ROM

Shaw K, Gennat H, O'Rourke P, Del Mar C. Exercise for overweight or obesity. Cochrane Database
Syst Rev. 2006;4:CD003817. Available at:
http://www.mrw.interscience.wiley.com/cochrane/clsysrev/articles/CD003817/frame.html

Summerbell CD, Waters E, Edmunds LD, Kelly S, Brown T, Campbell KJ. Interventions for preventing
obesity in children. Cochrane Database Syst Rev. 2005;3:CD001871. Available at:
http://www.mrw.interscience.wiley.com/cochrane/clsysrev/articles/CD001871/frame.html

Thomas DE, Elliott EJ, Baur L. Low glycaemic index or low glycaemic load diets for overweight and
obesity. Cochrane Database Syst Rev. 2007;3:CD005105. Available at:
http://www.mrw.interscience.wiley.com/cochrane/clsysrev/articles/CD005105/frame.html

page 415
2009 PREP SA on CD-ROM

Critique: 116

Prader-Willi syndrome is characterized by short stature, obesity,


hypogonadism, and small hands and feet. (Courtesy of Y. Lacassie)

page 416
2009 PREP SA on CD-ROM

Critique: 116

The striae observed in patients who have Cushing syndrome often have a
violaceous color. (Courtesy of M. Rimsza)

page 417
2009 PREP SA on CD-ROM

Question: 117
You are evaluating a 14-year-old girl in the clinic. She has had a fever for nearly 2 weeks, which she
has attributed to a "cold," although she has not had cough or upper respiratory tract symptoms. She is
concerned about some "spots" that she has noticed on her palms and soles. On physical examination,
you note splenomegaly and erythematous, nontender macules on her fingers, palms (Item Q117), and
soles of her feet. Additionally, she has lost 8 lb since her visit 6 months ago.

Of the following, the MOST appropriate next study for evaluation of this patient is

A. antinuclear antibody

B. echocardiography

C. Lyme titers

D. ophthalmologic examination

E. tuberculin skin test

page 418
2009 PREP SA on CD-ROM

Question: 117

(Courtesy of M. Rimsza)

page 419
2009 PREP SA on CD-ROM

Critique: 117 Preferred Response: B


The fever, Janeway lesions (erythematous, nontender macular lesions on the fingers and soles),
splenomegaly, and weight loss reported for the girl in the vignette strongly suggest infective
endocarditis. Infective endocarditis describes an infection involving the endocardial surface of the heart
that can occur in individuals who have structurally normal hearts, although it is more common in
children who have congenital heart disease. The most common sites are the cardiac valves, but
infection also may occur on the margins of a ventricular septal defect, along the chordae supporting the
atrioventricular valves, or along vascular grafts or foreign material such as a prosthetic valve. Although
a relatively rare occurrence in the pediatric population, infective endocarditis continues to be a major
source of morbidity and even mortality. Viridans streptococci (eg, S mitis, S bovis) as well as
Staphylococcus aureus are the most common bacterial pathogens causing endocarditis in children.
The clinical manifestations of infective endocarditis are many and can present variably in affected
individuals. The most common is fever, which may be associated with shaking chills. Constitutional
nonspecific manifestations include anorexia, weight loss, malaise, night sweats, arthralgias, myalgias,
and splenomegaly. In addition, a number of extracardiac manifestations are associated with infective
endocarditis, including petechiae and splinter hemorrhages seen under the nails, hematuria, and
glomerulonephritis. Roth spots are retinal hemorrhages that have a clear center and can be seen on
ophthalmologic examination (Item C117A). Janeway lesions may be evident on the fingers, palms, or
soles (Item C117B). Osler nodes are small, raised erythematous or purple nodules on the pads of the
digits that typically are very painful.
Among the cardiac manifestations of infective endocarditis is a murmur, which is heard in nearly
50% of affected patients. Such murmurs typically result from valvular insufficiency, and the left heart
valves are affected far more commonly than the right heart valves. Regurgitation of the mitral valve
produces a holosystolic murmur typically heard best at the cardiac apex, with radiation to the left axilla
(Item C117C). Regurgitation of the aortic valve produces a diastolic murmur that generally is heard best
at the mid-left or right sternal border with the patient in the sitting position, leaning forward (Item
C117D). If the regurgitation is severe, congestive heart failure also can be present.
The causative pathogen of infective endocarditis can be identified with blood cultures. Imaging of
cardiac valves and evaluation of myocardial function by echocardiography are important parts of an
assessment that can help guide management of valvular regurgitation and congestive heart failure, if
present. An ophthalmologic examination is an important component of the evaluation, but it is not
diagnostic and will not affect treatment. With a presentation such as that of the girl described in the
vignette, it is important to rule out other possible causes of systemic disease such as lupus
erythematosus, Lyme disease, and tuberculosis, but such diagnostic tests do not have the same
urgency or impact on management as echocardiography in a patient who clearly has infective
endocarditis.

References:

Baltimore RS. Infective endocarditis in children. Pediatr Infect Dis J. 1992;11:907-912. Abstract
available at: http://www.ncbi.nlm.nih.gov/pubmed/1454430

Hoyer A, Silberbach M. Infective endocarditis. Pediatr Rev. 2005;26:394-400. Available at:


http://pedsinreview.aappublications.org/cgi/content/full/26/11/394

Taubert KA, Dajani AS. Infective endocarditis. In: Moller JH, Hoffman JIE, eds. Pediatric Cardiovascular
Medicine. Philadelphia, Pa: Churchill Livingstone; 2000:768-779

page 420
2009 PREP SA on CD-ROM

Critique: 117

Roth spots are retinal hemorrhages that have a pale center (arrow). (Courtesy
of SY Lesnik Oberstein and eyetext.net)

page 421
2009 PREP SA on CD-ROM

Critique: 117

Janeway lesions are painless erythematous or hemorrhagic macules on the


fingers, palms, or soles. (Courtesy of M. Rimsza)

page 422
2009 PREP SA on CD-ROM

Question: 118
A mother brings her 8-year-old daughter to your office after the girl experiences a first unprovoked
generalized tonic-clonic seizure at school. The child had been seen in an emergency department, and
results of a head computed tomography scan performed there were normal. Her development, school
performance, and results of physical examination are normal. You review safety concerns (no
unsupervised time in bathtub or pools, wearing a bicycle helmet) and seizure first aid with the mother.
Following published guidelines, you obtain routine electroencephalography (EEG), which a neurologist
interprets as normal. The mother asks you about anticonvulsant therapy to prevent further seizures.

Of the following, you are MOST likely to advise her that

A. repeat sleep-deprived EEG is needed to select medication

B. treatment with carbamazepine should be initiated

C. treatment with phenobarbital should be initiated

D. treatment with phenytoin should be initiated

E. treatment with seizure medication should be deferred

page 423
2009 PREP SA on CD-ROM

Critique: 118 Preferred Response: E


For a child who has a first unprovoked seizure, as described in the vignette, safety concerns (no
unsupervised time in bathtub or pools, wearing a helmet while on a bike or scooter) and seizure first aid
should be reviewed with the parents and documented in the chart. Treatment after a single seizure in
childhood is not recommended, no matter what EEG shows.
In some cases, retrospective evaluation of children suggests that they have had subtle complex
partial seizures or previously have been found after an unwitnessed seizure. Therefore, it is critical for
the physician seeing the child after an apparent first seizure to obtain a very careful history to be certain
that the seizure is, indeed, the first.
The most important diagnostic decision after a first unprovoked seizure is whether to obtain
neuroimaging to diagnose a focal, treatable cause for seizures. Neuroimaging should be obtained after
a first unprovoked seizure when the seizure occurs in the first year after birth, is focal/asymmetric, or
begins with a stare and subsequently generalizes. Neuroimaging also should be obtained after a first
unprovoked seizure when the history or physical examination results indicate probable focal central
nervous system pathology, such as if the child has unilateral weakness or when postictal confusion is
prolonged, particularly if the child is febrile (to rule out herpes encephalitis).
Obtaining an EEG is recommended by published practice guidelines. There are two primary
problems to keep in mind when deciding whether to order an EEG:
1) Inter-reader agreement in the interpretation of EEGs is low. The level of training of neurologists
reading pediatric EEGs may be low in some communities, which is a particular problem for neonatal
EEGs, and even experienced EEG readers frequently differ in their visual interpretations.
2) Positive and negative predictive values are not very informative. The sensitivity of EEG varies
widely, depending on who reads it, but at best is approximately 50%; that is, about 50% of children who
have epilepsy and 50% of children who have a first unprovoked seizure that eventually will recur have
epileptiform EEGs, but the rest have normal EEGs. Accordingly, a normal EEG reading does not rule
out recurrence or epilepsy and only means that a recurrent seizure is somewhat less likely. The
specificity after first seizure is, at best, about 70%; that is, about 30% of children who do not have a
seizure recurrence still have epileptiform discharges on the EEG. Thus, a positive EEG also does not
predict epilepsy accurately.
Repeat EEGs with sleep deprivation have been advocated by neurology practice parameters for
many years in cases where the first EEG is read as normal and sleep was not present. The best
available evidence suggests, at most, modest benefit to this practice if used routinely in children. Any
benefits of the higher yield should be weighed against the potential for additional false-positive results,
the time for the additional trip to the EEG laboratory, missed school and work, cost, and the burden of
sleep deprivation. After a first seizure, the repeat EEG will not result in any additional information that
changes acute management.
Regardless of EEG results, two or more unprovoked seizures in a child are diagnostic of epilepsy
and warrant treatment and consultation with a neurologist for selection of medication. Because
phenobarbital causes sedation and cognitive and behavioral changes in young children, it would not be
used, except in countries where resources for medical care are very limited. Phenytoin has somewhat
erratic absorption and can cause gingival hyperplasia in children, so it is not first-line therapy for a new
diagnosis of epilepsy. Carbamazepine is a good choice for a child diagnosed with partial epilepsy, but
not for generalized epilepsy. Valproic acid is a good choice for both partial and generalized epilepsy in
children older than age 2 years. Each of these generic choices, along with the plethora of new
medications, has unique adverse event profiles, which physicians must understand.

References:

page 424
2009 PREP SA on CD-ROM

Gilbert DL, Buncher CR. An EEG should not be obtained routinely after first unprovoked seizure in
childhood. Neurology. 2000;54:635-641. Abstract available at:
http://www.ncbi.nlm.nih.gov/pubmed/10680796

Gilbert DL, DeRoos S, Bare MA. Does sleep or sleep deprivation increase epileptiform discharges in
pediatric electroencephalograms? Pediatrics. 2004;114:658-662. Available at:
http://pediatrics.aappublications.org/cgi/content/full/114/3/658

Gilbert DL, Sethuraman G, Kotagal U, Buncher CR. Meta-analysis of EEG test performance shows
wide variation among studies. Neurology. 2003;60:564-570. Abstract available at:
http://www.ncbi.nlm.nih.gov/pubmed/12601093

Hirtz D, Ashwal S, Berg A, et al. Practice parameter: evaluating a first nonfebrile seizure in children:
report of the Quality Standards Subcommittee of the American Academy of Neurology, the Child
Neurology Society, and the American Epilepsy Society. Neurology. 2000;55:616-623. Available at:
http://www.neurology.org/cgi/content/full/55/5/616

Hirtz D, Berg A, Bettis D, et al. Practice parameter: treatment of the child with a first unprovoked
seizure: report of the Quality Standards Subcommittee of the American Academy of Neurology and the
Practice Committee of the Child Neurology Society. Neurology. 2003;60:166-175. Available at:
http://www.neurology.org/cgi/content/full/60/2/166

Stroink H, van Donselaar CA, Geerts AT, Peters AC, Brouwer OF, Arts WF. The accuracy of the
diagnosis of paroxysmal events in children. Neurology. 2003;60:979-982. Abstract available at:
http://www.ncbi.nlm.nih.gov/pubmed/12654963

page 425
2009 PREP SA on CD-ROM

Question: 119
You receive a call from a local neonatologist because the mother of a newly admitted baby has
identified you as her pediatrician. The baby was born at term and had Apgar scores of 1, 2, and 4 at 1,
5, and 10 minutes, respectively. She is requiring significant ventilatory and fluid support, and she has
multiple congenital anomalies, including bilateral microtia, depressed nasal bridge, ocular
hypertelorism, cleft palate, and macrocephaly. Echocardiography shows truncus arteriosus, and head
ultrasonography reveals dilated ventricles consistent with hydrocephalus.

Of the following, the teratogenic agent that is MOST likely responsible for this infant’s features is

A. alcohol

B. isotretinoin

C. phenobarbital

D. phenytoin

E. thalidomide

page 426
2009 PREP SA on CD-ROM

Critique: 119 Preferred Response: B


The newborn described in the vignette has features consistent with retinoic acid embryopathy, also
called "Accutane embryopathy" (Accutane® is one of the brand names for isotretinoin [13-cis-retinoic
acid]). Soon after the drug was licensed for use in the United States, Lammer and associates published
an article describing 21 infants who had histories of prenatal isotretinoin exposure and negative
outcomes. It now is known that many retinoids are teratogenic in a number of species, with
malformations largely due to inhibited migration of cranial neural crest cells during embryogenesis. The
craniofacial, cardiovascular, and central nervous systems sustain the greatest deleterious effects.
Currently, a 35% risk for retinoic acid embryopathy is believed to exist in the infants of women who
take isotretinoin beyond 15 days after conception. Maternal use of isotretinoin prior to conception does
not appear to be associated with an increased risk for fetal anomalies. Paternal use of isotretinoin does
not cause birth defects.
Prenatal alcohol exposure is associated with fetal alcohol spectrum disorder. The most severe end
of the spectrum is fetal alcohol syndrome (FAS); affected infants typically exhibit intrauterine growth
restriction; microcephaly; and dysmorphisms that include short palpebral fissures, smooth philtrum, and
a narrow upper vermilion (Item C119A). Congenital heart defects are seen in some infants who have
FAS.
Phenobarbital and phenytoin use during pregnancy can result in a fetal anticonvulsant syndrome in
the exposed infant. Affected neonates also may have intrauterine growth restriction, ocular
hypertelorism, midface hypoplasia (Item C119B), cleft lip+/-cleft palate, and heart defects.
Thalidomide is a sedative that was marketed in the 1950s in Europe and was prescribed to pregnant
women to ameliorate morning sickness. It is probably the most notorious of all teratogens and was
taken off the market for many years but was reapproved by the United States Food and Drug
Administration in 1998 for use in the treatment of erythema nodosum leprosum and multiple myeloma.
The prescribing of thalidomide is strictly controlled, and physicians who do so must be registered in a
special safety program. Thalidomide exposure during pregnancy is associated with a variety of
anomalies, the most striking of which is severe limb reduction defects.

References:

Isotretinoin, alcohol, phenytoin, phenobarbital, thalidomide. Reprotox. Available for subscription at:
http://www.reprotox.org

Isotretinoin, alcohol, phenytoin, phenobarbital, thalidomide. Teris. Available for subscription at:
http://depts.washington.edu/terisweb/teris/

Jones KL. Retinoic acid embryopathy. In: Smith's Recognizable Patterns of Human Malformation. 6th
ed. Philadelphia, Pa; Elsevier Saunders; 2006:660-661

Lammer EJ, Chen DT, Hoar RM, et al. Retinoic acid embryopathy. N Engl J Med. 1985;313:837-841.
Abstract available at: http://www.ncbi.nlm.nih.gov/pubmed/3162101

page 427
2009 PREP SA on CD-ROM

Critique: 119

Characteristic facial appearance of a newborn who has fetal alcohol


syndrome includes midface hypoplasia, a relatively smooth philtrum, and a
narrow upper lip with a poorly defined "cupid's bow." (Courtesy of M. Rimsza)

page 428
2009 PREP SA on CD-ROM

Critique: 119

Arched eyebrows, wide-spaced eyes, and a short upturned nose are


characteristic of infants exposed to phenytoin during pregnancy. (Courtesy
of M. Rimsza)

page 429
2009 PREP SA on CD-ROM

Question: 120
You are evaluating a 14-year-old boy for his preparticipation sports physical examination before he tries
out for the freshman football team. He has no chronic health problems and no previous history of head
injuries. His mother expresses concern about recent reports of professional football players sustaining
cognitive damage due to repeated concussions.

Of the following, the MOST appropriate statement to include in your counseling regarding head injuries
in contact sports is that

A. baseline, detailed neuropsychological testing has been well established as a tool to use in the
management of head injuries in pediatric athletes

B. most athletic participants who sustain concussions lose consciousness

C. return-to-play guidelines for pediatric athletes are well established and evidence-based

D. seasonal and lifetime sports exclusion guidelines for pediatric athletes after head injury are well
established and evidence-based

E. the preparticipation visit history of an athlete should include the number of prior concussions, timing
and severity of each, and description of resulting symptoms

page 430
2009 PREP SA on CD-ROM

Critique: 120 Preferred Response: E


Although sports participation can have many physical and social benefits for children and adolescents,
there are risks, including athletic concussion, which is described as mild traumatic brain injury that is
caused by an impact to the head. Numerous guidelines and literature reviews have examined athletic
concussion, but few have focused specifically on the pediatric athlete.
Medical examination frequently is mandated by schools and organizations before children can
participate in sports, and preparticipation assessment of an athlete is an opportune time to focus on
how to prevent head trauma as well recognition of concussions. Loss of consciousness is not the only
indication that an athlete has suffered a concussion; in fact, most sports participants who sustain
concussions do not lose consciousness. Documenting the history of the number of prior concussions,
timing and severity of each, and the description of resulting symptoms can help establish a baseline for
medical care. The initial "grade" of the concussion generally is accepted as less important than the
systematic tracking of each athlete's recovery course over time.
Although conventional neuropsychological assessment is a well-established and recommended
methodology for the evaluation of children and adults who have sustained neurologic injury or who have
neurodevelopmental problems, the use of comprehensive neuropsychological evaluations at a
preparticipation visit is impractical and expensive because of the extensive time needed to complete an
evaluation. Abbreviated "baseline" neuropsychological testing for athletic purposes has been
introduced, but sufficient data regarding the reliability, validity, and clinical utility of neuropsychological
instruments used in this model are lacking for pediatric athletes. Similarly, the value of such baseline
testing in making individual decisions about return to play after a head injury has not been
demonstrated. However, neuropsychological testing still can play a valuable role in evaluating and
managing certain athletes after injury, especially those who experience poor recovery and multiple
injuries, and in developing appropriate posttraumatic educational management plans if cognitive
difficulties exist.
Return to play after concussion is a controversial area, and none of the current guidelines have
garnered sufficient scientific support to be recognized as the "gold standard." Experts agree that the
earliest an athlete should return to play is when no signs or symptoms of any type are apparent at rest
or exertion; neurologic examination results are normal; and neuroimaging, when performed, provides
unremarkable results. Some advocate for even more conservative standards with pediatric athletes,
including always removing them from the day's competition after a concussion and not allowing return
to play unless cleared medically. Seasonal and lifetime sports exclusion guidelines are not
well-established or evidence-based for pediatric athletes, but a conservative approach is inarguably
appropriate.

References:

Guskiewicz KM, Weaver NL, Padua DA, Garrett WE Jr. Epidemiology of concussion in collegiate and
high school football players. Am J Sports Med. 2000;28:643-650. Abstract available at:
http://www.ncbi.nlm.nih.gov/pubmed/11032218

Kirkwood MW, Yeates KO, Wilson PE. Pediatrics sport-related concussion: a review of the clinical
management of an oft-neglected population. Pediatrics. 2007;117:1359-1371. Available at:
http://pediatrics.aappublications.org/cgi/content/full/117/4/1359

page 431
2009 PREP SA on CD-ROM

Question: 121
You are evaluating a 1-year-old girl who was admitted to the pediatric intensive care unit following 3
days of diarrhea and decreased oral intake. Her heart rate is 160 beats/min, respiratory rate is 30
breaths/min, blood pressure is 70/40 mm Hg, and she has weak peripheral pulses. An arterial blood gas
evaluation on room air reveals pH of 7.08, Paco2 of 25 mm Hg, Pao2 of 100 mm Hg, and HCO3 of 5.0
mEq/L (5.0 mmol/L). Initial electrolyte values are: sodium of 130.0 mEq/L (130.0 mmol/l), potassium of
4.0 mEq/L (4.0 mmol/L), chloride of 95.0 mEq/L (95.0 mmol/L), bicarbonate of 6.0 mEq/L (6.0 mmol/L),
and glucose of 100.0 mg/dL (5.6 mmol/L).

Of the following, the MOST appropriate initial therapy is

A. 5 mcg/kg per minute dopamine infusion

B. 1 mEq/kg intravenous potassium chloride

C. 1 mEq/kg intravenous sodium bicarbonate

D. 20 mL/kg intravenous 0.9% sodium chloride

E. 1 unit/kg intravenous insulin

page 432
2009 PREP SA on CD-ROM

Critique: 121 Preferred Response: D


The child described in the vignette demonstrates severe metabolic acidosis, normally defined as a pH
of less than 7.10 or serum bicarbonate concentration of less than 8.0 mEq/L (8.0 mmol/L), with partial
respiratory compensation. Severe metabolic acidosis is a medical emergency that requires urgent
intervention via an organized approach. Because this patient's respiratory drive, oxygenation, and
ventilation are adequate, therapy to correct the metabolic acidosis and its underlying cause should be
the first priority.
Volume repletion using isotonic fluids (such as normal saline) at an initial dose of 20 mL/kg is the
first-line therapy to correct metabolic acidosis. Administration of 1 mEq/kg sodium bicarbonate to raise
the pH to above 7.2 may be indicated to correct the adverse effects of severe acidemia, such as
decreased myocardial contractility, increased pulmonary vascular resistance, sensitization to cardiac
arrhythmias, and decreased responsiveness to catecholamine administration, but volume
administration remains the initial therapy of choice. Adequate ventilation is imperative before
administering sodium bicarbonate to ensure that the additionally generated carbon dioxide is eliminated
and not retained, which would decrease cellular pH further. Children who have severe shock may
require vasopressor (eg, dopamine) administration if they do not respond to initial volume resuscitation.
Because metabolic acidosis produces an extracellular shift of potassium and subsequent increase in
serum potassium concentration, administration of potassium should be deferred until adequate urine
output is assured and correction of the underlying acidosis has begun. Insulin is the cornerstone of
treatment of the metabolic acidosis seen in diabetic ketoacidosis, but the child in the vignette has no
evidence of diabetes mellitus.

References:

Greenbaum LA. Electrolyte and acid-base disorders. In: Kliegman RM, Behrman RE, Jenson HB,
Stanton BF, eds. Nelson Textbook of Pediatrics. 18th ed. Philadelphia, Pa: Saunders Elsevier;
2007:267-308

Schwaderer AL, Schwartz GJ. Back to basics: acidosis and alkalosis. Pediatr Rev. 2004;25:350-357.
Available at: http://pedsinreview.aappublications.org/cgi/content/full/25/10/350

page 433
2009 PREP SA on CD-ROM

Question: 122
A 3-year-old girl presents to the emergency department in an almost unresponsive state. Her parents
say that she has become increasingly ill over the past 5 days and has been very thirsty, with increased
urination. This morning she began to vomit and could not keep down fluids. Findings on physical
examination in addition to unresponsiveness include rapid, sighing respirations and flushed cheeks.
You estimate that she is 10% dehydrated. Initial laboratory studies reveal a blood glucose concentration
of 700.0 mg/dL (38.9 mmol/L), sodium of 130.0 mEq/L (130.0 mmol/L), potassium of 4.6 mEq/L (4.6
mmol/L), chloride of 96.0 mEq/L (96.0 mmol/L), bicarbonate of 8.0 mEq/L (8.0 mmol/L), and a venous
pH of 7.0.

Of the following, the MOST appropriate action to decrease this child’s risk for cerebral edema during
treatment is to

A. avoid potassium replacement until the serum potassium value is less than 4.0 mEq/L (4.0 mmol/L)

B. correct acidosis rapidly with sodium bicarbonate

C. rehydrate initially with 3% saline

D. rehydrate slowly using 0.45% to 0.9% saline

E. replace continuing urinary fluid losses with 0.2% saline

page 434
2009 PREP SA on CD-ROM

Critique: 122 Preferred Response: D


Treatment of diabetic ketoacidosis (DKA) requires supplementation with fluid, electrolytes, insulin, and
carbohydrate to replenish losses of fluid and electrolytes and supply insulin for proper carbohydrate,
protein, and lipid metabolism and to repair acidosis. The major life-threatening complication of the
treatment of DKA in children is cerebral edema, which occurs in 1 in 100 to 1 in 400 episodes of DKA.
Slow rehydration with fluid containing adequate electrolytes may decrease the risk of cerebral edema.
The rate of replacement still is argued, but calculated steady replacement of fluid losses as 0.45% to
0.9% saline, with glucose and potassium added as necessary over 36 to 48 hours, generally is
considered appropriate.
Large urinary potassium losses occur during the development of DKA. Even children who present
with hyperkalemia have had potassium losses. Insulin and glucose drive potassium into cells and lower
circulating potassium concentrations relatively rapidly during treatment. Therefore, potassium
replacement should be started as soon as the child is urinating and there is no worrisome
hyperkalemia. Avoiding potassium replacement until the potassium is less than 4.0 mEq/L (4.0 mmol/L)
is unnecessary.
Peripheral acidosis is corrected rapidly by sodium bicarbonate, but bicarbonate dissociates in blood
into bicarbonate ion and carbon dioxide, and the carbon dioxide crosses cellular and blood-brain
barriers more rapidly than bicarbonate ion. Therefore, the net biologic intracellular effect of bicarbonate
administration is an increase in intracellular and central nervous system (as measured in cerebrospinal
fluid) acidosis. Several studies suggest that administration of bicarbonate may increase the chance for
an adverse outcome, including the increased risk of cerebral edema. On the other hand, it is possible
that only those who have the most severe acidosis receive bicarbonate and that it is disease severity
rather than treatment that worsens outcome.
Three percent saline is hypertonic and would worsen renal fluid losses. This is not a good
maintenance fluid for treatment of DKA, although some evidence suggests that hypertonic saline is as
effective as mannitol in reducing symptomatic cerebral swelling, should it occur. Urine sodium losses in
DKA usually are about 75.0 mEq/L (75.0 mmol/L). Therefore, 0.2% saline is a hypotonic replacement
solution whose use is ill advised.

References:

Dunger DB, Sperling MA, Acerini CL, et al. European Society for Paediatric Endocrinology/Lawson
Wilkins Pediatric Endocrine Society consensus statement on diabetic ketoacidosis in children and
adolescents. Pediatrics. 2004;113:e133-e140. Available at:
http://pediatrics.aappublications.org/cgi/content/full/113/2/e133

Edge JA, Jakes RW, Roy Y, et al. The UK case-control study of cerebral oedema complicating diabetic
ketoacidosis in children. Diabetologia. 2006;49:2002-2009. Abstract available at:
http://www.ncbi.nlm.nih.gov/pubmed/16847700

Wolfsdorf J, Craig ME, Daneman D, et al; International Society for Pediatriac and Adolescent Diabetes.
Diabetic ketoacidosis. Pediatr Diabetes. 2007;8:28-43

page 435
2009 PREP SA on CD-ROM

Question: 123
A 14-year-old boy has been receiving occupational therapy due to weakness in his graphomotor (eg,
handwriting) skills. During the school annual Individualized Education Plan (IEP) meeting, his mother
asks about alternative strategies that could be used to help him compensate for his area of weakness.

Of the following, the BEST alternative strategy is to

A. allow him to use print rather than cursive writing for his notes

B. have a class scribe write notes for him

C. have him use a word processor/laptop computer

D. have him use audio books

E. provide preferential seating near the blackboard

page 436
2009 PREP SA on CD-ROM

Critique: 123 Preferred Response: C


As students who have learning difficulties progress in their education, the emphasis changes from
remediation to accommodation. Accommodation encompasses modifications in how tasks are given to
students so that affected children may complete the same school work as other students. Such
accommodations allow students who have learning issues to present their knowledge without being
affected adversely by their disabilities.
The boy in the vignette, who has weakness in his graphomotor skills, should be allowed access to
electronic devices such as a laptop computer to help accommodate his area of weakness. The use of a
class scribe will not foster his independence in taking his own notes. Allowing him to print instead of
writing in script may make his notes neater to read, but it will slow his ability to take notes efficiently.
Preferential seating near the blackboard may be helpful for a student who has a visual acuity problem,
but it will not help this student's handwriting.
Audio texts are useful for students who read slowly. Other potential accommodations allow a change
in timing, formatting, setting, scheduling, response, or presentation of the assignment or test. Such
accommodations do not alter what the test or assignment measures in any significant manner.
Examples of such accommodations include a Braille version of a test for a student who is blind or taking
a test alone in a quiet room for one who has attention-deficit/hyperactivity disorder.

References:

Thorne G. Graphomotor Skills: Why Some Kids Hate to Write. Center for Development and Learning.
Covington, La. 2006. Available at: http://www.cdl.org/resource-library/articles/graphomotor.php

Shaywitz SE, Shaywitz BA. Dyslexia (specific reading disability) Pediatr Rev. 2003;24:147-153.
Available at: http://pedsinreview.aappublications.org/cgi/content/full/24/5/147

page 437
2009 PREP SA on CD-ROM

Question: 124
During hospital rounds, you are evaluating a 7-year-old boy who has been hospitalized for 1 month after
developing a perforated appendix. He recently started eating solid foods, but today he developed loose
stools and a mild amount of abdominal discomfort. He has had four bowel movements within the past 6
hours that are described as watery and nonbloody. His parents are very concerned that this is another
complication of his perforated appendix. On your physical examination, the boy is afebrile and has
active bowel sounds and only minor discomfort on palpation to his abdomen.

Of the following, the MOST likely organism to be causing this patient’s diarrhea is

A. Campylobacter jejuni

B. Clostridium difficile

C. Salmonella enteritidis

D. Shigella sonnei

E. Yersinia enterocolitica

page 438
2009 PREP SA on CD-ROM

Critique: 124 Preferred Response: B


Although any gastrointestinal pathogen, including Campylobacter jejuni, Salmonella enteritidis, Shigella
sonnei, or Yersinia enterocolitica, can be acquired in a nosocomial outbreak, hospitalized patients are
most likely to have gastrointestinal problems due to Clostridium difficile. Infection with this organism
usually occurs in the setting of altered bowel flora, such as in children who have a history of receiving
antimicrobial therapy or other medications (eg, chemotherapeutic agents), those who have required
repeated enemas, and those who have had prolonged nasal gastric tube insertions. Although
penicillins, clindamycin, and cephalosporins commonly are associated with C difficile infection, almost
every antimicrobial agent has been described as a potential cause. In fact, many clinical microbiology
laboratories will not accept a stool culture for the other pathogens from a patient hospitalized for longer
than 3 days unless testing for C difficile is negative.
Clinical symptoms of C difficile infection vary widely, and asymptomatic colonization is common in
infants and young children. Infants become colonized during the first postnatal week, with up to 50% of
infants younger than 1 year of age possibly being colonized. Carriage rates decrease to 1% to 5% by 2
years of age. Risk factors for becoming colonized in the hospital include having an infected roommate
and having symptomatic patients on the same ward. Symptomatic disease is due to the action of the
toxins (A and B) that are produced by the organism.
The clinical manifestations of a C difficile infection include diarrhea; abdominal cramps; abdominal
tenderness; and in some instances, fever, systemic toxicity, and bloody stools. Severe or even fatal
cases have been described, especially in patients who have problems with neutropenia or those who
have other gastrointestinal problems (eg, Hirschsprung disease).
The diagnosis of disease due to C difficile is made by documenting the presence of toxin A or B with
a commercially available kit or by endoscopic findings of pseudomembranous enterocolitis (Item C124).
Symptomatic infants younger than 1 year of age should be investigated for other causes of diarrhea
because the carriage rate is so high, and the presence of toxin may not be responsible for the patient's
clinical signs or symptoms.

References:

American Academy of Pediatrics. Clostridium difficile. In: Pickering LK, Baker CJ, Long SS, McMillan
JA, eds. Red Book: 2006 Report of the Committee on Infectious Diseases. 27th ed. Elk Grove Village,
Ill: American Academy of Pediatrics; 2006:261-263

Brook I. Pseudomembranous colitis in children. J Gastroenterol Hepatol. 2005;20:182-186. Abstract


available at: http://www.ncbi.nlm.nih.gov/pubmed/15683418

page 439
2009 PREP SA on CD-ROM

Critique: 124

Clostridium difficile pseudomembranous colitis. The pseudomebranes are


shown by arrows. (Courtesy of A. Bousvaros)

page 440
2009 PREP SA on CD-ROM

Question: 125
You are evaluating a 7-year-old boy who has human immunodeficiency virus (HIV) infection and failure
to thrive. Over the last 2 months, he has had intermittent temperatures to 102.5°F (39.2°C), anorexia,
abdominal pain, diarrhea, and a 6-lb weight loss. His HIV infection has been poorly controlled because
of noncompliance with medications. His viral load is 150,000 copies, and his CD4 count is 40
cells/mm3. Physical examination shows a thin, small-for-age boy who is playing quietly. He weighs 15.8
kg (<5th percentile). He has some oral thrush, cervical and inguinal adenopathy, and mild diffuse
abdominal tenderness to palpation.

Of the following, the MOST likely pathogen causing this patient’s illness is

A. Cryptosporidium

B. cytomegalovirus

C. Mycobacterium avium-intracellulare complex

D. Mycobacterium tuberculosis complex

E. Salmonella sp

page 441
2009 PREP SA on CD-ROM

Critique: 125 Preferred Response: C


The child described in the vignette displays clinical signs and symptoms that are classic for
disseminated Mycobacterium avium-intracellulare complex (MAC) disease. This infection occurs in
patients who have impaired cell-mediated immunity (eg, congenital immune defects or human
immunodeficiency virus [HIV] infection). The greatest risk factor for MAC in patients who have HIV
infection is a severe depression of the CD4 cell count. Patients whose CD4 counts are less than 100
cells/mm3 are at the highest risk for disease. Clinical manifestations of disseminated nontuberculous
mycobacterial infection may include high fever, night sweats, weight loss, lymphadenopathy, abdominal
pain, fatigue, hepatosplenomegaly, diarrhea, and anemia. Diagnosis is established by isolation of the
organism from blood or infected tissues.
The nontuberculous mycobacteria that most commonly cause disease in children are: MAC (eg, M
avium and M intracellulare), M fortuitum, M abscessus, M marinum, M chelonae, and M kansasii. These
organisms may cause a variety of clinical syndromes, including cervical lymphadenitis (MAC),
cutaneous infections (M fortuitum, M abscessus, M marinum, M chelonae), otitis media (M abscessus),
osteomyelitis (MAC, M fortuitum, M kansasii), central catheter infections (M chelonae, M fortuitum),
pulmonary disease (MAC, M kansasii, M fortuitum), and disseminated infection (MAC). In the
immunocompromised host, the most common manifestations of cytomegalovirus disease are
pneumonia, colitis, and retinitis, none of which are manifested in this patient. Disease caused by
Cryptosporidium is associated with a chronic diarrhea; fever usually is not seen. M tuberculosis
generally is associated with pulmonary manifestations. Salmonella sp is associated with acute diarrhea.

References:

American Academy of Pediatrics. Diseases caused by nontuberculous mycobacteria (atypical


mycobacteria, mycobacteria other than Mycobacterium tuberculous). In: Pickering LK, Baker CJ, Long
SS, McMillan JA, eds. Red Book: 2006 Report of the Committee on Infectious Diseases. 27th ed. Elk
Grove Village, Ill: American Academy of Pediatrics; 2006:698-704

Gordin FM, Horsburgh CR Jr. Mycobacterium avium complex. In: Mandell GL, Bennett JE, Dolin R, eds.
Mandell, Douglas, and Bennett's Principles and Practice of Infectious Diseases. 6th ed. Philadelphia,
Pa: Elsevier Churchill Livingstone; 2005:2897-2909

Horsburgh CR Jr, Caldwell MB, Simonds RJ. Epidemiology of disseminated nontuberculous


mycobacterial disease in children with acquired immunodeficiency syndrome. Pediatr Infect Dis J.
1993;12:219-222. Abstract available at: http://www.ncbi.nlm.nih.gov/pubmed/8095716

page 442
2009 PREP SA on CD-ROM

Question: 126
You are seeing a 7-year-old boy for occasional nocturnal enuresis. His weight and height are at the
50th percentile for age, his blood pressure is 110/66 mm Hg, and there are no unusual findings on
physical examination. Urinalysis shows a specific gravity of 1.030, pH of 6.5, 2+ blood, and no protein.
Urine microscopy reveals 10 to 20 red blood cells/high-power field and no casts or crystals. Results of a
repeat urine sample 3 weeks later are unchanged. Laboratory findings include:
· Blood urea nitrogen, 12.0 mg/dL (4.3 mmol/L)
· Creatinine, 0.4 mg/dL (35.4 mcmol/L)
· Complement component 3 (C3), 110.0 mg/dL (normal, 86.0 to 166.0 mg/dL)
· Complement component 4 (C4), 22.0 mg/dL (normal, 13.0 to 32.0 mg/dL)
· Antinuclear antibody, negative
· Erythrocyte sedimentation rate, 6 mm/hr

Of the following, the MOST appropriate next step is

A. abdominal computed tomography scan

B. referral for cystoscopy

C. referral for renal biopsy

D. renal/bladder ultrasonography

E. repeat urinalysis in 1 month

page 443
2009 PREP SA on CD-ROM

Critique: 126 Preferred Response: D


Microscopic hematuria is defined as a positive dipstick test for blood and more than 5 red blood
cells/high-power field on microscopy (Item C126). Persistent microscopic hematuria is defined as blood
on urinalysis detected on repeat samples over a 1-month period. Children who have persistent
microscopic hematuria, such as the boy described in the vignette, require investigation for an
underlying cause. The urgency of such an evaluation in the absence of symptoms is predicated on
whether the patient has accompanying proteinuria (>1+ on a dipstick). Those who have proteinuria
require urgent evaluation to look for an underlying glomerulopathy in which disruption of the glomerular
capillary barrier (as occurs in glomerulonephritis) results in red blood cells and albumin gaining access
to the urinary space.
Up to 4% of all children may have microscopic hematuria on a routine screening urinalysis. Based
on this high incidence, the initial recommendation for a child who has isolated, asymptomatic
microscopic hematuria is to undergo repeat urinalysis in 2 to 3 weeks time. If the hematuria persists, as
occurred in the patient in the vignette, the clinician should evaluate the patient for an underlying
genitourinary problem. This evaluation typically includes renal function tests (serum creatinine) and
serologic testing for an underlying immune complex-mediated glomerulonephritis (complement
components 3 and 4, antinuclear antibody). In addition, renal imaging (renal/bladder ultrasonography)
to screen for cysts, stones, and tumors is indicated. Abdominal computed tomography scan is not
indicated prior to a renal/bladder ultrasonography in this clinical setting. More invasive testing, such as
cystoscopy, should not be undertaken unless there is a strong suspicion of bladder pathology,
especially prior to screening with ultrasonography. Similarly, renal biopsy is not indicated in the
absence of proteinuria, hypertension, azotemia, or gross hematuria. Repeat urinalysis is redundant
because persistent hematuria already has been substantiated.

References:

Moxey-Mims M. Hematuria and proteinuria. In: Kher KK, Schnaper HW, Makker SP, eds. Clinical
Pediatric Nephrology. 2nd ed. London, England: Informa Healthcare; 2007:129-141

Patel HP, Bissler JJ. Hematuria in children. Pediatr Clin North Am. 2001;48:1519-1537. Abstract
available at: http://www.ncbi.nlm.nih.gov/pubmed/11732128

Vehaskari VM, Rapola J, Koskimies O, Savilahti E, Vilska J, Hallman N. Microscopic hematuria in


school children: epidemiology and clinicopathologic evaluation. J Pediatr. 1979;95:676-684. Abstract
available at: http://www.ncbi.nlm.nih.gov/pubmed/490233

page 444
2009 PREP SA on CD-ROM

Critique: 126

Numerous red blood cells (arrows) on microscopic examination of urine.


(Reprinted with permission from Schumann GB, Friedman SK. Wet
Urinalysis. Chicago, Ill: ASCP Press; 2007. ©2007, American Society for
Clinical Pathology.)

page 445
2009 PREP SA on CD-ROM

Question: 127
An 8-year-old girl presents with multiple episodes of "bronchitis." For the past 2 years, she has had
problems with coughing, wheezing, and difficulty catching her breath during vigorous exercise.
Treatment with a metered dose beta2 agonist inhaler has improved her symptoms. In your office, you
discuss the different tests to assess lung function.

Of the following, the BEST test to measure lung function for this girl is

A. arterial blood gas

B. exhaled breath condensate

C. exhaled nitric oxide

D. pulse oximetry

E. spirometry

page 446
2009 PREP SA on CD-ROM

Critique: 127 Preferred Response: E


Spirometry, also referred to as a pulmonary function test (PFT), measures inspiratory and expiratory
respiratory effort. Established pretest norms are based on the patient's height and ethnicity.
Measurements of lung function that can be obtained with spirometry include the forced vital capacity
(FVC), volume of air exhaled during the first second (FEV1), FEV1/FVC ratio, peak expiratory flow
(PEF), and airflow during the middle half of the effort (forced expiratory flow) (FEF25-75). Because ideal
test conditions require a patient to exhale for 6 seconds, children younger than 6 to 7 years of age often
are unable to complete the test. Also, spirometry does not measure total lung capacity (TLC) or residual
volume (RV).
An arterial blood gas is a gas diffusion measurement that provides insight into oxygenation (PO2)
and ventilation (PCO2), but it does not measure lung function. Pulse oximetry is a simple, noninvasive
method to measure oxygenation, but it also does not measure ventilation or lung function.
Exhaled nitric oxide (eNO) measurement was approved by the United States Food and Drug
Administration in 2003 for children ages 4 years and older. Although not a measure of lung function,
eNO is a useful noninvasive tool to measure nitric oxide, a marker of airway inflammation.
Exhaled breath condensate is a noninvasive method to measure the pH of the airway, another
marker of inflammation. Similar to eNO, exhaled breath condensate does not measure lung function but
is being developed as another tool to assess airway inflammation and assist with asthma management.

References:

Guill MF. Asthma update: clinical aspects and management. Pediatr Rev. 2004;10:335-344. Available
at: http://pedsinreview.aappublications.org/cgi/content/full/25/10/335

Liu AH, Covar RA, Spahn JD, Leung DYM. Childhood asthma. In: Kliegman RM, Behrman RE, Jenson
HB, Stanton BF, eds. Nelson Textbook of Pediatrics. 18th ed. Philadelphia, Pa: Saunders Elsevier;
2007:953-969

Silkoff PE, Carlson M, Bourke T, Katial R, Ogren E, Szefler SJ. The Aerocrine exhaled nitric oxide
monitoring system NIOX is cleared by the US Food and Drug Administration for monitoring therapy in
asthma. J Allergy Clin Immunol. 2004;114:1241-1256. Abstract available at:
http://www.ncbi.nlm.nih.gov/pubmed/15536442

page 447
2009 PREP SA on CD-ROM

Question: 128
A 4-year-old girl is brought to the emergency department after sticking a hair pin in a household
electrical outlet. The mother reports that she heard the child scream, and when she investigated, smoke
was coming from the outlet and the child was crying, holding her right hand. There was a black imprint
on her fingers in the shape of the hair pin. She washed the fingers with soap and water and drove the
child to the emergency department for further evaluation. On physical examination, the child is tearful
but awake and alert. Her right index finger and thumb have erythematous burn imprints with small
blisters surrounded by soot. She has no other burns or other findings of note on the remainder of her
examination.

Of the following, it is MOST important to evaluate this patient for

A. arrhythmias

B. compartment syndrome

C. immunization status

D. myoglobinuria

E. skin grafting

page 448
2009 PREP SA on CD-ROM

Critique: 128 Preferred Response: C


Electrical injuries account for approximately 5% of admissions to burn units in the United States
annually. Most injuries occur in adults and are related to high-voltage sources in the workplace.
Pediatric injuries typically are related to contact with household, low-voltage sources such as electric
cords or wall outlets. The primary determinant of tissue damage following electrical exposure is current
strength, which is directly proportional to voltage and inversely related to tissue resistance. Because
voltage is the only variable commonly known, electrical exposures are classified as low- (<1,000 V) and
high- (>1,000 V) voltage. Standard household electricity in the United States is 110 V; utility power lines
carry in excess of 14,000 V.
The most common injury following contact with electricity is burns. Excluding lightening injuries,
electrical burns may occur from any of the following mechanisms: electrothermal heating from direct
contact with the electrical source, arc exposure in which the body becomes part of the electrical circuit,
flash contact in which the current arc strikes the skin but does not enter the body, and flame burns that
result when clothing or other objects combust in the presence of electrical current. Oral burns due to
chewing on an electrical cord (Item C128) usually are electrothermal. Arc exposure can be associated
with deep tissue burns and internal organ involvement as the current flows through body; the extent of
injury may be underestimated by the appearance of the skin wounds. Flash contact is characterized by
surface burns accompanied by soot, as described for the girl in the vignette.
Although both high- and low-voltage exposures can lead to injuries in a variety of organ systems,
high-voltage exposures are responsible for most significant injuries. The most commonly affected organ
is the skin, with burns resulting as described previously. Arrhythmias, including asystole and ventricular
fibrillation, may occur, usually at the time of contact. Deep thermal injuries to bone and muscle may
occur after arc exposure and lead to compartment syndrome or rhabdomyolysis with subsequent renal
damage. Risk factors for these more serious injuries include extensive full-thickness burns, cardiac
arrest, or high-voltage exposure. It is rare for children exposed to household current to sustain injuries
other than burns.
The girl in the vignette has a surface thermal burn related to contact with the heated hair pin, and the
presence of soot on her finger is evidence of flash contact. She has no clinical features or risk factors
for rhabdomyolysis, myoglobinuria, compartment syndrome, or arrhythmias. The wound is small and
should not require skin grafting. Therefore, other than ascertaining her tetanus immunization status and
providing wound care, no further evaluation is indicated.

References:

Chen EH, Sareen A. Do children require ECG evaluation and inpatient telemetry after household
electrical exposures? Ann Emerg Med. 2007;49:64-67

Pinto DS, Clardy PF. Environmental electrical injuries. UpTo Date Online 15.3. 2008 Available for
subscription at:
http://www.utdol.com/utd/content/topic.do?topicKey=ad_emerg/2283&selectedTitle=1~150&source=sear
ch_result

Price TG, Cooper MA. Electrical and lightning injuries. In Marx JA, ed. Rosen's Emergency Medicine:
Concepts and Clinical Practice. 6th ed. Philadelphia, Pa: Mosby Elsevier; 2006:chap 140

Rosen CL, Adler JN, Rabban JT, et al. Early predictors of myoglobinuria and acute renal failure
following electrical injury. J Emerg Med. 1999;17:783-789. Abstract available at:
http://www.ncbi.nlm.nih.gov/pubmed/10499690

page 449
2009 PREP SA on CD-ROM

Critique: 128

Oral burns may result from chewing on an electrical cord. (Courtesy of M.


Wright)

page 450
2009 PREP SA on CD-ROM

Question: 129
A 12-year-old girl has had intermittent periumbilical abdominal pain for the past 4 years. Sometimes the
pain worsens when she drinks a glass of milk. A lactose breath hydrogen test demonstrates a breath
hydrogen of 40 ppm after 1 hour (normal, <20 ppm).

Of the following, the food that this girl is MOST likely to tolerate is

A. buttermilk

B. cheddar cheese

C. ice cream

D. skim milk

E. whole milk

page 451
2009 PREP SA on CD-ROM

Critique: 129 Preferred Response: B


Lactose intolerance (lactose maldigestion) is a common cause of pediatric recurrent abdominal pain.
Lactase is a digestive enzyme located on the intestinal villi that converts the disaccharide lactose to the
monosaccharides glucose and galactose. The monosaccharides can be absorbed across the intestinal
epithelial cells. In an individual who has low intestinal lactase (lactose intolerance), the lactose cannot
be broken down and passes into the lower intestine and colon. The malabsorbed lactose can cause
osmotic diarrhea or be fermented by bacteria, resulting in pain, gas, and bloating. Lactose intolerance
can either be primary (lactase activity that gradually declines with aging) or secondary (an infection or
enteropathy damages the villi, resulting in lactose intolerance). Primary lactose intolerance (adult-type
hypolactasia) is extremely common, occurring in approximately 20% of white adults, 80% of African
American adults, and 90% of Asian adults. It is uncommon in children younger 5 years of age, but as
children grow, those who are genetically predisposed to lactose intolerance may develop the condition
gradually during their school age years.
Lactose intolerance may be diagnosed clinically by elimination diet and rechallenge or through
lactose breath hydrogen testing. Breath testing offers the advantage of providing a more definitive
diagnosis, so milk is not withdrawn from the diet unnecessarily. In a breath hydrogen test, an adult is
given approximately 50 g of lactose (the equivalent of 4 cups of milk) as a single dose. Patients who
cannot digest and absorb this amount of lactose have the lactose fermented to hydrogen by intestinal
bacteria, which can be measured in the breath. Because intestinal bacterial fermentation is essential for
an accurate test, patients should not receive antibiotics for at least 2 weeks prior to the test.
Many patients who have lactose intolerance diagnosed on breath test can tolerate small amounts of
lactose in their diet. Specifically, hard cheeses, such as cheddar, which often have very little lactose
(<0.5 g/serving), can be eaten by many patients who have lactose intolerance. Other products, such as
whole or skim milk, ice cream, or buttermilk, have more lactose (approximately 10 g/serving) and are
less likely to be tolerated (Item C129).

References:

Heyman MB for American Academy of Pediatrics Committee on Nutrition. Lactose intolerance in


infants, children, and adolescents. Pediatrics. 2006;118:1279-1286. Available at:
http://pediatrics.aappublications.org/cgi/content/full/118/3/1279

Thiessen PN. Recurrent abdominal pain. Pediatr Rev. 2002;23:39-46. Available at:
http://pedsinreview.aappublications.org/cgi/content/full/23/2/39

page 452
2009 PREP SA on CD-ROM

Critique: 129

page 453
2009 PREP SA on CD-ROM

Question: 130
You are treating a former extremely low-gestational age newborn (ELGAN) who was born at 26 weeks’
gestation weighing 700 g. She is now 4 weeks old. Her nurse asks when the eye examination for
retinopathy of prematurity (ROP) will be performed and what risk for significant visual impairment exists
in this infant.

Of the following, the BEST time to obtain the first ROP screening eye examination in this infant is

A. 4 weeks after discharge from the hospital

B. 4 weeks after weaning from oxygen

C. 5 weeks after birth

D. 5 weeks after the expected delivery date

E. 5 weeks after weaning from the ventilator

page 454
2009 PREP SA on CD-ROM

Critique: 130 Preferred Response: C


Retinopathy of prematurity (ROP) is a multifactorial disorder that has been linked historically and
epidemiologically to the survival of low-birthweight preterm infants. As the survival of smaller and more
immature infants has increased throughout the past 5 decades, the incidence of ROP has shifted
toward the smallest, most immature, and sickest babies in neonatal intensive care units (NICUs) in the
United States and worldwide.
Prospective studies and critical reviews and analyses of extensive data confirm that the greatest
known risks for ROP among NICU patients include: very low birthweight (<1,500 g), severity of illness,
gestational immaturity (especially <28 weeks' gestational age), intrauterine growth restriction, male sex,
and systemic fungal infection. Among the additional possible risks are antenatal dexamethasone
exposure (compared to betamethasone) and late treatment with dexamethasone (treatment of infants
more than 3 weeks of age for 2 or more weeks).
The American Academy of Pediatrics recommends a specific timetable for testing at-risk preterm
infants (Item C130). In general, infants should be tested between 31 and 34 weeks gestational age.
Accordingly, the infant described in the vignette should be tested 5 weeks after birth.
Because most 26 weeks' gestation infants are discharged from the hospital at a gestational
equivalent of 34 to 36 weeks, or 8 to 10 postnatal weeks of age, this would be too late for an initial
examination, as would one occurring 4 weeks later. Many 26 weeks' gestation infants may have chronic
lung disease and be receiving assisted ventilation for days to weeks or be discharged from the hospital
on oxygen, so screening is not based on the time of weaning from assisted ventilation or oxygen. Five
weeks after the expected delivery date is too late for initial screening.

References:

Bharwani SK, Dhanireddy R. Systemic fungal infection is associated with the development of
retinopathy of prematurity in very low birth weight infants: a meta-review. J Perinatol. 2007;28:61-66.
Abstract available at: http://www.ncbi.nlm.nih.gov/pubmed/18046338

Darlow BA, Hutchinson JL, Henderson-Smart DJ, Donoghue DA, Simpson JM, Evans NJ on behalf of
the Australian and New Zealand Neonatal Network. Prenatal risk factors for severe retinopathy of
prematurity among very preterm infants of the Australian and New Zealand Neonatal Network.
Pediatrics. 2005;115:990-996. Available at:
http://pediatrics.aappublications.org/cgi/content/full/115/4/990

Hagadorn JI, Richardson DK, Schmid CH, Cole CH. Cumulative illness severity and progression from
moderate to severe retinopathy of prematurity. J Perinatol. 2007;27:502-509. Abstract available at:
http://www.ncbi.nlm.nih.gov/pubmed/17568754

Karlowicz MG, Giannone PJ, Pestian J, Morrow AL, Shults J. Does candidemia predict threshold
retinopathy of prematurity in extremely low birth weight (<1000 g) neonates? Pediatrics.
2000;105:1036-1040. Available at: http://pediatrics.aappublications.org/cgi/content/full/105/5/1036

Lee BH, Stoll BJ, McDonald SA, Higgins RD for the National Institute of Child Health and Human
Development Neonatal Research Network. Adverse neonatal outcomes associated with antenatal
dexamethasone versus antenatal betamethasone. Pediatrics. 2006;117:1503-1510. Available at:
http://pediatrics.aappublications.org/cgi/content/full/117/5/1503

Markestad T, Kaaresen PI, Rønnestad A, et al on behalf of the Norwegian Extreme Prematurity Study

page 455
2009 PREP SA on CD-ROM

Group. Early death, morbidity, and need of treatment among extremely premature infants. Pediatrics.
2005;115:1289-1298. Available at: http://pediatrics.aappublications.org/cgi/content/full/115/5/1289

Section on Ophthalmology, American Academy of Pediatrics, American Academy of Ophthalmology,


American Association for Pediatric Ophthalmology and Strabismus. Screening examination of
premature infants for retinopathy of prematurity. Pediatrics. 2006;117:572-576. Available at:
http://pediatrics.aappublications.org/cgi/content/full/117/2/572

page 456
2009 PREP SA on CD-ROM

Critique: 130

Reprinted with permission from the Section on Ophthalmology, American


Academy of Pediatrics, American Academy of Ophthalmology, American
Association for Pediatric Ophthalmology and Strabismus. Screening
examination of premature infants for retinopathy of prematurity. Pediatrics.
2006; 117:572-576.

page 457
2009 PREP SA on CD-ROM

Question: 131
A 15-year-old boy who has mild persistent asthma is brought to the emergency department because of
increased work of breathing of 1 day’s duration. He reports a low-grade fever and nonproductive cough
for the past 4 days, but this morning he developed difficulty breathing and a cough that produced a
small amount of yellowish sputum. His respiratory rate is 24 breaths/min, heart rate is 80 beats/min, and
temperature is 99.0°F (37.3°C). He appears in no respiratory distress, but his lung examination reveals
bilateral rales and occasional wheezes. A chest radiograph shows bilateral diffuse infiltrates with no
effusions.

Of the following, the MOST likely etiologic agent causing his symptoms is

A. Haemophilus influenzae

B. Mycobacterium tuberculosis

C. Mycoplasma pneumoniae

D. Staphylococcus aureus

E. Streptococcus pneumoniae

page 458
2009 PREP SA on CD-ROM

Critique: 131 Preferred Response: C


Mycoplasma pneumoniae is a common cause of community-acquired pneumonia in school-age
children and adolescents, but it is rare in infants and toddlers. Other clinical manifestations of M
pneumoniae infection include upper respiratory tract infection, pharyngitis, bullous myringitis, and
bronchitis. Occasionally, systemic symptoms such as malaise and fever are present. Among the rarer
clinical presentations are hemolytic anemia, aseptic meningitis, and Stevens-Johnson syndrome. M
pneumoniae is responsible for many cases of pneumonia and acute chest syndrome in children who
have sickle cell disease. Infection also may precipitate an exacerbation of asthma, as described for the
boy in the vignette. A recent study showed that children of different ages may respond differently to
infection with M pneumoniae. Children younger than 5 years of age were more likely to experience
coryza, vomiting and diarrhea, and tachypnea than were older children.
Typical radiographic findings are bilateral, diffuse infiltrates (Item C131A), but occasionally a lobar
infiltrate or pleural effusion may be seen. The diagnosis frequently can be made clinically, but serologic
testing is available. The complement fixation test and, more recently, immunofluorescence and enzyme
immunoassay can be used to measure levels of immunoglobulin M (IgM) and IgG antibodies to M
pneumoniae. High concentrations of IgM may be present for months after acute infection. Comparing
acute and convalescent IgG titers allows for more accurate diagnosis, although this may be impractical.
Serum cold hemagglutinin titers also may be positive in patients who have acute infection but have
limited sensitivity and specificity. Direct polymerase chain reaction is both sensitive and specific for the
diagnosis but is not widely available. Macrolide antibiotics generally are used for treatment, although
many infections are self-limited, and a recent review found no conclusive evidence that antibiotics are
effective in reducing symptoms.
Haemophilus influenzae and Streptococcus pneumoniae also cause community-acquired
pneumonia, but fever generally is present with these infections, and radiographs typically reveal lobar
infiltrates (Item C131B) with or without pleural effusions. Lobar infiltrates also are typical of pneumonia
caused by Staphylococcus aureus, and pneumatoceles (Item C131C) or lung abscesses are common
with this infection. The radiologic features of M tuberculosis infection are varied and include lobar
infiltrates, hilar adenopathy, and cavitary lesions (Item C131D).

References:

American Academy of Pediatrics. Mycoplasma pneumoniae infections. In: Pickering LK, Baker CJ,
Long SS, McMillan JA, eds. Red Book: 2006 Report of the Committee on Infectious Diseases. 27th ed.
Elk Grove Village, Ill: American Academy of Pediatrics; 2006:468-470

Gavranich JB, Chang AB. Antibiotics for community acquiredlower respiratory tract infections (LRTI)
secondary to Mycoplasma pneumoniae in children. Cochrane Database Syst Rev. 2005;3:CD004875.
Available at: http://www.mrw.interscience.wiley.com/cochrane/clsysrev/articles/CD004875/frame.html

Othman N, Isaacs D, Kesson A. Mycoplasma pneumoniae infections in Australian children. J Paediatr


Child Health. 2005;41:671-676. Abstract available at: http://www.ncbi.nlm.nih.gov/pubmed/16398873

page 459
2009 PREP SA on CD-ROM

Critique: 131

Anteroposterior radiograph of the chest in Mycoplasma pneumoniae


pneumonia shows widespread streaky lung opacities consistent with areas of
atelectasis and infiltrates. (Courtesy of D. Mulvihill)

page 460
2009 PREP SA on CD-ROM

Critique: 131

Lingular infiltrate in an 8-month-old infant who has bacterial pneumonia.


(Courtesy of D. Krowchuk)

page 461
2009 PREP SA on CD-ROM

Critique: 131

Anteroposterior radiograph of the chest demonstrates consolidation in the


right upper lobe and focal air collection within lung parenchyma consistent
with a pneumatocele (arrow). (Courtesy of D. Mulvihill)

page 462
2009 PREP SA on CD-ROM

Critique: 131

Anteroposterior chest radiograph in tuberculosis shows extensive


mediastinal, paratracheal (red arrow), and hilar adenopathy with cavitary
changes in the right lung (yellow arrow). (Courtesy of D. Krowchuk)

page 463
2009 PREP SA on CD-ROM

Question: 132
A resident in continuity clinic approaches you to review the laboratory values obtained at a patient’s
12-month health supervision visit. The fingerstick hemoglobin measurement was 10.5 g/dL (105.0 g/L),
and the lead concentration was 11.0 mcg/dL (0.53 mcmol/L).

Of the following, the next BEST step for this boy is to

A. admit him to the hospital for chelation therapy

B. call child protective services to move him to a shelter

C. call the health department to arrange for an environmental investigation

D. measure the venous lead concentration

E. refer him for formal developmental evaluation and neuropsychometric testing

page 464
2009 PREP SA on CD-ROM

Critique: 132 Preferred Response: D


Routine screening of children for exposure to environmental lead combined with environmental
abatement has resulted in an overall decrease in lead toxicity among children living in the United
States. However, the laboratory findings of mild anemia and a capillary blood lead concentration that is
marginally elevated, as described for the child in the vignette, are still common. Repeat measurement
of the blood lead by venipuncture to avoid skin contamination as well as further evaluation of the child's
mild anemia are appropriate next steps.
In 1997, as a result of the decrease in lead exposure and prevalence of lead poisoning, the Centers
for Disease Control and Prevention (CDC) updated its screening guidelines and incorporated, for the
first time, screening policies that use local blood lead concentration data or housing data collected by
the United States Bureau of Census to determine risk. In geographic areas where limited exposure to
lead and limited prevalence of elevated blood lead concentrations are documented, targeted (selective)
screening may be more appropriate than universal screening. However, one of the goals of Healthy
People 2010 is the elimination of childhood lead poisoning as a public health problem.
Current guidelines state that health-care professionals should use blood lead tests to screen
children at ages 1 and 2 years and children 36 to 72 months of age who previously had not been
screened if they meet specific criteria. The CDC and state and local health departments continue to
develop surveillance and intervention tools to determine the best practices for targeted screening.
Pediatricians may consult the CDC Web site and their local health departments for zip code-related risk
levels as well as questionnaires to assess individual children's risk factors based on history of housing
or other exposures. Screening of toddlers for lead exposure continues to be recommended and even
mandated by health-care organizations.
The American Academy of Pediatrics policy statement recommends the use of venous samples for
initial screening whenever possible. If capillary testing is performed and the lead concentration is
greater than 10.0 mcg/dL (0.5 mcmol/L), the lead concentration must be confirmed by a venous sample
because microlead sampling is more likely to yield false-positive results due to contamination from
environmental lead.
Lead ingestion may cause a microcytic anemia by interfering with iron absorption and utilization in
heme production and can inhibit enzymes required for heme synthesis directly. Children who have lead
poisoning may have pica either as a cause or symptom of lead poisoning. In these children, iron
supplementation should be initiated until the presence or absence of iron deficiency is determined.
In many communities, formal environmental evaluation of the child's home or removal of the child
and family from that home is not conducted until the blood lead value has been confirmed and is higher
than 20.0 mg/dL (1.0 mcmol/L). Although the child's developmental status should be monitored carefully
in the future as a part of recommended surveillance and screening, formal neurodiagnostic testing is
not indicated at this time unless there are other risk factors for developmental delay.
Counseling the family about potential sources of lead in the environment (eg, dirt, cosmetics, or
household items) and lead solder in old pipes may be beneficial.
Most asymptomatic children who have mildly elevated blood lead concentrations are not candidates
for chelation therapy with the currently available drugs because the toxicity of these drugs outweighs
the potential benefit of treatment, and chelation is unlikely to increase lead excretion significantly.
Chelation therapy should be considered, however, if lead concentrations are higher than 44.0 mcg/dL
(2.12 mcmol/L). The role of chelation is not clearly defined for children whose blood lead concentrations
range from 20.0 to 45.0 mcg/dL (0.97 to 2.17 mcmol/L). In this range, the clinician may choose to
pursue further environmental screening, attempt to eradicate lead from the child's environment, and
measure blood lead concentrations monthly. If the concentration remains in this range, despite
successful eradication of the lead source, the physician should institute behavior modification,
nutritional sufficiency, or chelation treatment.

page 465
2009 PREP SA on CD-ROM

Succimer is the drug of choice for children whose blood lead concentrations are 45.0 to 100.0
mcg/dL (2.17 to 4.8 mcmol/L). At values higher than 69.0 mcg/dL (3.3 mcmol/L), a second drug,
CaNa2EDTA, is added. For children who require two-drug treatment, the first dose always is succimer,
followed 4 hours later by CaNa2EDTA, because children who present with lead encephalopathy may
deteriorate when treated with CaNa2EDTA alone. D-penicillamine is not recommended as first-line
therapy because adverse effects and allergy are common (33%). Dimercaprol is not a first-line drug
because the rate of adverse effects approaches 50%. Serious adverse effects of chelation therapy may
occur due to chelation of other electrolytes. NaEDTA has been associated with fatal hypocalcemia and,
therefore, is contraindicated. The clinician should be careful not to confuse CaNa2EDTA with NaEDTA.
Chelation therapy should be conducted in conjunction with a pediatric toxicologist or pharmacist under
very close supervision. Oral agents may be used, but succimer is not palatable and must be emptied
from a capsule onto food.
Once lead has been ingested, the percentage absorbed may be modified by essential nutrients. A
healthy diet can be recommended for both lead-exposed and nonexposed children. Particular attention
must be given to calcium and iron intake. If iron deficiency is diagnosed, supplemental iron should be
prescribed. For children who are not iron-deficient, a multivitamin with iron can be recommended, but its
efficacy is unproven. Similarly, no published data support a role for therapeutic administration of
calcium or iron as treatment for lead poisoning in the absence of deficiency. Such studies are being
conducted.
Blood lead concentrations fall precipitously after completion of chelation, but rebound within weeks,
even if there is no further exposure to lead, due to release of lead from bone stores. In general, the
concentrations do not return to the high values seen before chelation, and a second course of chelation
rarely is indicated.

References:

Binns,HJ, Campbell,C, Brown,MJ for the Advisory Committee on Childhood Lead Poisoning Prevention.
Interpreting and managing blood lead levels of less than 10 mcg/dL in children and reducing childhood
exposure to lead: recommendations of the Centers for Disease Control and Prevention Advisory
Committee on Childhood Lead Poisoning Prevention. Pediatrics. 2007;120:e1285-e1298. Available at:
http://pediatrics.aappublications.org/cgi/content/full/120/5/e1285

Centers for Disease Control and Prevention. Lead program: state and local programs. Available at:
http://www.cdc.gov/nceh/lead/grants/contacts/CLPPP%20Map.htm

Laraque D, Trasande L. Lead poisoning: successes and 21st century challenges. Pediatr Rev.
2005;26:435-443. Available at: http://pedsinreview.aappublications.org/cgi/content/full/26/12/435

Markowitz M. Lead poisoning. Pediatr Rev. 2000;21:327-335. Available at:


http://pedsinreview.aappublications.org/cgi/content/full/21/10/327

Richardson M. Microcytic anemia. Pediatr Rev. 2007;28:5-14.

Rischitelli G, Nygren P, Bougatsos C, Freeman M, Helfand M. Screening for Elevated Lead Levels in
Childhood and Pregnancy: An Update of a 1996 U.S. Preventive Services Task Force Review.
Rockville, Md: Agency for Healthcare Research and Quality, U.S. Department of Health and Human
Services; 2006. Available at: http://www.ahrq.gov/clinic/uspstf06/lead/leadsum.htm

page 466
2009 PREP SA on CD-ROM

Yuan W, Holland SK, Cecil KM, et al. The impact of early childhood lead exposure on brain
organization: a functional magnetic resonance imaging study of language function. Pediatrics.
2006;118:971-977. Available at: http://pediatrics.aappublications.org/cgi/content/full/118/3/971

page 467
2009 PREP SA on CD-ROM

Question: 133
You are evaluating a 15-year-old boy in the emergency department who presents with fever, chills,
malaise, and blood in his urine. On physical examination, he appears comfortable and alert and has a
temperature of 102.7°F (39.3°C), a blood pressure of 110/40 mm Hg, no rashes, and clear breath
sounds. He has a diastolic murmur heard best in the sitting position (Item Q133). You elicit no
abdominal or flank tenderness.

Of the following, the BEST next step in the management of this patient is

A. administration of broad-spectrum antibiotics

B. blood cultures

C. renal ultrasonography

D. transesophageal echocardiography

E. urine culture

page 468
2009 PREP SA on CD-ROM

Critique: 133 Preferred Response: B


The patient described in the vignette has history and physical examination findings that are highly
suggestive of infective endocarditis. These include symptoms of chills and malaise; a history of fever;
and the findings of hematuria, a new murmur, and fever. Typically, the diagnosis is confirmed by
isolation of the offending organism from blood cultures. Blood cultures from three to five sites should be
obtained prior to initiation of antibiotic therapy. Because the bacterial shedding is constant, the
practitioner should not wait until the patient is febrile to obtain blood cultures. Viridans streptococci (eg,
S bovis, S mitis) as well as Staphylococcus aureus are the most common bacterial pathogens causing
endocarditis in children. However, clinicians must be concerned about organisms such as
Enterococcus, coagulase-negative Staphylococcus, fungi, and a group of bacteria referred to as the
HACEK organisms (Haemophilus sp, Actinobacillus actinomycetemcomitans, Cardiobacterium hominis,
Eikenella corrodens, and Kingella kingae). The HACEK organisms are gram-negative oral and
pharyngeal flora that are fastidious and slow-growing, often requiring growth factors and carbon dioxide
to be isolated in cultures.
Treatment of endocarditis depends on the isolated organism. In general, long-term antibiotic
treatment (4 to 6 weeks) is undertaken in an effort to eradicate completely the bacteria that have been
sequestered in a nonvascular vegetation. Surgery is reserved for patients who develop severe
congestive heart failure from severe valve regurgitation or deterioration.
The boy in the vignette requires intravenous antibiotic treatment, but blood cultures should be
obtained before therapy is begun. He also should undergo echocardiography, which may be performed
from the transesophageal approach to improve the sensitivity, but similar to renal ultrasonography, such
a study is performed after blood cultures have been obtained. The absence of vegetation at the time of
echocardiography does not rule out a diagnosis of infective endocarditis. Patients who have infective
endocarditis may exhibit hematuria from the deposition of immune complexes resulting in
glomerulonephritis. Although fever and hematuria may be associated with urinary tract infection, the
presence of a diastolic murmur and absence of urinary symptoms make such a diagnosis unlikely.

References:

Baltimore RS. Infective endocarditis in children. Pediatr Infect Dis J. 1992;11:907-912. Abstract
available at: http://www.ncbi.nlm.nih.gov/pubmed/1454430

Hoyer A, Silberbach M. Infective endocarditis. Pediatr Rev. 2005;26:394-400. Available at:


http://pedsinreview.aappublications.org/cgi/content/full/26/11/394

Taubert KA, Dajani AS. Infective endocarditis. In: Moller JH, Hoffman JIE, eds. Pediatric Cardiovascular
Medicine. Philadelphia, Pa: Churchill Livingstone; 2000:768-779

page 469
2009 PREP SA on CD-ROM

Question: 134
A 6-year-old boy has had difficulty walking and lower leg pain for 2 days. Five days ago, he had fever
and cough that had lasted for 3 days. On physical examination, the child has no fever, and vital signs
are normal, as are cranial nerves, speech, and language. Muscle bulk, tone, and reflexes are normal
and symmetric, but his lower legs are painful to palpation. Serum creatine kinase is 2,000 U/L, and
urine is negative for myoglobin.

Of the following, the MOST likely diagnosis is

A. dermatomyositis

B. Duchenne muscular dystrophy

C. Guillain-Barré syndrome

D. metabolic myopathy

E. viral myositis

page 470
2009 PREP SA on CD-ROM

Critique: 134 Preferred Response: E


A gait disturbance, such as described for the boy in the vignette, can result from a variety of potentially
serious disease processes and, therefore, requires urgent evaluation. The muscle pain and otherwise
normal neurologic findings prompted measurement of serum creatine kinase, which can help to localize
the problem rapidly. The prodrome of an upper respiratory tract illness and rapid onset of symptoms is
suggestive of viral myositis.
Dermatomyositis is a more indolent, chronic process and should not present acutely with muscle
pain. Moreover, dermatomyositis is characterized by specific skin findings such as the heliotrope rash
over the eyelids (Item C134A) and Gottron papules (Item C134B). Duchenne muscular dystrophy
presents with more chronic weakness. Although Guillain-Barré syndrome can present with pain and
weakness, the preserved reflexes, focal pain over the leg muscles, and elevated creatine kinase value
are not consistent with that diagnosis. Metabolic myopathies due to mitochondrial dysfunction can
present with acute pain, weakness, tender muscles, and rhabdomyolysis. However, the prevalence is
much lower than viral myositis.
Influenza A and B and enteroviruses may cause viral myositis. Other causes are in Item C134C.

References:

Moughan B. Musculoskeletal symptom complexes. In: Long SS, Pickering LK, Prober CG, eds.
Principles and Practice of Pediatric Infectious Diseases. New York, NY: Churchill Livingstone;
2003:150-159

Pasternack MS, Swartz MN. Myositis. In: Mandell GL, Bennett JE, Dolin R, eds. Mandell, Douglas, and
Bennett's Principals and Practice of Infectious Diseases. 6th ed. Philadelphia, Pa: Elsevier Churchill
Livingstone; 2005:1194-1203

Roos KL. Viral infections. In: Goetz CG, ed. Textbook of Clinical Neurology. 3rd ed. Saunders Elsevier;
2007:chapt 41

page 471
2009 PREP SA on CD-ROM

Critique: 134

Violaceous discoloration of the cheeks and eyelids (ie, heliotrope) in a child


who has dermatomyositis. (Courtesy of J. Jorizzo)

page 472
2009 PREP SA on CD-ROM

Critique: 134

Gottron papules: Erythematous scaling papules overlying the knuckles in a


child who has dermatomyositis. (Courtesy of D. Krowchuk)

page 473
2009 PREP SA on CD-ROM

Critique: 134

page 474
2009 PREP SA on CD-ROM

Question: 135
The pregnant mother of a child in your practice recently learned that her grandmother had a child who
died of "probable metabolic disease" at 2 days of age. She does not know details, and medical records
on that child no longer are available. The mother asks if her pregnancy can be tested to see if the fetus
could be affected with the same disorder.

Of the following, the MOST accurate statement regarding metabolic disease in the prenatal setting is
that

A. fetuses affected with metabolic diseases are unlikely to come to term

B. knowing the parents’ ethnic backgrounds aids in determining which tests should be offered

C. level 2 ultrasonography during the second trimester is likely to be helpful in detecting metabolic
disease

D. poor fetal growth is common in metabolic diseases

E. prenatal metabolic screening panels are widely available

page 475
2009 PREP SA on CD-ROM

Critique: 135 Preferred Response: B


Most inborn errors of metabolism do not become evident until after birth. Typically, affected fetuses are
normally formed and normally grown, and no abnormalities are detectable on prenatal ultrasonography.
With rare exception, maternal serum screening does not detect metabolic abnormalities in the fetus.
Usually, affected individuals are born at term. Although most of the United States has extended
newborn screening panels to detect a variety of inborn errors, no such panel exists for routine screening
during pregnancy.
The vignette highlights the importance of accessing as much information as possible regarding
family members who die from metabolic causes. Even if a diagnosis cannot be made in a particular
instance, medical records may be helpful in offering the expectant couple guidance or in making a
diagnosis posthumously. Taking a careful family history, with attention to familial ethnic, religious, and
geographic origins, may bring to light conditions that are likely to run in families. For example, because
of the high carrier rate for some metabolic diseases, parents of Ashkenazi Jewish descent, whose
ancestors originated from Eastern Europe, are at increased risk for having children affected by
Tay-Sachs disease (Item C135) and people of French-Canadian heritage are at increased risk for
having children affected by tyrosinemia. More than 100 metabolic disorders can be diagnosed
prenatally if clinicians have sufficient information available to know which disorder may occur in a
family.

References:

Driscoll DA, Sehdev HM, Marchiano DA. Prenatal carrier screening for genetic conditions. NeoReviews.
2004;5:e290-e295. Available for subscription at:
http://neoreviews.aappublications.org/cgi/content/full/5/7/e290

Prenatal diagnosis: emerging technologies for prenatal diagnosis. In: Nussbaum RL, McInnes RR,
Willard HF, eds. Thompson & Thompson Genetics in Medicine. 7th ed. Philadelphia, Pa: Elsevier
Saunders; 2007:456

page 476
2009 PREP SA on CD-ROM

Critique: 135

page 477
2009 PREP SA on CD-ROM

Question: 136
A 17-year-old young man comes to your office for a preparticipation sports physical examination for
high school wrestling. He reports that his coach would like him to wrestle in a weight category that is 10
lb less that his current weight. After you determine that his desired weight is in the range of healthy
weight for his height, you counsel him regarding safe weight loss.

Of the following, the MOST appropriate statement regarding healthy weight control practices for young
athletes is that

A. an appropriate diet for most athletes consists of a minimum of 1,500 kcal/day

B. dehydration causes greater body heat storage, reduces blood volume, and results in increased
exercise tolerance

C. most high school boys who participate in "weight-sensitive" sports practice unhealthy weight loss
behaviors

D. optimal values for body composition have been established for all sports

E. weight loss beyond 1.5% of body weight per week results in muscle weakness

page 478
2009 PREP SA on CD-ROM

Critique: 136 Preferred Response: E


Sports in which a lean body mass is desirable or competing at the lowest possible weight is seen as an
advantage include distance running, cross country skiing, swimming and diving, dance, figure skating,
gymnastics, and weight-class football and wrestling. Other sports, such as football, rugby, basketball,
and power lifting, emphasize weight gain in the form of lean body mass. Whether attempting to lose
weight and body fat or gain weight and muscle mass, some youth may resort to unhealthy dieting and
exercise practices, supplement use, and drug ingestion.
Some practices, including rapid weight loss with voluntary dehydration, the use of anabolic steroids,
and inappropriate use of stimulants or insulin, may be fatal. Inappropriate weight loss also may result in
medical complications such as delayed physical maturation and potential growth impairment;
oligomenorrhea and amenorrhea; development of eating disorders; depression; an increased incidence
of infectious diseases; and renal, endocrine, gastrointestinal, and thermoregulatory disturbances.
Disordered eating behaviors are prevalent in both female and male athletes. It is reported that 10% to
15% of high school boys in "weight-sensitive sports" practice unhealthy weight loss behaviors, with
numerous studies reporting such behaviors in wrestlers.
Voluntary dehydration frequently is used to lose weight, and techniques can include fluid restriction,
spitting, use of laxatives and diuretics, wearing of rubber suits during activities to cause sweating, and
excessive sweating in saunas and steam baths. Dehydration results in greater body heat storage,
causing an excessive increase in the core body temperature and a decreased blood volume, and
decreased exercise tolerance. It also increases the risk of heat-related illness.
No optimal values for body composition have been established for any sport. Rather than a specific
percentage of body fat for an individual athlete, a range that is realistic and appropriate should be
recommended. If it is appropriate that an adolescent loses weight, healthy weight loss should not
exceed 1.5% of the total body weight per week or 1 to 2 lb/wk. Weight loss beyond these guidelines
results in the breakdown and metabolism of muscle, causing muscular weakness. An appropriate diet
for most athletes consists of a minimum of 2,000 kcal/day. The ideal method of losing weight is to
consume 1,750 fewer kcal per week and expend 1,750 kcal more per week by exercising. A
well-balanced diet that contains approximately 55% to 65% of calories from carbohydrates, 15% to 20%
of calories from protein, and 20% to 30% of calories from fat is recommended.

References:

American Academy of Pediatrics Council on Sports Medicine and Fitness and Council on School
Health. Active healthy living: prevention of childhood obesity through increased physical activity.
Pediatrics. 2006;117:1834-1842. http://pediatrics.aappublications.org/cgi/content/full/117/5/1834

American Academy of Pediatrics Committee on Sports Medicine and Fitness. Promotion of healthy
weight-control practices in young athletes. Pediatrics. 2005;116:1557-1564. Available at:
http://pediatrics.aappublications.org/cgi/content/full/116/6/1557

McCrory P, Johnston K, Meeuwisse W, et al. Summary and agreement statement of the 2nd
International Conference on Concussion in Sport, Prague 2004. Br J Sports Med. 2005;39:196-204.
Abstract available at: http://www.ncbi.nlm.nih.gov/pubmed/15793085

page 479
2009 PREP SA on CD-ROM

Question: 137
You are evaluating a 2-week-old breastfed infant who is 15% below his birthweight and has been
lethargic and fed poorly over the past 4 days. You administer a normal saline fluid bolus. Laboratory
results include:
· Blood glucose of 126.0 mg/dl (7.0 mmol/L)
· Serum sodium of 170.0 mEq/L (170.0 mmol/L)
· Serum potassium of 5.0 mEq/L (5.0 mmol/L)
· Blood urea nitrogen of 31.0 mg/dL (11.1 mmol/L)
· Serum creatinine of 2.9 mg/dL (256.4 mcmol/L)

Of the following, the MOST appropriate initial fluid for correction is a solution containing 5% dextrose
and

A. NaCl (%): 0.20

KCl (mEq/L): 40
Duration of Infusion(hr): 12 to 24
B. NaCl (%): 0.45

KCl (mEq/L): 0
Duration of Infusion(hr): 48 to 72
C. NaCl (%): 0.45

KCl (mEq/L): 40
Duration of Infusion(hr): 12 to 24
D. NaCl (%): 0.9

KCl (mEq/L): 0
Duration of Infusion(hr): 12 to 24
E. NaCl (%): 0.9

KCl (mEq/L): 40
Duration of Infusion(hr): 48 to 72

page 480
2009 PREP SA on CD-ROM

Critique: 137 Preferred Response: B


Hypernatremia (serum sodium concentration greater than 145.0 mEq/L [145.0 mmol/L]) is common in
hospitalized children. Hypernatremia results from excessive sodium administration (incorrectly mixed
formula, sodium bicarbonate, hypertonic saline) or a deficit of water in relation to sodium. Such a deficit
may occur from decreased water intake (inadequate breastfeeding, fluid restriction, lack of access to
fluids), fluid losses (nephrogenic and central diabetes insipidus, increased insensible losses), or fluid
losses in greater proportion than sodium losses (diarrhea, vomiting, diuretic use, burns).
Treatment of hypernatremia is directed at correction of both the serum sodium concentration and the
circulatory volume. Initially, some children may require isotonic fluid boluses to restore circulation. It is
important to note that children who have hypernatremia often appear less dehydrated than they actually
are due to preservation of the extracellular volume.
The following equation can be used to estimate the free water deficit:

Water deficit (mL)= 4 mL x ideal body weight (kg) x desired change in serum sodium concentration

Hypernatremia, especially if chronic, should be corrected slowly, with a desired goal of decreasing
the serum sodium by 0.5 mEq/L per hour to avoid cerebral edema. Severe hypernatremia (serum
sodium >170.0 mEq/L [170.0 mmol/L]), as described for the child in the vignette, should be corrected
over 48 to 72 hours. Fluid administration generally consists of 1/4 to 1/2 normal saline solutions.
Symptoms of overcorrection, such as changes in mental status or onset of seizures, suggest the
development of cerebral edema and should be treated with hypertonic saline and slowing of the sodium
correction. In general, potassium administration should be withheld in cases of severe hypernatremic
dehydration until adequate urine output is assured.

References:

Greenbaum LA. Electrolyte and acid-base disorders. In: Kliegman RM, Behrman RE, Jenson HB,
Stanton BF, eds. Nelson Textbook of Pediatrics. 18th ed. Philadelphia, Pa: Saunders Elsevier;
2007:267-308

Moritz ML, Ayus JC. Disorders of water metabolism in children: hyponatremia and hypernatremia.
Pediatr Rev. 2002:23:371-380. Available at:
http://pedsinreview.aappublications.org/cgi/content/full/23/11/371

page 481
2009 PREP SA on CD-ROM

Question: 138
You are asked to see a 7-year-boy in whom medulloblastoma (primitive neuroectodermal tumor) was
diagnosed at age 3 years. Treatment at that time consisted of chemotherapy and craniospinal
irradiation. During the past year, he grew 2 cm, although he is eating normally, and his weight is
appropriate for height. Despite spinal irradiation, the upper-to-lower segment ratio is normal for his age.

Of the following, the MOST likely diagnosis is

A. acquired growth hormone deficiency

B. chemotherapy-induced renal failure

C. Cushing syndrome

D. irradiation-induced epiphyseal fusion

E. tumor recurrence

page 482
2009 PREP SA on CD-ROM

Critique: 138 Preferred Response: A


Cranial irradiation has gradual deleterious effects on pituitary hormone secretion due to damage to
hypothalamic releasing factors. Growth hormone, thyroid-stimulating hormone (TSH), and the
gonadotropins tend to be affected most, with adrenocorticotropic hormone (ACTH) relatively protected,
although all neuroendocrine function can be disturbed by cranial irradiation. The boy described in the
vignette has a normal upper-to-lower segment ratio, indicating that his spine still is growing despite
radiation. This suggests that his growth attenuation is due to an acquired endocrine deficiency of either
growth hormone or thyroid hormone. Early in the deficiency process, if this child were tested with
standard stimuli to growth hormone release, he might be able to release growth hormone, but if
continuous nocturnal growth hormone sampling were performed, diminished and disorderly
spontaneous growth hormone release might be documented. With time, secretory function to stimuli
also would be lost.
Renal failure from chemotherapy would be expected to have manifested earlier and been recognized
by this time. Cushing syndrome is a possible but rare cause of growth attenuation caused by adrenal or
pituitary oversecretion of cortisol or ACTH, respectively, and is not associated with cranial irradiation.
Irradiation-induced epiphyseal fusion at the level of the spine is common following spinal irradiation.
The epiphyseal fusion of the spine results in an increased lower segment (measured from the pubic
symphysis to the heel) compared with the upper segment because the legs continue to grow while the
spine does not. This results in an increased lower segment-to-upper segment ratio. Tumor recurrence
has more obvious manifestations than statural growth attenuation.

References:

Ballonoff A, Kavanagh B. Complications of cranial irradiation. UpToDate Online 15.3. 2008. Available
for subscription at: http://www.uptodateonline.com/utd/content/topic.do?topicKey=rad_ther/2462

Darzy KH, Pezzoli SS, Thorner MO, Shalet SM. The dynamics of growth hormone (GH) secretion in
adult cancer survivors with severe GH deficiency acquired after brain irradiation in childhood for
nonpituitary brain tumors: evidence for preserved pulsatility and diurnal variation with increased
secretory disorderliness. J Clin Endocrinol Metab. 2005;90:2794-2803. Available at:
http://jcem.endojournals.org/cgi/content/full/90/5/2794

Rose SR. Growth failure after childhood cancer: role of growth hormone deficiency. UpToDate online
15.3. 2008. Available for subscription at:
http://www.uptodateonline.com/utd/content/topic.do?topicKey=ped_onco/2817

page 483
2009 PREP SA on CD-ROM

Question: 139
A 24-month-old child has been evaluated and found eligible for early intervention services because of
language delay. His mother is reluctant to pursue therapy because she feels that his language will
improve without intervention.

Of the following, the MOST appropriate response is to

A. agree with the mother that his speech probably will improve without therapy

B. explain that therapy at this age is parent-based training to promote appropriate development in the
home setting

C. explain that therapy is critical for the child or the child will have regression in her language
development

D. postpone therapy pending re-evaluation in 6 months

E. warn the mother that if she does not enroll the child in therapy, she can be reported for child neglect

page 484
2009 PREP SA on CD-ROM

Critique: 139 Preferred Response: B


Federal legislation for the provision of services to infants, toddlers, and preschool children who have
disabilities has evolved since 1986. From its inception as part of PL99-457 (the Education of the
Handicapped Amendments) through the most recent changes defined in PL 105-17 (Individuals with
Disabilities Education Act, Part C), legislative efforts uphold the rights of students and parents to the
key components of a free and appropriate public education. Federal regulations define early
intervention services as services that "are designed to meet the developmental needs of each child
eligible under this part and the needs of the family related to enhancing the child's development."
The child described in the vignette will benefit from early intervention therapy, the goal of which is to
enhance the development of infants and toddlers who have disabilities and minimize their potential for
developmental delays. The services are designed to meet the needs of the child and family and
promote the child's development in natural environments. Therapy at this stage may be selected by the
parents, and parent-based training in the home is most appropriate for this child. Because language
delays have been identified for this boy, his parents should be encouraged to initiate services without
delay. His speech probably will improve more slowly without therapy, but it will not regress without
therapy.
Participation in the Part C early intervention system for infants and toddlers is voluntary for the
family; they have the right to accept or decline specific early intervention services. Thus, declining such
services does not constitute child neglect.
Early intervention services were established to allow children who have developmental problems to
reach their potential. Research studies have identified specific times in which a child's brain is
especially efficient at learning specific information. Coordinated, community-based multidisciplinary
programs for early intervention represent effective public policy because they not only help to improve
some children's cognitive outcome, but they also aid in family functioning.

References:

American Academy of Pediatrics Committee on Children With Disabilities. Role of the pediatrician in
family-centered early intervention services. Pediatrics. 2001;107:1155-1157. Available at:
http://pediatrics.aappublications.org/cgi/content/full/107/5/1155

American Academy of Pediatrics Committee on Children With Disabilities. The pediatrician's role in
development and implementation of an Individual Education Plan and/or and Individual Family Service
Plan. Pediatrics. 1999;104:124-127. Available at:
http://pediatrics.aappublications.org/cgi/content/full/104/1/124 Policy reaffirmed. Pediatrics.
2006;117:1846-1847. Available at: http://pediatrics.aappublications.org/cgi/content/full/117/5/1846

page 485
2009 PREP SA on CD-ROM

Question: 140
A 10-year-old boy presents with a 1-day history of fever and a swollen leg. According to his mother, the
boy developed a small abrasion on his leg while playing outside 3 days ago. Last night he began to
complain of pain in the area and had a low-grade fever. This morning his temperature was 102.4°F
(39.1°C) and the area around the abrasion looked very red and was tender to palpation. About 2 hours
later, the swelling had increased. Physical examination reveals a boy in no apparent distress who has a
temperature of 101.4°F (38.6°C), a heart rate of 93 beats/min, a respiratory rate of 23 breaths/min, and
a blood pressure of 95/65 mm Hg. All other findings are normal, except for a small erythematous
abrasion just above the medial malleolus that has no discharge. Erythema from this area extends to a
well-demarcated region of the mid-calf and is tender to touch (Item Q140).

Of the following, the MOST likely pathogen is

A. Pseudomonas aeruginosa

B. Staphylococcus aureus

C. Streptococcus pneumoniae

D. Streptococcus pyogenes

E. Vibrio vulnificans

page 486
2009 PREP SA on CD-ROM

Question: 140

Erythema, as described for the boy in the vignette. (Courtesy of D. Krowchuk)

page 487
2009 PREP SA on CD-ROM

Critique: 140 Preferred Response: D


Cellulitis is an infection and inflammation of the connective tissue that involves the dermis and often is
preceded by a break in the skin. Cellulitis also can be seen in patients who have an underlying
condition that predisposes them to such an illness (eg, lymphatic stasis, diabetes mellitus).
Streptococcus pyogenes and Staphylococcus aureus are the two most common bacterial agents
responsible for the development of this infection. Although distinction between the two pathogens is
difficult, the rapidly spreading erythema without purulence described for patient in the vignette is most
consistent with S pyogenes infection (Item C140A). Infections caused by S aureus tend to be more
localized and produce purulent material. Confirmation of the etiologic agent requires recovery of the
organism from an aspirate of the most erythematous area or a culture from purulent exudates or from
the blood.
For most patients, initial empiric therapy is with an antimicrobial agent that has activity against both
of these gram-positive cocci. Empiric treatment courses have changed recently, with the increased
prevalence of methicillin-resistant S aureus (MRSA). Mild forms of cellulitis may be treated with a
topical antimicrobial ointment (eg, mupirocin). More advanced cases require the addition of an oral
antimicrobial agent such as clindamycin, trimethoprim-sulfamethoxazole, or doxycycline in geographic
areas that are highly endemic for MRSA. Cephalexin or amoxicillin-clavulanate is an adequate initial
empiric agent in a community that has less than a 5% prevalence of MRSA. Severe forms of cellulitis
require intravenous antimicrobial agents and possibly even surgical debridement.
Pseudomonas aeruginosa can cause cellulitis but is more common when the patient has a history of
a penetrating injury to the skin that involves water (eg, cut on the leg from a stick in a creek), is
immunocompromised, or has diabetes mellitus. S pneumoniae occasionally causes cellulitis but is not a
common cause of this infection. Vibrio vulnificans can cause a life-threatening form of cellulitis termed
"necrotizing fasciitis," but such infections are associated with injuries from fish or objects in salt water in
an immunocompromised patient (Item C140B).

References:

Jaggi P, Shulman ST. Group A streptococcal infections. Pediatr Rev. 2006;27:99-105. Available at:
http://pedsinreview.aappublications.org/cgi/content/full/27/3/99

Todd JK. Staphylococcal infections. Pediatr Rev. 2005;26:444-450. Available at:


http://pedsinreview.aappublications.org/cgi/content/full/26/12/444

page 488
2009 PREP SA on CD-ROM

Critique: 140

Cellulitis caused by Streptococcus pyogenes is characterized by rapidly


spreading erythema. (Courtesy of D. Krowchuk)

page 489
2009 PREP SA on CD-ROM

Critique: 140

Necrotizing fasciitis is characterized by pain, erythema, and swelling that


progress to necrosis and bullae formation. It may be caused by infection with
a number of agents. (Courtesy of Bernard Cohen, MD; DermAtlas;
www.dermatlas.org)

page 490
2009 PREP SA on CD-ROM

Question: 141
You are evaluating a previously healthy 3-year-old boy for a 3-day history of nausea; vomiting; and
profuse watery, nonbloody diarrhea that has worsened over the last 24 hours. He can keep down water
and an oral electrolyte maintenance solution but has no interest in eating solid food. The family returned
5 days ago from a 1-week vacation at a resort in Acapulco, Mexico. On physical examination, the
tired-appearing little boy has a temperature of 100.8°F (38.3°C), moist mucous membranes, and a soft
abdomen with mild diffuse tenderness to palpation. Laboratory tests document a peripheral white blood
cell count of 6.7x103/mcL (6.7x109/L); hemoglobin of 12.0 g/dL (120.0 g/L); platelet count of
230.0x103/mcL (230.0x109/L); and a differential count of 50% neutrophils, 40% lymphocytes, and 10%
monocytes.

Of the following, the MOST likely pathogen causing this patient’s condition is

A. Campylobacter jejuni

B. Escherichia coli

C. Giardia lamblia

D. Salmonella sp

E. Shigella sp

page 491
2009 PREP SA on CD-ROM

Critique: 141 Preferred Response: B


The profuse watery, nonbloody diarrhea and history of vacationing in Mexico described for the child in
the vignette are consistent with traveler's diarrhea. By far, the most common cause of traveler's
diarrhea is enterotoxigenic Escherichia coli (ETEC), accounting for up to 75% of cases.
There are at least five types of diarrhea-producing E coli strains, which differ in epidemiologic
features and diarrhea type (Item C141).
The primary treatment of all diarrheal disease is to provide adequate fluid replacement by the oral or
parenteral routes. The use of additional antibiotic therapy depends on the type of E coli that is
suspected of causing the illness. Treatment of EHEC infections is entirely supportive. Antibiotics are
contraindicated because they can induce the expression and release of Shiga toxins, and their use has
been associated with a higher risk of hemolytic-uremic syndrome in children.
If a patient who has E coli diarrhea does not improve after several days of supportive care and
assays for Shiga toxin are negative, antimicrobial therapy can be considered. Antimicrobial agents that
may be used in the treatment of severe watery ETEC diarrhea in a traveler or for dysentery caused by
EIEC strains include trimethoprim-sulfamethoxazole, azithromycin, or ciprofloxacin. Selection of the
agent is based on patient age and antimicrobial susceptibility of the organism. Data regarding the
treatment of EAEC infections with antibiotics are sparse, and optimal therapy is unknown.
Shigella, Salmonella, and Campylobacter sp are much less common causes of traveler's diarrhea,
but may be responsible for up to 30% of cases, 15% of cases, and 15% of cases, respectively. Giardia
lamblia is not recognized as a cause of acute traveler's diarrhea.

References:

American Academy of Pediatrics. Escherichia coli diarrhea (including hemolytic-uremic syndrome). In:
Pickering LK, Baker CJ, Long SS, McMillan JA, eds. Red Book: 2006 Report of the Committee on
Infectious Diseases. 27th ed. Elk Grove Village, Ill: American Academy of Pediatrics; 2006:291-296

Donnenberg MS. Enterobacteriaceae. In: Mandell GL, Bennett JE, Dolin R, eds. Mandell, Douglas, and
Bennett's Principles and Practice of Infectious Diseases. 6th ed. Philadelphia, Pa: Elsevier Churchill
Livingstone; 2005:2567-2586

Ericsson CD, DuPont HL. Travelers' diarrhea: approaches to prevention and treatment. Clin Infect Dis.
1993;16:616-626

Guerrant RL, Bobak DA. Nausea, vomiting, and noninflammatory diarrhea. In: Mandell GL, Bennett JE,
Dolin R, eds. Mandell, Douglas, and Bennett's Principles and Practice of Infectious Diseases. 6th ed.
Philadelphia, Pa: Elsevier Churchill Livingstone; 2005:1236-1249

Nataro JP, Kaper JB. Diarrheogenic Escherichia coli. Clin Microbiol Rev. 1998;11:142-201. Available at:
http://cmr.asm.org/cgi/content/full/11/1/142?view=long&pmid=9457432

page 492
2009 PREP SA on CD-ROM

Critique: 141

page 493
2009 PREP SA on CD-ROM

Question: 142
A 5-year-old boy presents with dark red urine, fever, and rhinorrhea. He was well until 2 days ago, when
he developed rhinorrhea and mild cough. He denies urgency, frequency, dysuria, back pain, or
musculoskeletal complaints. On physical examination, the slightly ill-appearing boy has a temperature
of 99.5°F (37.5°C), heart rate of 130 beats/min, respiratory rate of 18 breaths/min, and blood pressure
of 104/58 mm Hg. He has pale conjunctivae, mild scleral icterus, a hyperdynamic precordium, and a I/VI
systolic murmur at the left upper sternal border. There is no edema, and musculoskeletal and
neurologic examination results are normal. Urinalysis results include: red appearance, a specific gravity
of 1.030, pH of 6.5, 3+ blood, and no protein. Microscopy reveals fewer than 5 red blood
cells/high-power field (HPF), fewer than 5 white blood cells/HPF, and no casts.

Of the following, the BEST initial test to obtain is

A. complete blood count with manual differential count

B. creatine phosphokinase measurement

C. renal/bladder ultrasonography

D. urine culture

E. viral culture of urine for adenovirus

page 494
2009 PREP SA on CD-ROM

Critique: 142 Preferred Response: A


The differential diagnosis for causes of red urine includes hematuria, hemoglobinuria, myoglobinuria,
and porphyrinuria. Some urinary tract pathogens (eg. Serratia marcescens) also can be associated with
red urine. In addition, red urine can result from ingestion of certain foods (beets, blackberries, or food
dyes) and medications (deferoxamine, rifampin, and phenolphthalein).
The initial test for evaluating a child who has red urine is the urinalysis to look for the presence or
absence of blood. The 3+ blood on a dipstick described for the child in the vignette narrows the
differential diagnosis to hematuria, hemoglobinuria, or myoglobinuria. The dipstick test incorporates an
indicator impregnated into the paper strip that, upon contact with free hemoglobin, myoglobin, or intact
red blood cells, oxidizes the indicator to result in a blue color change. This dipstick test is exquisitely
sensitive to blood to the level of 5 red blood cells/high-power field. The fewer than 5 red blood cells/high
power field on microscopy described for the patient in the vignette eliminates hematuria. The presence
of tachycardia, pale conjunctivae, and a systolic murmur is suggestive of anemia, and scleral icterus
supports a possible hemolytic anemia. Accordingly, a complete blood count to assess for anemia is the
most appropriate next laboratory test to obtain for this boy.
The absence of musculoskeletal complaints and normal musculoskeletal and neurologic
examination results make myoglobinuria (usually related to rhabdomyolyis) unlikely. If myoglobinuria
was suspected, measurement of creatine phosphokinase would be useful.
Renal/bladder ultrasonography is helpful in evaluating patients who have bright red urine, with clots
and red blood cells noted on urinalysis. Such findings are suggestive of a structural cause for the red
urine, such as renal or bladder mass or stones.
Red urine caused by bacterial or viral infections is due to hematuria. Thus, culture of the urine for
bacteria or viral culture for adenovirus is inappropriate because this boy does not have hematuria.

References:

Kalia A, Travis LB. Hematuria, leukocyturia, and cylindruria. In: Edelmann CM Jr, Bernstein J, Meadow
SR, Spitzer A, Travis LB, eds. Pediatric Kidney Disease. 2nd ed. Boston, Ma: Little, Brown and
Company; 1992:553-563

Moxey-Mims M. Hematuria and proteinuria. In: Kher KK, Schnaper HW, Makker SP, eds. Clinical
Pediatric Nephrology. 2nd ed. London, England: Informa Healthcare; 2007:129-141

Schwartz G. Clinical assessment of renal function. In: Kher KK, Schnaper HW, Makker SP, eds. Clinical
Pediatric Nephrology. 2nd ed. London, England: Informa Healthcare; 2007:71-93

page 495
2009 PREP SA on CD-ROM

Question: 143
You are evaluating a 16-year-old girl during her biannual asthma follow-up visit. She has a history of
mild persistent asthma that is well-controlled on a low-dose inhaled corticosteroid. You review the
asthma guidelines and recommend that she receive the influenza vaccine. Her mother immediately
replies, "Oh no, my daughter has a severe egg allergy and was told to never get the influenza vaccine."

Of the following, the vaccine that is contraindicated in a patient who has a severe immunoglobulin
E-mediated egg allergy is

A. human papillomavirus vaccine

B. measles-mumps-rubella

C. tetanus diphtheria

D. varicella

E. yellow fever

page 496
2009 PREP SA on CD-ROM

Critique: 143 Preferred Response: E


Of currently available vaccines, the measles-mumps-rubella (MMR) vaccine, trivalent influenza vaccine
(TIV), live attenuated intranasal influenza vaccine (LAIV), and yellow fever vaccine all contain small
amounts of egg protein. All of these vaccines are contraindicated in patients who have egg allergies
except the MMR. Despite being developed in chick embryos, the MMR vaccine contains only 0.5 to 1
ng of ovalbumin per dose, a quantity far lower than that required to cause an immunoglobulin (Ig)
E-mediated reaction. Patients who have experienced severe anaphylaxis to egg in the past generally
should not receive the TIV, LAIV, or yellow fever vaccine.
Children who have mild egg allergies (eg, atopic dermatitis, cutaneous-only symptoms) and who are
at high risk for complications from influenza should be offered the TIV. Protocols for skin testing,
two-step dose challenge, single-dose challenge, and desensitization are available for such patients.
Clinicians should remember that many patients who have experienced adverse food reactions in the
past are not truly allergic to that food. Pursuing skin or serum allergy testing to egg, particularly if
negative, will aid in counseling the families on the risks of future influenza vaccination and therapeutic
options.
Although diphtheria and tetanus toxoids and acellular pertussis, human papillomavirus, and varicella
vaccines do not contain egg protein, other vaccine components may result in a reaction.

References:

American Academy of Pediatrics. Active immunization. In: Pickering LK, Baker CJ, Long SS, McMillan
JA, eds. Red Book: 2006 Report of the Committee on Infectious Diseases. 27th ed. Elk Grove Village,
Ill: American Academy of Pediatrics; 2006:9-10

American Academy of Pediatrics. Reporting of adverse events. Reporting of vaccine-preventable


diseases. In: Pickering LK, Baker CJ, Long SS, McMillan JA, eds. Red Book: 2006 Report of the
Committee on Infectious Diseases. 27th ed. Elk Grove Village, Ill: American Academy of Pediatrics;
2006:41-50

Cerecedo Carballo I, Dieguez Pastor MC, Bartolomé Zavala B, Sánchez Cano M, de la Hoz Caballer B.
Safety of measles-mumps-rubella vaccine (MMR) in patients allergic to eggs. Allergol Immunopathol
(Madr). 2007;35:105-109. Available at:
http://db.doyma.es/cgi-bin/wdbcgi.exe/doyma/mrevista.pubmed_full?inctrl=05ZI0102&rev=105&vol=35&
num=3&pag=105

Cox JE, Cheng TL. In brief: egg-based vaccines. Pediatr Rev. 2006;27:118-119. Available at:
http://pedsinreview.aappublications.org/cgi/content/full/27/3/118

Piquer-Gibert M, Plaza-Martín A, Martorell-Aragonés A, et al. Recommendations for administering the


triple viral vaccine and antiinfluenza vaccine in patients with egg allergy. Allergol Immunopathol (Madr).
2007;35:209-212. Available at:
http://db.doyma.es/cgi-bin/wdbcgi.exe/doyma/mrevista.pubmed_full?inctrl=05ZI0102&rev=105&vol=35&
num=5&pag=209

page 497
2009 PREP SA on CD-ROM

Question: 144
A 3-month-old infant is brought to the office for fussiness, increased sleeping, and poor feeding.
According to his mother, he was doing well until 4 days ago, when his formula intake decreased from 6
oz every 3 to 4 hours to 1 to 2 oz every 4 hours and she had to awaken him to feed. He has had no
vomiting, diarrhea, or fever. He was born at term, and the mother had no antenatal infections. On
physical examination, the infant is difficult to console and has a high-pitched cry. His temperature is
98.2°F (36.8°C), heart rate is 160 beats/min, and respiratory rate is 30 breaths/min. His anterior
fontanelle is flat, pupils are 4 mm and equally reactive, and there is no evidence of corneal abrasions.
His lungs are clear, heart sounds are normal, and abdominal evaluation findings are benign. His
extremities are warm, well-perfused, and have normal tone. Results of the initial laboratory evaluation,
including a complete blood count with differential count, electrolytes, and urinalysis, are normal. The
fecal occult blood test result is negative.

Of the following, the MOST appropriate next study is

A. abdominal ultrasonography

B. chest radiography

C. computed tomography scan of the brain

D. serum ammonia determination

E. urine organic acid screen

page 498
2009 PREP SA on CD-ROM

Critique: 144 Preferred Response: C


The differential diagnosis of the irritable infant is extensive and includes conditions that affect all organ
systems (Item C144A). The evaluation should be based on a complete history and physical examination
as well as a high index of suspicion for serious occult causes. For the patient described in the vignette,
the concern should be high for nonaccidental head injury, despite the lack of trauma history or external
or cutaneous findings on physical examination, and should prompt the physician to obtain neuroimaging
(eg, computed tomography scan or magnetic resonance imaging). Of note, recent studies have
reconfirmed the incidence of occult head injuries and the importance of neuroimaging in the evaluation
of suspected child abuse. Almost 30% of children undergoing child abuse evaluation in one study had
occult brain injury despite the absence of neurologic symptoms, and as many as 10% of brain injuries
may be missed if only skeletal surveys and ophthalmologic examinations are performed.
The presenting signs and symptoms of nonaccidental head trauma due to inflicted traumatic brain
injury (also known as "shaken baby syndrome") often are nonspecific. It is estimated that as many of
30% of cases are not diagnosed initially, in part because the findings may be attributed to other
conditions, such as viral syndrome, colic, or formula intolerance. The history also may be misleading,
with most caretakers reporting no trauma. In some instances, the sole finding may be a
disproportionately large head circumference.
The absence of external findings is related largely to the biomechanics of the injurious event.
Vigorous shaking, with or without impact, leads to traction on the dural bridging veins. Shearing of these
veins causes bleeding into the subdural space (Item C144B). Especially in those cases where there
was no impact of the infant's head during the shaking episode, bruising or swelling is likely to be
absent.
Abdominal ultrasonography to evaluate for intussusception or hydronephrosis, serum ammonia and
urine organic acid determinations to evaluate for metabolic errors, or chest radiography to look for
cardiomegaly or pulmonary infiltrates may be indicated in the evaluation of the irritable infant. However,
in contrast to abusive head trauma, it is likely that signs, symptoms, or abnormalities on screening
laboratory evaluations would provide clues to these other diagnoses.

References:

Herman M, Le A. The crying infant. Emerg Med Clin North Am. 2007;25:1137-1159. Abstract available
at: http://www.ncbi.nlm.nih.gov/pubmed/17950139

Keenan HT, Runyan DK, Marshall SW, Nocerna MA, Merten DF. A population-based comparison of
clinical and outcome characteristics of young children with serious inflicted and noninflicted traumatic
brain injury. Pediatrics. 2004;114:633-639. Available at:
http://pediatrics.aappublications.org/cgi/content/full/114/3/633

Laskey AL, Holsti M, Runyan DK, Socolar RRS. Occult head trauma in young suspected victims of
physical abuse. J Pediatr. 2004;144:719-722. Abstract available at:
http://www.ncbi.nlm.nih.gov/pubmed/15192615

Newton AW, Vandeven AM. Update on child maltreatment with a special focus on shaken baby
syndrome. Curr Opin Pediatr. 2005;17:246-251. Abstract available at:
http://www.ncbi.nlm.nih.gov/pubmed/15800421

Sirotnak AP, Grigsby T, Krugman R. Physical abuse of children. Pediatr Rev. 2004;25:264-277.
Available at: http://pedsinreview.aappublications.org/cgi/content/full/25/8/264

page 499
2009 PREP SA on CD-ROM

Critique: 144

page 500
2009 PREP SA on CD-ROM

Critique: 144

Computed tomography scan without contrast in a 3-year-old who was the


victim of nonaccidental trauma shows a large left subdural hematoma
(arrows) with 4-mm left-to-right midline shift. (Courtesy of D. Krowchuk)

page 501
2009 PREP SA on CD-ROM

Question: 145
A 13-year-old boy who has a 1-year history of abdominal pain in the epigastric and periumbilical regions
presents for further evaluation. According to his history, the pain occurs one to three times per week
and sometimes interferes with school attendance and physical activity. Findings on physical
examination are normal. You review the diagnostic studies that have been performed in the past year.

Of the following, the finding that MOST warrants referral for upper endoscopy is

A. abnormal lactose breath hydrogen test result

B. elevated serum alkaline phosphatase value

C. elevated serum amylase value

D. mild anemia (hematocrit, 33% [0.33])

E. positive tissue transglutaminase antibody

page 502
2009 PREP SA on CD-ROM

Critique: 145 Preferred Response: E


Most children who have recurrent abdominal pain (RAP) have functional abdominal pain. Therefore,
most affected children do not require gastrointestinal endoscopy. However, if certain conditions are
strongly suspected, endoscopy can be an essential part of the diagnostic evaluation. Such conditions
include peptic ulcer disease, bleeding esophageal varices, reflux esophagitis, eosinophilic esophagitis,
inflammatory bowel disease, and celiac disease. Most of these conditions present with symptoms
considered "red flags," including chronic diarrhea, weight loss, unexplained fever, hematemesis, or
lower gastrointestinal tract bleeding. The choice of either upper gastrointestinal endoscopy or lower
gastrointestinal endoscopy (colonoscopy) is based on the patient's clinical symptoms. Tissue forceps
biopsies can be taken through the endoscope and tissue sent to the pathologist for identification of
subtle lesions (eg, esophagitis, celiac disease, giardiasis, Helicobacter pylori gastritis).
The strongest indication for upper endoscopy for the patient described in the vignette is a positive
tissue transglutaminase antibody test. This antibody is a sensitive and specific test for celiac disease,
although some false-positive results can be seen. Because a lifelong gluten-free diet is the only
treatment for celiac disease, it is still recommended that patients undergo endoscopy with duodenal
biopsies to establish the diagnosis. The endoscopic visual appearance may be normal in patients who
have celiac disease, but in severe cases, "scalloped" duodenal folds may be seen. Generally, three to
four biopsies of the duodenum are needed to evaluate for celiac disease; histologic features include
intraepithelial lymphocytosis and villous atrophy.
Elevated alkaline phosphatase values suggest normal adolescent bone growth, bone inflammation,
osteopenia, or hepatobiliary disease. Elevated serum amylase values suggest pancreatitis, and routine
upper endoscopy is rarely helpful in affected patients (although endoscopic retrograde
cholangiopancreatography may be useful in evaluating recurrent pancreatitis). Mild anemia is a
nonspecific finding that can be seen for many different reasons. An abnormal lactose breath test result
suggests lactose intolerance, which does not require endoscopic confirmation.

References:

Fox VL. Gastrointestinal endoscopy: patient preparation and surgical considerations. In: Walker WA,
Goulet O, Kleinman RE, Sherman PM, Shneider BL, Sanderson IR, eds. Pediatric Gastrointestinal
Disease. 4th ed. Hamilton, Ontario, Canada: BC Decker; 2004:1666-1673

Thakkar K, Gilger MA, Shulman RJ, El Serag H. Esophagogastroduodenoscopy in children with


abdominal pain: a systematic review. Am J Gastroenterol. 2007;102:654-661. Abstract available at:
http://www.ncbi.nlm.nih.gov/pubmed/17222318

page 503
2009 PREP SA on CD-ROM

Question: 146
You are called to the newborn nursery to see a 2.1-kg term infant whose bedside glucose screening
test value is 30 mg/dL (1.7 mmol/L). The nurse describes the baby as being generally lethargic, jittery
with stimulation, and intolerant of oral feeding attempts at 4 hours of age (poor oral suckling and emesis
of the small volumes of formula taken). He was born at 41 weeks’ gestation to a mother who had poor
weight gain, smoked cigarettes, and had hypertension. The Apgar scores following a vaginal delivery
were 6 and 8 at 1 and 5 minutes, respectively. There is no history of maternal diabetes, illicit drug use,
or intrapartum difficulties. On physical examination, the baby’s vital signs are normal except for
tachypnea (respiratory rate of 80 breaths/min), with pulse oximetry of 90% on room air. The infant has
plethora, acrocyanosis, and generalized low tone. He exhibits rapid, shallow tachypnea, with clear
lungs bilaterally on auscultation. There is a soft I/VI systolic murmur along the lower left sternal border
and no gallop. Upon stimulation, he has jittery hand movements. Laboratory findings include:
· Serum glucose, 45.0 mg/dL (2.5 mmol/L)
· White blood cell count, 7.0x103/mcL (7.0x109/L) with a normal differential count
· Platelet count, 150.0x103/mcL (150.0x109/L)
· Hematocrit, 70% (0.70)
An arterial blood gas reveals a pH of 7.40, Pao2 of 75 mm Hg, Paco2 of 30 mm Hg, and base excess of
-7 mEq/L.

Of the following, the MOST appropriate treatment for this infant’s underlying problem is

A. administration of amphotericin B

B. double-volume exchange transfusion

C. intubation and assisted ventilation

D. partial exchange transfusion

E. phototherapy

page 504
2009 PREP SA on CD-ROM

Critique: 146 Preferred Response: D


By definition, polycythemia exists when the hematocrit (HCT) is greater than 65% (0.65). This condition
occurs in newborns who are small for gestational age, infants of diabetic mothers, the recipient twin in a
twin-twin transfusion syndrome-affected pregnancy, or infants who have delayed clamping of the
umbilical cord after delivery. The decision to treat polycythemia is contingent upon the presence of
symptoms associated with the state of hyperviscosity of the circulating blood conferred upon it by the
increased HCT or an HCT of greater than 70% (0.70). Such symptoms include those described for the
infant in the vignette as well as those described in the Table (Item C146). Although blood hyperviscosity
also may occur due to markedly elevated white blood cell numbers (generally, >100.0x103/mcL
[100.0x109/L]) or with elevations of certain plasma protein fractions, in the newborn, it is almost
exclusively related to an increase in the red blood cell mass, as reflected in the HCT.
The treatment of choice for symptomatic polycythemia, as seen in this infant, is a partial exchange
transfusion, which reduces the red blood cell mass and maintains a euvolemic state by the
administration of crystalloid (eg, normal saline). The partial exchange transfusion is partial in that it
removes only a portion of the circulating volume of blood (as opposed to a complete or double-volume
exchange transfusion) and replaces (exchanges) it with crystalloid. The volume of such an exchange is
based on the following formula:

Volume of exchange (mL) =


[Infant's blood volume] x [Observed HCT-Desired HCT]/Observed HCT

The blood volume of an infant who has polycythemia is 100 mL/kg.


For a 3-kg infant who has an observed HCT of 70% (0.70), the volume of exchange is:

= [3 kg x 100 mL/kg] x [0.70-0.55]/0.70


= 300 mL x 0.15/0.70
= 300 mL x 0.214
= 64 mL

Polycythemia cannot be treated solely with intravenous crystalloid because this fluid leaves the
circulatory compartment easily. Because the patient does not have evidence of systemic fungal
infection, amphotericin B is not indicated and would not treat polycythemia. The infant in the vignette
does not have hypoxemia or hypercarbia that warrants intubation and assisted ventilation. Phototherapy
does not treat polycythemia, only the hyperbilirubinemia that follows. A double-volume exchange
transfusion is used to treat severe hyperbilirubinemia.

References:

Ceriani Cernadas JM, Carroli G, Pellegrini L, et al. The effect of timing of cord clamping on neonatal
venous hematocrit values and clinical outcome at term: a randomized, controlled trial. Pediatrics.
2006;117:e779-e786. Available at: http://pediatrics.aappublications.org/cgi/content/full/117/4/e779

Linderkamp O. Blood viscosity of the neonate. NeoReviews. 2004;5:e406-e416. Available for


subscription at: http://neoreviews.aappublications.org/cgi/content/full/5/10/e406

Luchtman-Jones L, Schwartz AL, Wilson DB. Blood component therapy for the neonate. In: Martin RJ,
Fanaroff AA, Walsh MC, eds. Fanaroff and Martin's Neonatal-Perinatal Medicine. 8th ed. Philadelphia,
Pa: Mosby Elsevier; 2006:1344-1356

page 505
2009 PREP SA on CD-ROM

Philip AGS, Saigal S. When should we clamp the umbilical cord? NeoReviews. 2004;5:e142-e154.
Available for subscription at: http://neoreviews.aappublications.org/cgi/content/full/5/4/e142

page 506
2009 PREP SA on CD-ROM

Critique: 146

page 507
2009 PREP SA on CD-ROM

Question: 147
A worried mother brings her 4-year-old son to your office because his right eye has been red for 3 days.
She assumed it was pink eye that he contracted at child care, but she now is concerned because he
has developed swelling in front of his right ear, and his eye has become redder. They live in a wooded
area and got a new kitten 6 weeks ago, but there is no history of the kitten scratching the child. Physical
examination reveals a well-appearing child who has obvious conjunctival injection (Item Q147A) of the
right eye but no discharge or pain. You palpate a 2x2-cm tender, mobile preauricular lymph node (Item
Q147B) and a 2x3-cm anterior cervical lymph node on the right. The remainder of the physical
examination findings are normal.

Of the following, the MOST likely pathogen causing this boy’s symptoms is

A. Bartonella henselae

B. Francisella tularensis

C. Haemophilus influenzae

D. Pasteurella multocida

E. Staphylococcus aureus

page 508
2009 PREP SA on CD-ROM

Question: 147

(Courtesy of M. Rimsza)

page 509
2009 PREP SA on CD-ROM

Question: 147

(Courtesy of M. Rimsza)

page 510
2009 PREP SA on CD-ROM

Critique: 147 Preferred Response: A


The boy described in the vignette has signs of Parinaud oculoglandular syndrome, with painless,
nonpurulent conjunctivitis and ipsilateral preauricular lymphadenopathy (Item C147A). These findings,
combined with the cervical lymphadenopathy and his exposure to a kitten, make cat-scratch disease, or
Bartonella henselae infection, the most likely possibility. Although most patients who have cat-scratch
disease report some contact with kittens or cats, many do not recall being scratched. Cats younger than
1 year of age are most likely to transmit the organism to humans; human-to-human transmission does
not occur.
The most common clinical manifestation of cat-scratch disease is regional lymphadenopathy, with
cervical and axillary areas affected most (Item C147B). A papule (Item C147C) at the site of the scratch
often precedes the development of lymphadenopathy. Lymph nodes may enlarge considerably and may
be tender, warm, and erythematous; suppuration can occur in up to 30% of patients. Nodes may remain
enlarged for several months. Other clinical manifestations of cat-scratch disease include oculoglandular
syndrome, fever of unknown origin, painful osteolytic lesions, and hepatic or splenic microabscesses.
Among the less common manifestations are encephalopathy, thrombocytopenic purpura, and
neuroretinitis.
Cat-scratch disease can be diagnosed by obtaining an indirect immunofluorescent antibody assay
for B henselae. Biopsy of an affected node also may show the bacilli or granulomas but should be
reserved for atypical cases. The course of cat-scratch disease is self-limited. The efficacy of antibiotic
therapy is controversial, but some studies have shown improvement in those who have severe systemic
symptoms and those who are immunocompromised. Several oral agents have shown efficacy:
azithromycin, erythromycin, ciprofloxacin, trimethoprim-sulfamethoxazole, and rifampin. Needle
aspiration of suppurative lymph nodes may hasten recovery and reduce pain, but incision and drainage
is not recommended because of the risk of sinus tract formation and persistent drainage.
Tularemia, infection with Francisella tularensis, also can cause an oculoglandular syndrome, but the
conjunctivitis typically is painful. A history of exposure to ticks or wild rabbits would make this diagnosis
more likely. Haemophilus influenzae can cause conjunctivitis with ipsilateral preauricular
lymphadenopathy, but the conjunctivitis typically is purulent, and ipsilateral otitis media might be
expected. Pasteurella multocida is the most common organism causing infection after a cat bite. Typical
findings include cellulitis and suppuration at the area of the bite; conjunctivitis is rare. Staphylococcus
aureus also can cause purulent conjunctivitis, but the history of kitten exposure for the patient in the
vignette makes this a less likely causative pathogen.

References:

American Academy of Pediatrics. Cat-scratch disease. In: Pickering LK, Baker CJ, Long SS, McMillan
JA, eds. Red Book: 2006 Report of the Committee on Infectious Diseases. 27th ed. Elk Grove Village,
Ill: American Academy of Pediatrics; 2006:246-248.

English R. Cat-scratch disease. Pediatr Rev. 2006;27:123-128. Available at:


http://pedsinreview.aappublications.org/cgi/content/full/27/4/123

page 511
2009 PREP SA on CD-ROM

Critique: 147

Parinaud oculoglandular syndrome (inoculation of the conjunctiva with


ipsilateral preauricular adenopathy) in a 6-year-old boy. (Courtesy of the Red
Book® Online)

page 512
2009 PREP SA on CD-ROM

Critique: 147

Axillary lymphadenopathy in cat-scratch disease. (Courtesy of M. Rimsza)

page 513
2009 PREP SA on CD-ROM

Critique: 147

A papule at the site of inoculation may be observed and precedes the


development of lymphadenopathy by 1 to 2 weeks. (Courtesy of M. Rimsza)

page 514
2009 PREP SA on CD-ROM

Question: 148
A mother brings in her 3-month-old boy because he has had a worsening cough over the past 2 days.
She reports that he has been afebrile but not feeding as well as he normally does. You note a few
coarse breath sounds, rare wheezing, and intermittent subcostal retractions on physical examination.
His respiratory rate is 56 breaths/min and temperature is 101.5°F (38.6°C). There is a family history of
asthma. A trial of albuterol results in minimal improvement in his chest findings. His oxygen saturation is
89% on room air.

Of the following, the next BEST course of action is to

A. arrange for home administration of albuterol

B. hospitalize the boy for intravenous steroid and antibiotic administration

C. hospitalize the boy for observation and supportive care

D. order a complete blood count

E. refer the boy to a pulmonologist

page 515
2009 PREP SA on CD-ROM

Critique: 148 Preferred Response: C


The child described in the vignette clearly exhibits tachypnea for age and has clinical examination
findings consistent with respiratory distress. The indications for hospital admission for a child who has
bronchiolitis are similar to those for any child who has any lower respiratory tract disorder. Respiratory
distress that is indicated by tachypnea or hypoxemia; signs of accessory muscle use; poor feeding or
inability to maintain the airway; signs of respiratory failure such as cyanosis, listlessness, or fatigue; and
dehydration necessitate immediate hospitalization.
At the age of 3 months, the boy is at risk for dehydration and poor feeding due to his inability to suck
and swallow from breast and bottle in conjunction with the reported signs and symptoms.
A trial of bronchodilator did not improve his condition, which is not unusual in acute bronchiolitis. For
the minority of children (<35%) who do respond to albuterol, a trial in the home with very close follow-up
and revisit might be considered. Although nebulized racemic epinephrine has been found to be effective
in some children who have bronchiolitis, home use never has been shown to be safe or effective.
Viral bronchiolitis has a natural course of worsening over the first 3 to 5 days, which represents an
additional concern for the child in the vignette.
The use of antibiotics has not been proven to be helpful in viral bronchiolitis, and the use of steroids
is controversial and not clearly efficacious. For children who have classic signs and symptoms of
bronchiolitis, such as described for this child, a complete blood count is not indicated.
Most cases of viral bronchiolitis resolve without sequelae, although there is some suggestion of an
increased risk for future development of recurrent wheezing in affected children. Bronchiolitis alone is
not an indication for a pediatric pulmonary consultation unless the child fails to improve or exhibits other
signs of underlying pulmonary disease. Once a decision has been made to admit a child who has
bronchiolitis, whether the child should be admitted to an observation, inpatient, or intensive care unit
depends on local or practice preference. There is evidence that the sequential use of respiratory
scoring systems may allow for early discharge or the use of observation units.

References:

Gadomski AM, Bhasale AL. Bronchodilators for bronchiolitis. Cochrane Database Syst Rev.
2006;3:CD001266. Available at:
http://www.mrw.interscience.wiley.com/cochrane/clsysrev/articles/CD001266/frame.html

Lozan JM. Bronchiolitis (updated). BMJ Clinical Evidence. 2007. Available for subscription at:
http://clinicalevidence.bmj.com/ceweb/conditions/chd/0308/0308.jsp

Pelletier AJ, Mansbach JM, Camargo CA Jr. Direct medical costs of bronchiolitis hospitalizations in the
United States.Pediatrics 2006;118:2418-2423. Available at:
http://pediatrics.aappublications.org/cgi/content/full/118/6/2418

Subcommittee on Diagnosis and Management of Bronchiolitis. Clinical practice guideline:diagnosis and


management of bronchiolitis. Pediatrics. 2006;118:1774-1793. Available at:
http://pediatrics.aappublications.org/cgi/content/full/118/4/1774

Willson DF, Horn SD, Hendley JO, Smout R, Gassaway J. Effect of practice variation on resource
utilization in infants hospitalized for viral lower respiratory illness. Pediatrics. 2001;108;851-855.
Available at: http://pediatrics.aappublications.org/cgi/content/full/108/4/851

page 516
2009 PREP SA on CD-ROM

Question: 149
A 15-year-old patient is brought to your office with a complaint of chest pain. She had been healthy until
3 days ago, when she developed a fever. The pain is precordial, referred to the epigastrium, and
exacerbated by deep breathing and coughing. She refuses to lie down and prefers to sit leaning
forward.

Of the following, the MOST likely expected finding on electrocardiography is

A. elevation of the S-T segments

B. first-degree heart block

C. pre-excitation with a delta wave

D. tall peaked T waves

E. T-wave flattening

page 517
2009 PREP SA on CD-ROM

Critique: 149 Preferred Response: A


Chest pain in children and adolescents is a common problem for which patients and parents frequently
seek medical care. The causes of chest pain in the pediatric population are varied and can be
considered by organ systems: musculoskeletal, respiratory, gastrointestinal, psychological, and cardiac.
Among the musculoskeletal causes are chest wall strain, trauma, costochondritis, and the precordial
catch syndrome. Respiratory causes include asthma, pneumonia, pneumothorax, pneumomediastinum,
and chronic cough. Chest pain may result from gastritis, esophagitis, or indigestion. Psychogenic
processes, including anxiety, fear, and attention-seeking behaviors, may cause or exacerbate chest
pain. Perhaps the most common causes of chest pain in pediatrics are those referred to as idiopathic.
Such a diagnosis often is given to the patient who presents with a 1- to 2-week history of intermittent,
brief, sharp, or stabbing pain that is not associated with exercise or exertion.
The cardiac causes of chest pain are important to recognize because they can be associated with
significant morbidity and mortality. Pericardial pain (resulting from inflammation and often associated
with pericarditis), angina and myocardial ischemia, arrhythmias, or aortic dissection may cause chest
pain. The common causes of pericarditis are viral, inflammatory, and rheumatologic. The typical pain of
pericarditis frequently is substernal, positional, and can be severe. Patients often prefer to sit leaning
forward, as described for the girl in the vignette, and may refuse to lie down. The pain worsens with
deep inspiration, coughing, or movement of the upper torso. Because the pericardium is inflamed,
pericardial effusion may occur in affected patients, which may lead to pericardial tamponade. Some, but
not all, pericardial effusions in pericarditis have the associated finding of a friction rub noted on
auscultation. The absence of a rub does not exclude pericardial effusion or pericarditis.
Electrocardiographic findings can include S-T segment elevation, low voltage, or in cases of large
pericardial effusion, electrical alternans. The latter finding is a beat-to-beat variation in voltage that likely
results from the pendular motion of the heart in the effusion.
First-degree heart block (Item C149A) is not likely in pericarditis. Pre-excitation is seen in patients
who have Wolff-Parkinson-White syndrome (Item C149B) rather than pericarditis. Abnormalities of the T
waves usually implicate an electrolyte abnormality. Tall, peaked T waves are seen with hyperkalemia
(Item C149C), and flattened T waves are a nonspecific finding that may be seen with hypokalemia (Item
C149D) or certain ventricular strain patterns.

References:

Cava J, Sayger PL. Chest pain in children and adolescents. Pediatr Clin North Am. 2004;51:1553-1568.
Abstract available at: http://www.ncbi.nlm.nih.gov/pubmed/15561173

Demmler GJ. Infectious pericarditis in children. Pediatr Infect Dis J. 2006;25:165-166

Fahey J. Chest pain. In: Rudolph C, Rudolph A, eds. Rudolph's Pediatrics. 21st ed. New York, NY:
McGraw Hill Medical Publishing Division; 2003:1894-1897

Nowlen TT, Bricker JT. Pericardial disease. In: Moller JH, Hoffman JIE, eds. Pediatric Cardiovascular
Medicine. Philadelphia, Pa: Churchill Livingstone; 2000:780-792

page 518
2009 PREP SA on CD-ROM

Critique: 149

First-degree heart block represents a prolongation of the PR interval; it is not


a feature of pericarditis. (Courtesy of A. Friedman)

page 519
2009 PREP SA on CD-ROM

Critique: 149

Pre-excitation (early depolarization of the QRS complex) manifests as delta


waves (arrows) and is a feature of Wolff-Parkinson-White syndrome.
(Courtesy of A. Friedman)

page 520
2009 PREP SA on CD-ROM

Critique: 149

The electrocardiogram in hyperkalemia shows peaked T waves. (Courtesy of


A. Friedman)

page 521
2009 PREP SA on CD-ROM

Critique: 149

In hypokalemia, the electrocardiogram shows ST segment depression,


flattening of T waves, and the appearance of a U wave (arrow). (Courtesy of A.
Friedman)

page 522
2009 PREP SA on CD-ROM

Question: 150
A 14-year-old boy who has epilepsy presents to the emergency department after a generalized
tonic-clonic seizure that began on the playground at school. He continued to convulse en route in the
ambulance, where he received 15 mg diazepam rectally and intravenous access was achieved. In the
emergency department, he continues to be unresponsive, exhibiting tachycardia and nonsuppressable
bilateral synchronous rhythmic clonic jerks.

Of the following, the MOST appropriate medication to administer next is

A. fosphenytoin 20 mg/kg intravenously

B. pentobarbital 5 mg/kg intravenously

C. phenobarbital 20 mg/kg intravenously

D. phenytoin 7 mg/kg orally

E. valproic acid 15 mg/kg intravenously

page 523
2009 PREP SA on CD-ROM

Critique: 150 Preferred Response: A


Standard recommendations for pharmacologic treatment of status epilepticus and refractory status
epilepticus in adults were established in the 1990s. They also represent the standard of care for
children and remain valid at the time of this writing (Item C150).
Because the boy described in the vignette continues to have seizures after administration of a
reasonable dose of diazepam (step 2), he now should receive either phenytoin 20 mg/kg intravenously
or fosphenytoin 20 mg phenytoin equivalents/kg intravenously. Use of phenobarbital before phenytoin is
not standard. Pentobarbital is reserved for use when other treatments have failed to induce a medical
coma, a state of complete unresponsiveness manifested by a characteristic electroencephalographic
pattern of "burst suppression" (bursts interspersed with flat or nearly flat tracing). Intravenous valproic
acid and levetiracetam are new options for use in selected cases, but they are not standard at this time.

References:

Johnston MV. Seizures in childhood. In: Kliegman RM, Behrman RE, Jenson HB, Stanton BF, eds.
Nelson Textbook of Pediatrics. 18th ed. Philadelphia, Pa: Saunders Elsevier; 2007:2457-2475

Riviello JJ Jr, Ashwal S, Hirtz D, et al. Practice parameter: diagnostic assessment of the child with
status epilepticus (an evidence-based review): report of the Quality Standards Subcommittee of the
American Academy of Neurology and the Practice Committee of the Child Neurology Society.
Neurology. 2006;67:1542-1550. Available at: http://www.neurology.org/cgi/content/full/67/9/1542

Riviello JJ Jr, Holmes GL. The treatment of status epilepticus. Semin Pediatr Neurol. 2004;11:129-138.
Abstract available at: http://www.ncbi.nlm.nih.gov/pubmed/15259866

page 524
2009 PREP SA on CD-ROM

Critique: 150

page 525
2009 PREP SA on CD-ROM

Question: 151
A 2-day-old male is approaching hospital discharge from the regular nursery. You receive an urgent call
from the nurse caring for him, who says that he would not awaken for his last feeding and is now
difficult to arouse. A blood glucose determination is normal. You arrange for laboratory tests and call the
neonatologist to evaluate the baby while you make plans to leave your office. When you arrive at the
hospital 45 minutes later, the baby has been transferred to the neonatal intensive care unit, is now
comatose, and has irregular breathing. Results of a complete blood count with differential count,
platelets, and a chemistry panel, including renal and liver function tests, are normal.

Of the following, the MOST appropriate laboratory test for diagnosis and formulating a treatment plan
for this baby is

A. acylcarnitine profile

B. serum ammonia

C. total and free carnitine

D. urine organic acids

E. very long-chain fatty acids

page 526
2009 PREP SA on CD-ROM

Critique: 151 Preferred Response: B


The differential diagnosis for any infant who has reduced arousability or coma must include inborn
errors of metabolism, particularly those that cause elevations of serum ammonia. This is especially true
when the infant has previously been alert and feeding well because the change in mental status
suggests the possible accumulation of a substance that impairs consciousness.
In addition to evaluation for sepsis, initial laboratory tests should include measurement of serum
ammonia, electrolytes, and urine ketones. The anion gap also must be calculated. Elevated ammonia
concentrations are associated with a variety of inborn errors, including urea cycle defects, transient
hyperammonemia, maple syrup urine disease, organic acidopathies, fatty acid oxidation defects, and
nonketotic hyperglycinemia (Item C151). Normally, ammonia values are below 110.0 mcmol/L in
newborns. Because elevated ammonia concentrations can cause irreversible brain damage, the
measurement of serum ammonia and initiation of treatment for hyperammonemia must be the first
steps in evaluating the infant in a coma. In a metabolic emergency involving an unknown diagnosis and
a serum ammonia concentration of 200.0 mcmol/L or greater, benzoate and phenylacetate (or one of
the recently marketed preparations for this purpose) should be administered intravenously to lower the
concentration. If the ammonia concentration is 600.0 mcmol/L or greater, hemodialysis is recommended.
The infant described in this vignette has laboratory values suggestive of a urea cycle defect, such as
ornithine transcarbamylase deficiency. If he had a low glucose value or acidosis, evaluation for other
causes of hyperammonemia would be appropriate and should include measurement of total and free
carnitine, urine organic acids, and very long-chain fatty acids as well as acylcarnitine profile.

References:

Burton BK. Inborn errors of metabolism in infancy: a guide to diagnosis. Pediatrics. 1998;102:e69-e78.
Available at: http://pediatrics.aappublications.org/cgi/content/full/102/6/e69

Hoffmann GF, Nyhan WL, Zschocke J, Kahler SG, Mayatepek E. Approach to the patient with metabolic
disease. In: Inherited Metabolic Diseases. Philadelphia, Pa: Lippincott Williams & Wilkins; 2002:19-94

Niemi A-K, Enns GM. Pharmacology review: sodium phenylacetate and sodium benzoate in the
treatment of neonatal hyperammonemia. NeoReviews. 2006;7:e486-e495. Available for subscription at:
http://neoreviews.aappublications.org/cgi/content/full/7/9/e486

Nyhan WL, Barshop NA, Ozand PT. Hyperammonemia and disorders of the urea cycle: introduction to
hyperammonemia and disorders. In: Atlas of Metabolic Diseases. 2nd ed. London, England: Hodder
Arnold; 2005:191-198

page 527
2009 PREP SA on CD-ROM

Critique: 151

page 528
2009 PREP SA on CD-ROM

Question: 152
During the annual health supervision visit of a 16-year-old patient, he reports smoking a pack of
cigarettes daily. He plans to become a vocal music major in college and is concerned that smoking may
affect his voice, but he is uncertain if he wants to stop smoking at this time.

Of the following, the MOST appropriate statement to include in your counseling regarding smoking
cessation is that

A. chronic obstructive lung disease is the first pulmonary problem to arise in cigarette smokers

B. cigarettes contain few other toxic chemicals aside from nicotine

C. erectile dysfunction is not associated with cigarette smoking

D. initial symptoms of nicotine dependence occur in some teens after only a few cigarettes

E. lung cancer is the only cancer associated with cigarette smoking

page 529
2009 PREP SA on CD-ROM

Critique: 152 Preferred Response: D


Tobacco use has decreased for adolescents in the United States since peaks in the late 1990s, but it
still remains prevalent, with 23% of 9th through 12th graders reporting cigarette use on one or more of
the past 30 days preceding the Centers for Disease Control and Prevention (CDC) National Youth Risk
Behavior Survey. Although relatively few studies have examined smoking cessation techniques specific
to adolescents, brief counseling interventions modeled after similar interventions demonstrated to be
effective with adults are recommended. The "5 A's" (Ask, Advise, Assess, Assist, and Arrange) are
advocated for use by the United States Public Health Service Task Force practice guidelines in
counseling adolescents to stop tobacco use.
Among the physiologic consequences of tobacco use are ischemic heart disease; numerous
cancers, including lung, head and neck, esophageal, gastric, colorectal, bladder, renal, prostate, and
cervical; premature wrinkling of the skin; low-birthweight infants associated with maternal smoking in
pregnancy; adverse effects on natural killer cells and other immune modulators; and chronic obstructive
pulmonary disease, with small airway disease present before evidence of chronic disease is
demonstrable. Cigarettes usually contain thousands of chemicals aside from nicotine, including
cyanide, carbon monoxide, formaldehyde, and tar, that may have long-term detrimental effects on
health.
Emphasizing the immediate and short-term effects of tobacco use rather than the long-term effects
is believed to be a better counseling technique for teens. Discussing the effects on athletic performance
and on vocal and dramatic performance where voice is important, the potential for erectile dysfunction,
and the possibility of symptoms of nicotine dependence occurring in some teens after only a few
cigarettes may be more salient to a young person and more important in making a decision to stop
tobacco use.

References:

Ammerman SD. Tobacco. In: Neinstein LS, ed. Adolescent Health Care: A Practical Guide. 5th ed.
Philadelphia, Pa: Lippincott Williams & Wilkins;, 2008:888-907

Centers for Disease Control and Prevention. Youth Risk Behavior Survey: 2007. Available at:
http://www.cdc.gov/Features/RiskBehavior/

Centers for Disease Control and Prevention. Youth risk behavior surveillance-United States, 2005.
MMWR Surv Summ. 2006;55(No.SS-5). Available at: http://www.cdc.gov/mmwr/PDF/SS/SS5505.pdf

Klein JD, Camenga DR. Tobacco prevention and cessation in pediatric patients. Pediatr Rev.
2004;25:17-26. Available at: http://pedsinreview.aappublications.org/cgi/content/full/25/1/17

page 530
2009 PREP SA on CD-ROM

Question: 153
You are evaluating a 2-month-old girl who has suspected infantile botulism. On physical examination,
she has a weak cry, poor head control, dilated pupils, and a markedly decreased gag reflex. Her
respiratory rate is 30 breaths/min, and she has decreased breath sounds bilaterally at her lung bases.
Her oxygen saturation on room air is 85%, but has increased to 90% on 3 L/min of oxygen administered
by nasal cannula. An arterial blood gas evaluation on 3 L/min oxygen shows a pH of 7.24, a Paco2 of
60 mm Hg, and a Pao2 of 70 mm Hg.

Of the following, the MOST appropriate immediate step is

A. intubation with mechanical ventilation

B. oxygen administration via a nonrebreathing face mask

C. oxygen administration via a high-flow nasal cannula

D. placement of a nasal trumpet

E. serial measurements of negative inspiratory force

page 531
2009 PREP SA on CD-ROM

Critique: 153 Preferred Response: A


The patient described in the vignette demonstrates respiratory failure, which is defined as the inability to
maintain adequate oxygenation (hypoxemia, as defined by a PaO2 less than 60 mm Hg) and ventilation
(hypercarbia, as defined by a PaCO2 greater than 50 mm Hg). Although her oxygen saturation has
improved with oxygen administration, findings of her neurologic examination include decreased tone,
impaired airway protection, and decreased aeration in the setting of a progressive neurologic disease.
In addition, her elevated PaCO2 reflects her inadequate respiratory effort with subsequent impaired
ventilation. Impending respiratory failure in pediatric patients typically is characterized by tachypnea,
increased use of accessory muscles, or retractions. Hypoxemia, grunting, and fatigue are late signs and
worrisome for respiratory collapse.
Oxygen should be administered to patients who have evidence of impending respiratory failure in
their position of comfort with a goal of maintaining the oxygen saturation at greater than 90%. A variety
of factors, including the cause of the respiratory failure, speed of progression, patient age, associated
morbidities, and anticipated procedures such as the need for imaging studies or transport, can guide
the clinician in managing respiratory failure and deciding whether to proceed with intubation. The
impending respiratory failure seen for the patient in the vignette mandates placement of an
endotracheal tube and assistance with ventilation.
Careful monitoring is imperative for patients experiencing respiratory failure, especially those who
may have chronic respiratory failure and an impaired respiratory drive, although the first priority is
correction of the hypoxemia. Administration of oxygen to patients who have chronic respiratory failure
should be at the lowest concentration needed to maintain an oxygen saturation of 90%, with frequent
arterial blood gas monitoring to ensure that their PaCO2 is not rising.
Failure to respond to oxygen administration indicates the need for more aggressive intervention.
Bag-valve-mask ventilation can support the oxygenation and ventilation needs of most pediatric
patients while appropriate personnel trained in endotracheal intubation are assembled. Patients who
have underlying or acute neurologic or neuromuscular processes that interfere with their ability to
protect their airways or generate adequate tidal volumes are candidates for early intubation.
Serial negative inspiratory force measurements are an excellent method of following respiratory
effort in patients who have underlying neuromuscular disorders, but they normally require a cooperative
patient of at least 5 years of age. Administration of higher concentrations of oxygen (eg, by high-flow
nasal cannula or nonrebreathing face mask) has the potential of increasing the patient's oxygenation
but would not improve the ventilation. Placement of a nasal trumpet is indicated for patients who have
an intact respiratory drive and evidence of upper airway obstruction.

References:

Frankel LR. Respiratory distress and failure. In: Kliegman RM, Behrman RE, Jenson HB, Stanton, BF,
eds. Nelson Textbook of Pediatrics. 18th ed. Philadelphia, Pa: Saunders Elsevier; 2007:421-423

Pope J, McBride J. Consultation with the specialist: respiratory failure in children. Pediatr Rev.
2004;25:160-167. Available at: http://pedsinreview.aappublications.org/cgi/content/full/25/5/160

page 532
2009 PREP SA on CD-ROM

Question: 154
A 12-year-old girl who developed type 1 diabetes at age 3 years comes in with her parents for a health
supervision visit. Her diabetes control has been excellent. Physical examination reveals Sexual Maturity
Rating 2 pubic hair and breast development and a palpable and somewhat firm thyroid gland.

Of the following, the MOST likely cause of the thyroid enlargement is

A. Graves disease

B. Hashimoto thyroiditis

C. iodine deficiency

D. multinodular goiter

E. pubertal thyroid enlargement

page 533
2009 PREP SA on CD-ROM

Critique: 154 Preferred Response: B


Approximately 10% of people who have type 1 diabetes develop autoantibodies against the thyroid or
chronic lymphocytic thyroiditis, sometimes referred to as Hashimoto thyroiditis. This is more common in
children in whom the onset of type 1 diabetes occurs before the age of 10 years, such as the girl
described in the vignette. A smaller percentage, perhaps 50% or more of individuals who have chronic
lymphocytic thyroiditis, eventually develop hypothyroidism, and a very small number may develop other
autoimmune thyroid disorders, including Graves disease.
Iodine deficiency is not very common in the United States because iodine is freely available from
sources such as iodized salt, seafood, iodophors used to clean stainless steel milk storage containers,
and sodium alginate thickeners derived from seaweed that are found in many fast foods. Multinodular
goiter would be unusual in an iodine-sufficient child of this age without pre-existing thyroid problems.
On physical examination, a multinodular goiter has a bumpy or irregular firm surface. Although the
thyroid becomes somewhat more palpable during puberty, perhaps because of increased need for
thyroid hormone during rapid growth, it is soft and not firm.
Thyroid autoimmunity may be associated with other autoimmune disorders. Approximately 3% to 4%
of individuals who have chronic lymphocytic thyroiditis develop celiac disease. Vitiligo also can be
found. Less frequently, adrenal insufficiency, pernicious anemia, ovarian failure, type 1 diabetes, or
other autoimmune disorders can develop in an individual who initially has chronic lymphocytic
thyroiditis. Some have classified these disorders as polyglandular autoimmune syndromes type 1, 2, or
3. Type 1 is caused by a known gene defect in AIRE, an immune regulator. Individuals who have this
rare severe autosomal recessive disorder can exhibit candidiasis, hypoparathyroidism, Addison
disease, and pernicious anemia in addition to type 1 diabetes, vitiligo, and other autoimmunities. A
specific underlying gene defect has not been identified in those who have type 2 and type 3
autoimmune endocrinopathies, but these conditions have linkages to human lymphocyte antigen types
related to the autoimmune response. Type 2 autoimmune endocrinopathy includes autoimmune thyroid
disease or diabetes with adrenal insufficiency. The type 3 disorder usually occurs in adults and is
comprised of variable combinations of autoimmune endocrine disorders, but not adrenal insufficiency.

References:

Aldasouqi SA, Akinsoto OPA, Jabbour SA. Polyglandular autoimmune syndrome type 1. eMedicine
Specialties, Endocrinology, Multiple Endocrine Disease and Miscellaneous Endocrine Disease. 2006.
Available at: http://www.emedicine.com/med/topic1867.htm

Kaloumenou I, Alevizaki M, Ladopoulos C, et al. Thyroid volume and echostructure in schoolchildren


living in an iodine-replete area: relation to age, pubertal stage, and body mass index. Thyroid.
2007;17:875-881. Abstract available at: http://www.ncbi.nlm.nih.gov/pubmed/17956161

Kordonouri O, Hartmann R, Deiss D, Wilms M, Grüters-Kieslich A. Natural course of autoimmune


thyroiditis in type 1 diabetes: association with gender, age, diabetes duration, and puberty. Arch Dis
Child. 2005;90:411-414. Abstract available at: http://www.ncbi.nlm.nih.gov/pubmed/15781936

LaFranchi S. Thyroiditis. In: Kliegman RM, Behrman RE, Jenson HB, Stanton BF, eds. Nelson
Textbook of Pediatrics. 18th ed. Philadelphia, Pa: Saunders Elsevier; 2007:2327-2329

page 534
2009 PREP SA on CD-ROM

Question: 155
A 9-year-old child has been struggling in his regular third-grade classroom and has not yet received
additional educational support. A comprehensive psychoeducational evaluation reveals a significant
discrepancy between cognitive testing scores and academic performance for reading and writing.

Of the following, the BEST educational intervention for this boy is

A. after-school private tutoring for language arts

B. mainstream classroom with preferential seating

C. resource services for language arts and reading

D. self-contained classroom for all subjects

E. summer school so he can catch up

page 535
2009 PREP SA on CD-ROM

Critique: 155 Preferred Response: C


The Individuals with Disabilities Education Act (IDEA) of 1990 (PL 101-476) defines the guidelines for
education of children in the United States who have specific learning disabilities. According to the "least
restrictive clause" in IDEA, children who have learning disorders should be integrated into the
mainstream classroom as much as possible.
The child described in the vignette is having learning issues in the area of reading and writing. The
most appropriate setting for him is in the mainstream class for all of his subjects except language arts
and reading. For these skills, he should receive extra educational support. Typically, this is
accomplished by having the student go to a specialized classroom (resource room) staffed by a special
education teacher.
Preferential seating close to the teacher in a regular classroom will not address his areas of
academic weakness. Although he may benefit from additional tutoring and summer school, these
strategies could be offered in addition to resource services. Children who have more significant learning
issues (eg, autism spectrum disorder, cognitive impairment) may require a self-contained classroom
that provides more individualized and intensive educational support.

References:

American Academy of Pediatrics Committee on Children With Disabilities. The pediatrician's role in
development and implementation of an Individual Education Plan and/or an Individual Family Service
Plan. Pediatrics. 1999;104:124-127. Available at:
http://pediatrics.aappublications.org/cgi/content/reprint/104/1/124

Council on Children With Disabilities. Policy statement: provision of educationally related services for
children and adolescents with chronic diseases and disabling conditions. Pediatrics.
2007;119:1218-1223. Available at: http://pediatrics.aappublications.org/cgi/content/full/119/6/1218

Ewen JB, Shapiro BK. Specific learning disabilities. In: Accardo PJ. Capute & Accardo's
Neurodevelopmental Disabilities in Infancy and Childhood. Volume II: The Spectrum of
Neurodevelomental Disabilities. 3rd ed. Baltimore, Md: Paul H. Brookes Publishing Co; 2008:553-577

Fessler MA, Plourde PA. Psychoeducational assessment. In: Accardo PJ. Capute & Accardo's
Neurodevelopmental Disabilities in Infancy and Childhood. Volume II: The Spectrum of
Neurodevelomental Disabilities. 3rd ed. Baltimore, Md: Paul H. Brookes Publishing Co; 2008:591-610

page 536
2009 PREP SA on CD-ROM

Question: 156
You are the physician for the child care center that your child attends. When an outbreak of diarrhea
occurs at the center, the director calls you. She knows infections due to Giardia lamblia are common in
child care centers, but asks if she needs to be worried about anything else.

Of the following, the organism MOST likely to cause a child care outbreak is

A. Aeromonas hydrophila

B. Campylobacter jejuni

C. Salmonella sp

D. Shigella sp

E. Yersinia enterocolitica

page 537
2009 PREP SA on CD-ROM

Critique: 156 Preferred Response: D


Outbreaks of diarrheal illness can occur in child care facilities when caretakers do not adhere to hand
hygiene policies and the pathogen requires a small inoculum for disease production. Giardia lamblia
and Shigella sp commonly cause child care center outbreaks. Shigella are gram-negative bacilli that are
spread readily person-to-person through fecal-oral and oral-anal contamination or by houseflies or other
contaminated fomites. Among the common bacterial causes of gastroenteritis, Shigella is unique
because ingestion of as few as 10 organisms can cause human disease. There are four primary
Shigella pathogens: S sonnei, S flexneri, S boydii, and S dysenteriae.
The typical incubation period is 2 to 4 days, and the illness caused by Shigella can range from a mild
diarrhea to life-threatening dysentery. Fever and abdominal cramps with high-volume, watery stools
followed 24 to 48 hours later by small-volume, bloody mucoid stools with tenesmus is a common
presentation. Bloody mucoid stools are more common with diarrhea due to S dysenteriae and S
flexneri; S boydii and S sonnei usually cause only watery diarrhea. Seizures can occur, with an
incidence that is higher than would be expected from febrile seizures alone. Other complications can
include Reiter syndrome, hemolytic-uremic syndrome, colonic perforation, and toxic encephalopathy.
Stool microscopy revealing a large number of neutrophils in a patient who has clinical findings
suggestive of shigellosis supports the diagnosis but is not specific for any species of Shigella.
Aeromonas hydrophila, Campylobacter jejuni, Salmonella sp, and Yersinia enterocolitica are not
commonly involved in outbreaks of gastrointestinal disease in child care centers due to their large
inoculum requirements.

References:

American Academy of Pediatrics. Children in out-of-home child care. In: Pickering LK, Baker CJ, Long
SS, McMillan JA, eds. Red Book: 2006 Report of the Committee on Infectious Diseases. 27th ed. Elk
Grove Village, Ill: American Academy of Pediatrics; 2006:130-145.

American Academy of Pediatrics. Shigella infections. In: Pickering LK, Baker CJ, Long SS, McMillan
JA, eds. Red Book: 2006 Report of the Committee on Infectious Diseases. 27th ed. Elk Grove Village,
Ill: American Academy of Pediatrics; 2006:589-591

Spence JT, Cheng TL. In brief: Shigella species. Pediatr Rev. 2004;25:329-330. Available at:
http://pedsinreview.aappublications.org/cgi/content/full/25/9/329

page 538
2009 PREP SA on CD-ROM

Question: 157
A frantic mother brings in her three children, all of whom suddenly developed fevers, red and sore eyes,
headaches, and sore throats 2 days after attending a swimming party at a country club. The mother
states that five other children and 10 adults who attended the party have similar symptoms. On physical
examination, all of the children have temperatures higher than 102.0°F (38.9°C), bilateral conjunctivitis,
nasal congestion, and exudative pharyngitis.

Of the following, the MOST likely cause of these findings is

A. adenovirus

B. Chlamydia trachomatis

C. Epstein-Barr virus

D. Leptospira sp

E. Mycoplasma pneumoniae

page 539
2009 PREP SA on CD-ROM

Critique: 157 Preferred Response: A


The respiratory tract is the most common site of adenoviral infections. Clinical presentations of infection
include symptoms of the common cold (cough, fever, sore throat, and rhinorrhea), pharyngitis, tonsillitis,
otitis media, pharyngoconjunctival fever, acute hemorrhagic conjunctivitis, croup, bronchiolitis, and a
pertussislike syndrome. Pharyngoconjunctival fever is characterized by conjunctivitis (Item C157),
pharyngitis, rhinitis, cervical adenitis, and fever. Community outbreaks of pharyngoconjunctival fever
have been attributed to exposure to water from contaminated swimming pools and fomites from shared
towels. The onset is acute, and fever and symptoms last for 3 to 5 days.
The children described in the vignette have physical findings consistent with adenovirus-associated
pharyngoconjunctival fever. Conjunctivitis is not seen with disease caused by Mycoplasma
pneumoniae. Conjunctivitis is one of the most distinct findings with Leptospira infection, but pharyngitis
usually is not seen, and patients are much more ill, exhibiting chills, myalgias, uveitis, rash, and aseptic
meningitis. Conjunctivitis is seen in a small percentage of persons infected with Epstein-Barr virus, but
community outbreaks associated with contaminated swimming pools do not occur and do not represent
a mode of disease transmission. Chlamydia trachomatis is associated with neonatal conjunctivitis and
trachoma, but trachoma is rare in the United States.

References:

American Academy of Pediatrics. Adenovirus infections. In: Pickering LK, Baker CJ, Long SS, McMillan
JA, eds. Red Book: 2006 Report of the Committee on Infectious Diseases. 27th ed. Elk Grove Village,
Ill: American Academy of Pediatrics; 2006:202-204

American Academy of Pediatrics. Chlamydia trachomatis. In: Pickering LK, Baker CJ, Long SS,
McMillan JA, eds. Red Book: 2006 Report of the Committee on Infectious Diseases. 27th ed. Elk Grove
Village, Ill: American Academy of Pediatrics; 2006:252-257

American Academy of Pediatrics. Leptospirosis. In: Pickering LK, Baker CJ, Long SS, McMillan JA,
eds. Red Book: 2006 Report of the Committee on Infectious Diseases. 27th ed. Elk Grove Village, Ill:
American Academy of Pediatrics; 2006:424-426

American Academy of Pediatrics. Mycoplasma pneumoniae infections. In: Pickering LK, Baker CJ,
Long SS, McMillan JA, eds. Red Book: 2006 Report of the Committee on Infectious Diseases. 27th ed.
Elk Grove Village, Ill: American Academy of Pediatrics; 2006:468-470

Singh-Naz N, Rodriguez W. Adenoviral infections in children. Adv Pediatr Infect Dis. 1996;11:365-388

page 540
2009 PREP SA on CD-ROM

Critique: 157

Adenovirus infection may produce conjunctivitis. (Courtesy of Red Book®


Online)

page 541
2009 PREP SA on CD-ROM

Question: 158
A 16-year-old girl presents with symptoms of burning with micturition and back pain. Her temperature is
101.3°F (38.5°C), heart rate is 88 beats/min, respiratory rate is 14 breaths/min, and blood pressure is
108/64 mm Hg. You can elicit costovertebral angle tenderness on the left side and suprapubic
tenderness. Her urinalysis demonstrates a urine specific gravity of 1.025, pH of 6.5, 2+ blood, 1+
protein, 3+ leukocyte esterase, and positive for nitrite. Microscopy reveals 5 to 10 red blood
cells/high-power field (HPF), 50 to 100 white blood cells/HPF, 4+ bacteria, and occasional squamous
epithelial cells.

Of the following, the MOST likely diagnosis is

A. acute bacterial cystitis

B. acute glomerulonephritis

C. acute pyelonephritis

D. bacterial urethritis

E. nephrolithiasis

page 542
2009 PREP SA on CD-ROM

Critique: 158 Preferred Response: C


The adolescent described in the vignette presents with classic symptoms of acute pyelonephritis (upper
urinary tract infection [UTI]): fever, back pain, and dysuria associated with physical findings of
costovertebral angle and suprapubic tenderness.
UTIs typically are classified as lower urinary tract (involving the bladder and urethra) and upper
urinary tract (involving the ureters, renal pelvis, and kidneys). In an adolescent, a bladder infection
(cystitis) is not accompanied by flank pain or fever. Urethritis, which may be caused by a sexually
transmitted infection, is accompanied by symptoms of dysuria, but not symptoms of flank pain or fever.
Distinguishing between upper and lower UTI may be more difficult in younger children. In neonates
and infants, the presence of fever in the setting of a UTI characterizes an upper tract infection
(pyelonephritis). In contrast, the absence of fever suggests lower tract infection. The diagnosis often is
made during the evaluation of an infant for fever and irritability. A study from Taiwan revealed that a UTI
was present in 13.6% of infants younger than 8 weeks of age who presented with fever. Urine white
blood cell counts greater than 10/high-power field (pyuria) were the best indicators of UTI in this setting.
Bacteremia accompanies UTI in 31% of pediatric patients younger than 1 month of age and 21% of
those 1 to 2 months of age. Febrile infants who have no apparent source of fever had UTI in 7.5% of
cases in another study. Older infants present with fever and vomiting, and the diagnosis is based on a
positive urine culture collected by transurethral or suprapubic catheterization. The incidence of
bacteremia falls to 3% in children 2 to 36 months of age.
As children develop verbal skills, it becomes easier to distinguish between upper and lower tract
infection because they can express their symptoms of dysuria and frequency.
Acute glomerulonephritis does not present with dysuria and flank pain. Nephrolithiasis can present
with flank pain and possibly dysuria (if accompanied by crystalluria), but the presence of suprapubic
pain, fever, and disproportionate pyuria to hematuria in the setting of a positive nitrite result make this
diagnosis unlikely.

References:

Ginsburg CM, McCracken GH Jr. Urinary tract infections in young infants. Pediatrics. 1982;69;409-412.
Available at: http://pediatrics.aappublications.org/cgi/content/abstract/69/4/409

Hoberman A, Chao HP, Keller DM, Hickey R, Davis HW, Ellis D. Prevalence of urinary tract infection in
febrile infants. J Pediatr. 1993;123:17-23. Abstract available at:
http://www.ncbi.nlm.nih.gov/pubmed/8320616

Johnson CE. New advances in childhood urinary tract infections. Pediatr Rev. 1999:20:335-342.
Available at: http://pedsinreview.aappublications.org/cgi/content/full/20/10/335

Raszka WV Jr, Khan O. Pyelonephritis. Pediatr Rev. 2005;26:364-370. Available at:


http://pedsinreview.aappublications.org/cgi/content/full/26/10/364

page 543
2009 PREP SA on CD-ROM

Question: 159
A 17-year-old boy presents to the emergency department with respiratory distress and hypoxia (room
air Po2, 86%). His parents called 911 after their son started "gasping for air." The boy has a history of
moderate persistent asthma that recently worsened after a viral infection. He uses a daily inhaled
corticosteroid and an as-needed beta2 agonist inhaler. During the past 2 days, he has not been able to
go to school because of his breathing problems, and his parents have used his nebulizer every 2 hours
over the past 12 hours. On physical examination, the boy is awake and responsive to questions, but his
respiratory rate is 34 breaths/min, and he has nasal flaring and intercostal retractions. Lung
examination demonstrates equal breath sounds bilaterally but obvious expiratory wheezing with a
prolonged expiratory phase. The results of an arterial blood gas on a nonrebreather with 100% oxygen
are: pH of 7.35 (normal, 7.35 to 7.45), Pco2 of 45 mm Hg (normal, 35 to 45 mm Hg), bicarbonate of
24.0 mEq/L (24.0 mmol/L)(normal, 22.0 to 26.0 mEq/L [22.0 to 26.0 mmol/L]), and Po2 of 90 mm Hg
(normal, 75 to 100 mm Hg).

Of the following, a TRUE statement regarding the management of this boy’s condition is that

A. a normal carbon dioxide value on an arterial blood gas measurement makes an asthma exacerbation
unlikely

B. if assisted ventilation is required, the ventilator rate should be set at 34 breaths/min

C. initial corticosteroid therapy can be administered either orally or intravenously


D. the parents should have doubled his inhaled corticosteroid dose when his beta2 agonist was not
working

E. the patient should blow into a paper bag to help calm him

page 544
2009 PREP SA on CD-ROM

Critique: 159 Preferred Response: C


Regardless of the level of asthma severity, affected patients can experience exacerbations. The
National Heart, Lung, and Blood Institute 2007 Expert Panel 3 Asthma Guidelines have outlined in
detail the importance of recognition and treatment of asthma exacerbations both at home and in the
emergency department (available at http://www.nhlbi.nih.gov/guidelines/asthma/asthgdln.pdf).
The teenager described in the vignette is having a severe asthma exacerbation that has not
improved with multiple short-acting beta2 agonist (SABA) nebulization treatments administered at
home. Although he is able to answer questions and is alert, he exhibits hypoxia, tachypnea, and
accessory muscle use. Additional signs of severe obstruction include peak expiratory flow rates less
than 50% of predicted, quiet breath sounds, an inability to speak whole phrases, and cyanosis. He
should receive supplemental oxygen, an arterial blood gas, a SABA nebulizer treatment, and
corticosteroids as soon as possible. Most patients, including this boy, should be able to tolerate oral
corticosteroids, which should be continued for 3 to 10 days. Intravenous corticosteroids typically are
reserved for life-threatening asthma exacerbations. Most experts and published experience support oral
and intravenous corticosteroids as being equally effective. One commonly used technique during an
asthma exacerbation is to double the inhaled steroid dose at the onset of the asthma exacerbation.
However, this recommendation is not supported by results of a controlled trial or the 2007 Expert Panel
3 asthma guidelines.
One of the primary concerns during an acute exacerbation is whether the patient has impending
respiratory failure. A patient who is too dyspneic to speak, perspiring, or has a peak expiratory flow of
less than 25% may need to be intubated. An initial blood gas measurement can be helpful in assessing
the patient's ventilatory status. However, a normal or elevated PCO2 in the context of significant
tachypnea is worrisome and requires aggressive treatment. Indeed, a patient who has a respiratory rate
of 34 breaths/min should have a low PCO2 (respiratory alkalosis) unless there is airway obstruction.
Patients who have respiratory failure due to asthma and require intubation should be treated differently
from other patients who have respiratory failure requiring intubation. Because the expiratory phase
during an asthma exacerbation is prolonged, setting the ventilatory rate to match the patient's
tachypnea does not allow sufficient time to expel carbon dioxide. An inappropriately high ventilator rate
also can lead to increased thoracic pressure, increased positive end-expiratory pressure, and possibly
a pneumothorax. A slower rate (8 to 12 breaths/min) is recommended if such a patient is intubated.
Blowing in a paper bag has no role in the treatment of asthma.
An asthma action plan should be developed for patients and their families to prevent asthma
exacerbations and emergency department visits. The action plan can help patients understand the
symptoms associated with asthma exacerbation, the role of peak expiratory flows in determining the
severity of an exacerbation, and specific action(s) to take during an exacerbation.

References:

Harrison TW, Oborne J, Newton S, Tattersfield AE. Doubling the dose of inhaled corticosteroid to
prevent asthma exacerbations: randomised controlled trial. Lancet. 2004;363:271-275. Abstract
available at: http://www.ncbi.nlm.nih.gov/pubmed/14751699

Keeley D, McKean M. Asthma and other wheezing disorders in children. BMJ Clinical Evidence. 2006.
Available for subscription at: http://clinicalevidence.bmj.com/ceweb/conditions/chd/0302/0302.jsp#Q1

National Asthma Education and Prevention Program. Expert Panel Report 3 (EPR-3): Guidelines for the
Diagnosis and Management of Asthma-Summary Report 2007. J Allergy Clin Immunol.
2007;120:S94-S138. Abstract available at: http://www.ncbi.nlm.nih.gov/pubmed/17983880

page 545
2009 PREP SA on CD-ROM

Question: 160
You are camping with a group of boys at a rural campground in the southeastern United States when
one of the campers is bitten by a snake. His tent mates kill the snake (Item Q160). The victim is crying
and guarding his right hand. On examination of the boy’s hand, you note several small, erythematous
abrasions but no swelling or ecchymosis.

Of the following, the MOST appropriate course of action is to

A. apply a tourniquet above the bite

B. apply ice to the wound

C. incise and suction the wound

D. provide local wound care

E. transport the boy to the hospital for antivenom

page 546
2009 PREP SA on CD-ROM

Question: 160

Snake, as described in the vignette. (Courtesy of D. Krowchuk, wih special


thanks to Sci Works, Winston-Salem, NC)

page 547
2009 PREP SA on CD-ROM

Critique: 160 Preferred Response: D


Each year approximately 7,000 to 8,000 venomous snake bites occur in the United States that result in
five to six deaths and significant morbidity. Although most bites occur in young adult males, children are
disproportionately represented among the fatalities because their smaller size leads to a higher venom
dose per kilogram. In the United States, almost all venomous snakes are pit vipers (cottonmouths,
rattlesnakes [Item C160A], copperheads [Item C160B]). Of these, eastern and western diamondback
rattlesnakes are responsible for most fatal bites. Besides pit vipers, the only other venomous snake in
the United States is the coral snake, which is found in the southern United States.
Management of a snake bite is predicated on whether the bite was caused by a venomous or
nonvenomous snake, which can be determined either by examining the (preferably dead) snake or by
the appearance of the wound. A pit viper can be identified by its triangular-shaped head, elliptical
pupils, and presence of fangs; a nonvenomous snake has a round head and pupils and no fangs or
rattle (Item C160C). Although the coral snake also has a round head, it is recognizable by its unique
striped coloration in which red and yellow bands are adjacent ("red on yellow, kill a fellow") (Item C160D
and Item C160E). Because the snake often is not available for inspection, examination of the wound
should yield findings that can guide further treatment. A pit viper bite generally has visible fang marks; a
nonvenomous snake typically leaves an abrasion or scratches at the site, such as described in the
vignette. If the wound has been envenomated (approximately 25% of pit viper bites are "dry" or
nonenvenomated), pain at the site is immediate, followed by rapid development of erythema, swelling,
and regional spread of the pain. Over the next several hours, hemorrhagic bulla and lymphangitis may
be noted (Item C160F). Early systemic symptoms include nausea, vomiting, hypotension, and
tachycardia. In contrast to pit viper venom, which contains a mixture of hemotoxic, cardiotoxic,
myotoxic, and proteolytic enzymes, coral snake venom is primarily a neurotoxin. The bite typically is
painless, but neurologic and autonomic symptoms, including hypersalivation, weakness, altered mental
status, cranial nerve palsies, and respiratory paralysis, may develop within 12 hours.
Both the snake and wound appearance for the patient described in the vignette indicate that he was
bitten by a nonvenomous snake. Local wound care and tetanus immunization update are indicated. Had
the wound been envenomated, field treatment would include maintenance of the victim's respiratory
and cardiovascular functions; immobilization and dependent positioning of the extremity; and removal of
watches, rings, and tight clothing. Use of tourniquets, wound incision and suction, or application of ice is
not recommended and may lead to increased tissue injury. Rapid transport to the nearest appropriate
facility where antivenom can be given is critical.
The currently available pit viper antivenom is the sheep-derived Fab antibody-segment-based
product (crotaline Fab). In six small studies performed to evaluate safety and efficacy, crotaline Fab
controlled both local and systemic symptoms effectively and caused fewer cases of acute allergic
reactions or serum sickness than the previously available but no longer manufactured horse-derived
crotaline antivenom. Although the studies included few children, the limited data support its use.

References:

Gold BS, Dart RC, Barish RA. Bites of venomous snakes. N Engl J Med. 2002;347:347-356. Extract
available at: http://content.nejm.org/cgi/content/extract/347/5/347

Schmidt JM. Antivenom therapy for snakebites in children: is there evidence? Curr Opin Pediatr.
2005;17:234-238. Abstract available at: http://www.ncbi.nlm.nih.gov/pubmed/15800419

Singletary EM, Rochman AS, Bodmer JCA, Holstege CP. Envenomations. Med Clin North Am.
2005;89:1195-1224. Abstract available at: http://www.ncbi.nlm.nih.gov/pubmed/16227060

page 548
2009 PREP SA on CD-ROM

Critique: 160

Blacktailed rattlesnake. Note the triangular-shaped head characteristic of


venomous snakes. (Courtesy of J. Brashears and M. Feldner)

page 549
2009 PREP SA on CD-ROM

Critique: 160

Southern copperhead. (Courtesy of the Centers for Disease Control and


Prevention, Public Health Image Library, Edward J. Wozniak)

page 550
2009 PREP SA on CD-ROM

Critique: 160

Comparison of venomous snakes (pit vipers) and nonvenomous snakes in the


United States. (Reprinted with permission from Gold BS, Dart RC, Barish RA.
Bites of venoumous snakes. New Engl J Med. 2002;347:347-356 Copyright ©
2002 Massachusetts Medical Society. All rights reserved.

page 551
2009 PREP SA on CD-ROM

Critique: 160

Arizona coral snake: Note the wide red and black rings separated by narrow
pale yellow rings. The coral snake can be distinguished from similarly colored
harmless snakes by the adjacent red and yellow bands ("red on yellow, kill a
fellow"). See item C160E. (Courtesy of J Brashears and M Feldner)

page 552
2009 PREP SA on CD-ROM

Critique: 160

Nonvenomous milk snake: Although the markings are similar to those of a


coral snake, red and black (not yellow) bands are adjacent ("red on black,
friend of Jack"). (Courtesy of the Centers for Disease Control and
Prevention, Public Health Image Library, Edward J. Wozniak)

page 553
2009 PREP SA on CD-ROM

Critique: 160

Ecchymosis, swelling, and ruptured hemorrhagic bullae may be observed


following the bite of a venomous snake. (Courtesy of M. Rimsza)

page 554
2009 PREP SA on CD-ROM

Question: 161
An 8-year-old girl presents to the emergency department with a history of recurrent severe vomiting.
According to the family, she has had four similar episodes in the past 6 months. Each time, the child
awakens from sleep, vomits every 20 minutes for 6 hours, and then goes back to bed. Between
episodes, which occur approximately every 4 to 6 weeks, the child is happy and playful. Normal results
have been found on prior head magnetic resonance imaging, upper gastrointestinal radiograph series,
and renal ultrasonography. In the emergency department, the girl is quiet, somewhat listless, and
prefers to be in a dark room with an emesis basin. Results of physical examination are unremarkable,
and optic discs are sharp. After 8 hours of intravenous hydration, the symptoms resolve, and she is
discharged.

Of the following, the MOST appropriate medication to treat her underlying condition is

A. amitriptyline

B. lubiprostone

C. omeprazole

D. ranitidine

E. sucralfate

page 555
2009 PREP SA on CD-ROM

Critique: 161 Preferred Response: A


The history described for the girl in the vignette is most consistent with cyclic vomiting syndrome (CVS),
a symptom complex usually considered a migraine variant. The hallmarks of this syndrome are intense
periods of vomiting that usually last between 8 and 48 hours. The typically affected patient has an aura
of intense nausea or malaise that is followed by vomiting of undigested food, gastric fluid, and ultimately
bile. During the peak of the attack, patients may vomit as frequently as three to six times per hour.
Physical examination findings usually are benign, but patients characteristically are listless and in a
state characterized as "conscious coma." Abdominal pain, dehydration, headache, and electrolyte
abnormalities can accompany the episode. Most episodes are self-limited and resolve spontaneously.
In between episodes, the patient usually feels completely well and has no significant symptoms of
nausea or vomiting. Precipitating factors for episodes include infection, psychological stress, and
menstrual periods (catamenial migraine), but stressors cannot be identified in most episodes.
Although most patients who have CVS have a migraine variant, other conditions can present with
recurrent episodic vomiting, including pancreatitis, ureteropelvic junction obstruction, malrotation,
eosinophilic gastroenteritis, and even intracranial lesions. Most notably, certain metabolic defects (urea
cycle defects, fatty acid oxidation defects) can present with recurrent vomiting. Therefore, a thorough
evaluation should exclude other causes. Helpful diagnostic tests include upper gastrointestinal
radiographic series (to exclude malrotation), abdominal ultrasonography (to exclude ureteropelvic
junction obstruction), and magnetic resonance imaging of the head (to exclude intracranial lesions). In
addition, in the middle of a CVS episode, electrolytes should be measured and metabolic studies
(ammonia, urinary organic acids and acylcarnitine, plasma amino acids, pyruvate, and lactate)
obtained.
Supportive care during a vomiting episode of CVS includes intravenous fluids, antiemetics
(ondansetron), and antianxiety medications (lorazepam). If the episodes are frequent, prophylactic
antimigraine therapy is effective in reducing their frequency and severity. This therapy should be
instituted in consultation with a specialist (pediatric neurologist or gastroenterologist). Daily amitriptyline
is used commonly as first-line prophylactic therapy for children older than age 5 years; cyproheptadine
often is used in younger children. Lubiprostone is used in the treatment of refractory constipation.
Omeprazole, ranitidine, and sucralfate are used to treat reflux or gastritis and have not been shown to
be useful in treating CVS.

References:

Cyclic Vomiting Syndrome Association of America web site. Available at: http://www.cvsaonline.org

Li BU, Misiewicz L. Cyclic vomiting syndrome: a brain-gut disorder. Gastroenterol Clin North Am.
2003;32:997-1019. Abstract available at: http://www.ncbi.nlm.nih.gov/pubmed/14562585

page 556
2009 PREP SA on CD-ROM

Question: 162
You are examining a preterm infant who was delivered at 34 weeks’ gestation due to premature labor
and a maternal urinary tract infection. His mother is 23 years old and has had insulin-dependent
diabetes mellitus for many years. Her diabetes reportedly was well managed during the pregnancy, and
a recent hemoglobin A1c measurement was 7.0%. On physical examination, the infant is appropriately
grown for gestational age, weighs 2 kg, and has a gestational age assessment that equates to 34
weeks. He requires supplemental oxygen with an FiO2 of 0.40 administered by continuous nasal
positive airway pressure.

Of the following, the GREATEST concerns for this infant related to his mother’s diabetes are

A. anemia, hypotension, and hypercalcemia

B. apnea, bradycardia, and hypercalcemia

C. hyperglycemia, hyperacusis, and aortic stenosis

D. hyperglycemia, hypocalcemia, and sepsis

E. hypoglycemia, hypocalcemia, and polycythemia

page 557
2009 PREP SA on CD-ROM

Critique: 162 Preferred Response: E


The infant described in the vignette is an appropriately grown infant of a diabetic mother (IDM). In this
instance, the mother has longstanding insulin insufficiency (type 1 diabetes mellitus). Other
classifications of diabetes in pregnancy include previously existing insulin resistance (type 2 diabetes
mellitus) and gestational diabetes (insulin resistance occurring only in pregnancy). Common sequelae
of maternal diabetes seen in the fetus include fetal macrosomia, polyhydramnios, and congenital
anomalies. Neonatal problems requiring evaluation and management in the IDM include:
* Birth defects (congenital anomalies)
* Septal hypertrophy of the myocardium and possible heart failure
* Hypoglycemia (not hyperglycemia)
* Hypocalcemia (not hypercalcemia)
* Hyperbilirubinemia
* Macrosomia with or without visceromegaly
* Polycythemia (not anemia)
* Polyhydramnios
* Renal vein thrombosis
* Neonatal small left colon
Sepsis is not more common in IDMs, nor is hypotension, except in the presence of septal
hypertrophy. Apnea and bradycardia may occur in a preterm 34-week gestation infant, but it is not
related to maternal diabetes. Aortic stenosis is not a common cardiac sequela in IDMs. Hyperacusis is
not related to maternal diabetes.

References:

Cowett RM. Neonatal care of the infant of the diabetic mother. NeoReviews. 2002;3:e190-e196.
Available for subscription at: http://neoreviews.aappublications.org/cgi/content/full/3/9/e190

Cowett RM. The infant of the diabetic mother. NeoReviews. 2002;3:e173-e189. Available for
subscription at: http://neoreviews.aappublications.org/cgi/content/full/3/9/e173

Kalhan SC, Parimi PS. Disorders of carbohydrate metabolism. In: Martin RJ, Fanaroff AA, Walsh MC,
eds. Fanaroff and Martin's Neonatal-Perinatal Medicine. 8th ed. Philadelphia, Pa: Mosby Elsevier;
2006:1467-1490

page 558
2009 PREP SA on CD-ROM

Question: 163
You are discussing the pharmacokinetics and potential interactions of drugs used in the pediatric
population with a group of medical students. One of them asks you if medications should be taken with
food, and you respond that interactions between food and drugs may either reduce or increase the drug
absorption, depending on the type of medication.

Of the following, the MOST accurate advice regarding taking medications with food is that

A. amoxicillin should not be given with meals

B. ciprofloxacin should be taken with dairy products

C. griseofulvin should be taken with high-protein meals

D. isoniazid should be taken with food

E. tetracycline should not be taken with dairy products

page 559
2009 PREP SA on CD-ROM

Critique: 163 Preferred Response: E


It is important for the clinician to recognize that the concomitant use of foods and medications may
result in an undesired response to the drug, either through decreased efficacy or increased risk of
toxicity. Such interactions may result from a feature of the drug or food. For example, some
medications, such as isotretinoin or griseofulvin, are more bioavailable when taken with fatty foods
because they are lipophilic agents. Others, such as digoxin, have lower bioavailability with high-fiber
meals because the fiber binds the drug. In fact, most food-drug interactions occur because some
characteristic of the food affects the bioavailability of the drug.
Components in some foods and dairy products chelate a portion of the drug and reduce
bioavailability, which occurs with many antibiotics. Other antibiotics are acid-labile agents, so the
increased gastric acid secretion that occurs with food intake may reduce their bioavailability. For some
medications, the effect is minimal unless the dose is inadequate, but for others, the risk is higher and
warrants careful attention to avoid treatment failure. Item C163 lists some antibiotics commonly used in
the pediatric population that have potential food-drug interactions. In general, for medications that
should not be taken with food, the drug should be given either 1 to 2 hours before or 2 hours after the
ingestion of the food in question.
Many other medications should be administered with caution to ensure proper bioavailability and
efficacy. Patients should be advised to pay close attention to any instructions offered by their
pharmacist to avoid adverse reactions or treatment failures.

References:

Gal P, Reed M. Principles of drug therapy. In: Kliegman RM, Behrman RE, Jenson HB, Stanton BF,
eds. Nelson Textbook of Pediatrics. 18th ed. Philadelphia, Pa: Saunders Elsevier; 2007:331-338

Schmidt LE, Dalhoff K. Food-drug interactions. Drugs. 2002;62:1481-1502. Abstract available at:
http://www.ncbi.nlm.nih.gov/pubmed/12093316

page 560
2009 PREP SA on CD-ROM

Critique: 163

page 561
2009 PREP SA on CD-ROM

Question: 164
During the health supervision visit of a term newborn boy, his mother relates that a cousin's child died
at age 4 months from sudden infant death syndrome. She asks what she can do to prevent such an
occurrence in her son.

Of the following, the single MOST important preventive measure is to

A. avoid use of a pacifier

B. cosleep in the parental bed for close observation

C. ensure side sleep positioning to prevent aspiration

D. ensure supine sleep positioning

E. use home apnea and bradycardia monitoring

page 562
2009 PREP SA on CD-ROM

Critique: 164 Preferred Response: D


Sudden infant death syndrome (SIDS) remains a significant cause of death in the first postnatal year.
However, the rate of death has diminished by more than 50% since the recommendation was made
more than a decade ago for supine rather than prone sleep position for children younger than 1 year of
age.
In the United States, infant cosleeping with parents increases the risk of SIDS, possibly related to
frank suffocation. However, the recommendation to avoid cosleeping remains controversial when
providing support for breastfeeding and considering the prevalence of cosleeping worldwide. Further
risk factors associated with cosleeping include parental obesity and sleeping on a very soft surface,
such as a waterbed. Tobacco smoke exposure increases the risk of SIDS, especially in the setting of
cosleeping.
Although initial recommendations suggested placing the child either supine or on the side to prevent
SIDS, subsequent research has shown that the side position also can increase the risk of SIDS,
possibly due to the propensity for a bundled infant placed on his or her side to roll forward into the
prone position. Therefore, parents should be urged to place the infant supine instead of on the side for
sleep.
Home apnea and bradycardia monitoring has not been shown to reduce the risk of SIDS, even in
families where a previous child has died from SIDS. Its use is limited to preterm infants who have
apnea of prematurity and infants in whom central apnea, known cardiac arrhythmia, or other identifiable
cause of events that may respond to monitoring and cardiorespiratory resuscitation is a distinct
possibility.
Pacifier use may reduce the incidence of SIDS but is not without controversy due to possible
interference with breastfeeding in the early weeks after birth. However, the peak incidence of SIDS
occurs between 2 and 6 months, which is primarily after the establishment of successful breastfeeding
during the first 2 postnatal weeks.
Modeling of appropriate infant sleep position and good patient education in the hospital may be
contributing factors to adoption of these practices.

References:

Colson ER, Levenson S, Rybin D, et al. Barriers to following the supine sleep recommendation among
mothers at four centers for the Women, Infants, and Children Program. Pediatrics.
2006;118:e243-e250. Available at: http://pediatrics.aappublications.org/cgi/content/full/118/2/e243

Creery D, Mikrogianakis A. Sudden infant death syndrome. BMJ Clinical Evidence. 2006. Available for
subscription at: http://clinicalevidence.bmj.com/ceweb/conditions/chd/0315/0315.jsp

Farrell PA, Weiner GM, Lemons JA. SIDS, ALTE, apnea, and the use of home monitors. Pediatr Rev.
2002;23:3-9. Available at: http://pedsinreview.aappublications.org/cgi/content/full/23/1/3

Fu LY, Moon RY. Apparent life-threatening events (ALTES) and the role of home monitors. Pediatr Rev.
2007;28:203-208. Available at: http://pedsinreview.aappublications.org/cgi/content/full/28/6/203

Hauck FR, Omojokun OO, Siadaty MS. Do pacifiers reduce the risk of sudden infant death syndrome?
A meta-analysis. Pediatrics. 2005;116:e716-e723. Available at:
http://pediatrics.aappublications.org/cgi/content/full/116/5/e716

Hein HA, Pettit SF. Back to Sleep: good advice for parents but not for hospitals? Pediatrics.

page 563
2009 PREP SA on CD-ROM

2001;107:537-539. Available at: http://pediatrics.aappublications.org/cgi/content/full/107/3/537

Moon RY, Fu LY. Sudden infant death syndrome. Pediatr Rev. 2007;28:209-214. Available at:
http://pedsinreview.aappublications.org/cgi/content/full/28/6/209

page 564
2009 PREP SA on CD-ROM

Question: 165
You are leading teaching rounds with the residents at the hospital. They present an 18-month-old boy
who has had 6 days of a temperature to at least 102.3°F (39.1°C). He also has nonexudative
conjunctivitis, a polymorphous rash, erythema of his lips, and swelling of his hands and feet. The
residents ask you to comment on the use of echocardiography in this condition.

Of the following, the MOST accurate statement about echocardiography in this disease is that

A. abnormal results at diagnosis suggest a poor outcome

B. it should be performed only if C-reactive protein concentrations are elevated

C. it should be performed to confirm the diagnosis

D. normal results at diagnosis obviate the need to repeat the study

E. the study may be useful in confirming atypical cases

page 565
2009 PREP SA on CD-ROM

Critique: 165 Preferred Response: E


The diagnostic criteria for Kawasaki disease (KD) are fever for at least 5 days and at least four of the
following five features:
* Changes in the extremities (edema, erythema, desquamation)
* Polymorphous exanthema
* Conjunctival injection
* Erythema and/or fissuring of the lips and oral cavity
* Cervical lymphadenopathy
In addition, to meet the diagnosis of KD, such findings should not be explainable by another disease
process. The patient described in the vignette has had fever for more than 5 days and has four of the
criteria of KD. He requires echocardiography for assessment and to obtain baseline measurements of
the coronary arteries. Echocardiography is an important component of the evaluation of the patient who
is suspected to have KD because it is a noninvasive, portable, and virtually risk-free method of
assessing cardiac structure and function. The complete echocardiographic assessment of the patient
who has acute KD includes an evaluation of cardiac function, determination of the presence of valvular
dysfunction, assessment for pericardial effusion, and baseline measurement of the coronary arteries.
Myocarditis, with or without valvular dysfunction and with or without pericardial effusion, can occur in
those who have KD and usually is not significant. However, myocarditis must be suspected in the
patient who has abnormal hemodynamic findings, and echocardiography should be pursued. By the
end of the first week of the illness, as many as 50% of children who have KD exhibit echocardiographic
evidence of coronary arteritis. This usually manifests as a "brightness" of the perivascular component of
the coronary artery and is the result of edema and thickening of the vascular wall. In addition, there may
be dilation, also referred to as ectasia, of the coronary arteries during this phase of the illness. Such
dilation, typically transient and asymptomatic, is distinct from the coronary artery aneurysms that can
develop in the second stage (2 to 4 weeks) of the illness (Item C165).
Echocardiography also can play a role diagnosing KD that presents in an incomplete or atypical
pattern (with fewer of the findings described previously at presentation). For example, a child who has
had a fever of unknown origin for 5 or more days but only two or three of the clinical criteria of KD may
have atypical KD. Newburger and colleagues have published an extensive protocol to facilitate the
evaluation of such patients. For those who have suspected atypical KD, further laboratory evaluation,
including measurement of C-reactive protein (CRP) and erythrocyte sedimentation rate (ESR), can be
helpful. If the CRP is 3.0 mg/dL or greater and the ESR is 40 mm/hr or greater, supplemental laboratory
tests should include echocardiography and assessment of serum albumin, hemoglobin, alanine
aminotransferase, white blood cell count, and platelet count. If the CRP is less than 3.0 mg/dL and the
ESR is less than 40 mm/hr, the child should be followed daily; if the fever persists for at least 7 days,
echocardiography should be performed. Echocardiography also should be obtained, even if the fever
resolves, if typical skin peeling occurs under the nail beds of the fingers and toes.
Abnormal findings, such as coronary arteritis, in patients who have KD do not portend a poor
outcome; rather, they are consistent with the diagnosis of KD. For this patient, who meets the criteria for
the diagnosis, echocardiography is performed regardless of the findings on laboratory tests.
Echocardiography neither confirms nor dispels the diagnosis for patients who meet the clinical criteria
of KD, and normal results do not obviate the need to repeat the study to assess disease progression.

References:

American Academy of Pediatrics. Kawasaki disease. In: Pickering LK, Baker CJ, Long SS, McMillan JA,
eds. Red Book: 2006 Report of the Committee on Infectious Disease. 27th ed. Elk Grove Village, Ill:
American Academy of Pediatrics; 2006:412-415

page 566
2009 PREP SA on CD-ROM

Miyashita M, Karasawa K, Taniguchi K, et al. Usefulness of real-time 3-dimensional echocardiography


for the evaluation of coronary artery morphology in patients with Kawasaki disease. J Am Soc
Echocardiogr. 2007;20:930-933. Abstract available at: http://www.ncbi.nlm.nih.gov/pubmed/17555938

Newburger JW, Takahashi M, Gerber MA, et al. AHA scientific statement. Diagnosis, treatment, and
long-term management of Kawasaki disease. A statement for health professionals from the Committee
on Rheumatic Fever, Endocarditis and Kawasaki Disease, Council on Cardiovascular Disease in the
Young, American Heart Association. Circulation. 2004;110:2747-2771. Available at:
http://circ.ahajournals.org/cgi/content/full/110/17/2747

Satou GM, Giamelli J, Gewitz MH. Kawasaki disease: diagnosis, management, and long-term
implications. Cardiol Rev. 2007;15:163-169. Abstract available at:
http://www.ncbi.nlm.nih.gov/pubmed/17575479

page 567
2009 PREP SA on CD-ROM

Critique: 165

Computed tomography angiography of the heart with three-dimensional


reconstruction shows a large aneurysm of the left coronary artery in a patient
who has Kawasaki disease. (Courtesy of D. Mulvihill)

page 568
2009 PREP SA on CD-ROM

Question: 166
A 6-year-old boy presents in late summer to the emergency department with a severe headache and
muscle pains. He recently returned from a camping trip. On physical examination, he is febrile and has
no focal weakness, but he suffers a prolonged tonic-clonic seizure and becomes unresponsive. Head
computed tomography scan reveals no abnormalities. Acyclovir and fosphenytoin are administered.
Magnetic resonance imaging shows subtle, diffuse signal change and thickening in the cerebral cortex,
no signal changes in temporal lobes, and no meningeal enhancement.

Of the following, the MOST likely cause of the boy’s symptoms is

A. arbovirus

B. Borrelia burgdorferi

C. herpes simplex virus 1

D. Listeria monocytogenes

E. Taenia solium

page 569
2009 PREP SA on CD-ROM

Critique: 166 Preferred Response: A


The child described in the vignette has symptoms of an acute viral infection, with fever, muscle pains,
and a prolonged seizure. Involvement of the central nervous system should be suspected because of
the seizure. The magnetic resonance imaging (MRI) findings suggest focal encephalitis that could be
due to an arbovirus or herpes simplex infection. Arbovirus infection is more likely to occur in late
summer because arbovirus is transmitted to humans by mosquitoes. In addition, herpes simplex
infection results in necrotizing focal encephalitis in the temporal lobes (Item C166A) rather than the
subtle thickening described in the vignette. In both cases, focal infection and inflammation in the brain's
cortex can lead to seizures. In the case of herpes, outside the neonatal period, the temporal lobe
infection occurs because of reactivation of the herpes simplex virus (HSV) in the trigeminal nerve and
spread directly into adjacent brain tissue.
Virus also can enter the brain via the blood, in which case generalized encephalitis is more likely.
Examples include HSV in the neonatal period or measles, although this is much rarer because of the
measles-mumps-rubella vaccine. Other viruses prone to cause encephalitis include human
immunodeficiency virus, cytomegalovirus, and rabies.
The other infectious etiologies are less likely, given the details of this case. Ingestion of the eggs of
Taenia solium (pork tapeworm) can lead to neurocysticercosis. This can present with focal-onset
seizures, but affected patients usually do not appear ill, and computed tomography scan or MRI
typically reveal isolated or multiple cysts (Item C166B). Borrelia burgdorferi is transmitted by ticks, and
early infection presents with the characteristic rash (erythema migrans). Late-stage disease can present
with central nervous system findings, the most common of which is facial nerve palsies. Meningitis is
less likely but can occur. Focal encephalitis is extremely unlikely. Listeria monocytogenes is not likely to
present in a child of this age, and the characteristic central nervous system infection is brainstem
encephalitis.

References:

Halstead S. Arborvirus encephalitis in North America. In: Kliegman RM, Behrman RE, Jenson HB,
Stanton BF, eds. Nelson Textbook of Pediatrics. 17th ed. Philadelphia, Pa: Saunders Elsevier;
2007:1405-1408

Mazzulli T. Laboratory diagnosis of infection due to viruses, Chlamydia, and Mycoplasma. In: Long SS,
Pickering LK, Prober CG, eds. Principles and Practice of Pediatric Infectious Diseases. 2nd ed. New
York, NY: Churchill Livingstone; 2003:1392-1408

Willoughby RE Jr. Encephalitis, meningoencephalitis, and postinfectious encephalomyelitis. In: Long


SS, Pickering LK, Prober CG, eds. Principles and Practice of Pediatric Infectious Diseases. 2nd ed.
New York, NY: Churchill Livingstone; 2003:291-295

page 570
2009 PREP SA on CD-ROM

Critique: 166

Axial computed tomography scan in herpes simplex virus encephalitis shows


necrosis in the right temporal lobe (arrow). (Courtesy of D. Gilbert)

page 571
2009 PREP SA on CD-ROM

Critique: 166

Neurocysticercosis: Coronal T1-weighted magnetic resonance imaging


following contrast reveals a 1-cm bilobed lesion in the left parietal area with
ring enhancement (arrow). (Courtesy of D. Gilbert)

page 572
2009 PREP SA on CD-ROM

Question: 167
A 7-month-old boy presents to the emergency department with vomiting and diarrhea. Findings on
physical examination are normal except for dehydration and lethargy. Laboratory tests reveal a serum
glucose concentration of 30.0 mg/dL (1.7 mmol/L). The mother tells you that she recently had the flu.
Family history is negative for any serious or chronic illnesses. You are considering an inborn error of
metabolism.

Of the following, the MOST helpful next laboratory test is measurement of

A. serum calcium

B. serum lipids

C. serum sodium

D. urine ketones

E. urine reducing substances

page 573
2009 PREP SA on CD-ROM

Critique: 167 Preferred Response: D


In the absence of sepsis, hypoglycemia in infancy most commonly is associated with disorders of
carbohydrate metabolism, such as glycogen storage diseases (GSDs) or fatty acid oxidation disorders.
A disorder of glycogen storage should be suspected for the infant who presents with hypoglycemia,
massive hepatomegaly without splenomegaly, and lactic acidosis; in these conditions, glycogen stores
in the liver cannot be broken down to supply necessary glucose. Symptoms of tremulousness and
irritability with fasting may be present for some time prior to diagnosis.
The infant described in the vignette is unlikely to have GSD because he has normal findings on
physical examination. His hypoglycemia is in association with symptoms of vomiting and diarrhea and a
recent exposure to influenza, raising the question of whether he is unable to create energy from fat
stores during this hypermetabolic state. The next step in making a diagnosis is to measure urine
ketones. If he is able to break down fatty acids for energy, he should have large ketones in his urine; if
this is not the case, he will have little to no ketones in his urine. Should the latter be the case, total and
free carnitine concentrations should be measured and an acylcarnitine profile be obtained to determine
which type of fatty acid oxidation defect is present.
Individuals who have GSD often have marked hyperlipidemia with apparent hyponatremia
(correction must be made for the increased serum solids), but the absence of these findings is not very
helpful in making a diagnosis for the child described in the vignette.
Serum calcium values typically are normal in both fatty acid oxidation and glycogen storage
disorders. Urine reducing substances characteristically are elevated in infants who have galactosemia,
which presents in the newborn period with hepatomegaly and jaundice.

References:

Burton BK. Inborn errors of metabolism in infancy: a guide to diagnosis. Pediatrics. 1998;102:e69-e78.
Available at: http://pediatrics.aappublications.org/cgi/content/full/102/6/e69

Hoffmann GF, Nyhan WL, Zschocke J, Kahler SG, Mayatepek E. Approach to the patient with metabolic
disease. In: Inherited Metabolic Diseases. Philadelphia, Pa. Lippincott Williams & Wilkins; 2002:19-94

Nyhan WL, Barshop BA, Ozand PT. Disorders of carbohydrate metabolism. In: Atlas of Metabolic
Diseases. 2nd ed. London, England: Hodder Arnold; 2005:371-372

Nyhan WL, Barshop BA, Ozand PT. Hyperammonemia and disorders of the urea cycle: introduction to
hyperammonemia and disorders of the urea cycle. In: Atlas of Metabolic Diseases. 2nd ed. London,
England: Hodder Arnold; 2005:191-192

Nyhan WL, Barshop BA, Ozand PT. Organic acidemia: introduction. In: Atlas of Metabolic Diseases.
2nd ed. London, England: Hodder Arnold; 2005:1-3, 191-192,371-372

page 574
2009 PREP SA on CD-ROM

Question: 168
A 15-year-old young woman has had joint pain for the past 3 days. She developed fever, chills, and
fatigue 4 days ago, but the fever has resolved. In addition, she explains that her left elbow, right knee,
and right wrist are all painful, red, and swollen, and she has a rash on her hands and feet that looks like
pus-filled bumps. She is sexually active, with inconsistent condom use for contraception. Physical
examination reveals an afebrile young woman who has swelling, tenderness, and mild erythema of the
left elbow, right knee, and right wrist. She has a few pustules and vesicles on the right palm and
bilateral soles (Item Q168). The abdomen is not tender and is without masses.

Of the following, the MOST likely pathogen causing this patient’s symptoms is

A. Chlamydia trachomatis

B. group A beta-hemolytic streptococci

C. Neisseria gonorrhoeae

D. parvovirus B19

E. Treponema pallidum

page 575
2009 PREP SA on CD-ROM

Question: 168

(Courtesy of Bernard Cohen, MD, DermAtlas; www.dermatlas.org)

page 576
2009 PREP SA on CD-ROM

Critique: 168 Preferred Response: C


Although probably most recognizable as causing urethritis or cervicitis, Neisseria gonorrhoeae can be
implicated in a number of other infections, including neonatal conjunctivitis, salpingitis, proctitis, pelvic
inflammatory disease, and disseminated gonococcal infection (DGI). DGI produces gonococcal
bacteremia and may result in petechial or pustular acral skin lesions, asymmetric arthralgia,
tenosynovitis, and septic arthritis, as described for the young woman in the vignette. Less frequently,
perihepatitis, endocarditis, and meningitis can occur. Patients who have DGI typically present with a
triad of tenosynovitis, dermatitis, and polyarthralgias without purulent arthritis or with purulent arthritis
without associated skin lesions. There may be crossover between these two syndromes, however. The
dermatitis is described as painless lesions that often are few in number and usually pustular or
vesiculopustular (Item C168A). Such lesions frequently are transient, lasting for 3 to 4 days.
The differential diagnosis of DGI may include group A beta-hemolytic streptococcal infection with
acute rheumatic fever, but the rash of this infection is very transient and almost never pustular or
vesicopustular. Parvovirus B19 also may cause a rash (fifth disease or erythema infectiosum) and
arthritis, but the rash usually is reticular on the extremities (Item C168B) and has a "slapped cheeks"
appearance on the face (Item C168C). Chlamydia trachomatis infection is not associated with a rash or
arthralgias. Secondary syphilis, caused by Treponema pallidum, is characterized by rash,
mucocutaneous lesions, and lymphadenopathy. The rash is described as polymorphic, generalized
maculopapular (Item C168D) and typically involves the palms and soles (Item C168E).

References:

American Academy of Pediatrics. Syphilis. In: Pickering LK, Baker CJ, Long SS, McMillan JA, eds. Red
Book: 2006 Report of the Committee on Infectious Diseases. 27th ed. Elk Grove Village, Ill: American
Academy of Pediatrics; 2006:631-644

Goldenberg DL, Sexton DJ. Disseminated gonococcal infection. UpToDate Online 15.3. 2008. Available
for subscription at: http://www.utdol.com/utd/content/topic.do?topicKey=stds/9841

Workowski KA, Berman SM, Centers for Disease Control and Prevention. Sexually transmitted
diseases treatment guidelines, 2006. MMWR Recomm Rep. 2006;55(RR11):1-94. Available at:
http://www.cdc.gov/mmwr/preview/mmwrhtml/rr5511a1.htm

page 577
2009 PREP SA on CD-ROM

Critique: 168

Patients who have disseminated gonococcal infection often exhibit necrotic


pustules that have a gray appearance. (Courttesy of D. Krowchuk)

page 578
2009 PREP SA on CD-ROM

Critique: 168

Fifth disease produces a lacy, reticulated erythema on the extremities.


(Courtesy of D. Krowchuk)

page 579
2009 PREP SA on CD-ROM

Critique: 168

Erythematous ("slapped") cheeks are a feature of erythema infectiosum.


(Courtesy of D. Krowchuk)

page 580
2009 PREP SA on CD-ROM

Critique: 168

Secondary syphilis produces a generalized eruption composed of


erythematous macules and papules. (Courtesy of C. Haverstock)

page 581
2009 PREP SA on CD-ROM

Critique: 168

The eruption of secondary syphilis often involves the palms and soles.
(Courtesy of C. Haverstock)

page 582
2009 PREP SA on CD-ROM

Question: 169
A 5-year-old boy has been receiving mechanical ventilation in the pediatric intensive care unit for 1
week due to complicated adenoviral pneumonia. Over the past several days, he has developed
markedly increased oxygen requirements and progressive opacification of his bilateral lung fields on
chest radiography (Item Q169). He now meets the clinical criteria for acute respiratory distress
syndrome (ARDS). His parents ask about the prognosis.

Of the following, the MOST accurate statement regarding the natural history of ARDS is that

A. complete resolution usually occurs by 1 month

B. development of other organ failure significantly increases the mortality rate

C. mortality is unrelated to the underlying cause

D. the major cause of death is unremitting respiratory failure

E. the overall mortality rate in children is 80%

page 583
2009 PREP SA on CD-ROM

Question: 169

Chest radiography findings, as observed for the child in the vignette.


(Courtesy of D. Mulvihill)

page 584
2009 PREP SA on CD-ROM

Critique: 169 Preferred Response: B


Acute respiratory distress syndrome (ARDS) represents the severe end of the acute lung injury
spectrum. Classically, ARDS is defined as a syndrome of lung inflammation with increased vascular
permeability characterized by an acute onset, bilateral infiltrates on chest radiography (Item C169),
absence of left atrial hypertension, and a PaO2/FiO2 of less than 200 mm Hg. The exact incidence of
ARDS in the pediatric population has been difficult to quantify because of changes in disease definition
and limitations of epidemiologic studies (both in methodology and age ranges studied). Most experts
agree that ARDS accounts for 2% to 4% of pediatric intensive care unit admissions. It has been
estimated that affected patients account for 8% of total patient days and 33% of deaths in the pediatric
intensive care unit.
Historically, risk factors for development of ARDS included sepsis, trauma, aspiration, pneumonia,
near-drowning, and multiorgan dysfunction. ARDS normally progresses through several phases. The
initial exudative phase is characterized by pulmonary edema, profound hypoxemia, and atelectasis.
Some patients recover in 3 to 7 days, but others progress to the fibroproliferative stage, which is
characterized by persistent hypoxemia, decreased lung compliance, and development of
pneumothoraces. The final recovery phase consists of resolution of hypoxemia and improved lung
compliance. Complete resolution may take 6 to 12 months, and some patients develop lifelong
restrictive lung disease, lung cysts, or decreased exercise tolerance.
Overall mortality in ARDS has been reported to be 30% to 40%, although exact rates in pediatric
patients remain uncertain. Patients rarely die solely from lung failure; multiorgan system failure or
withdrawal of life support is the most common cause of death. Mortality rates vary, depending on the
underlying disease process, even when corrected for severity of illness. Patients who develop ARDS
following trauma, for example, tend to have much better survival rates than those who develop ARDS
following bone marrow transplantation or gram-negative sepsis.

References:

Frankel LR. Respiratory distress and failure. In: Kliegman RM, Behrman RE, Jenson HB, Stanton BF,
eds. Nelson Textbook of Pediatrics. 18th ed. Philadelphia, Pa: Saunders Elsevier; 2007:421-423

Vish M, Shanley TP. Acute lung injury and acute respiratory distress syndrome. In: Wheeler DS, Wong
HR, Shanley TP, eds. Pediatric Critical Care Medicine: Basic Science and Clinical Evidence. New York,
NY: Springer-Verlag; 2007:395-411

page 585
2009 PREP SA on CD-ROM

Critique: 169

Anteroposterior chest radiograph in acute respiratory distress syndrome


demonstrates diffuse air space disease bilaterally without focal consolidation
or effusions. (Courtesy of D. Mulvihill)

page 586
2009 PREP SA on CD-ROM

Question: 170
A 6-month-old baby whose family has just emigrated from Ecuador is brought to your office by the
maternal grandmother. Physical examination reveals length at the 5th percentile, weight at the 10th
percentile, head circumference at the 25th percentile, a sallow complexion with jaundice, hoarse cry,
dry skin, and large tongue. The anterior fontanelle measures 3x4 cm. You diagnose primary
hypothyroidism and start appropriate thyroid hormone replacement therapy.

Of the following, the MOST likely long-term outcome in this baby is

A. adrenal insufficiency

B. microcephaly

C. normal adult height

D. normal intellectual function

E. precocious puberty

page 587
2009 PREP SA on CD-ROM

Critique: 170 Preferred Response: C


The baby described in the vignette has congenital hypothyroidism and has not received early treatment
with thyroid hormone. Fortunately, congenital hypothyroidism now usually is diagnosed soon after birth
by newborn screening programs in the United States, but this screening may not be available in
resource-limited countries. Children who have severe hypothyroidism at birth tend to have poorer
intellectual outcomes than their siblings, even if therapy is started in the first few days after birth.
However, earlier therapy with larger doses of thyroid hormone leads to improved intellectual outcomes,
even in the most high-risk athyreotic group of infants. Despite the guarded prognosis for intellectual
outcome, with continued and appropriate thyroid hormone therapy, this child should grow normally in
height, and adult height should be appropriate for the family.
Congenital hypothyroidism is not associated with adrenal insufficiency, although acquired
hypothyroidism because of chronic lymphocytic thyroiditis may be associated with the development of
autoimmune adrenal insufficiency (Addison disease). Microcephaly is an unusual outcome for children
treated appropriately with thyroid hormone. Indeed, brain catch-up growth in the first postnatal year and
a somewhat more flattened basal skull can give affected children larger head circumferences, split
sutures, and the appearance of macrocephaly with increased intracranial pressure. On the other hand,
overtreatment with thyroid hormone can lead to more rapid fusion of the cranial sutures and
microcephaly, so treatment must be monitored with careful and frequent laboratory measurements of
thyroid-stimulating hormone and thyroid hormone (free thyroxine [T4] or total T4 if free T4 is not
available). The potential for normal intellectual function in this child is guarded. At the least, he is likely
to have learning disabilities and some cerebellar problems, including clumsiness and decreased fine
motor skills. Precocious puberty is not an outcome of congenital hypothyroidism unless a child is
overtreated with thyroid hormone and develops premature maturation, which is very unusual. Some
children who have severe acquired hypothyroidism have manifested signs of sexual precocity that
disappear after treatment of the hypothyroidism (Van Wyk-Grumbach syndrome).

References:

American Academy of Pediatrics, Rose SR and the Section on Endocrinology and Committee on
Genetics, American Thyroid Association, Brown RS and the Public Health Committee, Lawson Wilkins
Pediatric Endocrine Society. Update of newborn screening and therapy for congenital hypothyroidism.
Pediatrics. 2006;117:2290-2303. Available at:
http://pediatrics.aappublications.org/cgi/content/full/117/6/2290

Kempers MJ, van der Sluijs Veer L, Nijhuis-van der Sanden RW, et al. Neonatal screening for
congenital hypothyroidism in the Netherlands: cognitive and motor outcome at 10 years of age. J Clin
Endocrinol Metab. 2007;92:919-924. Abstract available at:
http://www.ncbi.nlm.nih.gov/pubmed/17164300

Postellon DJ, Bourgeois MJ, Varma S. Congenital hypothyroidism. eMedicine Specialties, Pediatrics:
General Medicine, Endocrinology. 2007. Available at: www.emedicine.com/ped/topic501.htm

page 588
2009 PREP SA on CD-ROM

Question: 171
You are meeting with a family that recently moved to the United States. The two children, ages 11
months and 24 months, show evidence of global developmental delays. The parents ask what services
are available for their children. You explain that United States federal law provides children with early
intervention services.

Of the following, the BEST explanation about the provisions of the law is that

A. eligibility depends on the socioeconomic level of the family

B. family participation is mandated by law

C. the provision of services requires referral by a physician

D. the services are provided only to children who have autism

E. services for infants and toddlers must be family-based and culturally competent

page 589
2009 PREP SA on CD-ROM

Critique: 171 Preferred Response: E


Public Law 94-142, the Education of the Handicapped Act, passed in 1975, gave children between the
ages of 5 and 18 years access to a free, appropriate public education. Services to children between the
ages of 3 and 5 years of age were optional. In 1986, the law was amended as 99-457, which
established early intervention programs for children from birth to 3 years who had developmental delay.
The law also mandated that preschool children receive services. The law was amended again in 1990
with 101-476, the "Individuals with Disabilities Education Act" (IDEA). The portion entitled Part H, the
Program for Infants and Toddlers with Disabilities, required states to develop and implement
community-based systems of care that are coordinated, family-based, and culturally competent,
involving greater interagency collaboration. It mandated early identification and provision of services to
infants and toddlers who have developmental delays and established conditions. In 1997, the IDEA
amendment, PL 105-17 (Part C, formerly Part H), encouraged the states that did not serve the at-risk
population to track and monitor such children so that they could be referred when needed.
Eligibility for such programs is dependent on whether a child has any developmental disability, not
just autism. Families do not need a physician's referral for participation, they are not required to
participate, and such participation is not dependent on income.

References:

American Academy of Pediatrics Committee on Children With Disabilities. Role of the pediatrician in
family-centered Early Intervention Services. Pediatrics. 2001;107:1155-1157. Available at:
http://pediatrics.aappublications.org/cgi/content/full/107/5/1155

Davidson L. In brief: law and the child. Pediatr Rev. 2003;24:213-214. Available at:
http://pedsinreview.aappublications.org/cgi/content/full/24/6/213

Individuals with Disabilities Education Act 1997. Office of Special Education and Rehabilitative
Services. U.S. Department of Education. Available at:
http://www.ed.gov/about/offices/list/osers/policy.html

page 590
2009 PREP SA on CD-ROM

Question: 172
Results of a stool culture from a 2-year-old boy who has been hospitalized with bloody diarrhea indicate
that the causative agent is Shigella sp. The boy is allergic to trimethoprim-sulfamethoxazole.

Of the following, the MOST appropriate antimicrobial agent to use for this patient is

A. amoxicillin

B. azithromycin

C. cefdinir

D. ciprofloxacin

E. linezolid

page 591
2009 PREP SA on CD-ROM

Critique: 172 Preferred Response: B


Shigella are gram-negative bacilli that are readily spread person-to-person through fecal-oral
contamination. They usually are recovered from bacterial stool cultures using MacConkey and
Hektoen-Enteric agars. The four primary Shigella species (S sonnei, S flexneri, S boydii, and S
dysenteriae) cannot be distinguished clinically; they must be identified in the microbiology laboratory.
Shigella sp primarily infect the large intestine and may cause a range of manifestations from mild
diarrhea to life-threatening dysentery.
Unlike the nontyphoidal forms of Salmonella for which the indications for antimicrobial therapy are
limited to young (<3 months of age) or immunocompromised patients who have gastroenteritis, all
patients who have shigellosis benefit from therapy. Antimicrobial therapy can shorten the duration of
diarrhea and eradicate organisms from the stool. Although trimethoprim-sulfamethoxazole is a good
agent for empiric therapy of shigellosis, antimicrobial susceptibility testing always should be performed
due to increasing problems with resistance. If ampicillin and trimethoprim-sulfamethoxazole resistance
is verified, parenteral ceftriaxone, a fluoroquinolone (eg, ciprofloxacin), or azithromycin can be used.
Choosing an appropriate antimicrobial agent to treat the boy described in the vignette, who is allergic
to trimethoprim-sulfamethoxazole and is 2 years of age, is a challenge. Although ciprofloxacin is
effective, it should not be used routinely in children younger than 18 years of age unless no other
options are available. Surprisingly, the clinical usefulness of the oral cephalosporins (eg, cefdinir), with
the exception of cefixime, have not been demonstrated. Amoxicillin is less effective than ampicillin
because of its rapid absorption from the gastrointestinal tract, and linezolid is an antimicrobial agent
from a class of drugs called the oxazolidinones that are indicated for the treatment of resistant
gram-positive organisms (eg, methicillin-resistant Staphylococcus aureus). Therefore, the agent of
choice for this boy is azithromycin. The duration of antibiotic therapy for shigellosis typically is 5 days.

References:

American Academy of Pediatrics. Shigella infections. In: Pickering LK, Baker CJ, Long SS, McMillan
JA, eds. Red Book: 2006 Report of the Committee on Infectious Diseases. 27th ed. Elk Grove Village,
Ill: American Academy of Pediatrics; 2006:589-591

Niyogi SK. Increasing antimicrobial resistance-an emerging problem in the treatment of shigellosis. Clin
Microbiol Infect. 2007;13:1141-1143. Abstract available at:
http://www.ncbi.nlm.nih.gov/pubmed/17953700

Spence JT, Cheng TL. In brief: Shigella species. Pediatr Rev. 2004;25:329-330. Available at:
http://pedsinreview.aappublications.org/cgi/content/full/25/9/329

page 592
2009 PREP SA on CD-ROM

Question: 173
You are called to the delivery room to evaluate a term female infant born by precipitous normal
spontaneous vaginal delivery to an 18-year-old young woman who received no prenatal care. The
mother reports using marijuana and alcohol early in her pregnancy and was seen in the emergency
room on two occasions for urinary tract infections. She had several "colds" late in her pregnancy. She
lives with her boyfriend and has two dogs, a cat, and a turtle as pets. Physical examination of the infant
reveals a 2-kg lethargic, jaundiced infant who has a weak cry, microcephaly, and a distended abdomen.
Her liver is palpable 6 cm below the right costal margin, and her spleen is palpable 4 cm below the left
costal margin. She has a diffuse petechial rash with areas of purpura on her extremities (Item Q173).
Laboratory tests show a peripheral white blood cell count of 10.6x103/mcL (10.6x109/L), hemoglobin of
12.0 mg/dL (120.0 g/L), and platelet count of 60.0x103/mcL (60.0x109/L). The alanine aminotransferase
measurement is 300 U/L, and the aspartate aminotransferase value is 420 U/L. Head ultrasonography
shows scattered intracerebral calcifications.

Of the following, the MOST rapid test for making the diagnosis in this infant is

A. blood culture

B. cerebrospinal fluid polymerase chain reaction

C. nasopharyngeal viral culture

D. serology

E. urine culture

page 593
2009 PREP SA on CD-ROM

Question: 173

Purpura, as exhibited by the infant described in the vignette. (Reprinted with


permission from Stehel E, Sanchez PJ. NeoReviews. 2005;6:e38-e45.)

page 594
2009 PREP SA on CD-ROM

Critique: 173 Preferred Response: E


The newborn described in the vignette has signs and symptoms suggestive of congenital
cytomegalovirus (CMV) infection. CMV is a ubiquitous DNA virus that may be transmitted vertically from
mother to infant in utero by transplacental passage of maternal bloodborne virus, at birth by passage
through an infected maternal genital tract, or postnatally by ingestion of CMV-positive human milk.
Approximately 1% of all liveborn infants are infected in utero and excrete CMV at birth. The risk for
infection is greatest during the first half of gestation. In utero fetal infection can occur after maternal
primary infection or after reactivation of infection during pregnancy, but sequelae are much more
common in infants exposed to maternal primary infection, with 10% having manifestations evident at
birth.
Symptomatic congenital CMV disease is characterized by intrauterine growth restriction; jaundice;
hepatosplenomegaly; hepatitis; thrombocytopenia with petechiae and purpura (Item C173A); and
severe central nervous system involvement that can be characterized by microcephaly, intracerebral
calcifications, chorioretinitis (Item C173B), or sensorineural hearing loss.
Viral culture is the test of choice for confirming the diagnosis of congenital CMV infection. The
diagnosis is established by isolation of the virus from urine, stool, cerebrospinal fluid (CSF), or saliva in
the first 1 to 2 postnatal weeks. CSF polymerase chain reaction testing can be used to detect CMV
DNA, but it is less sensitive than viral isolation by culture and is not used routinely. Routine blood
culture and nasopharyngeal viral culture are not helpful for detecting CMV. Standard serologic testing is
a cumbersome approach to diagnosing congenital CMV disease; serial samples need to be obtained to
make the diagnosis clearly. The presence of CMV immunoglobulin M antibodies at birth is highly
suggestive of a congenital CMV infection, but a confirmatory urine culture for CMV is recommended to
establish the diagnosis definitively.

References:

American Academy of Pediatrics. Cytomegalovirus infection. In: Pickering LK, Baker CJ, Long SS,
McMillan JA, eds. Red Book: 2006 Report of the Committee on Infectious Diseases. 27th ed. Elk Grove
Village, Ill: American Academy of Pediatrics; 2006:273-277

Demmler GJ. Congenital cytomegalovirus infection and disease. Adv Pediatr Infect Dis.
1996;11:135-162

Demmler GJ. Cytomegalovirus. In: Feigin RD, Cherry JD, Demmler GJ, Kaplan SL, eds. Textbook of
Pediatric Infectious Diseases. 5th ed. Philadelphia, Pa: Saunders; 2004:1912-1932

Modlin JF, Grant PE, Makar RS, Roberts DJ, Krishnamoorthy KS. Case records of the Massachusetts
General Hospital. Weekly clinicopathological exercises. Case 25-2003: a newborn boy with petechiae
and thrombocytopenia. N Engl J Med. 2003;349:691-700

page 595
2009 PREP SA on CD-ROM

Critique: 173

Congenital cytomegalovirus infection may produce petechiae and purpura, as


seen on the face of this infant. (Reprinted with permission from Stehel E,
Sanchez PJ. NeoReviews. 2005;6:e38-e45.)

page 596
2009 PREP SA on CD-ROM

Critique: 173

Cytomegalovirus retinitis: White areas, often arranged along veins, represent


edema due to inflammation. Areas of early scarring (hyperpigmentation) are
present. (Courtesy of R.G. Weaver, Jr)

page 597
2009 PREP SA on CD-ROM

Question: 174
A 6-year-old girl is experiencing daytime and nighttime enuresis of 1 month’s duration. She achieved
daytime continence at age 3 and has been dry at night since age 4. She has no history of fever, but
does have some dysuria. The physical examination is remarkable only for suprapubic tenderness.
Urinalysis demonstrates a specific gravity of 1.015, pH of 6.5, 1+ blood, trace protein, 3+ leukocyte
esterase, and positive for nitrite. Microscopy reveals 2 to 5 red blood cells/high-power field (HPF), 20 to
50 white blood cells/HPF, and 3+ bacteria. Results of a urine culture are pending.

Of the following, the MOST appropriate empiric treatment for this patient is

A. amoxicillin

B. cefixime

C. cephalexin

D. ciprofloxacin

E. trimethoprim-sulfamethoxazole

page 598
2009 PREP SA on CD-ROM

Critique: 174 Preferred Response: E


The child described in the vignette has dysuria, incontinence, and suprapubic tenderness, but she is
otherwise well and has no fever or systemic symptoms. Based on the urinary findings of positive
leukocyte esterase and nitrite with pyuria on microscopy, she most likely has a lower urinary tract
infection (UTI) or cystitis.
Cystitis is treated with empiric outpatient oral antibiotic therapy directed at the most likely urinary
pathogens until the urine culture and antibiotic susceptibilities are available. Approximately 90% of UTIs
are caused by Escherichia coli. Because E coli are usually sensitive to trimethoprim-sulfamethoxazole
(TMP-SMX), it is an excellent agent for initial treatment of cystitis. In some regions of the country,
bacterial resistance to this agent may be increasing, but it still is considered the best first-line agent
because it may be more effective than beta-lactam antibiotics (eg, amoxicillin) in treating UTI, is
inexpensive and readily available, and allows the clinician to reserve more broad-spectrum antibiotics
for treatment failures. Due to high resistance rates, amoxicillin should not be used as first-line empiric
treatment for cystitis. Broader-spectrum antibiotics such as first-generation (cephalexin) and
third-generation (cefixime) cephalosporins or fluoroquinolones (ciprofloxacin) should be reserved for
second-line treatment unless the child has recurrent UTI and a history of resistance or allergy to
TMP-SMX.
The standard duration of treatment is 7 to 14 days. A recent meta-analysis supported the use of a
short course (2 to 4 days) of antibiotics, which compared favorably with a standard course (7 to 14
days) in terms of treatment failures and recurrence of lower UTI. Single-dose antibiotic regimens have
been used in adults, but pediatric studies demonstrate an increased number of treatment failures and
UTI recurrences compared with standard 7- to 14-day treatment regimens.

References:

Jantausch B, Kher K. Urinary tract infection. In: Kher KK, Schnaper HW, Makker SP, eds. Clinical
Pediatric Nephrology. 2nd ed. London, England: Informa Healthcare; 2007:553-573

Keren R, Chan E. A meta-analysis of randomized, controlled trials comparing short- and long-course
antibiotic therapy for urinary tract infections in children. Pediatrics. 2002;109:e70. Available at:
http://pediatrics.aappublications.org/cgi/content/full/109/5/e70

Larcombe J. Urinary tract infection in children. BMJ Clinical Evidence. 2007. Available for subscription
at: http://clinicalevidence.bmj.com/ceweb/conditions/chd/0306/0306.jsp

Michael M, Hodson EM, Craig JC, Martin S, Moyer VA. Short versus standard duration oral antibiotic
therapy for acute urinary tract infection in children. Cochrane Database Syst Rev. 2003;1:CD003966.
Available at: http://www.mrw.interscience.wiley.com/cochrane/clsysrev/articles/CD003966/frame.html

Raszka WV Jr, Khan O. Pyelonephritis. Pediatr Rev. 2005;26:364-370. Available at:


http://pedsinreview.aappublications.org/cgi/content/full/26/10/364

page 599
2009 PREP SA on CD-ROM

Question: 175
An 8-year-old boy presents with wheezing, coughing, and difficulty breathing of 6 months’ duration.
Findings on his history and pulmonary function tests are suggestive of moderate persistent asthma. In
preparation for asthma management, you have reviewed the current asthma guidelines, educated the
patient on peak flow monitoring, and discussed possible therapeutic options. You decide to start him on
a daily inhaled corticosteroid.

Of the following, the MOST likely adverse event he may experience from inhaled corticosteroids is

A. acne

B. decreased adult height

C. mood swings

D. oral candidiasis

E. weight gain

page 600
2009 PREP SA on CD-ROM

Critique: 175 Preferred Response: D


Despite remaining the cornerstone therapy for persistent asthma, many parents are reluctant to have
their children begin using inhaled corticosteroids. Addressing parents' concerns and outlining the
potential adverse effects may improve understanding and compliance.
Local oropharyngeal symptoms, including dysphonia, oral candidiasis, and cough, are commonly
encountered adverse effects of inhaled corticosteroids. Local symptoms appear to be dose-related and
can be lessened by using a "spacer" and rinsing out the mouth after use. A number of studies have
documented a transient decrease in growth velocity during the first year of inhaled corticosteroid
therapy, but a daily cumulative dose up to 800 mcg of inhaled budesonide has been shown not to affect
predicted adult height.
Less common adverse effects of inhaled steroids include acne, mood swings, weight gain,
decreased serum immunoglobulin G concentrations, and rarely, posterior subcapsular cataracts and
adrenal suppression.
Inhaled steroids are used in persistent asthma to help halt lung remodeling by decreasing bronchial
inflammation, reducing inflammatory mediators, and decreasing bronchial hyperresponsiveness.

References:

Allen DB, Bielory L, Derendorf H, Dluhy R, Colice GL, Szefler SJ. Inhaled corticosteroids: past lessons
and future issues. J Allergy Clin Immunol. 2003;112(3 suppl):S1-S40. Abstract available at:
http://www.ncbi.nlm.nih.gov/pubmed/14515117

Keeley D, McKean M. Asthma and other wheezing disorders in children. BMJ Clinical Evidence. 2006.
Available for subscription at: http://clinicalevidence.bmj.com/ceweb/conditions/chd/0302/0302.jsp#Q1

Schielmer RP, Spahn JD, Covar R, Szefler SJ. Glucocorticoids. In: Adkinson NF, Jr, Yunginger JW,
Busse WW, Bochner BS, Holgate ST, Simons FE, eds. Middleton's Allergy Principles and Practice. 6th
ed. Philadelphia, Pa: Mosby Inc; 2003:870-913

page 601
2009 PREP SA on CD-ROM

Question: 176
Your first patient of the day is a 2-year-old girl who is brought in by her mother after a brown spider was
found in the child’s bed. The mother has brought the spider for you to inspect (Item Q176A). On
physical examination, there is a 2-cm bulla with 4 cm of surrounding erythema on the medial aspect of
the girl’s calf (Item Q176B). The child otherwise appears well and occasionally scratches at the lesion.

Of the following, the MOST appropriate course of action for this patient is to

A. begin dapsone therapy

B. begin local wound care

C. prescribe a 5-day course of prednisone

D. refer the child to a surgeon for excision of the bite area

E. transfer the patient to the emergency department for antivenom

page 602
2009 PREP SA on CD-ROM

Question: 176

Type of spider brought by the family for inspection. (Courtesy of the Centers
for Disease Control and Prevention, Public Health Image Library)

page 603
2009 PREP SA on CD-ROM

Question: 176

Bulla with surrouding erythema, as described for the child in the vignette.
(Courtesy of M. Rimsza)

page 604
2009 PREP SA on CD-ROM

Critique: 176 Preferred Response: B


In the United States, two spider species are responsible for most spider bite-related illness and injury:
Lactrodectus sp (black widow spiders) (Item C176A) and Loxosceles sp. Black widow spider bites
cause a systemic syndrome characterized by autonomic dysfunction, muscle cramping, and rigidity due
to neurotoxins in the venom. Spiders of the genus Loxosceles, of which the brown recluse spider
(Loxosceles recluse) is best known, are recognized primarily as a cause of necrotic skin lesions,
although systemic symptoms, including a flulike illness, hemolytic anemia, and renal failure, may occur.
The spider responsible for the skin lesion described for the child in the vignette is a brown recluse,
which can be recognized by its brown color, violin-shaped marking on the thorax (Item C176B), and
three sets of eyes. Found throughout the central and southern United States, they live in dark, protected
environments such as wood piles or storage sheds but can be found indoors in bedsheets and clothing
hampers. They are nocturnal, and human bites typically occur at night. Brown recluse spider venom
contains a variety of proteolytic enzymes that can cause extensive tissue damage. The bite itself
usually is painless and may go unnoticed until pain, erythema, and pruritus develop at the site. The
initial erythematous maculopapular lesion becomes bullous (Item C176C) and increases in size over
the subsequent 48 hours. After that time period, an eschar develops (Item C176D) and subsequently
separates, leaving a deep ulcer at the site (Item C176E). Systemic symptoms include fever, chills,
vomiting, and arthralgias. Because children are smaller and receive a larger per kilogram venom dose,
they are affected more commonly by the systemic syndrome, known as loxoscelism.
Black widow spider bites also are painless and leave little more than two small red, pinpoint marks
on the skin. The venom causes severe muscular cramping, tremors, and autonomic symptoms such as
drooling and sweating. The symptoms seen after black widow spider bites frequently are mistaken for a
variety of other conditions, including cholinergic crisis and acute abdominal processes.
Treatment of brown recluse spider bites, black widow spider bites, and bites of all other spiders
indigenous to North America is largely supportive and includes conscientious wound care and tetanus
immunization, if indicated. The muscular cramping caused by a black widow spider bite is controlled
readily with benzodiazepines, narcotics, and calcium gluconate. An antivenom is available, although its
use is indicated only when autonomic symptoms or pain cannot be managed with usual measures. Skin
grafting may be necessary if tissue necrosis occurs following a brown recluse spider bite. Many
therapeutic modalities have been advocated to limit the extent of the brown recluse bite wound,
including dapsone, early wound excision, hyperbaric oxygen, and steroids, but none have proven
effective. Loxosceles antivenom is not available in the United States.
Skin findings caused by a wide variety of other pathologic conditions often are attributed to brown
recluse spider bites. The differential diagnosis is extensive and includes conditions such as
staphylococcal and streptococcal skin lesions, cutaneous anthrax, atypical mycobacterial infection,
sporotrichosis, ecthyma gangrenosum, herpes simplex and zoster, vasculitic lesions, erythema
chronicum migrans, and erythema nodosum. Because necrotic skin lesions can be caused by a number
of serious conditions, and documented spider bites are rare, clinicians should consider alternative
diagnoses when faced with a necrotic skin lesion, if the spider is not available for inspection.

References:

Singletary EM, Rochman AS, Bodmer JCA, Holstege CP. Envenomations. Med Clin North Am.
2005;89:1195-1224. Abstract available at: http://www.ncbi.nlm.nih.gov/pubmed/16227060

Sjogren R, MacGregor RS, Zenel J. Visual diagnosis: an infant who has a red papule on a swollen,
tender arm. Pediatr Rev. 2004;25:182-185. Available at:
http://pedsinreview.aappublications.org/cgi/content/full/25/5/182

page 605
2009 PREP SA on CD-ROM

Swanson DL, Vetter RS. Bites of brown recluse spiders and suspected necrotic arachnidism. N Engl J
Med. 2005;352:700-707. Extract available at: http://content.nejm.org/cgi/content/extract/352/7/700

page 606
2009 PREP SA on CD-ROM

Critique: 176

Female black widow spider: Note the red hourglass-shaped marking on the
ventral abcominal surface (the color of this marking is variable, however, and
may be yellow, orange, or white). (Courtesy of the Centers for Disease
Control and Prevention, Public Health Image Library, James Gathany)

page 607
2009 PREP SA on CD-ROM

Critique: 176

Brown recluse spider: Note the violin-shaped marking on the thorax (arrow).
(Courtesy of the Centers for Disease Control and Prevention, Public Health
Image Library)

page 608
2009 PREP SA on CD-ROM

Critique: 176

Following the bite of a brown recluse spider, a bulla with surrounding


erythema may develop. (Courtesy of M. Rimsza)

page 609
2009 PREP SA on CD-ROM

Critique: 176

An eschar with surrounding erythema may occur following the bite of a brown
recluse spider. (Courtesy of M. Smith)

page 610
2009 PREP SA on CD-ROM

Critique: 176

Folliowing the bite of a brown recluse spider, an eschar may develop and
subsequently separate, leaving a deep ulcer. (Courtesy of M. Rimsza)

page 611
2009 PREP SA on CD-ROM

Question: 177
A 12-year-old boy has had intermittent heartburn for the past several years. Results of an upper
gastrointestinal radiographic series performed at age 11 years were normal. Over the past year, he has
had several episodes of "food getting stuck in his chest." The most common foods that cause him
difficulty are hot dogs, steak tips, and chicken strips. Physical examination findings are unremarkable.
He has been treated with omeprazole for 3 months, but symptoms persist.

Of the following, the MOST appropriate next diagnostic test is

A. esophageal manometry study

B. esophagogastroduodenoscopy with biopsy

C. radionuclide gastric emptying scan

D. serum immunoglobulin E measurement

E. 24-hour esophageal pH probe study

page 612
2009 PREP SA on CD-ROM

Critique: 177 Preferred Response: B


The chronic heartburn and dysphagia to solid foods, primarily meats, described for the boy in the
vignette strongly suggest either reflux esophagitis (possibly with an esophageal stricture) or eosinophilic
esophagitis (EE) (Item C177A). The most appropriate next test to evaluate the dysphagia is
esophagogastroduodenoscopy with biopsies. The endoscopy can characterize the severity of
inflammation, exclude Barrett esophagus (a premalignant complication of gastroesophageal reflux
[GER]), and potentially differentiate between GER and EE. Esophageal manometry, radionuclide gastric
emptying scan, measurement of serum immunoglobulin E, and 24-hour esophageal pH probe study are
potentially useful in the evaluation of GER or allergy, but they are not the appropriate initial tests in
evaluating solid food dysphagia.
GER is defined as excessive regurgitation of gastric contents (both acid and nonacid) into the
esophagus. The most common gastrointestinal symptoms are heartburn (pyrosis), dysphagia (difficulty
swallowing), and odynophagia (painful swallowing). GER also may result in extraintestinal symptoms,
including hoarseness, chronic cough, exacerbation of asthma, apnea, and aspiration pneumonia. GER
is extremely common in childhood. According to one cross-sectional study, 3% to 5% of healthy
children report reflux symptoms at any given time. Because GER frequently is mild and self-limited,
empiric therapy with an antacid, histamine receptor blocker, or proton pump inhibitor is appropriate and
can alleviate symptoms. Prolonged, persistent reflux (ie, more than 3 months) or complicated reflux (ie,
associated with hematemesis or respiratory symptoms) warrants further evaluation. Among the useful
tests to evaluate GER are upper gastrointestinal radiographic series (which excludes malrotation and
hiatal hernia), pH probe study or esophageal impedance (which determine the extent and duration of
reflux over a 24-hour period), gastric emptying scan (which rules out gastroparesis), and esophageal
motility evaluation (which assesses peristalsis and esophageal sphincter pressure). Upper endoscopy
with biopsy generally is the most valuable test because it can determine if chronic GER has caused any
esophageal injury (Item C177B).
In the past 10 years, researchers have recognized that not all heartburn or dysphagia is caused by
GER. EE presents similarly to reflux, but is believed to be an atopic condition. Affected patients tend to
have more dysphagia symptoms than heartburn. Often, there is a history of meat impaction in the
esophagus. As noted previously, upper endoscopy with biopsies frequently can differentiate between
the two conditions. GER usually is characterized by ulceration and injury limited to the distal 5 cm of the
esophagus. In contrast, EE is characterized by inflammation of the proximal, middle, and distal
esophagus, with large numbers of eosinophils seen on biopsy. Differentiating between these two
conditions is important because treatment of GER involves acid blockade. In contrast, the therapies for
EE include anti-inflammatory therapy (eg, fluticasone, budesonide), acid blockade, or elimination diets.

References:

Furuta GT, Straumann A. Review article: the pathogenesis and management of eosinophilic
oesophagitis. Aliment Pharmacol Ther. 2006;24:173-182. Available at:
http://www.blackwell-synergy.com/doi/full/10.1111/j.1365-2036.2006.02984.x

Kumar Y, Sarvananthan R. Gastro-oesophageal reflux in children. BMJ Clinical Evidence. 2007.


Available for subscription at: http://clinicalevidence.bmj.com/ceweb/conditions/chd/0310/0310.jsp

Nelson SP, Chen EH, Syniar GM, Kaufer Christoffer K; for the Pediatric Practice Research Group.
Prevalence of symptoms of gastroesophageal reflux during childhood. Arch Pediatr Adolesc Med.
2000;154:150-154.

page 613
2009 PREP SA on CD-ROM

Critique: 177

Eosinophilic esophagitis: Findings at endoscopy may include mucosal rings


(A), furrows (B), and exudates (C). (Reprinted with permission from Hopp R,
Natarajan N. Index of suspicion (case 1). Pediatr Rev. 2007;28:389-394 and
courtesy of Doernbecher Children’s Hospital)

page 614
2009 PREP SA on CD-ROM

Critique: 177

In severe esophagitis, the areas of erythema represent inflammation, and


white exudates cover erosions. (Courtesy of A. Bousvaros)

page 615
2009 PREP SA on CD-ROM

Question: 178
You are seeing a 12-month-old infant who was born at 26 weeks’ gestation. He receives daily diuretics
and nasal cannula oxygen with a baseline flow of 0.1 L/min, but his mother called this morning reporting
that he had a temperature of 100.5°F (37.8°C), nasal congestion, increased work of breathing with a
rapid respiratory rate, and a "wheezing" cough. You instructed her to increase the oxygen flow rate to
0.5 L/min and come directly to the clinic. Physical examination reveals intercostal and subcostal
retractions, a respiratory rate of 80 breaths/min, and a prolonged expiratory phase with audible
wheezing. A copious, cloudy, green nasal discharge is present. No heart murmur is audible. Pulse
oximetry while receiving 0.5 L/min oxygen reveals an oxygen saturation of 85% at rest.

Of the following, the BEST explanation for this child’s presenting signs of respiratory distress is

A. acute cor pulmonale

B. foreign body aspiration

C. gastroesophageal reflux

D. lower respiratory tract infection

E. upper respiratory tract infection

page 616
2009 PREP SA on CD-ROM

Critique: 178 Preferred Response: D


The infant described in the vignette has symptoms of a lower respiratory tract infection. The presence of
nasal discharge suggests a concomitant or preceding upper respiratory tract infection, but an upper
respiratory tract infection alone would not result in tachypnea, retractions, or hypoxemia. It is unlikely
that the infant has aspirated a foreign body because pulmonary findings are bilateral. There is no
history of emesis or prolonged concerns for gastroesophageal reflux. Although chronic hypoxemia may
contribute to cor pulmonale, this is not an acute condition and is inconsistent with the findings described
in the vignette.
Chronic lung disease of prematurity (CLD), also referred to as bronchopulmonary dysplasia, remains
a continuing concern for graduates of the neonatal intensive care unit (NICU) who have required
intubation, assisted ventilation, and oxygen therapy for extended periods of time. Primarily affecting
very low-birthweight (VLBW) infants (<1,500 g), CLD is characterized by a sustained oxygen
requirement beyond 36 weeks' gestation corrected age; obstructive small airways disease that may be
reversible with bronchodilators; and inflammatory characteristics that include increased airway
secretions, smooth muscle bronchoreactivity, and increased interstitial lung water. The infant who has
CLD generally has an increased work of breathing, with an associated increased caloric expenditure,
and a decreased pulmonary reserve capacity. Hence, he or she has little ability to meet increased
metabolic and pulmonary work associated with exertion (eg, physical exercise) or increased respiratory
work (associated with lower respiratory tract infection). As a result, up to 50% of VLBW infants who
have CLD are rehospitalized for pulmonary decompensation in the first year after discharge from the
NICU.
The infant who has CLD requires increased calories and protein to grow compared with unaffected
age- and birthweight-matched preterm infants. Good nutrition can help the at-risk growing preterm
infant who has CLD in attaining good overall physical growth, pulmonary development, and
neurodevelopment as well as optimizing immune function.

References:

American Academy of Pediatrics Committee on Fetus and Newborn. Postnatal corticosteroids to treat
or prevent chronic lung disease in preterm infants. Pediatrics. 2002;109;330-338. Available at:
http://pediatrics.aappublications.org/cgi/content/full/109/2/330

Bancalari EH. Bronchopulmonary dysplasia and neonatal chronic lung disease. In: Martin RJ, Fanaroff
AA, Walsh MC, eds. Fanaroff and Martin's Neonatal-Perinatal Medicine. 8th ed. Philadelphia, Pa:
Mosby Elsevier; 2006:1155-1167

Baraldi E, Filippone M. Chronic lung disease after premature birth. N Engl J Med. 2007;357:1946-1955.
Extract available at: http://content.nejm.org/cgi/content/extract/357/19/1946

Bhandari A, Panitch HB. Pulmonary outcomes in bronchopulmonary dysplasia. Semin Perinatol.


2006;30:219-226. Abstract available at: http://www.ncbi.nlm.nih.gov/pubmed/16860162

Biniwale MA, Ehrenkranz RA. The role of nutrition in the prevention and management of
bronchopulmonary dysplasia. Semin Perinatol. 2006;30:200-208. Abstract available at:
http://www.ncbi.nlm.nih.gov/pubmed/16860160

Shaw NJ, Kotecha S. Management of infants with chronic lung disease of prematurity in the United
Kingdom. Early Hum Dev. 2005;81:165-170. Abstract available at:

page 617
2009 PREP SA on CD-ROM

http://www.ncbi.nlm.nih.gov/pubmed/15748971

Vaucher YE. Bronchopulmonary dysplasia: an enduring challenge. Pediatr Rev. 2002;23:349-358.


Available at: http://pedsinreview.aappublications.org/cgi/content/full/23/10/349

page 618
2009 PREP SA on CD-ROM

Question: 179
You are evaluating a 10-month-old boy brought to the emergency department because of fussiness for
1 day. His mother reports that she was carrying him while answering the phone yesterday and that he
fell from her arms onto the linoleum floor. Physical examination reveals a thin boy who is crying. He
resists weight-bearing on the left leg, but you cannot elicit specific tenderness. He has bruises on the
left temporal region, upper arm, and thighs. You suspect nonaccidental trauma and order a skeletal
survey.

Of the following, the skeletal survey finding that is MOST specific for nonaccidental trauma is

A. linear nondisplaced skull fracture

B. long bone nondisplaced shaft fracture

C. metaphyseal chip fracture

D. spiral tibial fracture

E. subperiosteal new bone formation

page 619
2009 PREP SA on CD-ROM

Critique: 179 Preferred Response: C


Whenever child abuse is suspected, it is critical for the clinician to evaluate the child fully for any and all
potential manifestations of such abuse. The evaluation begins with a thorough trauma history and social
history, paying close attention to the child-parent interaction, and a complete physical examination.
When bruises or other lesions are present, or the history is inconsistent with the mechanism of action
for the lesions, child abuse should be strongly suspected, and further evaluation is required. Such
evaluation includes a complete skeletal survey to look for fractures.
A skeletal survey consists of radiographs of the skull, lateral cervical spine, ribs, pelvis,
thoracolumbar spine, arms, hands, legs, and feet. Anteroposterior and lateral views should be obtained
to examine all aspects of the bones fully. A skeletal survey can provide information about the age of a
fracture, although some fractures, such as those of the ribs, may not be recognized early. A repeat
survey 2 weeks later can be helpful to identify healing fractures that exhibit new bone formation.
Certain types of fractures have a high specificity for nonaccidental trauma. Among the most specific
are metaphyseal chip fractures (Item C179A) and metaphyseal bucket handle fractures (Item C179B),
both of which are seen in children younger than 1 year of age. Other concerning findings include
fractures of the posterior ribs (Item C179C), scapula, spinous process, and sternum as well as multiple
fractures, fractures of different ages, femur fractures in children younger than 1 year of age (Item
C179D), and complex skull fractures.
Common skeletal survey findings that have a low specificity for child abuse include simple linear
skull fractures, nondisplaced long bone fractures, and subperiosteal new bone formation. A
nondisplaced spiral fracture of the tibia, also known as a "toddler's fracture" (Item C179E), is the result
of a twisting motion of the tibia and can occur in young children who have just learned to walk. Even in
these types of fractures, however, it is important to obtain a complete trauma history. If the history is
inconsistent with the injury, a full evaluation for nonaccidental trauma is indicated. Further evaluation
includes imaging of the head (either computed tomography scan or magnetic resonance imaging) to
look for hematomas, ophthalmologic examination to look for retinal hemorrhages (Item C179F), and
urinalysis and liver and pancreatic enzyme measurement to look for evidence of blunt abdominal
trauma. Any suspicious findings warrant mandatory reporting to child protective services.

References:

Jenny C; Committee on Child Abuse and Neglect. Evaluating infants and young children with multiple
fractures. Pediatrics. 2006;118:1299-1303. Available at:
http://pediatrics.aappublications.org/cgi/content/full/118/3/1299

Kellogg ND and the Committee on Child Abuse and Neglect. Evaluation of suspected child physical
abuse. Pediatrics. 2007;119:1232-1241. Available at:
http://pediatrics.aappublications.org/cgi/content/full/119/6/1232

Sirotnak AP, Grigsby T, Krugman RD. Physical abuse of children. Pediatr Rev. 2004;25:264-277.
Available at: http://pedsinreview.aappublications.org/cgi/content/full/25/8/264

page 620
2009 PREP SA on CD-ROM

Critique: 179

Metaphyseal fracture (arrow) of the left proximal humerus in an infant who


had been physically abused. (Courtesy of D. Krowchuk)

page 621
2009 PREP SA on CD-ROM

Critique: 179

Bucket handle metaphyseal fracture of the radius (arrow) in a child who has
been physically abused. (Courtesy of S. Sinal)

page 622
2009 PREP SA on CD-ROM

Critique: 179

Healing posterior rib fractures (arrows) in an infant who has been physically
abused. (Courtesy of D. Krowchuk)

page 623
2009 PREP SA on CD-ROM

Critique: 179

Right femur fracture in a 2-month-old child who had been physically abused.
(Courtesy of D. Krowchuk)

page 624
2009 PREP SA on CD-ROM

Critique: 179

Oblique (left) and anteroposterior (right) views of the distal tibia show a
nondisplaced spiral (toddler's) fracture. (Courtesy of D. Mulvihill)

page 625
2009 PREP SA on CD-ROM

Critique: 179

Retinal hemorrhages and papilledema in an infant who had been shaken.


(Courtesy of M. Rimsza)

page 626
2009 PREP SA on CD-ROM

Question: 180
A medical student notes on rounds that a 2-year-old girl admitted for pneumonia has a complete blood
count (CBC) that includes a hematocrit of 35% (0.35), hemoglobin of 11.5 g/dL (115.0 g/L), mean
corpuscular volume of 68.0 fL, and platelet and white blood cell counts that are normal for age. During
the bedside encounter with the child’s mother, you advise her to start the child on a multivitamin with
iron and have her primary care physician obtain another CBC in a month or so. The medical student
asks why you recommended iron supplementation when the child has a normal hematocrit.

Of the following, the BEST reason to prescribe supplemental iron therapy for this child at this time is to
prevent

A. diminished cognitive abilities

B. fatigue

C. rapid progression to anemia

D. recurrent infections

E. short stature

page 627
2009 PREP SA on CD-ROM

Critique: 180 Preferred Response: A


The child described in the vignette likely has iron deficiency, as evidenced by her low mean corpuscular
volume. Providing iron supplementation may improve her cognition.
It is not clear whether effects on cognition and behavior caused by iron deficiency are completely
reversible with iron therapy. Of the two studies that treated children for 2 months or longer, one reported
dramatic benefits for development and the other did not. Evidence suggests that children who had iron
deficiency as toddlers may have slightly impaired cognition in elementary school, even if they were
treated with iron and the anemia resolved. Finally, according to some evidence, iron deficiency may
cause or be associated with symptoms of attention-deficit/hyperactivity disorder that may be improved
with iron therapy.
For many children, such as the girl described in the vignette, iron deficiency is revealed on a
complete blood count obtained because of a febrile illness. Fever may cause transient anemia and
microcytosis due to hemolysis, and repeat screening is recommended when the child recovers. Routine
iron supplementation with a multivitamin with iron should be prescribed until it is determined whether
iron therapy is required.
Iron deficiency has not been proven to cause fatigue in the absence of anemia, and there is no
definitive association between isolated iron deficiency or anemia and short stature. Iron deficiency does
not cause recurrent infection. Iron deficiency does not lead rapidly to anemia; rather, anemia may
develop over weeks to months.

References:

Konofal E, Lecendreux M, Arnulf I, Mouren MC. Iron deficiency in children with


attention-deficit/hyperactivity disorder. Arch Pediatr Adolesc Med. 2004;158:1113-1115.

Lozoff B, De Andraca I, Castillo M, Smith JB, Walter T, Pino P. Behavioral and developmental effects of
preventing iron-deficiency anemia in healthy full-term infants. Pediatrics. 2003;112:846-854. Available
at: http://pediatrics.aappublications.org/cgi/content/full/112/4/846

Lozoff B, Jimenez E, Smith JB. Double burden of iron deficiency in infancy and low socioeconomic
status: a longitudinal analysis of cognitive test scores to age 19 years. Arch Pediatr Adolesc Med.
2006;160:1108-1113.

Martins S, Logan S, Gilbert R. Iron therapy for improving psychomotor development and cognitive
function in children under the age of three with iron deficiency anaemia. Cochrane Database Syst Rev.
2001;2;CD001444. Available at:
http://www.mrw.interscience.wiley.com/cochrane/clsysrev/articles/CD001444/frame.html

Wu AC, Lesperance L, Bernstein H. Screening for iron deficiency. Pediatr Rev. 2002;23:171-178.
Available at: http://pedsinreview.aappublications.org/cgi/content/full/23/5/171

page 628
2009 PREP SA on CD-ROM

Question: 181
The youngest child in a family affected by neurofibromatosis 1, who is 5 years old, has just had the
diagnosis confirmed. You begin the process of counseling the family.

Of the following, the MOST accurate statement about potential medical complications in affected
children is that they

A. are at increased risk of developing cataracts

B. have an increased incidence of acoustic neuroma

C. have an increased incidence of childhood hypertension

D. should be restricted from contact sports

E. should be screened annually for optic gliomas using computed tomography scan

page 629
2009 PREP SA on CD-ROM

Critique: 181 Preferred Response: C


Neurofibromatosis (NF) is an autosomal dominant neurocutaneous disorder of which there are at least
two types. Type 1 has an incidence of approximately 1 in 3,500 people, and its characteristic features
are the cutaneous café au lait macules and benign neurofibromas (Item C181). Type 2 has an
incidence of approximately 1 in 40,000, and a characteristic feature is vestibular schwannoma.
Individuals affected by NF-1 are at risk for the development of a number of complications, including
optic pathway tumors, skeletal deformations, speech impediments, and learning disabilities. In addition,
there is variable association with hypertension, short stature, constipation, and chronic headache.
Patients who have NF-2 are not at increased risk for such complications.
The child described in the vignette is at risk for the development of hypertension, particularly during
ages 2 through 10 years. Although most patients have idiopathic hypertension, NF-1 can be associated
with renal artery stenosis and pheochromocytoma. This child's health supervision visits should include a
detailed history and accurate blood pressure measurements to ensure early identification of this
complication. The child is not at an increased risk for the development of cataracts, and there is no
reason to restrict him from contact sports if he has no other complications. Because approximately 15%
of individuals who have NF-1 develop optic pathway tumors, periodic ophthalmologic evaluation is
important. However, the potential high cumulative radiation dose of annual computed tomography
scanning likely outweighs the benefits; the necessity and frequency of such imaging should be
determined in consultation with the ophthalmologist. Acoustic neuroma, now referred to as vestibular
schwannoma, is seen in NF-2, not NF-1.

References:

Hersh JH, Committee on Genetics. Health supervision for children with neurofibromatosis. Pediatrics.
2008;121:633-642. Available at: http://pediatrics.aappublications.org/cgi/content/full/121/3/633

Lama G, Graziano L, Calabrese E, et al. Blood pressure and cardiovascular involvement in children
with neurofibromatosis type 1. Pediatr Nephrol. 2004;19:413-418. Abstract available at:
http://www.ncbi.nlm.nih.gov/pubmed/14991390

Viskochil DH. Neurocutaneous disorders. In: Rudolph C, Rudolph A, eds. Rudolph's Pediatrics. 21st ed.
New York, NY: McGraw Hill Medical Publishing Division; 2003:769-774

page 630
2009 PREP SA on CD-ROM

Critique: 181

Café au lait macules and neurofibromas (arrow) are observed in


neurofibromatosis type 1. (Courtesy of the Media Lab at Doernbecher)

page 631
2009 PREP SA on CD-ROM

Question: 182
During the routine health supervision visit for a 4-month-old infant, you note low tone and poor visual
interaction. His head shape is symmetric and his head circumference is 36 cm (<2nd percentile). He
had been born at term with a head circumference of 32 cm (2nd percentile).

Of the following, the MOST helpful initial diagnostic test to explain the cause of the infant’s abnormal
examination findings is

A. brain magnetic resonance imaging

B. electroencephalography

C. head ultrasonography

D. high-resolution karyotyping

E. three-dimensional head computed tomography scan

page 632
2009 PREP SA on CD-ROM

Critique: 182 Preferred Response: A


The child described in the vignette had microcephaly at birth and has continued to develop below the
standard head growth curve, increasing by just 4 cm, instead of 7 cm, during the first 4 months. Such
slow growth most likely is due to a primary brain problem rather than any skull anomaly (Item C182).
Microcephaly in the presence of abnormal neurodevelopment, as described for this infant, indicates a
need for a thorough evaluation to determine the neurologic cause of symptoms. Magnetic resonance
imaging (MRI) allows detailed assessment of all brain structures, including the brainstem and gray and
white matter integrity, features that can guide additional evaluation.
Head circumference should be measured and plotted at birth and at all subsequent health
supervision visits, particularly for the first 3 postnatal years, because head growth normally corresponds
with rapid brain growth in early childhood. Suboptimal head growth during the first year and
microcephaly at 1 year of age are associated with subnormal developmental outcomes in children who
had hypoxic-ischemic injuries at term birth.
Head ultrasonography is a reasonable alternative to MRI because it does not require sedation and,
therefore, is easier on the child (and family). However, it has lower resolution for peripheral gray matter
and deep brainstem structures. This is a better choice for evaluation of a large head (macrocephaly)
because it can diagnose hydrocephalus accurately. Electroencephalography findings are too
nonspecific to guide diagnosis or treatment.
High-resolution karyotyping may be reasonable, particularly if the infant displays other dysmorphic
features or congenital brain structural anomalies, because chromosomal abnormalities can result in
microcephaly. However, this evaluation can be deferred until after additional information about brain
structure has been obtained. For example, if there is evidence of a stroke, hypoxic-ischemic injury, or a
congenital infection, karyotyping may not be needed.
Three-dimensional computed tomography scan is used to diagnose craniosynostosis.
Craniosynostosis usually presents with an unusual head shape due to closure of one or more skull
sutures. This infant's symmetric microcephaly plus an abnormal neurodevelopmental history make a
bony abnormality in the skull unlikely.

References:

Kinsman SL, Johnston MV. Congenital anomalies of the central nervous system. In: Kliegman RM,
Behrman RE, Jenson HB, Stanton BF, eds. Nelson Textbook of Pediatrics. 18th ed. Philadelphia, Pa:
Saunders Elsevier; 2007:2443-2456

Shevell M, Ashwal S, Donley D, et al. Practice parameter: evaluation of the child with global
developmental delay. Report of the Quality Standards Subcommittee of the American Academy of
Neurology and The Practice Committee of the Child Neurology Society. Neurology. 2003;60:367-380.
Available at: http://www.neurology.org/cgi/content/full/60/3/367

Shevell M, Majnemer A, Platt RW, Webster R, Birnbaum R. Developmental and functional outcomes in
children with global developmental delay or developmental language impairment. Dev Med Child
Neurol. 2005;47:678-683. Available at:
http://www.blackwell-synergy.com/doi/abs/10.1111/j.1469-8749.2005.tb01053.x

page 633
2009 PREP SA on CD-ROM

Critique: 182

page 634
2009 PREP SA on CD-ROM

Question: 183
A 2-day-old infant is transferred from the regular nursery to the neonatal intensive care unit for
evaluation and management of poor feeding and lethargy. A serum metabolic panel reveals a carbon
dioxide concentration of 12.0 mEq/L (12.0 mmol/L) and a borderline low white blood cell count. After
stopping all feedings, a septic evaluation is performed, and intravenous antibiotics are started. During
this time, she becomes alert and vigorous, and her carbon dioxide value normalizes. Three days later,
results of the septic evaluation are negative, and the infant resumes human milk feedings. Initially she
does well, but after 2 days, she begins to vomit and becomes less active. Serum metabolic panel
shows a glucose concentration of 35.0 mg/dL (1.9 mmol/L), a carbon dioxide concentration of 8.0
mEq/L (8.0 mmol/L), and an anion gap of 25; the serum ammonia value is twice the upper limit of
normal. The baby is again made NPO and given intravenous fluids.

Of the following, the MOST critical diagnostic test for this baby is

A. lysosomal enzyme analysis

B. peroxisomal enzyme analysis

C. serum galactose-1-phosphate measurement

D. serum toxicology screen

E. urine organic acids measurement

page 635
2009 PREP SA on CD-ROM

Critique: 183 Preferred Response: E


The finding of metabolic acidosis with an increased anion gap in an infant or child who is acutely ill,
such as the infant described in the vignette, is an important clue to the possible diagnosis of an inborn
error of metabolism. Even so, it is important to remember that any condition that produces a metabolic
acidosis, such as infection and necrosis, can present similarly. Of the inborn metabolic errors, those
characteristically associated with overwhelming metabolic acidosis in the young infant are the organic
acid disorders, especially propionic, methylmalonic, and isovaleric acidemias. The infant in the vignette
has propionic acidemia.
Evaluation of the infant or child who has acidosis and an elevated anion gap should include
measurement of plasma lactate, plasma pyruvate, complete blood and platelet counts, serum ammonia,
urine organic and amino acids, plasma amino acids, and total and free carnitine as well as an
acylcarnitine profile. Metabolic acidosis can interfere with normal urea cycle function, leading to
elevated serum ammonia concentrations; it also can depress bone marrow production.
Lysosomal storage disorders, such as Hurler and Hunter syndromes, typically do not present in the
newborn period with metabolic imbalance; they become apparent in toddlers who exhibit decreased
acquisition of skills and progressive deformation.
Peroxisomal enzyme disorders that present in the newborn period, such as Zellweger syndrome, are
associated with altered sensorium, hypotonia, and hepatomegaly and are not responsive to the
withholding of feedings.
Serum galactose-1-phosphate concentrations are elevated in the most common form of
galactosemia. Galactosemia presents after days to weeks of human or cow milk feedings and is
associated with jaundice, hepatomegaly, and Escherichia coli sepsis.
A serum toxicology screen in this neonate would not be expected to reveal the source of her illness,
and the infant's presentation is not characteristic of maternal substance abuse.

References:

Burton BK. Inborn errors of metabolism in infancy: a guide to diagnosis. Pediatrics. 1998;102:e69-e78.
Available at: http://pediatrics.aappublications.org/cgi/content/full/102/6/e69

Enns GM. Inborn errors of metabolism masquerading as hypoxic-ischemic encephalopathy.


NeoReviews. 2005;6:e549-e558. Available for subscription at:
http://neoreviews.aappublications.org/cgi/content/full/6/12/e549

Hoffmann GF, Nyhan WL, Zschocke J, Kahler SG, Mayatepek E. Approach to the patient with metabolic
disease. In: Inherited Metabolic Diseases. Philadelphia, Pa: Lippincott Williams & Wilkins; 2002:19-94

Nyhan WL, Barshop BA, Ozand PT. Organic acidemias. In: Atlas of Metabolic Diseases. 2nd ed.
London, England: Hodder Arnold; 2005:1-108

page 636
2009 PREP SA on CD-ROM

Question: 184
A 17-year-old young man comes to the clinic in the juvenile detention center with a penile discharge. He
has no other symptoms. He was tested 1 week ago at a sexually transmitted infections clinic, and
results of the rapid urine testing by nucleic acid amplification are positive for Neisseria gonorrhoeae and
negative for Chlamydia trachomatis.

Of the following, the MOST appropriate treatment regimen is

A. benzathine penicillin G 2.4 million units intramuscularly in a single dose

B. ceftriaxone 125 mg intramuscularly in a single dose

C. ciprofloxacin 500 mg orally in a single dose

D. ofloxacin 400 mg orally in a single dose

E. spectinomycin 2 g intramuscularly in a single dose

page 637
2009 PREP SA on CD-ROM

Critique: 184 Preferred Response: B


Gonorrhea is the second most commonly reported notifiable disease in the United States. Although the
Centers for Disease Control and Prevention (CDC) Sexually Transmitted Diseases Treatment
Guidelines, 2006, recommended treating uncomplicated gonococcal infections of the cervix, urethra,
and rectum with either ceftriaxone, cefixime, ciprofloxacin, ofloxacin, or levofloxacin as single-dose
therapy, in April 2007, the CDC issued an update to the guidelines, indicating that fluoroquinolones no
longer are recommended for the treatment of gonococcal infections. Current recommendations for
uncomplicated gonococcal infections of the cervix, urethra, and rectum include ceftriaxone 125 mg
intramuscularly or cefixime 400 mg orally, both in a single dose, plus treatment for chlamydia if
chlamydial infection has not been ruled out. These recommendations apply to all adult and adolescent
patients, regardless of travel history or sexual behavior.
Alternative regimens include spectinomycin 2 g in a single intramuscular dose or parenteral
cephalosporin single-dose regimens, including ceftizoxime 500 mg intramuscularly, cefoxitin 2 g
intramuscularly with probenecid 1 g orally, or cefotaxime 500 mg intramuscularly. Spectinomycin is
recommended for persons who have penicillin or cephalosporin allergies. Benzathine penicillin is not
recommended due to increased resistance to this drug.

References:

Centers for Disease Control and Prevention. Update to CDC's sexually transmitted diseases treatment
guidelines, 2006: fluoroquinolones no longer recommended for treatment of gonococcal infections.
MMWR Morbid Mortal Wkly Rep. 2007;56:332-336. Available at:
http://www.cdc.gov/mmwr/preview/mmwrhtml/mm5614a3.htm

Workowski KA, Berman SM, Centers for Disease Control and Prevention. Sexually transmitted
diseases treatment guidelines, 2006. MMWR Recomm Rep. 2006;55(RR11):1-94. Available at:
http://www.cdc.gov/mmwr/preview/mmwrhtml/rr5511a1.htm

page 638
2009 PREP SA on CD-ROM

Question: 185
A 3-year-old boy is admitted to the hospital for fever, cough, and increasing respiratory insufficiency of 2
days’ duration. Chest radiography demonstrates a right middle lobe and lower lobe pneumonia with a
significant pleural effusion (Item Q185). You aspirate a sample of pleural fluid and send it to the
laboratory for analysis.

Of the following, the MOST likely expected laboratory findings are

A. White Blood Cells: Moderate to high

Lactate Dehydrogenase (U/L): 2 to 3 times the serum concentration


Protein (g/dL): <3
Gram Stain: Gram-positive cocci
B. White Blood Cells: Moderate to high

Lactate Dehydrogenase (U/L): Equal to the serum concentration


Protein (g/dL): <3
Gram Stain: Negative

C. White Blood Cells: Moderate to high

Lactate Dehydrogenase (U/L): 2 to 3 times the serum concentration


Protein (g/dL): >3
Gram Stain: Gram-positive cocci

D. White Blood Cells: Rare

Lactate Dehydrogenase (U/L): Equal to the serum concentration


Protein (g/dL): >3
Gram Stain: Negative

E. White Blood Cells: Rare

Lactate Dehydrogenase (U/L): Equal to the serum concentration


Protein (g/dL): <3
Gram Stain: Gram-positive cocci

page 639
2009 PREP SA on CD-ROM

Question: 185

(Courtesy of B. Poss)

page 640
2009 PREP SA on CD-ROM

Critique: 185 Preferred Response: C


The child described in the vignette has a pleural effusion and lobar pneumonia. Pleural effusions in
pediatric patients have a variety of causes, including pneumonia, trauma, malignancy, renal disease,
liver failure, and congestive heart disease. Parapneumonic effusions (those associated with an
underlying pneumonia) account for up to 70% of pediatric pleural effusions. Pleural effusions may be
seen in up to 40% of bacterial pneumonias; progression to pus (empyema) occurs in more than 50% of
these cases and is increasing in both incidence and associated complications.
Classification of pleural fluid into transudates and exudates may assist the clinician in determining
causes and guiding treatment (Item C185). Transudates are plasma ultrafiltrates and usually result from
renal and liver disease or congestive heart failure. Exudates arise from inflammatory processes (such
as pneumonia, malignancies, trauma, or systemic inflammatory diseases) or impaired lymphatic
drainage, often following thoracic surgery (eg, chylothorax). Empyema, a type of exudate, is
characterized by the presence of white blood cells, a positive Gram stain, or frank pus.
The child described in the vignette most likely has an exudate based on the lactate dehydrogenase
and protein values. A Gram stain may reveal bacteria (in this case, a gram-positive cocci), although
bacterial culture results may be negative, especially if the patient has received prior antibiotic therapy.
The pH value of the fluid should be low due to consumption of glucose by the bacteria, and there should
be moderate-to-high white blood cell counts, consistent with an empyema. Lactate dehydrogenase
values often are two to three times the serum concentrations, and protein values frequently are 3.0 g/dL
(30.0 g/L) or greater in pleural effusions. Tube thoracostomy, potentially aided by video-assisted
thoracic surgery, and antibiotics are the recommended treatment.

References:

Efrati O, Barak A. Pleural effusions in the pediatric population. Pediatr Rev. 2002:23:417-426.
Available at: http://pedsinreview.aappublications.org/cgi/content/full/23/12/417

Schultz KD, Fan LL, Pinksy J, et al. The changing face of pleural empyemas in children: epidemiology
and management. Pediatrics. 2004;113:1735-1740. Available at:
http://pediatrics.aappublications.org/cgi/content/full/113/6/1735

Winnie GB. Pleurisy, pleural effusions, and empyema. In: Kliegman RM, Behrman RE, Jenson HB,
Stanton BF, eds. Nelson Textbook of Pediatrics. 18th ed. Philadelphia, Pa: Saunders Elsevier;
2007:1832-1834

page 641
2009 PREP SA on CD-ROM

Question: 186
You are seeing a short 9-year-old boy. He is growing steadily in height just below the third percentile on
the growth curve. His parents ask if he will be very short when he finishes growing.

Of the following, the MOST important information needed to answer this question is

A. ethnicity of family

B. parent heights

C. sibling heights

D. upper-to-lower segment ratio

E. weight-for-age curve

page 642
2009 PREP SA on CD-ROM

Critique: 186 Preferred Response: B


Short stature is a statistical diagnosis representing a stature less than the third percentile for height (3%
of all children). Most children growing along lower percentiles, such as the boy described in the
vignette, are not growing slowly by the time they reach middle childhood; they dropped to this percentile
some time in the first 2 years after birth and now have a normal growth rate. Most either have short
parents or will advance through puberty slowly (constitutional delay) and end up taller than anticipated
by their childhood position on the growth curve. Accordingly, parent heights are the most pertinent facts
needed to answer the question about this child's adult height.
Family ethnicity is only generally useful in evaluating short stature; within-family heights are much
more important for the individual child. Sibling height, unless the siblings have reached full adult height,
are not as useful as adult parental height because the sibling adult heights still are unknown. Children
who have particularly short legs or short spines may have an underlying chondrodystrophy or skeletal
dysplasia. Measurement of upper-to-lower segment ratio can be a useful clinical examination when
considering these conditions, although they are very rare. Children who are particularly underweight for
height or very overweight for height may grow poorly. However, most overweight children grow
exceedingly well because of their overnourished status. A very malnourished child may have a
decreased growth rate. The steady growth in the child in the vignette suggests that nutritional status is
not the most important determinant of his present height.
Children who have normal growth rates are unlikely to have underlying endocrine or metabolic
disorders or serious underlying illnesses to explain their stature. In contrast, children who have growth
attenuation are much more likely to have underlying and perhaps specifically remediable causes for
short stature.

References:

Ferry RJ Jr. Short stature. eMedicine Specialties, Pediatrics: General Medicine, Endocrinology. 2007.
Available at: www.emedicine.com/ped/topic2087.htm

Plotnick LP, Miller RS. Growth, growth hormone, and pituitary disorders. In: McMillan JA, Feigin RD,
DeAngelis C, Jones MD Jr. Oski's Pediatrics, Principles & Practice. Philadelphia, Pa: Lippincott,
Williams & Wilkins; 2006:2084-2092

Rogol AD. Causes of short stature. UpToDate Online 15.3. 2008. Available for subscription at:
http://www.uptodateonline.com/utd/content/topic.do?topicKey=pediendo/2279

page 643
2009 PREP SA on CD-ROM

Question: 187
A 4-year-old boy cannot attend a local nursery school because he is not toilet trained. His development
is otherwise normal. His parents explain that when they attempt to put him on the toilet, he refuses and
runs out of the bathroom. They ask how they can train him to use the toilet.

Of the following, the MOST appropriate approach is to

A. develop a behavioral modification program to encourage him to use the toilet

B. have the parents gently scold him when he has accidents

C. insist he sit on the toilet every 2 hours during the day

D. recommend the family find a different nursery school that allows children who are not toilet trained

E. tell the parents to have him clean his own clothes after toilet accidents

page 644
2009 PREP SA on CD-ROM

Critique: 187 Preferred Response: A


In the United States, 98% of children are continent during the day by the time they are 36 months old.
Toilet training usually requires about 3 to 6 months for successful completion. For children such as the
boy described in the vignette, who appears to be late in achieving toilet training, a behavioral
modification program can be established to encourage use of the toilet. It is best to have the child take
responsibility for being toilet trained, and a reward system using a star chart to earn a desired object
may help him to meet this goal. Punishing him for not using the toilet or insisting that he clean his own
clothes could lead to noncompliance and adversely affect his self-esteem. Insisting a child sit on the
potty chair or forcing him or her to sit may increase the child's resistance. Often, a parent pushing a
child to become toilet trained due to a preschool requirement leads to a power struggle, and the child
does not achieve continence. Recommending that the parents find a different nursery school that does
not require a child to be toilet trained may alleviate some of their stress, but it does not address how to
toilet train a child.
Toilet training begins with the parent encouraging the child to practice running to the potty chair. The
parent should praise or reward the child for complying with the practice session. The child should be
changed after any accidents, and the parents should avoid use of physical or verbal punishment. The
use of underwear, time-in (provide the child with positive reinforcement), and incentives may help to
increase a child's motivation.

References:

Parker S. Toilet training. In: Parker S, Zuckerman B, Augustyn M, eds. Developmental and Behavioral
Pediatrics: A Handbook for Primary Care. 2nd ed. Philadelphia, Pa: Lippincott Williams & Wilkins;
2005:355-357

Schmitt B. Toilet training: getting it right the first time. Contemp Pediatr. 2004;21:105

Wolraich ML, Tippins S, ed. Guide to Toilet Training. Elk Grove Village, Ill: American Academy of
Pediatrics; 2003

page 645
2009 PREP SA on CD-ROM

Question: 188
The nurse caring for a 5-day-old infant you have hospitalized calls your office to report that the infant’s
blood culture is growing gram-positive rods. You admitted the infant to the hospital because of a rectal
temperature of 102.0°F (38.9°C) measured by his mother at home.

Of the following, the MOST likely pathogen is

A. Enterococcus sp

B. Escherichia coli

C. Listeria monocytogenes

D. Proteus mirabilis

E. Staphylococcus epidermidis

page 646
2009 PREP SA on CD-ROM

Critique: 188 Preferred Response: C


Listeria monocytogenes is a small, motile, gram-positive rod. Three major serotypes of Listeria infect
humans: 1a, 1b, and 4b. Infections in the newborn result from asymptomatic fecal or vaginal carriage in
pregnant women, with the organism colonizing the baby during passage through the birth canal.
Although bacterial infections due to group B Streptococcus and gram-negative organisms such as
Escherichia coli or Klebsiella pneumoniae are more common, neonatal sepsis and meningitis due to
Listeria represent a major risk to the neonate, especially in certain geographic regions of the United
States (eg, the southwestern area).
Illness with Listeria can present as early-onset disease (from birth to 7 days) or late-onset disease
(>7 days after birth). The typical characteristics of early-onset infections are bacteremia or pneumonia
and preterm birth; late-onset disease usually results in meningitis. For older infants and children,
listerial infection is associated with ingestion of contaminated foods; unpasteurized milk, soft cheese,
and prepared meats (eg, hot dogs, deli meats) are the sources of many infections. Most of these
infections present with fever, malaise, headache, gastrointestinal tract symptoms, or back pain.
Bacteremia and meningitis also can occur. Proteus mirabilis and E coli are gram-negative rods, and
Enterococcus sp and Staphylococcus epidermidis are gram-positive cocci.

References:

American Academy of Pediatrics. Listeria monocytogenes infections (listeriosis). In: Pickering LK, Baker
CJ, Long SS, McMillan JA, eds. Red Book: 2006 Report of the Committee on Infectious Diseases. 27th
ed. Elk Grove Village, Ill: American Academy of Pediatrics; 2006:426-428

Posfay-Barbe KM, Wald ER. Listeriosis. Pediatr Rev. 2004;25:151-159. Available at:
http://pedsinreview.aappublications.org/cgi/content/full/25/5/151

page 647
2009 PREP SA on CD-ROM

Question: 189
A mother calls you to report that her 7-year-old son came home with a notice from school stating that a
child in his class was diagnosed with mumps. The mother does not know the immunization status of the
infected child but states that her son has received two measles-mumps-rubella (MMR) vaccines and is
up to date on all his other immunizations. Her son has been asymptomatic, with no fever or other
systemic complaints.

Of the following, the MOST appropriate action is to

A. administer a dose of mumps immune globulin to her son

B. confirm that her son has received two doses of MMR vaccine

C. keep her son home from school for 9 days to observe for the development of symptoms

D. treat her son with a course of ribavirin

E. vaccinate her son immediately with another dose of MMR to prevent infection from this exposure

page 648
2009 PREP SA on CD-ROM

Critique: 189 Preferred Response: B


In the United States, mumps vaccine is administered as part of the measles-mumps-rubella (MMR)
vaccine routinely to children at 12 to 15 months of age, with a second dose of MMR typically
administered at 4 to 6 years of age. Protective efficacy of the vaccine is estimated to be more than
95%. In cases of exposure, such as described in the vignette, it is important to ensure that the exposed
person has received the recommended number of doses of MMR vaccine because mumps outbreaks
have occurred in people in highly immunized populations who previously have received only a single
dose of mumps-containing vaccine. Therefore, the most appropriate action is to confirm with the mother
that her son has received two doses of MMR vaccine.
Mumps vaccine has not been demonstrated to be effective in preventing infection after exposure.
However, the vaccine can be administered after exposure to provide protection against subsequent
exposures in persons who are not fully vaccinated. Fully immunized persons do not need to be
excluded from school after exposure. Students who are not fully immunized are excluded from school
until they are immunized, after which they can be readmitted immediately to school. Students who
refuse mumps vaccination because of medical, religious, or other reasons should be excluded from
school for at least 26 days after the onset of parotitis in the last person who has mumps in the affected
school. Persons who have mumps are excluded from school for 9 days from the onset of their parotid
swelling.
Immune globulin (IG) and mumps IG are not effective as postexposure prophylaxis measures. In
fact, mumps IG no longer is available in the United States. Treatment of the disease is supportive; no
effective antiviral agents are available.

References:

American Academy of Pediatrics. Mumps. In: Pickering LK, Baker CJ, Long SS, McMillan JA, eds. Red
Book: 2006 Report of the Committee on Infectious Diseases. 27th ed. Elk Grove Village, Ill: American
Academy of Pediatrics; 2006:464-468

Centers for Disease Control and Prevention (CDC). Notice to readers: updated recommendations of the
Advisory Committee on Immunization Practices (ACIP) for the control and elimination of mumps.
MMWR Morb Mortal Wkly Rep. 2006;55:629-630. Available at:
http://www.cdc.gov/mmwr/preview/mmwrhtml/mm55e601a1.htm

page 649
2009 PREP SA on CD-ROM

Question: 190
A 4-year-old boy presents with periorbital edema. He is receiving no medications, and his family history
is negative for renal disease. On physical examination, he is afebrile; his heart rate is 88 beats/min,
respiratory rate is 18 breaths/min, and blood pressure is 106/62 mm Hg; and he has periorbital (Item
Q190A) and pitting pretibial edema (Item Q190B). Laboratory evaluation shows normal electrolyte
values, blood urea nitrogen of 14.0 mg/dL (5.0 mmol/L), creatinine of 0.3 mg/dL (26.5 mcmol/L), and
albumin of 1.6 g/dL (16.0 g/L). Urinalysis demonstrates a specific gravity of 1.020; pH of 6.5; 3+ protein;
and negative blood, leukocyte esterase, and nitrite. Microscopy results are normal. Additionally,
complement component (C3 and C4) values are normal, and results of serologic testing for antinuclear
antibody, hepatitis B and C, and human immunodeficiency virus are negative.

Of the following, you are MOST likely to advise the parents that

A. a renal biopsy is warranted to determine the optimal treatment

B. disease relapse can be expected in fewer than 25% of those achieving remission

C. patients who relapse have a similar prognosis as those who do not respond to steroids

D. remission is expected in more than 75% of patients who receive corticosteroid treatment

E. tacrolimus is the preferred treatment for patients who do not respond to corticosteroids

page 650
2009 PREP SA on CD-ROM

Question: 190

Periorbital edema, as exhibited by the boy in the vignette. (Courtesy of M.


Rimsza)

page 651
2009 PREP SA on CD-ROM

Question: 190

Pretibial pitting edema (arrow), as exhibited by the boy in the vignette.


(Reprinted with permission from Cavanaurgh RM Jr. Orthostatic edema in
adolescents: more than walking on water. Pediatr Rev. 2005;26:115-124.)

page 652
2009 PREP SA on CD-ROM

Critique: 190 Preferred Response: D


The boy described in the vignette has signs and symptoms of nephrotic syndrome. Causes of nephrotic
syndrome can be subdivided into primary glomerulopathies (eg, minimal change disease, focal
segmental glomerulosclerosis, mesangial proliferative glomerulonephritis) and secondary
glomerulopathies (eg, lupus nephritis, membranoproliferative glomerulonephritis due to hepatitis C, and
human immunodeficiency virus nephropathy).
The initial evaluation of new-onset nephrotic syndrome centers on establishing whether it is due to a
primary glomerulopathy and warrants an empiric trial of corticosteroids. Clinical features that suggest
secondary glomerulopathy include moderate-to-severe hypertension or gross hematuria. Laboratory
markers suggestive of a secondary glomerulopathy include azotemia, hypocomplementemia, a positive
antinuclear antibody finding, or a positive serologic test for human immunodeficiency virus or hepatitis B
or C.
The normal blood pressure and renal function as well as the absence of hematuria,
hypocomplementemia, or a positive serologic test for lupus or viral infection described for the boy in the
vignette are consistent with primary glomerulopathy. Accordingly, this child should receive a 4- to
6-week trial of daily corticosteroids because nearly 90% of childhood nephrotic syndrome cases are
sensitive to steroids. Although 90% of pediatric patients achieve a state of remission with
corticosteroids, 50% to 70% experience at least one relapse during their disease course.
Unless clinical or serologic findings raise the possibility of an underlying glomerulonephritis, renal
biopsy is not warranted. The steroid response, not renal biopsy findings, is the best predictor of the
prognosis for childhood nephrotic syndrome. Steroid-sensitive patients, even those who have a
relapsing course, have a better prognosis than steroid-resistant patients. The approximately 10% of
patients who are steroid-resistant have a guarded prognosis, with a risk of developing chronic renal
failure. Steroid-resistant patients require a renal biopsy to establish histologic information.
In most circumstances, the first-line medication for steroid-resistant nephrotic syndrome is
cyclosporine. More recently, another calcineurin inhibitor, tacrolimus, has been used in children who fail
to respond to cyclosporine. At present, tacrolimus is considered a second- or third-line treatment for
steroid-resistant nephrotic syndrome.

References:

Niaudet P. Steroid-sensitive idiopathic nephrotic syndrome in children. In: Avner ED, Harmon WE,
Niaudet P, eds. Pediatric Nephrology. 5th ed. Philadelphia, Pa: Lippincott Williams & Wilkins;
2004:543-556

Niaudet P. Steroid-resistant idiopathic nephrotic syndrome in children. In: Avner ED, Harmon WE,
Niaudet P, eds. Pediatric Nephrology. 5th ed. Philadelphia, Pa: Lippincott Williams & Wilkins;
2004:557-573

Valentini RP, Smoyer WE. Nephrotic syndrome. In: Kher KK, Schnaper HW, Makker SP, eds. Clinical
Pediatric Nephrology. 2nd ed. London, England: Informa Healthcare; 2007:155-194

page 653
2009 PREP SA on CD-ROM

Question: 191
You are evaluating a 14-year-old girl for seasonal allergic rhinitis. Despite a regimen of multiple allergy
medications, she continues to have significant sneezing, rhinorrhea, and nasal congestion. You decide
to evaluate for possible allergic triggers and discuss the advantages and disadvantages of allergy skin
testing and blood testing.

Of the following, a TRUE statement regarding allergy skin and blood testing is that

A. infants younger than 1 year of age cannot undergo skin testing

B. patients may experience anaphylaxis during aeroallergen or food skin testing

C. patients need to fast prior to blood allergy testing

D. patients need to stop their antihistamines prior to blood allergy testing

E. the negative predictive value of aeroallergen skin testing is poor

page 654
2009 PREP SA on CD-ROM

Critique: 191 Preferred Response: B


Two primary diagnostic tools are used to determine the role of indoor and outdoor aeroallergens as
triggers for allergic rhinitis or allergic asthma: skin testing and blood testing. Aeroallergen skin testing
involves the application of specific allergens (eg, oak, Bermuda grass, cat, ragweed) on the skin,
typically using a prick or puncture method. Although sometimes uncomfortable for infants and toddlers,
allergy skin testing is tolerated extremely well by most children and adolescents and can be performed
at any age. The advantages of skin testing are that a broad array of allergens can be tested, testing
materials are inexpensive, and results are immediately evident to the patient. One disadvantage is that
patients must stop their antihistamine medication(s) 1 week prior to skin testing. Also, although most
patients tolerate the local pruritus experienced at "positive" skin test sites, those who are very sensitive
(eg, severe food anaphylaxis) may experience a systemic reaction with even a simple skin test. For
patients who have a history of severe anaphylaxis to a specific allergen, allergists may choose to
perform serum immunoglobulin (Ig) E testing instead of skin testing because blood testing does not
have a risk for anaphylaxis.
In the past, serum IgE testing employed primarily the radioallergosorbent test (RAST) method.
Because of the significant variability in results between laboratories, RAST has been replaced in most
institutions with the more sensitive and reproducible CAP-system fluorescein enzyme immunoassay.
This system uses a cellulose matrix system. The advantage of serum IgE testing is that it is not affected
by medications (ie, patients do not need to stop an antihistamine). Patients do not need to fast prior to
either allergy skin or blood testing.
While ongoing studies are comparing the sensitivity and specificity of skin testing compared with the
CAP system fluorescein enzyme immunoassay, skin testing is regarded as more sensitive and specific.
Finally, although skin testing is considered "inexpensive," most general pediatricians find the cost of an
allergy consultation with skin testing to be more expensive than a routine battery of serum IgE tests for
aeroallergens or food. The availability and clinical application of serum IgE testing continues to expand,
but clinicians who do not seek allergy consultation should be comfortable with interpretation and
application of test results for a specific clinical scenario (eg, a wheat IgE of 10 kU/L in a patient who has
atopic dermatitis has little to no clinical significance).

References:

Cartwright RC, Dolen WK. Consultation with the specialist: who needs allergy testing and how to get it
done. Pediatr Rev. 2006;27:140-146. Available at:
http://pedsinreview.aappublications.org/cgi/content/full/27/4/140

Mahr TA, Sheth K. Update on allergic rhinitis. Pediatr Rev. 2005;26:284-289. Available at:
http://pedsinreview.aappublications.org/cgi/content/full/26/8/284

page 655
2009 PREP SA on CD-ROM

Question: 192
A 3-year-old boy is brought to the emergency department at 8 am after his parents found him
unresponsive in bed. The last time they had seen him awake was at 2 am, when they found him playing
in the living room as they were cleaning up after a cocktail party. On physical examination, the child has
diaphoresis and moans to painful stimuli. His vital signs include a temperature of 96.4°F (35.8°C), heart
rate of 145 beats/min, respiratory rate of 20 breaths/min, blood pressure of 84/34 mm Hg, and oxygen
saturation of 97% in room air. His pupils are mid-sized and sluggishly reactive.

Of the following, the MOST important test to obtain at this time is

A. acetylcholinesterase determination

B. bedside glucose determination

C. blood alcohol determination

D. serum osmolality

E. urine toxicology screen

page 656
2009 PREP SA on CD-ROM

Critique: 192 Preferred Response: B


The approach to any pediatric patient who presents with acute alteration in mental status should include
assessment of airway patency, adequacy of oxygenation, ventilation, and perfusion as well as a
bedside glucose determination. Hypoglycemia may result from a variety of causes, including inborn
errors of metabolism, poisonings, sepsis, shock, liver failure, and starvation. Signs and symptoms of
hypoglycemia are related to counterregulatory epinephrine release and cerebral glucopenia. These
include diaphoresis, tachycardia, tremulousness, pallor, irritability, somnolence, and coma.
Unrecognized hypoglycemia can cause mortality as well as significant morbidity from seizure activity
and if chronic, impaired brain development. Because hypoglycemia is treated easily with intravenous
dextrose, oral glucose, or in some settings, glucagon, rapid diagnosis is critical.
A number of poisonings are well known causes of hypoglycemia in children and include oral
hypoglycemic agents, insulin, beta-blockers, salicylates, and ethanol. Ethanol intoxication with
hypoglycemia is a likely cause of the symptoms described for the patient in the vignette because of the
parents' report of unsupervised activity in an environment where ethanol was accessible. Other clues to
ethanol ingestion in children who have hypoglycemia include an anion gap acidosis, an osmolar gap,
and an elevated lactate value. The hypoglycemia caused by ethanol is related to gluconeogenesis
inhibition and is not responsive to glucagon.
Although acetylcholinesterase and blood alcohol determinations, measurement of serum osmolality,
and a urine toxicology screen may be indicated in the evaluation of a patient who has acute alteration of
mental status, the bedside glucose determination is the most appropriate initial test because it permits
rapid identification of hypoglycemia and prompt treatment.

References:

Ernst AA, Jones K, Nick TG, Sanchez J. Ethanol ingestion and related hypoglycemia in a pediatric and
adolescent emergency department population. Acad Emerg Med. 1996;3:46-49. Abstract available at:
http://www.ncbi.nlm.nih.gov/pubmed/8749967

Sperling M. Hypoglycemia. In: Kleigman RM, Behrman RE, Jenson HB, Stanton BF, eds. Nelson
Textbook of Pediatrics. 18th ed. Philadelphia, Pa: Saunders Elsevier; 2007:655-670

Sunehag A, Haymond MW. Etiology of hypoglycemia in infants and children. UpToDate Online 15.3.
2008. Available for supscription at:
http://www.utdol.com/utd/content/topic.do?topicKey=pediendo/11162&selectedTitle=4~29&source=sear
ch_result

page 657
2009 PREP SA on CD-ROM

Question: 193
A 3-year-old child presents to your office with chronic recurrent diarrhea of 3 months’ duration. He
attends child care during the week. He is one of four children in the family, the oldest of whom is 8
years old. Stool microscopic analysis identifies Giardia lamblia. You treat the boy with metronidazole for
10 days. On a follow-up visit 30 days after initiating treatment, the mother states that the symptoms
initially improved, but have recurred.

Of the following, the BEST explanation for the persistent symptoms is

A. Clostridium difficile infection

B. metronidazole resistance

C. persistent giardiasis

D. superimposed milk protein allergy

E. undiagnosed celiac disease

page 658
2009 PREP SA on CD-ROM

Critique: 193 Preferred Response: C


Based on the documented history of giardiasis and ongoing diarrhea for the patient described in the
vignette, the most likely cause of his symptoms is persistent giardiasis, most likely from reinfection from
one of his siblings or a family pet. Many Giardia infections are asymptomatic, and siblings, other
children in child care, or family pets frequently can be reservoirs for the organism. Therefore, a patient
treated with a course of metronidazole is susceptible to reinfection if he or she is exposed to
asymptomatic carriers. Metronidazole resistance is extremely rare in Giardia parasites, and based on
the history provided, there is no reason to suspect Clostridium difficile infection, celiac disease, or food
allergy at this time.
Giardia is a flagellated enteric-coated protozoan that is the most common cause of intestinal
parasitic infections in the United States. The parasite exists in two forms, cysts and trophozoites. Cysts
(Item C193A) are extremely hardy and can survive in cold water for weeks to months. The cysts also
are resistant to chlorination. Once animals (eg, dogs, cats, beavers) or humans ingest contaminated
water, the cysts mature into trophozoites (Item C193B) in the intestine, where they cause small
intestinal inflammation, villous atrophy, and malabsorption. Asymptomatic carriage is more common
than symptomatic disease. However, in patients who do develop symptoms, the most common features
are diarrhea, nausea, distention, cramping, and anorexia. Because Giardia is a small bowel pathogen, it
does not cause rectal bleeding.
The most important aspect to diagnosing Giardia infection is having a strong clinical suspicion. A
number of good diagnostic tests of the stool are available, including microscopic examination for ova
and parasites (stool O and P), enzyme-linked immunofluorescence assays, and direct fluorescence
antibody tests. The microscopic examination ("O and P" test) has poorer sensitivity (50%) from a single
sample and ideally requires microscopic examination of three separate samples. In contrast, the
Giardia antigen-based tests have a sensitivity of approximately 90% from a single stool sample.
However, the O and P examination can identify other potential pathogens (Blastocystis, ameba) that are
not identified with the antigen tests. In some cases, endoscopy with biopsy may be useful to exclude
Giardia or other pathogens and to characterize the extent of villous atrophy.
Suspected or established giardiasis can be treated with either metronidazole (5 mg/kg per dose
administered three times a day for 7 days) or nitazoxanide for 3 days (100 mg bid for children younger
than 3 years, 200 mg bid for children ages 4 through 11 years, 500 mg bid for patients older than 12
years of age). If asymptomatic carriers are suspected in the family, it may be prudent to evaluate and
treat the entire family.

References:

American Academy of Pediatrics. Giardia intestinalis infections (giardiasis). In: Pickering LK, Baker CJ,
Long SS, McMillan JA, eds. Red Book: 2006 Report of the Committee on Infectious Diseases. 27th ed.
Elk Grove Village, Ill: American Academy of Pediatrics; 2006:296-301

Huang DB, White AC. An updated review on Cryptosporidium and Giardia. Gastroenterol Clin North
Am. 2006;35:291-314. Abstract available at: http://www.ncbi.nlm.nih.gov/pubmed/16880067

page 659
2009 PREP SA on CD-ROM

Critique: 193

Photomicrograph of a Giardia lamblia cyst using an iodine stain. (Courtesy of


the Public Health Image Library, Centers for Disease Control and Prevention)

page 660
2009 PREP SA on CD-ROM

Critique: 193

Scanning electron micrograph demonstrates a Giardia lamblia trophozoite.


(Courtesy of the Public Health Image Library, Centers for Disease Control and
Prevention)

page 661
2009 PREP SA on CD-ROM

Question: 194
You admit a 39 weeks’ gestation male who has respiratory distress to the intensive care nursery. His
mother had a negative group B Streptococcus screening culture and did not receive antibiotics in labor.
She did not have chorioamnionitis or prolonged rupture of the fetal membranes. However, the amniotic
fluid was meconium-stained at the time of delivery, and the infant required tracheal intubation, with
resultant meconium suctioned from below the vocal cords. Apgar scores were 3 and 7 at 1 and 5
minutes, respectively. On physical examination, he has marked work of breathing with tachypnea and
retractions and episodic cyanosis when agitated. Breath sounds are coarse and equal. There is no
heart murmur. While receiving hood oxygen at an FiO2 of 0.50, his oxygen saturation by pulse oximetry
is 85%. You obtain a chest radiograph.

Of the following, the radiographic findings MOST expected for this infant are

A. air bronchograms, diffusely hazy lung fields, and low lung volume

B. cardiomegaly, hazy lung fields, and pulmonary vascular engorgement

C. fluid density in the horizontal fissure, hazy lung fields with central vascular prominence, and normal
lung volume

D. gas-filled loops of bowel in the left hemithorax and opacification of the right lung field

E. patchy areas of diffuse atelectasis, focal areas of air-trapping, and increased lung volumes

page 662
2009 PREP SA on CD-ROM

Critique: 194 Preferred Response: E


The infant described in the vignette, who was born at term following exposure to meconium-stained
amniotic fluid (MSAF), has respiratory distress and hypoxemia. Radiographic evidence of meconium
aspiration syndrome (MAS), atelectasis and hyperinflation, is seen in Item C194A.
Meconium aspiration may follow in utero fetal gasping and aspiration of MSAF in the fetus that has
intrauterine growth restriction or asphyxia or after delivery of the newborn who has meconium in the
oropharynx, nasopharynx, or trachea. If respiratory distress follows, the condition often is described as
MAS.
The prevalence of MAS among infants born following MSAF is less than 10%, but tends to rise
beyond 40 weeks' gestation (Item C194B), especially in cases of nonreassuring fetal heart rate
monitoring or perinatal asphyxia. Affected infants present with clinical manifestations of impaired
pulmonary compliance and hypoxemia. The chest radiograph is essential in confirming the diagnosis. In
the classic case of MAS, a diffuse pneumonitis ensues, the lung often is hyperinflated, and there are
patchy infiltrates and opacification of the pulmonary parenchyma. Complication with air-leak syndromes
such as pneumothorax or pneumomediastinum is not uncommon.
In some cases, chest radiography demonstrates pulmonary hypoperfusion without any apparent
infiltrates or air leaks, and these cases generally represent hypoxia-mediated pulmonary vascular
reactivity and pulmonary hypertension that may have been initiated in utero.
Air bronchograms in diffusely hazy low volume lungs are most consistent with surfactant
deficiency-related respiratory distress syndrome (Item C194C) in preterm infants. The same findings in
a term infant might be seen in congenital pneumonia. Cardiomegaly with hazy lung fields and
pulmonary vascular engorgement is seen in left-sided obstructive cardiac disease states or infants who
have pulmonary overcirculation (eg, truncus arteriosus, aortic stenosis, anomalous pulmonary venous
return, transposition of the great arteries) (Item C194D). Fluid densities in the horizontal fissure with
hazy lung fields, normal lung volumes, and central vascular prominence are seen in retained fetal lung
fluid, otherwise known as transient tachypnea of the newborn (Item C194E). When gas-filled loops of
bowel are seen in the chest, diaphragmatic hernia is the diagnosis (Item C194F).

References:

Aly H. Respiratory disorders in the newborn: identification and diagnosis. Pediatr Rev.
2004;25:201-208. Available at: http://pedsinreview.aappublications.org/cgi/content/full/25/6/201

Dargaville PA, Copnell B for the Australian and New Zealand Neonatal Network. The epidemiology of
meconium aspiration syndrome: incidence, risk factors, therapies, and outcome. Pediatrics.
2006;117:1712-1721. Available at: http://pediatrics.aappublications.org/cgi/content/full/117/5/1712

Miller MJ, Fanaroff AA, Martin RJ. Respiratory disorders in preterm and term infants. In: Martin RJ,
Fanaroff AA, Walsh MC, eds. Fanaroff and Martin's Neonatal-Perinatal Medicine. 8th ed. Philadelphia,
Pa: Mosby Elsevier; 2006:1122-1145

Ross MG. Meconium aspiration syndrome-more than intrapartum meconium. N Engl J Med.
2005;353:946-948. Extract available at: http://content.nejm.org/cgi/content/extract/353/9/946

page 663
2009 PREP SA on CD-ROM

Critique: 194

Meconium aspiration: There are areas of atelectasis (arrows) and


hyperinflation (seen best at the bases). (Reprinted with permission from Aly
H. Respiratory disorders in the newborn: identification and diagnosis. Pediatr
Rev. 2004;25:201-208.)

page 664
2009 PREP SA on CD-ROM

Critique: 194

Incidence of meconium aspiration syndrome related to gestational age.


(Reprinted with permission from Dargaville PA, Copnell B for the Australian
and New Zealand Neonatal Network. The epidemiology of meconium
aspiration syndrome: incidence, risk factors, therapies, and outcome.
Pediatrics. 2006;117:1712-1721.)

page 665
2009 PREP SA on CD-ROM

Critique: 194

Underinflation, a "ground-glass" appearance, and air bronchograms (arrows)


are characteristic of respiratory distress syndrome. (Courtesy of B. Carter)

page 666
2009 PREP SA on CD-ROM

Critique: 194

Chest radiograph from an infant who has transposition of the great vessels
revealing prominent pulmonary vessels (arrows), suggesting pulmonary
overcirculation. (Reprinted with permission from:Aly H. Respiratory disorders
in the newborn: identification and diagnosis. Pediatr Rev. 2004;25:201-208.)

page 667
2009 PREP SA on CD-ROM

Critique: 194

Chest radiograph in transient tachypnea of the newborn shows increased


pulmonary interstitial markings and fluid in the interlobar fissures (arrows).
(Reprinted with permission from Aly H. Respiratory disorders in the newborn:
identification and diagnosis. Pediatr Rev. 2004;25:201-208.)

page 668
2009 PREP SA on CD-ROM

Critique: 194

Plain radiograph of the chest and abdomen in a patient who has congenital
diaphragmatic hernia shows bowel in the left chest, with displacement of the
heart to the right. (Courtesy of B. Carter)

page 669
2009 PREP SA on CD-ROM

Question: 195
A mother brings her 5-year-old girl to your office because she noticed a "lump" in her daughter’s neck
over the past several weeks. The girl appears well and has normal vital signs and no fever. A 1x1-cm
slightly soft mass (Item Q195) is apparent in the middle of her neck, and when she swallows, the mass
moves vertically. There is no drainage or overlying erythema.

Of the following, the MOST likely diagnosis is

A. branchial cleft cyst

B. cystic hygroma

C. reactive lymphadenopathy

D. thyroglossal duct cyst

E. thyroid nodule

page 670
2009 PREP SA on CD-ROM

Question: 195

Mass, as described for the child in the vignette. (Courtesy of M. Rimsza)

page 671
2009 PREP SA on CD-ROM

Critique: 195 Preferred Response: D


The differential diagnosis of neck masses in children is extensive and includes inflammatory masses,
neoplasms, and congenital lesions. The most common masses are inflammatory and include reactive
lymphadenopathy and lymphadenitis. Reactive lymphadenopathy occurs when an area drained by the
lymph node becomes infected, as in anterior cervical adenopathy with streptococcal or viral pharyngitis.
The affected lymph nodes are enlarged, mobile, and usually tender, but overt signs of infection, such as
erythema and warmth, are absent. Lymphadenitis occurs when the node itself becomes infected, and
erythema, tenderness, warmth, and occasionally fluctuance are present.
Congenital lesions, although present from birth, may not become evident until later in childhood,
often after a simple upper respiratory tract infection. These result from failure of branchial clefts to close
properly during embryonic development. The most common is the branchial cleft cyst (Item C195A),
which may be unilateral or bilateral and is nonmobile. A thyroglossal duct cyst is a soft midline mass
(Item C195B) that moves vertically when the child swallows or protrudes the tongue, as described for
the girl in the vignette. Drainage onto the skin may be seen with these lesions. Both types of lesions
may become infected and appear indistinguishable from lymphadenitis (especially unilateral branchial
cleft cysts), but their persistence after appropriate antibiotic treatment suggests the diagnosis.
Neoplasms, such as leukemia and lymphoma, may present with enlarged lymph nodes in the
cervical area. Characteristics include nontender, firm nodes that are matted to the adjacent tissues. A
cystic hygroma (Item C195C), or lymphangioma, is a soft mass of dilated lymph vessels. These benign
tumors of the lymphatic system can grow to a large size and infiltrate local structures and blood
vessels. Thyroid nodules are infrequent in children. They may be firm or soft masses and are usually
lateral rather than midline. Movement on swallowing is not typical.

References:

Camitta BM. The lymphatic system. In: Kliegman RM, Behrman RE, Jenson HB, Stanton BF, eds.
Nelson Textbook of Pediatrics. 18th ed. Philadelphia, Pa: Elsevier Saunders; 2007:2092-2096

Tracy TF Jr, Muratore CS. Management of common head and neck masses. Semin Pediatr Surg.
2007;16:3-13. Abstract available at: http://www.ncbi.nlm.nih.gov/pubmed/17210478

Waldhausen JH. Branchial cleft and arch anomalies in children. Semin Pediatr Surg. 2006;15:64-69.
Abstract available at: http://www.ncbi.nlm.nih.gov/pubmed/16616308

page 672
2009 PREP SA on CD-ROM

Critique: 195

Branchial cleft cysts do not appear in the midline. Typically, they are located
laterally along the anterior border of the sternocleidomastoid muscle.
(Courtesy of D. Epstein)

page 673
2009 PREP SA on CD-ROM

Critique: 195

A thyroglossal duct cyst is a soft midline mass that moves vertically when the
child swallows or protrudes the tongue. (Courtesy of M. Rimsza)

page 674
2009 PREP SA on CD-ROM

Critique: 195

Cystic hygromas represent macrocystic lymphatic malformations. They


typically are soft and not localized only to the midline neck. (Courtesy of the
Media Lab at Doernbecher)

page 675
2009 PREP SA on CD-ROM

Question: 196
You have been treating a 2-year-old girl for 1 month with ferrous sulfate after her hemoglobin was 10.0
g/dL (100.0 g/L) and hematocrit was 29% (0.29). Today, her reticulocyte count is 4.2% (0.042),
hemoglobin is 11.5 g/dL (115.0 g/L), and hematocrit is 33% (0.33). The nurse practitioner student with
whom you are working asks if she can stop the iron supplement.

Of the following, the BEST reason for continuing iron therapy in this child is to

A. increase her appetite

B. prevent infection

C. prevent lead poisoning

D. reinforce vitamin supplementation

E. replenish iron stores

page 676
2009 PREP SA on CD-ROM

Critique: 196 Preferred Response: E


Treatment of iron deficiency anemia involves iron replacement, usually with ferrous sulfate, for at least 2
months after the anemia has been corrected to replenish iron stores.
Most children who have mild iron deficiency anemia are asymptomatic and are diagnosed following
routine screening as part of health supervision visits, screening based on dietary risk factors, or a
complete blood count obtained for evaluation of illness, particularly fever without a focus.
Contrary to popular belief, there is no evidence that treatment of iron deficiency stimulates appetite.
Treatment of iron deficiency also does not prevent lead poisoning, although lead poisoning and iron
deficiency may coexist because iron deficiency may lead to pica.
Neither iron supplementation nor therapy has been shown to prevent clinical infection. However,
studies in children of low socioeconomic status have shown that iron supplementation may improve
T-cell counts, delayed hypersensitivity skin reactions, and interleukin-2 production. Vitamin and mineral
supplementation is not recommended for children who are eating normal diets and have no evidence
for deficiency; such supplementation may increase the risk of ingesting toxic amounts of iron.

References:

Glader B. Iron deficiency anemia. In: Kliegman RM, Behrman RE, Jenson HB, Stanton BF, eds. Nelson
Textbook of Pediatrics. 18th ed. Philadelphia, Pa: Elsevier Saunders; 2007:2014-2016

Kleinman RE. Nutrition and immunity. In: Pediatric Nutrition Handbook. 5th ed. Elk Grove Village, Ill:
American Academy of Pediatrics; 2004:609-628

Richardson M. Microcytic anemia. Pediatr Rev. 2007;28:5-14. Available at:


http://pedsinreview.aappublications.org/cgi/content/full/28/1/5

page 677
2009 PREP SA on CD-ROM

Question: 197
You are performing screening sports participation examinations at the local high school. One of the
students, a 16-year-old boy, reports that his father has hypertrophic cardiomyopathy but that none of his
three older brothers has it. He also reports that he was seen by a cardiologist at age 10 years and was
"fine." As you take his history, you find that he has never had shortness of breath, chest pain, exercise
intolerance, dizziness, or fainting. He has always participated in sports and has excelled.

Of the following, the BEST plan of management for this boy is

A. chest radiography

B. electrocardiography

C. genetic testing for hypertrophic cardiomyopathy

D. referral to a cardiologist

E. unrestricted sports participation unless symptoms develop

page 678
2009 PREP SA on CD-ROM

Critique: 197 Preferred Response: D


Hypertrophic cardiomyopathy (HCM) typically is inherited as an autosomal dominant trait due to a gene
abnormality on chromosome 15. However, multiple genes have been found to be responsible for this
disorder, and this fact contributes to the variability of severity. Interestingly, HCM may not be apparent
on echocardiography until adolescence and even young adulthood in some patients, necessitating
regular, repeat evaluation for asymptomatic children who have an affected first-degree relative.
Alternatively, genetic testing may be an option for some, but genetic variability and the expense of
testing may make this impractical in some cases. Accurate diagnosis of HCM in children and
adolescents is essential because the condition can be the source of morbidity and mortality. Thoughtful
counseling is an important component of care for the affected child; some sports and activities, such as
competitive soccer, basketball, and football, are believed to be particularly high risk for the development
of arrhythmia and even sudden death. In fact, HCM is the most common reason for a young person to
experience sudden death during sports activities, occurring with nearly twice the frequency of coronary
artery anomalies.
The boy in the vignette has a family history of HCM (present in his father); each of his siblings has
as great as a 50% chance of having inherited the HCM gene from the father. Current recommendations
suggest evaluating such children regularly (perhaps as often as yearly) with detailed history, physical
examination, and diagnostic testing that includes electrocardiography and echocardiography (Item
C197). Therefore, the boy in the vignette and his siblings should be referred to a cardiologist for further
evaluation.
Chest radiography, a relatively nonspecific imaging study for cardiac structure, may yield normal
results in patients who have HCM and would not be used either to confirm or rule out the diagnosis.
Similarly, electrocardiography tracings can appear normal in a small percentage of patients who have
HCM, making this study inappropriate for confirmation or ruling out of the diagnosis. Genetic testing for
the HCM gene can be very effective if the specific gene mutation is well-identified in the index case of
the family, but undertaking such testing for the boy in the vignette without testing his father first is not
productive. Recognition of HCM and referral to a cardiologist for thorough preparticipation screening are
critical.

References:

Berger S, Utech L, Hazinski MF. Sudden death in children and adolescents. Pediatr Clin North Am.
2004;51:1653-1677. Abstract available at: http://www.ncbi.nlm.nih.gov/pubmed/15561179

Corrado D, Basso C, Schiavon M, Thiene G. Screening for hypertrophic cardiomyopathy in young


athletes. N Engl J Med. 1998;339:364-369. Abstract available at:
http://www.ncbi.nlm.nih.gov/pubmed/9691102

Ho CY, Seidman CE. A contemporary approach to hypertrophic cardiomyopathy. Circulation.


2006;113:e858-e862. Available at: http://circ.ahajournals.org/cgi/content/full/113/24/e858

Maron BJ, Chaitman BR, Ackerman MJ, et al. Recommendations for physical activity and recreational
sports participation for young patients with genetic cardiovascular diseases. Circulation.
2004;109:2807-2816. Available at: http://circ.ahajournals.org/cgi/content/full/109/22/2807

Maron BJ, Thompson PD, Ackerman MJ, et al. AHA scientific statements. Recommendations and
considerations related to preparticipation screening for cardiovascular abnormalities in competitive
athletes: 2007 update. A scientific statement from the American Heart Association Council on Nutrition,

page 679
2009 PREP SA on CD-ROM

Physical Activity, and Metabolism: endorsed by the American College of Cardiology Foundation.
Circulation. 2007;115:1643-1655. Available at: http://circ.ahajournals.org/cgi/content/full/115/12/1643

page 680
2009 PREP SA on CD-ROM

Question: 198
A 4-year-old boy recently underwent hematopoietic stem cell transplantation for acute myelogenous
leukemia. Fourteen days after his transplant, he experiences a seizure and confusion. He is receiving
cyclosporine, prednisone, ganciclovir, fluconazole, cefotaxime, tobramycin, and omeprazole. Magnetic
resonance imaging shows signal changes in bilateral occipital lobes.

Of the following, the MOST likely cause of the seizures is

A. cyclosporine

B. fluconazole

C. ganciclovir

D. prednisone

E. tobramycin

page 681
2009 PREP SA on CD-ROM

Critique: 198 Preferred Response: A


Seizures in children are not commonly iatrogenic, but medications used in certain disciplines, most
commonly oncology and psychiatry, may precipitate them. Chemotherapy for childhood cancers or
organ transplantation often has adverse effects on the central nervous system, causing neuronal cell
death, changes in white matter, or vascular lesions. Patients receiving cyclosporine or intrathecal
methotrexate may develop diffuse or predominantly occipital white matter changes.
The child described in the vignette has characteristic changes associated with cyclosporine. In
addition to seizures, occipital blindness or strokelike weakness may occur. The changes seen on
magnetic resonance imaging can be reversible. Fluconazole, ganciclovir, prednisone, and tobramycin
are unlikely causes of seizures. Other medications that may cause or exacerbate seizures in children
include isoniazid, theophylline, cocaine, psychostimulants, bupropion, insulin, and oral hypoglycemic
agents.

References:

Abbott MB, Levin RH, Wu S. Medication potpourri. Pediatr Rev. 2006;27:283-288. Available at:
http://pedsinreview.aappublications.org/cgi/content/full/27/8/283

Norman JK, Parke JT, Wilson DA, McNall-Knapp RY. Reversible posterior leukoencephalopathy
syndrome in children undergoing induction therapy for acute lymphoblastic leukemia. Pediatr Blood
Cancer. 2007;49:198-203. Abstract available at: http://www.ncbi.nlm.nih.gov/pubmed/16123992

Pound CM, Keene DL, Udjus K, Humphreys P, Johnston DL. Acute encephalopathy and cerebral
vasospasm after multiagent chemotherapy including PEG-asparaginase and intrathecal cytarabine for
the treatment of acute lymphoblastic leukemia. J Pediatr Hematol Oncol. 2007;29:183-186. Abstract
available at: http://www.ncbi.nlm.nih.gov/pubmed/17356399

page 682
2009 PREP SA on CD-ROM

Question: 199
During the health supervision visit of a 2-week-old infant, you note that his weight remains below his
birthweight. The baby was delivered at term by a midwife in the parents’ home. There were no
complications, and the parents have declined all perinatal testing. His mother says that he breastfeeds
well, and her milk supply is good compared with that for her previous two children. Recently, though,
the infant has been vomiting after feedings. On physical examination, he has total body jaundice, and
his liver is enlarged to palpation. He is alert.

Of the following, the test that is MOST likely to aid in diagnosis is

A. abdominal ultrasonography

B. serum transaminases measurement

C. total and direct bilirubin measurement

D. urine organic acids measurement

E. urine reducing substances measurement

page 683
2009 PREP SA on CD-ROM

Critique: 199 Preferred Response: E


The infant described in the vignette has a history and physical findings suggestive of galactosemia, an
autosomal recessive inborn error of metabolism that results in the inability to oxidize galactose.
Galactose dimerizes with glucose to form lactose, which is abundant in human and cow milk.
Individuals who have classic galactosemia are deficient in galactose-1-phosphate uridyltransferase
(GALT) activity, resulting in an elevated concentration of galactose, a reducing substance, in the urine.
Although the presence of reducing substances in the urine is not specific for galactosemia, this finding,
especially in the presence of poor weight gain, jaundice, and hepatomegaly, is strongly suggestive of
galactosemia. Galactosemia can be detected in almost 100% of affected infants by newborn screening
programs that test for it.
The infant who has classic galactosemia typically presents within days of the initiation of milk
feedings with vomiting and jaundice. If untreated, the child develops hepatomegaly and failure to thrive.
Escherichia coli sepsis may occur, and approximately 10% of infants have cataracts at the time of
diagnosis. Treatment is aimed at removing lactose from the diet, usually by feeding the baby a
soy-based formula.
The natural history of galactosemia is more complicated than once believed. Studies are showing
that, despite early diagnosis and strict dietary management, affected older children and adults can have
learning disabilities, neurologic disorders, verbal dyspraxia, and ovarian failure.
For infants presenting with signs of galactosemia, abdominal ultrasonography may show
hepatomegaly. Serum transaminases are elevated, as are total and direct bilirubin values. However,
these abnormalities are not sufficiently informative to point to a diagnosis. Urine organic acids
assessment may document elevated lactate, which is a nonspecific finding.

References:

Elsas LJ. Galactosemia. GeneReviews. 2007. Available at:


http://www.geneclinics.org/servlet/access?db=geneclinics&site=gt&id=8888891&key=xAcWBcrjmZrVo&
gry=&fcn=y&fw=4Vlc&filename=/profiles/galactosemia/index.html

Nyhan WL, Barshop BA, Ozand PT. Organic acidemias. In: Atlas of Metabolic Diseases. 2nd ed.
London, England: Hodder Arnold; 2005:1-108

page 684
2009 PREP SA on CD-ROM

Question: 200
A 16-year-old young man presents to the emergency department with a 12-hour history of pain in the
scrotal area. He states that the pain started gradually and describes it as on the left side and moderate
in intensity. He is sexually active and uses condoms. He has some burning pain with urination, but no
penile discharge. He has felt warm but has not taken his temperature. He has had no vomiting or
diarrhea. He has had no previous similar symptoms. On physical examination, the young man is
afebrile and has normal findings on abdominal evaluation. He has moderate swelling of the left scrotum
without erythema and marked tenderness that involves more of the posterolateral area. The testicular
position is lower on the left than on the right. The left spermatic cord is very tender. Urinalysis shows
more than 10 white blood cells per high-power field on a first-void specimen.

Of the following, the MOST likely diagnosis is

A. epididymitis

B. testicular torsion

C. testicular tumor

D. urinary tract infection

E. varicocele

page 685
2009 PREP SA on CD-ROM

Critique: 200 Preferred Response: A


The differential diagnosis for a painful scrotal mass or swelling in an adolescent includes testicular
torsion, torsion of the spermatic cord or appendix testis, epididymitis (Item C200), orchitis, trauma with
hematoma, and incarcerated hernia. Other causes could be Henoch-Schönlein purpura or other trauma
such as an infected piercing or insect sting. The features exhibited by the young man in the vignette
that support the diagnosis of epididymitis include: sexual activity, gradual pain onset, localization of pain
to the spermatic cord and posterolateral area of the scrotum, dysuria, and pyuria.
With testicular torsion, a true surgical emergency, pain often is more abrupt in onset, and nausea
and vomiting are frequent. The affected testis frequently is located more superiorly than the
contralateral testis. Testicular tumors and varicoceles generally are not painful and are not
accompanied by spermatic cord tenderness. A urinary tract infection is unlikely in an adolescent who
has no congenital urinary tract abnormalities and would not produce a painful, swollen testis.

References:

Adelman WP, Joffe A. Scrotal disorders. In: Neinstein, LS, ed. Adolescent Health Care: A Practical
Guide. 5th ed. Philadelphia, Pa: Lippincott Williams & Wilkins; 2008:401-410

Centers for Disease Control and Prevention. Update to CDC's sexually transmitted diseases treatment
guidelines, 2006: fluoroquinolones no longer recommended for treatment of gonococcal infections.
MMWR Morbid Mortal Wkly Rep. 2007;56:332-336. Available at:
http://www.cdc.gov/mmwr/preview/mmwrhtml/mm5614a3.htm

Workowski AK, Berman SM, Centers for Disease Control and Prevention. Sexually transmitted
diseases treatment guidelines, 2006. MMWR Recomm Rep. 2006;55(RR11):1-94. Available at:
http://www.cdc.gov/mmwr/preview/mmwrhtml/rr5511a1.htm

page 686
2009 PREP SA on CD-ROM

Critique: 200

Epididymitis is characterized by erythema and swelling of the scrotum.


(Courtesy of J. Zenel)

page 687
2009 PREP SA on CD-ROM

Question: 201
A 1-year-old boy who is intubated for severe asthma is demonstrating significant acute respiratory and
cardiac deterioration, as evidenced by tachycardia, tachypnea, decreased blood pressure, and oxygen
saturation of 75%. During your examination, you note a marked shift of the trachea to the left and
markedly decreased aeration on the right side.

Of the following, the MOST likely cause of this boy’s sudden respiratory deterioration is

A. endotracheal tube obstruction

B. exacerbation of the underlying asthma

C. intubation of the right mainstem bronchus

D. tension pneumothorax

E. ventilator-associated pneumonia

page 688
2009 PREP SA on CD-ROM

Critique: 201 Preferred Response: D


The tachypnea, tachycardia, decreased breath sounds on the involved side, and decreased
oxygenation described for the boy in the vignette are strongly suggestive of pneumothorax, the
accumulation of air within the extrapleural space. Pneumothoraces typically are classified as simple,
communicating, or tension. Simple pneumothoraces are the most common and have no communication
with the atmosphere. They usually are a result of a spontaneous pneumothorax with or without
underlying lung pathology, blunt chest trauma, or mechanical ventilation. A communicating
pneumothorax normally is caused by open chest trauma, although rapid accumulation of pleural air is
possible in the spontaneously breathing patient due to entrainment of air with inspiration. Tension
pneumothoraces represent a medical emergency and develop when continued accumulation of air
produces a mediastinal shift to the contralateral side and subsequent compression and decreased
function of the lung and vascular structures. The tracheal shift and hypotension reported for the boy in
the vignette indicate that he has a tension pneumothorax.
Rapid diagnosis and treatment of symptomatic pneumothoraces is critical. Chest radiography (Item
C201) is the gold standard for diagnosis, but clinical conditions may necessitate rapid treatment with
thoracentesis or thoracic tube placement prior to imaging. Development of a pneumothorax along with
other potential ventilator complications always should be considered promptly when a mechanically
ventilated patient deteriorates.
Endotracheal tube obstruction normally presents with bilateral decreased aeration, difficulty in
passing a suction catheter, and lack of tracheal shift. A right bronchus intubation is characterized by
decreased aeration on the left side and oxygen desaturation, but cardiovascular compromise is less
common than with a tension pneumothorax. Progression of underlying disease processes such as
asthma or development of ventilator-associated pneumonia is more gradual than described in the
vignette.

References:

Chase MA, Wheeler DS. Disorders of the pediatric chest. In: Wheeler DS, Wong HR, Shanley T, eds.
Pediatric Critical Care Medicine: Basic Science and Clinical Evidence. New York, NY: Springer-Verlag;
2007:361-375

Winnie GB. Pnuemothorax. In: Kliegman RM, Behrman RE, Jenson HB, Stanton BF, eds. Nelson
Textbook of Pediatrics. 18th ed. Philadelphia, Pa: Saunders Elsevier; 2007:1835-1836

page 689
2009 PREP SA on CD-ROM

Critique: 201

Chest radiography demonstrating tension pneumothorax: There is collapse of


the right lung, flattening of the right hemidiaphragm, and shift of the heart and
mediastinum to the left. (Courtesy of D. Mulvihill)

page 690
2009 PREP SA on CD-ROM

Question: 202
During the health supervision visit of a 9}-year-old girl, you note that her height is just above the 97th
percentile for age and her weight is at the 85th percentile. Her mother is 5 feet 5 inches tall and father is
5 feet 10 inches. Her parents ask if she will be very tall when she has finished growing.

Of the following, the MOST important element of the physical examination to help answer this question
is

A. arm span measurement

B. body mass index

C. eye examination

D. Sexual Maturity Rating

E. upper-to-lower segment ratio

page 691
2009 PREP SA on CD-ROM

Critique: 202 Preferred Response: D


Most children of tall stature either have tall parents or have matured early and are tall in late childhood,
although adult height after early puberty will be well within the mid-parental range. Therefore, if the girl
described in the vignette is in puberty, an explanation for her tall stature at age 9 years will be apparent,
and a bone age radiograph can be used to predict her ultimate adult height. Accordingly, it is most
important to determine her Sexual Maturity Rating. Lack of puberty suggests that she will, indeed, be
tall.
Further studies might be indicated to look for other reasons for her tall stature, particularly if she has
had growth acceleration. Rarely, children of tall stature have Marfan syndrome, which is characterized
by long arm span and a decreased body mass index. Increased body mass index and obesity may be
associated with tall stature and early puberty. If there is worry about growth hormone excess and a
pituitary tumor, eye examination and careful assessment of visual fields should be undertaken, but
growth hormone-producing pituitary tumors often are too small to produce visual symptoms. They are
detected best by physical examination showing signs of growth hormone excess: widened spaces
between the teeth, large hands and feet, and increased soft-tissue thickening. Laboratory measurement
of insulin-like growth factor I and magnetic resonance imaging of the pituitary may be helpful.
Measurement of the upper-to-lower segment ratio may be useful in the clinical diagnosis of Marfan
syndrome because long legs compared with the trunk result in a lower ratio than expected for age.
Other congenital causes of tall stature, such as Sotos syndrome, are associated with specific physical
and behavioral findings, including large head circumference and decreased intellectual functioning.
Diagnostic genetic testing is available for this disorder.

References:

Boom JA. Normal pediatric growth. UpToDate Online 15.3. 2008.Available for subscription at:
http://www.uptodateonline.com/utd/content/topic.do?topicKey=gen_pedi/13648

Cohen P, Shim M. Hyperpituitarism, tall stature, and overgrowth syndromes. In: Kliegman RM,
Behrman RE, Jenson HB, Stanton BF, eds. Nelson Textbook of Pediatrics. 18th ed. Philadelphia, Pa:
Saunders Elsevier; 2007:2303-2307

Richmond EJ, Rogol AD. The child with abnormally rapid growth. UpToDate Online 15.3. 2008.
Available for subscription at:
http://www.uptodateonline.com/utd/content/topic.do?topicKey=pediendo/7226

page 692
2009 PREP SA on CD-ROM

Question: 203
A mother brings in her 10-year-old daughter and 8-year-old son because they are fighting constantly.
The son says he hates having a sister and complains that his parents favor her and give her everything
she wants. The daughter says that her brother is spoiled and always touches her stuff. The mother is
frustrated by their constant fighting and asks for assistance in handling the children.

Of the following, the BEST initial guidance for the mother is to

A. explain that this is typical of siblings and she should ignore the behavior

B. give her a list of books on parenting

C. refer her to a behavioral therapist to improve her parenting skills

D. suggest she use behavioral modification techniques to diminish fighting

E. tell her to return in 6 months if the siblings are still fighting

page 693
2009 PREP SA on CD-ROM

Critique: 203 Preferred Response: D


Although sibling rivalry is common, the actions described for the children in the vignette indicate the
need for the mother to employ behavioral techniques to decrease their fighting in the home. Children
initially should be allowed to resolve their differences, but parents need to intervene if one sibling is
being abused either physically or verbally. When the fighting is heading toward a dangerous situation,
the parents need to describe the actions of the siblings, establish limits, and separate the siblings. The
purpose of the behavioral intervention is to open lines of communication so the siblings can begin to
work out their differences. If the behavior continues to be challenging and not responsive to such initial
parental interventions, referral to a therapist may be considered. Telling the mother either to ignore the
sibling rivalry or wait 6 months will not aid in changing the children's behaviors. A book on sibling rivalry
may supplement an initial therapeutic plan, but the family should be provided direct guidance on
implementing a behavior intervention strategy.
A common time for the emergence of sibling rivalry is after a recent birth. It is common for a child to
be upset and have some regression in behavioral skills following the birth of a new sibling. Parents
need to intervene more with younger children to prevent any injuries.

References:

Faber A, Mazlish E. When the kids fight: how to step in so we can step out. In: Siblings Without Rivalry:
How to Help Your Children Live Together So You Can Live Too. New York, NY: Quill; 2002:146-177

Needlman R. Sibling rivalry. In: Parker S, Zuckerman B, Augustyn M, eds. Developmental and
Behavioral Pediatrics: A Handbook for Primary Care. 2nd ed. Philadelphia, Pa: Lippincott Williams &
Wilkins; 2005:412-415

page 694
2009 PREP SA on CD-ROM

Question: 204
You are evaluating an 8-month-old boy who is having multiple "coughing spells." During these spells,
the boy sometimes turns blue and even vomits. You inform the mother that you are going to prescribe
an antimicrobial agent. She wants to know why you are giving her infant an antimicrobial agent when he
needs something for the cough.

Of the following, the BEST reason to prescribe an antimicrobial agent for this boy is that treatment will
decrease the

A. chance of death

B. cough

C. hypoxic episodes

D. infectivity

E. posttussive vomiting

page 695
2009 PREP SA on CD-ROM

Critique: 204 Preferred Response: D


The clinical findings for the child described in the vignette indicate the diagnosis of pertussis (whooping
cough). The clinical course of pertussis can be ameliorated with antimicrobial agents only in patients
whose disease is limited to mild upper respiratory tract symptoms (catarrhal stage). Once patients
develop the paroxysmal cough (paroxysmal stage), the use of antimicrobial agents does not shorten the
disease course or affect the cough, posttussive vomiting, hypoxic episodes, or chance of death.
However, antimicrobial therapy is used for patients in the paroxysmal stage to decrease the spread of
Bordetella pertussis to others.
Erythromycin products have been the treatment of choice for pertussis until recently. Data now
demonstrate that azithromycin and clarithromycin have microbiologic effectiveness similar to
erythromycin in individuals 6 months of age or older. The advantages of the newer macrolides over
erythromycin for the treatment of pertussis are: 1) higher tissue concentrations, 2) longer half-life, 3)
fewer daily doses required (one or two per day), and 4) short treatment courses (5 to 7 days).
Azithromycin and clarithromycin also are approved as first-line agents for prophylaxis of exposed
individuals. In most situations of treatment or prophylaxis, patients tolerate and are more compliant with
azithromycin or clarithromycin than erythromycin products. Erythromycin should not be used in infants
younger than 1 month of age due to the agent's association with infantile pyloric stenosis. There are no
safety data for the use of clarithromycin in neonates, making azithromycin the macrolide of choice for
this age group. Trimethoprim-sulfamethoxazole is an alternative agent that can be used for children
older than 2 months of age who cannot take macrolides for pertussis treatment or prophylaxis.

References:

American Academy of Pediatrics. Pertussis (whooping cough). In: Pickering LK, Baker CJ, Long SS,
McMillan JA, eds. Red Book: 2006 Report of the Committee on Infectious Diseases. 27th ed. Elk Grove
Village, Ill: American Academy of Pediatrics; 2006:498-520

Tiwari T, Murphy RV, Moran J. Recommended antimicrobial agents for the treatment and postexposure
prophylaxis of pertussis. 2005 CDC guidelines. MMWR Recomm Rep. 2005;54(RR14):1-16. Available
at: http://www.cdc.gov/mmwr/preview/mmwrhtml/rr5414a1.htm

Waseem M, Kin LL. Index of suspicion: case 6. Pediatr Rev. 2005;26:23-33. Available at:
http://pedsinreview.aappublications.org/cgi/content/full/26/1/23

page 696
2009 PREP SA on CD-ROM

Question: 205
You are evaluating a 17-month-old previously healthy girl who presents with an 8-month history of
recurrent cellulitis and abscesses on her lower right abdomen. Eight months ago, she developed a
"pimple" on her abdomen that rapidly enlarged to the size of a golf ball and became very red, hard, and
tender over 2 days. The lesion spontaneously drained a purulent material and resolved. Over the last 8
months, the girl has had nine similar episodes. She was seen on several occasions in an urgent care
center and each time was placed on a course of cephalexin, which resulted in no improvement until the
lesion drained spontaneously. The patient has no fever with the episodes. Physical examination shows
a 2x3-cm erythematous, indurated, very tender, fluctuant lesion on the patient’s right flank.

Of the following, the MOST likely organism causing this patient’s recurrent infections is

A. methicillin-resistant Staphylococcus aureus

B. methicillin-sensitive Staphylococcus aureus

C. Staphylococcus epidermidis

D. Streptococcus pneumoniae

E. Streptococcus pyogenes

page 697
2009 PREP SA on CD-ROM

Critique: 205 Preferred Response: A


The first-generation cephalosporins (eg, cephalexin) have a spectrum of activity focused primarily on
gram-positive bacteria. They have been used widely as alternatives to penicillin for the treatment of
staphylococcal and nonenterococcal streptococcal infections. These agents are effective in the
treatment of infections caused by susceptible strains of groups A, B, C, and G streptococci; S
pneumoniae; viridans streptococci; methicillin-susceptible Staphylococcus aureus (MSSA); and some
strains of S epidermidis. Historically, first-generation cephalosporins have been the mainstay of therapy
for skin and soft-tissue infections. The gram-negative spectrum of the first-generation cephalosporins is
much more limited. These drugs are effective against susceptible strains of Neisseria gonorrhoeae,
Escherichia coli, Klebsiella sp, and Proteus mirabilis. However, they have poor activity against
penicillin-resistant pneumococci, Haemophilus influenzae, and Moraxella catarrhalis and are not
recommended in the treatment of sinusitis, otitis media, and lower respiratory tract infections. All
isolates of methicillin-resistant S aureus (MRSA) are resistant to the cephalosporin class of antibiotics.
Antibiotics used to treat MRSA include clindamycin, trimethoprim-sulfamethoxazole, and linezolid.
The patient described in the vignette has had episodes of recurrent cellulitis and abscesses that
improve only after spontaneous drainage, despite receiving a first-generation cephalosporin for
treatment. The most common organisms associated with skin and soft-tissue infections are S aureus
and S pyogenes (group A streptococci). The organism most likely to be the cause of this patient's
infection is MRSA, which is associated with recurrent episodes of cellulitis and abscesses. MRSA is
resistant to first-generation cephalosporins such as cephalexin. MSSA and S pyogenes both are
susceptible to the first-generation cephalosporins, and infection caused by these agents should improve
with treatment. S pneumoniae is a cause of preseptal cellulitis and rarely lymphadenitis, but it is not
associated with cellulitis and abscesses. S epidermidis usually does not cause cellulitis or soft-tissue
infections.

References:

Andes DR, Craig WA. Cephalosporins. In: Mandell GL, Bennett JE, Dolin R, eds. Mandell, Douglas, and
Bennett's Principles and Practice of Infectious Diseases. 6th ed. Philadelphia, Pa: Elsevier Churchill
Livingstone; 2005:294-310

Jantausch BA. Peripheral brain: cephalosporins. Pediatr Rev. 2003;24:128-136. Available at:
http://pedsinreview.aappublications.org/cgi/content/full/24/4/128

page 698
2009 PREP SA on CD-ROM

Question: 206
A mother brings in her 4-year-old daughter because of decreased energy following a 3-day history of
diarrhea without vomiting. On physical examination, the girl’s temperature is 100.2°F (37.9°C), heart
rate is 130 beats/min, respiratory rate is 18 breaths/min, and blood pressure is 122/84 mm Hg. She has
pale conjunctivae, a hyperdynamic precordium, and mild pretibial edema. Laboratory evaluation
reveals:
· Sodium, 133.0 mEq/L (133.0 mmol/L)
· Potassium, 5.2 mEq/L (5.2 mmol/L)
· Chloride, 100.0 mEq/L (100.0 mmol/L)
· Bicarbonate, 16.0 mEq/L (16.0 mmol/L)
· Albumin, 2.5 g/dL (25.0 g/L)
· Blood urea nitrogen, 40.0 mg/dL (14.3 mmol/L)
· Creatinine, 1.4 mg/dL (123.8 mcmol/L)
· Hemoglobin, 6.1 g/dL (610.0 g/L)
· White blood cell count, 21.5x103/mcL (21.5x109/L)
· Platelet count, 90.0x103/mcL (90.0x109/L)

Of the following, the MOST likely additional laboratory abnormality is

A. a low reticulocyte count

B. elevated lactate dehydrogenase concentration

C. elevated parathyroid hormone concentration

D. positive Coombs test

E. prolonged prothrombin time

page 699
2009 PREP SA on CD-ROM

Critique: 206 Preferred Response: B


The diarrheal prodrome, tachycardia, hyperdynamic precordium, anemia, thrombocytopenia, and
elevated creatinine value described for the girl in the vignette suggests the diagnosis of
hemolytic-uremic syndrome (HUS). HUS is a microangiopathic process often linked to Shiga
toxin-producing enterohemorrhagic Escherichia coli, which results in the triad of microangiopathic
hemolytic anemia, thrombocytopenia, and acute renal failure (ARF). HUS is categorized as a thrombotic
microangiopathy that has a predilection for arterioles and capillaries, especially those involving the
renal circulation.
When HUS is preceded by diarrhea (90% of cases), it is designated D(+). The diarrheal prodrome
may be bloody or nonbloody and is associated most commonly with enterohemorrhagic E coli, with
serotype O157:H7 being the most common in the United States. This Shiga toxin-producing E coli often
is acquired by ingesting undercooked hamburger, unpasteurized apple cider, or other contaminated
foods. Patients who have this organism in their gastrointestinal tracts can develop diarrhea (bloody or
nonbloody) alone or HUS. Interestingly, only 5% to 8% of children who have hemorrhagic colitis from a
Shiga toxin-producing enterohemorrhagic E coli actually develop HUS, which suggests that virulence
and host factors likely play a role in who develops HUS. Those who exhibit HUS develop endothelial
cell injury and small vessel thromboses involving the glomerular capillary, arteriolar, and interlobular
arteries. The resultant ischemic injury affects the glomeruli and tubules, leading to ARF. The endothelial
injury results in von Willebrand factor release, platelet adhesion, and microthrombi formation. This
thrombotic microangiopathic process leads to a hemolytic anemia and thromobocytopenia.
Children who have D(+) HUS may have symptoms mimicking those of ulcerative colitis or an acute
abdomen. To detect HUS in its early stages, it is important to monitor hemoglobin, platelet count, and
serum creatinine as well as urinalysis in a child who has severe colitis. Because oliguria due to
dehydration is common in children who have severe diarrhea, the diagnosis of ARF due to HUS can be
delayed. When HUS is suspected, a peripheral blood smear should be obtained. A smear that shows
evidence of fragmented red blood cells (schistocytes) (Item C206), low platelet number, and
reticulocytosis is diagnostic. Of note, the reticulocyte count may not be markedly elevated in severe
renal dysfunction due to impaired erythropoietin production. Leukocytosis is common, with higher white
blood cell counts associated with a worse renal prognosis. In addition, due to the severity of the
hemolysis, other laboratory abnormalities include elevated lactate dehydrogenase and low haptoglobin
values. The Coombs test is negative because this is not an antibody-mediated process. The
prothrombin and partial thromboplastin times are normal because HUS is not a consumptive
coagulopathy. The parathyroid hormone concentration would not be expected to be elevated in the
setting of acute renal failure; this is a manifestation of renal osteodystrophy seen in severe forms of
chronic kidney disease.
Treatment of HUS is largely supportive, including intravenous volume expansion in those who do not
have oliguria and dialysis in those who have oligoanuria. Of note, empiric antibiotics should be avoided
in children who have bloody diarrhea because if the diarrhea is due to enterohemorrhagic E coli,
antibiotic therapy is associated with an increased risk of developing HUS.

References:

Ake JA, Jelacic S, Ciol MA, et al. Relative nephroprotection during Escherichia coli O157:H7 infections:
association with intravenous volume expansion. Pediatrics. 2005;115:e673-e680. Available at:
http://pediatrics.aappublications.org/cgi/content/full/115/6/e673

Oakes RS, Siegler RL, McReynolds MA, Pysher T, Pavia AT. Predictors of fatality in postdiarrheal
hemolytic uremic syndrome. Pediatrics. 2006;117:1656-1662. Available at:

page 700
2009 PREP SA on CD-ROM

http://pediatrics.aappublications.org/cgi/content/full/117/5/1656

Mahan JD. Hemolytic uremic syndrome. In: Kher KK, Schnaper HW, Makker SP, eds. Clinical Pediatric
Nephrology. 2nd ed. London, England: Informa Healthcare; 2007:235-244

Wong CS, Jelacic S, Habeeb RL, Watkins SL, Tarr PI. The risk of the hemolytic-uremic syndrome after
antibiotic treatment of Escherichia coli O157:H7 infections. N Engl J Med. 2000;342:1930-1936.
Available at: http://content.nejm.org/cgi/content/full/342/26/1930

page 701
2009 PREP SA on CD-ROM

Critique: 206

Schistocytes (arrows) are observed on the peripheral blood smear of patients


who have hemolytic-uremic syndrome. (Courtesy of S. Dabbagh)

page 702
2009 PREP SA on CD-ROM

Question: 207
A 16-year-old girl who has moderate persistent asthma presents to the emergency department with
coughing, wheezing, and increasing dyspnea. She states that she was feeling fine until she was
exposed to cologne that one of her classmates was wearing. An ambulance was called after her
symptoms did not improve following administration of two puffs of her beta2 agonist inhaler. On physical
examination, the teenager has a respiratory rate of 30 breaths/min, heart rate of 90 beats/min, and
pulse oximetry of 98% on room air. She has difficulty completing a sentence and points to her neck,
saying it is "hard to get air in." Her lungs are clear to auscultation, and rhinolaryngoscopy demonstrates
adduction of one of the vocal cords during inspiration. Pulmonary function testing shows a blunted
inspiratory loop (Item Q207).

Of the following, the MOST likely cause for this patient’s symptoms is

A. allergic rhinitis

B. asthma exacerbation

C. habit cough

D. sinusitis

E. vocal cord dysfunction

page 703
2009 PREP SA on CD-ROM

Question: 207

Blunted inspiratory flow curve in pink (arrow), as exhibited by the patient in


the vignette. A normal curve is shown in blue. (Courtesy of K. Waibel)

page 704
2009 PREP SA on CD-ROM

Critique: 207 Preferred Response: E


The teenager described in the vignette has signs and symptoms consistent with vocal cord dysfunction
(VCD), a condition that can mimic or coexist with asthma. In contrast to an asthma exacerbation, the
key features of VCD exhibited by this girl include a normal room air pulse oximetry reading, failure to
improve with her beta2 agonist inhaler, clear lungs, and difficulty with inspiration instead of expiration. A
blunted inspiratory loop on spirometry also is supportive of VCD, although affected patients usually
have normal spirometry readings when not experiencing symptoms. Triggers for VCD can include viral
upper respiratory tract infections, chemicals, fumes, pollution, emotional changes, laughing, exercise,
gastroesophageal reflux (GER), and cold air.
GER can cause cough and be a trigger for asthma. It may worsen during exercise, eating, or when
supine. Although GER is a cause of chronic cough, the patient in the vignette does not admit to GER
symptoms, making this diagnosis unlikely.
Postnasal drip syndrome, now termed upper airway cough syndrome, can result in coughing due to
allergic rhinitis, nonallergic rhinitis, or sinusitis. The lack of nasal congestion, rhinorrhea, or postnasal
drip for this girl makes this an unlikely cause of her acute symptoms.
Psychogenic cough, also called habit cough syndrome, is a well-described chronic cough that may
begin after a viral upper respiratory tract infection. The cough usually is nonproductive and does not
occur during sleep. Teenagers who have asthma may use coughing as a method to avoid school
(factitious or malingering), but the girl in the vignette is not having symptoms during a specific class or
time of day.

References:

Liu AH, Covar RA, Spahn JD, Leung DYM. Childhood asthma. In: Kleigman RM, Behrman RE, Jenson
HB, Stanton BF, eds. Nelson Textbook of Pediatrics. 18th ed. Philadelphia, Pa: Saunders Elsevier;
2007:953-969

Weinberger M, Abu-Hasan M. Pseudo-asthma: when cough, wheezing, and dyspnea are not asthma.
Pediatrics. 2007;120:855-864. Available at:
http://pediatrics.aappublications.org/cgi/content/full/120/4/855

page 705
2009 PREP SA on CD-ROM

Question: 208
A 7-year-old boy is brought to the emergency department because of altered mental status. His parents
report that he was well when he came home from school today, but when he came in the house for
dinner after playing outside with his friends, he complained of abdominal pain and had an episode of
nonbilious and nonbloody emesis. Over the next 30 minutes, he became increasingly lethargic until his
parents could not arouse him. They called emergency medical services, and he was transported to the
emergency department by ambulance. On physical examination, he is unresponsive and drooling, his
temperature is 98.8°F (37.1°C), heart rate is 50 beats/min, respiratory rate is 36 breaths/min, blood
pressure is 100/60 mm Hg, and oxygen saturation is 82% on room air. His pupils are mid-size and
sluggishly reactive, and his breath sounds are coarse bilaterally, with increased work of breathing. You
suspect a toxin exposure.

Of the following, the MOST appropriate treatment of this patient is

A. atropine

B. N-acetylcysteine

C. naloxone

D. octreotide

E. physostigmine

page 706
2009 PREP SA on CD-ROM

Critique: 208 Preferred Response: A


The patient described in the vignette is exhibiting the classic symptoms of cholinergic poisoning. These
symptoms can be remembered using the mnemonic "SLUDGE" (salivation, lacrimation, urination,
diarrhea, gastric emesis) and are due to irreversible inhibition of acetylcholinesterase and excess
acetylcholine at the neuromuscular junction. The resulting overstimulation of cholinergic receptors
causes the muscarinic symptoms noted previously. Nicotinic symptoms, which include muscle
twitching, weakness, and paralysis, also may result. Organophosphate exposure can occur though
ingestion as well as dermal absorption and inhalation. These compounds are found commonly in the
home environment as components of lawn and garden care products, scabicides, and interior
insecticides.
The treatment of cholinergic poisoning involves patient stabilization, decontamination, and
administration of antidotes. Significantly symptomatic patients may require intubation for airway
protection and pulmonary toilet as well as assisted ventilation with 100% oxygen for respiratory muscle
weakness and hypoxia. Systemic decontamination with activated charcoal as well as dermal
decontamination should be performed. Dermal decontamination may be facilitated by cleansing the skin
with a dilute bleach solution. Contaminated clothes should be discarded because laundering may not
remove the toxin.
The mainstay of stabilization and treatment for cholinergic poisonings is the administration of
atropine. Atropine competes with acetylcholine at the cholinergic receptors and decreases the
muscarinic cholinergic effects. The doses of atropine used in this setting are higher than those used for
symptomatic bradycardia from other causes. An initial atropine dose of 0.05 mg/kg should be
administered and doubled every 3 to 5 minutes until the pulmonary muscarinic symptoms
(bronchorrhea, bronchospasm) are controlled. Nicotinic neuromuscular symptoms are treated by adding
pralidoxime, a cholinesterase reactivating agent, to atropine therapy.
N-acetylcysteine is used to treat acetaminophen poisoning, which causes few, if any, acute
symptoms. Naloxone is an opiate antagonist used to treat symptomatic opiate overdoses, which are
characterized by miosis, bradycardia, and respiratory depression without bronchorrhea. Octreotide is a
somatostatin analog that inhibits insulin release and is indicated in the treatment of sulfonylurea
overdoses, which cause profound hypoglycemia unresponsive to dextrose administration.
Physostigmine is a cholinergic agent that may be used in significantly symptomatic anticholinergic
poisonings. Because the patient's symptoms are not consistent with overdoses of any of these drugs,
treatment with these agents is not indicated.

References:

Bird S. Organophosphate and carbamate toxicity. UpToDate Online 15.3. 2008. Available for
subscription at:
http://www.utdol.com/utd/content/topic.do?topicKey=ad_tox/9425&selectedTitle=1~150&source=search
_result

Karr CJ, Solomon GM, Brock-Utne AC. Health effects of common home, lawn, and garden pesticides.
Pediatr Clin North Am. 2007;54:63-80. Abstract available at:
http://www.ncbi.nlm.nih.gov/pubmed/17306684

Peter JV, Moran JL, Graham PL. Advances in the management of organophosphate poisoning. Expert
Opin Pharmacother. 2007;8:1451-1464. Abstract available at:
http://www.ncbi.nlm.nih.gov/pubmed/17661728

page 707
2009 PREP SA on CD-ROM

Question: 209
You are evaluating a 2-day-old term infant because of abdominal distention. He fed normally the first
day after birth, but has had progressively increasing vomiting, which now is bilious. Physical
examination demonstrates upslanted palpebral fissures, a prominent tongue, and mild hypotonia. Upon
passage of a nasogastric tube, you aspirate 80 mL of green-yellow material from his stomach.
Abdominal radiographs, including a left lateral decubitus film, reveal dilated loops of bowel and air-fluid
levels but no evidence of pneumatosis (Item Q209).

Of the following, the condition that BEST explains this baby’s clinical findings is

A. duodenal atresia

B. Hirschsprung disease

C. meconium ileus

D. necrotizing enterocolitis

E. neonatal intussusception

page 708
2009 PREP SA on CD-ROM

Question: 209

(Courtesy of D. Mulvihill)

page 709
2009 PREP SA on CD-ROM

Critique: 209 Preferred Response: B


The infant described in the vignette has clinical features of Down syndrome (Item C209A). Infants who
have Down syndrome are at risk for a number of gastrointestinal malformations, most notably duodenal
atresia and Hirschsprung disease. The air in the distal small bowel apparent in the radiograph obtained
for the infant in the vignette excludes duodenal atresia and annular pancreas. The lack of pneumatosis
on the radiograph for this term infant makes necrotizing enterocolitis unlikely. Neonatal intussusception
is extremely rare, and meconium ileus is associated with cystic fibrosis, not Down syndrome.
Hirschsprung disease is characterized by congenital absence of a portion of the enteric nervous
system (aganglionosis). The aganglionic segment of bowel typically begins at the anal verge and
extends proximally. Disease limited to the rectosigmoid colon ("short segment" Hirschsprung) accounts
for 80% to 90% of cases. The remainder of cases can involve a larger portion of the distal colon ("long
segment Hirschsprung"), the entire colon ("total colonic Hirschsprung"), or the colon and small bowel.
The aganglionic segment of bowel is unable to contract, leading to either severe constipation or a
functional bowel obstruction.
It can be difficult to distinguish between functional constipation and Hirschsprung disease in
childhood. Some distinguishing features are shown in Item C209B. In general, patients who have
Hirschsprung disease have a history of delayed passage of meconium; 95% of affected infants fail to
pass meconium in the first day after birth. Stools of infants and toddlers who have Hirschsprung
disease often are very thin. Because of the hypertensive anal sphincter and aperistaltic distal colon,
affected children almost never have encopresis. On rectal examination, an infant or child who has short
segment Hirschsprung may have an "explosion of stool" after the examiner's finger is removed.
If Hirschsprung disease is suspected strongly, the patient should undergo further diagnostic testing.
Anorectal manometry demonstrates a hypertensive anal sphincter that fails to relax. Barium enema may
demonstrate a narrowed rectosigmoid (aganglionic segment) (Item C209C) and a dilated proximal
colon, with a "transition zone" (area of caliber change between the normal and aganglionic segment).
The definitive test for Hirschsprung disease is rectal biopsy, which demonstrates absent ganglion cells
in the submucosa and muscularis propria of the rectum. If Hirschsprung disease is identified, surgical
resection of the aganglionic colon and anastomosis of the normal colon to the anorectal canal is the
treatment of choice.

References:

de Lorijn F, Kremer LC, Reitsma JB, Benninga MA. Diagnostic tests in Hirschsprung disease: a
systematic review. J Pediatr Gastroenterol Nutr. 2006;42:496-505. Abstract available at:
http://www.ncbi.nlm.nih.gov/pubmed/16707970

Imseis E, Gariepy CE. Hirschsprung disease. In: Walker WA, Goulet O, Kleinman RE, Sherman PM,
Shneider BL, Sanderson IR, eds. Pediatric Gastrointestinal Disease. 4th ed. Hamilton, Ontario, Canada:
BC Decker; 2004:1031-1043

page 710
2009 PREP SA on CD-ROM

Critique: 209

Facial features of Down syndrome in infants include upslanted palpebral


fissures, a flat face, and a broad nasal root. (Courtesy of M. Rimsza)

page 711
2009 PREP SA on CD-ROM

Critique: 209

page 712
2009 PREP SA on CD-ROM

Critique: 209

Spot radiograph from a barium enema series in a patient who has


Hirschsprung disease showing a contracted rectum (representing the
aganglionic segment) with a short transition zone to a normal dilated proximal
rectum. (Courtesy of D. Mulvihill)

page 713
2009 PREP SA on CD-ROM

Question: 210
You are called to the neonatal intensive care unit to examine a newborn who has abdominal distention
and respiratory distress. She was born at 38 weeks’ gestation and weighs 4 kg. Apgar scores were 3
and 6 at 1 and 5 minutes, respectively. She required tracheal intubation and assisted ventilation. On
physical examination, she has a large, distended, and tense abdomen without bowel sounds. The
abdominal wall is not erythematous, and there is no clearly palpable mass. She does not display other
evidence of body wall or scalp edema. The breath sounds are coarse and equal bilaterally. There is no
heart murmur. Radiograph of the chest appears normal, but abdominal radiography shows background
granular density, paucity of intraluminal bowel gas, and a calcified mass in the left lower quadrant (Item
Q210).

Of the following, the BEST explanation for this infant’s abdominal findings is

A. congenital lymphangioma

B. erythroblastosis fetalis

C. meconium peritonitis

D. ovarian cyst

E. urinary ascites

page 714
2009 PREP SA on CD-ROM

Question: 210

(Courtesy of B. Carter)

page 715
2009 PREP SA on CD-ROM

Critique: 210 Preferred Response: C


The respiratory distress described for the infant in the vignette may be related to diaphragmatic
impingement from abdominal distention. Evaluation of the cause of abdominal distention includes a
plain radiograph that, in this case, has findings consistent with ascites and a calcified mass (Item
C210A). Subsequent abdominal ultrasonography may prove to be valuable diagnostically. The mass is
a meconium pseudocyst, indicative of a bowel rupture with contained (walled-off) meconium that has
become calcified. This condition, seen in meconium peritonitis, usually is associated with meconium
ileus and is a result of cystic fibrosis (CF).
An autosomal recessive disease, CF is believed to affect 1 in 2,000 to 4,000 live births of white
children. It occurs much less frequently in African Americans (estimated at 1 in 17,000 live births) and is
rare in infants of Asian descent (estimated at 1 in 90,000 live births). The disease is related to altered
epithelial cell ion transport and is associated with exocrine pancreatic insufficiency and pulmonary
decompensation in later life. More than 1,000 mutations (mapped to the 7th chromosome) have been
identified, but the most common defect is in a chloride channel transmembrane transport protein
regulated by the delta-F508 mutation, which accounts for more than 70% of all CF cases. Problems that
may present in the neonatal period include:
1. Meconium ileus, a thick meconium obstruction of the distal ileum characteristically presenting
clinically as a small bowel obstruction. This condition occurs in approximately 17% of infants who have
CF. A history may indicate polyhydramnios, a prenatal ultrasonographically observed bowel dilation, or
delayed postnatal passage of stool. Physical examination may reveal abdominal distention and
bile-stained emesis. A plain abdominal radiograph may show stacked loops of variably dilated bowel,
soap-suds bubbly-like appearance of meconium stool (most often in the right lower quadrant), and a
ground-glass character of the distal bowel with a relative paucity of distal bowel gas. A contrast enema
may reveal a microcolon and failure to pass contrast beyond the ileocecal valve. Surgical exploration
and removal of inspissated meconium is generally necessary, as may be a temporizing ileostomy and
later reanastamosis.
2. Meconium peritonitis is associated with bowel obstruction leading to perforation and spillage of
meconium into the peritoneal cavity, occasionally involving a walled-off calcified meconium
"pseudocyst" apparent on plain abdominal radiograph or abdominal ultrasonography. The perforation
may have occurred in utero and may be associated with fetal ascites or hydrops, pulmonary hypoplasia
and respiratory failure, and impaired bowel or liver function. The newborn may have ascites or anasarca
on physical examination, with a tense, distended abdomen. Plain abdominal radiography may reveal a
diffuse ground-glass density and little intraluminal bowel gas; there also may be diffuse or focal
calcification (Item C210B). Medical management is directed at stabilizing pulmonary function, fluid and
electrolyte balance, and hepatic function before addressing the surgical condition.
3. Prolonged jaundice with a predominance of conjugated hyperbilirubinemia also is seen in infants
who have CF and may result from inspissated bile or prolonged use of parenteral nutrition.
Congenital lymphangiomas may present with ascites and hydrothoraces requiring drainage; analysis
of the fluid confirms the diagnosis. Erythroblastosis fetalis is an isoimmune hemolytic condition that
involves fetal hydrops (neonatal anasarca) and profound anemia, but it is not related to bowel
obstruction and does not result in meconium peritonitis. Ovarian cysts may be massive, occasionally
resulting in bowel compression and extrinsic obstruction that can require surgical excision. They are
discernible by abdominal and pelvic ultrasonography as being distinct from the bowel lumen. Urinary
ascites develops following urinary tract obstruction, with perforation and leakage of urine into the
peritoneal space. It occurs most commonly in boys who have posterior urethral valves. Analysis of the
ascites fluid reveals an elevated creatinine concentration.

References:

page 716
2009 PREP SA on CD-ROM

Albanese CT, Sylvester KG. Pediatric surgery. In: Doherty GM, Way LW. Current Surgical Diagnosis
and Treatment. 12th ed. New York, NY: The McGraw-Hill Companies, Inc; 2006:chap 45

Chaudry G, Navarro OM, Levine DS, Oudjhane K. Abdominal manifestations of cystic fibrosis in
children. Pediatr Radiol. 2006;36:233-240. Abstract available at:
http://www.ncbi.nlm.nih.gov./pubmed/16391928

Davis PB. Cystic fibrosis. Pediatr Rev. 2001;22:257-264. Available at:


http://pedsinreview.aappublications.org/cgi/content/full/22/8/257

Kaye CI and the Committee on Genetics. Newborn screening fact sheets. Pediatrics.
2006;118:e934-e963. Available at: http://pediatrics.aappublications.org/cgi/content/full/118/3/e934

page 717
2009 PREP SA on CD-ROM

Critique: 210

Abdominal radiograph in meconium peritonitis reveals a hazy appearance to


the abdomen that suggests ascites and a calcified mass (arrow) representing
a meconium pseudocyst. (Courtesy of B. Carter)

page 718
2009 PREP SA on CD-ROM

Critique: 210

Anteroposterior radiograph of the chest and abdomen demonstrates a paucity


of gas in the abdomen, with scattered calcifications typical of meconium
peritonitis. (Courtesy of D. Mulvihill)

page 719
2009 PREP SA on CD-ROM

Question: 211
A 13-year-old girl comes to your office with a 1-day history of right eye pain and tearing. She denies
trauma, but says she rubbed her eyes a lot the day before because it was windy outside. Her right
bulbar and palpebral conjunctivae are very injected, and copious clear discharge is present. There is no
hyphema, and the pupils are normal. She complains of pain with the eye examination. After applying
fluorescein to the eye, you see a single linear abrasion on the cornea. When you evert the eyelid, you
find no foreign body.

Of the following, the MOST appropriate management for this condition is

A. oral analgesic

B. oral antistaphylococcal antibiotic

C. tight patching of the eye

D. topical anesthetic drops

E. topical steroid drops

page 720
2009 PREP SA on CD-ROM

Critique: 211 Preferred Response: A


Corneal abrasions occur commonly in children as a result of vigorous rubbing of the eye or a foreign
body. Symptoms are tearing, photophobia, and pain. The physical examination may reveal injection of
the conjunctivae, copious clear discharge, or a dull corneal light reflex. It is important to perform
fluorescein dye staining in children suspected of having a corneal abrasion. This dye stains the
damaged cornea but is not taken up by intact corneal epithelium. The staining is seen best with a Wood
lamp (Item C211), but large abrasions may be visible with regular white light or an ophthalmoscope.
The presence of a linear abrasion, as described for the girl in the vignette, suggests the possibility of a
retained foreign body underneath the eyelid, and care should be taken to evert the eyelid to evaluate for
this.
Topical anesthetic drops, such as tetracaine, may be used to facilitate the eye examination and may
provide significant relief to the patient, but providing these for home use is not recommended because
they may slow healing and mask persistence of symptoms. Oral analgesics usually are all that is
needed to control pain while the abrasion heals. Topical nonsteroidal anti-inflammatory drops also may
provide relief, but topical steroids are not indicated in the management of uncomplicated corneal
abrasions (due to concerns about adverse effects on wound healing and exacerbation of coexisting
viral infection). Antibiotic ointment may provide lubrication until the abrasion has healed, but the risk of
secondary infection is low, so routine use of antibiotics is not needed. Patching does not speed the
healing process and has been shown to interfere with activities of daily living; it no longer is
recommended for routine care of simple corneal abrasions.
Most corneal abrasions heal within 2 to 3 days, so if symptoms continue beyond this period,
evaluation by an ophthalmologist is warranted to rule out secondary infection or retained foreign body.

References:

Calder LA, Balasubramanian S, Fergusson D. Topical nonsteroidal anti-inflammatory drugs for corneal
abrasions: meta-analysis of randomized trials. Acad Emerg Med. 2005;12: 467-473. Abstract available
at: http://www.ncbi.nlm.nih.gov/pubmed/15860701

Michael JG, Hug D, Dowd MD. Management of corneal abrasion in children: a randomized clinical trial.
Ann Emerg Med. 2002;40:67-72. Abstract available at: http://www.ncbi.nlm.nih.gov/pubmed/12085075

Stout AU. Technical tip: corneal abrasions. Pediatr Rev. 2006;27:433-434. Available at:
http://pedsinreview.aappublications.org/cgi/content/full/27/11/433

Turner A, Rabiu M. Patching for corneal abrasion. Cochrane Database Syst Rev. 2006;2:CD004764.
Available at: http://www.mrw.interscience.wiley.com/cochrane/clsysrev/articles/CD004764/frame.html

page 721
2009 PREP SA on CD-ROM

Critique: 211

Corneal abrasion (arrow) demonstrated after flourescein staining and


examination with a Wood lamp. (Courtesy of Wake Forest University Eye
Center)

page 722
2009 PREP SA on CD-ROM

Question: 212
A previously healthy 15-year-old girl returns from summer camp in the mountains complaining of
dysuria, frequency, and urgency. You diagnose cystitis and prescribe trimethoprim-sulfamethoxazole.
Her mother phones 3 days later to report that the girl is very tired and appears pale. You advise her
mother to bring her to your office. On examination, she appears pale and your order laboratory tests.
The girl’s hemoglobin is 8.5 g/dL (85.0 g/L), a decrease from the value of 11.5 g/dL (115.0 g/L) that
was measured during her pre-camp physical examination. Her reticulocyte count is 5.0% (0.050), and
the red cell indices are normal except for mild microcytosis with a mean corpuscular volume of 76 fL.
You review a smear (Item Q212).

Of the following, the MOST likely cause of this girl’s rapid onset of anemia is

A. glucose-6-phosphate dehydrogenase deficiency

B. hemoglobin SC disease

C. hereditary elliptocytosis

D. inadequate dietary iron

E. pyelonephritis

page 723
2009 PREP SA on CD-ROM

Question: 212

(Courtesy of Gulati G, Caro J. Blood Cells - An Atlas of Morphology with


Clinical Relevance. Chicago, Ill: ASCP Press; 2007. ©2007, American Society
for Clinical Pathology.)

page 724
2009 PREP SA on CD-ROM

Critique: 212 Preferred Response: A


The girl described in the vignette has a peripheral blood smear and clinical signs and symptoms
consistent with hemolytic anemia following the administration of trimethoprim-sulfamethoxazole. In
people who have glucose-6-phosphatase dehydrogenase (G6PD) deficiency, hemolysis can occur after
the administration of sulfonamides and other antibiotics.
G6PD deficiency is the most common disease-producing enzymopathy in humans. Inherited as an
X-linked disorder, primarily as single-base mutations in the G6PD locus at Xq28, G6PD deficiency
affects 400 million people worldwide. The highest prevalence rates (with gene frequencies from 5% to
25%) are found in tropical Africa, the Middle East, tropical and subtropical Asia, some areas of the
Mediterranean, and Papua New Guinea. The defect occurs in approximately 13% of African American
males. Homozygous women are found in populations in which the frequency of G6PD deficiency is
high. Heterozygous (carrier) women may develop hemolytic attacks. The gene confers some protection
against malaria, which probably accounts for its high gene frequency in modern populations.
The G6PD enzyme catalyzes the oxidation of glucose-6-phosphate to 6-phosphogluconate while
reducing the oxidized form of nicotinamide adenine dinucleotide phosphate (NADP+) to nicotinamide
adenine dinucleotide phosphate (NADPH). NADPH, a required cofactor in many biosynthetic reactions,
maintains glutathione in its reduced form.
Reduced glutathione is a scavenger for dangerous oxidative metabolites in the cell. Red blood cells
depend on G6PD activity as the only protection against oxidative stresses. People deficient in G6PD,
therefore, are at risk for hemolysis and its sequelae when exposed to oxidative stress. The degree of
hemolysis varies with dose of the inciting agent.
The enzymatic defect in Americans of African descent is statistically less severe than that of people
of Mediterranean descent. The enzyme also is found in lower quantities in older red blood cells due to
senescence of the enzyme.
Affected individuals typically do not display symptoms or signs in the absence of oxidative stress.
However, those who have very low enzyme concentrations may present with neonatal jaundice and
acute hemolytic anemia. Neonatal jaundice usually appears by age 1 to 4 days, at the same time as or
slightly earlier than so-called physiologic jaundice; kernicterus is rare. Acute hemolytic anemia results
from stress factors such as oxidative drugs or chemicals, infection, or ingestion of fava beans (common
to a Mediterranean diet).
Gallstones may be a prominent feature in affected individuals. As with hemolysis of different causes,
jaundice and splenomegaly may be present during a crisis. Immediately after an episode, younger red
blood cells are released that contain a higher concentration of enzyme. Accordingly, testing should be
delayed in the event of undiagnosed hemolysis in children for a few weeks after the hemolysis to allow
G6PD to decrease to normal concentrations.
The clinician should have a high suspicion for G6PD deficiency in immigrants of the ethnic groups
noted previously and consider testing potentially affected males who exhibit hemolysis as part of a
significant illness (such as diabetic acidosis, hepatitis) or trauma, especially prior to administering
medications that may precipitate hemolysis. Precipitants include antibacterials (especially
sulfonamides), antimalarials (chloroquine, primaquine), and other medications (aspirin, vitamin K
analogs). Chemicals that may induce hemolysis include naphthalene, which is found in mothballs.
Neonatal screening and health education have been effective in some countries at detecting
individuals at risk and preventing hemolysis. Widespread screening of asymptomatic individuals is rare
in the United States because most affected individuals have only a mild form of the disease.
Hemoglobin SC disease and hereditary elliptocytosis are unlikely diagnoses in a previously healthy
15-year-old girl because these conditions are associated with clinical disease in early childhood. In
addition, hemoglobin SC disease often is diagnosed in the neonatal period because of the availability of
neonatal screening. Hereditary elliptocytosis (Item C212) also is detected in infancy and early childhood

page 725
2009 PREP SA on CD-ROM

based on hemolysis without stressors. Iron deficiency does not cause hemolysis or jaundice.
Pyelonephritis may result from ascending infection from cystitis but normally is associated with fever
and other constitutional symptoms in addition to fatigue. Hemolysis can result from infection in persons
unaffected by G6PD deficiency but usually not to the degree noted in the child described in the vignette.

References:

Carter SM, Gross SJ. Glucose-6-phosphate dehydrogenase deficiency. eMedicine Specialties,


Medicine, Ob/Gyn, Psychiatry, and Surgery, Hematology. 2005. Available at:
http://www.emedicine.com/med/topic900.htm

Frank JE. Diagnosis and management of G6PD deficiency. Am Fam Physician. 2005;72:1277-1282.
Available at: http://www.aafp.org/afp/20051001/1277.html

Segal GB. Enzymatic defects. In: Kliegman RM, Behrman RE, Jenson HB, Stanton BF, eds. Nelson
Textbook of Pediatrics. Philadelphia, Pa: Saunders Elsevier; 2007:2039-2041

Segel GB, Hirsh MG, Feig SA. Managing anemia in a pediatric office practice: part 2. Pediatr Rev.
2002;23:111-122. Available at: http://pedsinreview.aappublications.org/cgi/content/full/23/4/111

page 726
2009 PREP SA on CD-ROM

Critique: 212

Elliptocytes (oval-shaped red blood cells), as seen in hereditary elliptocytosis.


(Courtesy of Gulati G, Caro J. Blood Cells - An Atlas of Morphology with
Clinical Relevance. Chicago, Ill: ASCP Press; 2007. ©2007, American Society
for Clinical Pathology.)

page 727
2009 PREP SA on CD-ROM

Question: 213
A 13-year-old boy who has a bicuspid aortic valve and aortic stenosis with a 20-mm Hg (mild) gradient
by echocardiography (Item Q213) is interested in participating in sports. He asks for your advice.

Of the following, the BEST response is that

A. aerobic activities should be well tolerated

B. football is contraindicated

C. he should not participate in weight training

D. he should refrain from highly competitive sports

E. wrestling is contraindicated

page 728
2009 PREP SA on CD-ROM

Critique: 213 Preferred Response: A


Congenital heart disease is the most commonly encountered malformation, with an incidence of
approximately 8 in 1,000 live births or slightly less than 1%. Today, congenital heart disease is
diagnosed early, often in utero, and medical, catheter-based, and surgical management strategies have
progressed to a point that most affected children lead active and healthy lives. Exercise and sports
participation are healthy and normal components of many children's lives, including those affected by
congenital heart disease. Various professional organizations have recommended levels of sports
participation based on the underlying cardiac malformation and its degree of severity.
The most common congenital heart abnormality is a bicommissural (bicuspid) aortic valve (Item
C213), as described for the boy in the vignette. Although many of the children who have bicommissural
valves do not have associated aortic stenosis during childhood, it is the most common cause of aortic
stenosis in children. The pressure gradient across the aortic valve during systole determines the
severity of the stenosis. Pressure gradients that are less than 25 mm Hg generally are considered mild,
those between 25 and 50 mm Hg are considered moderate, and those in excess of 50 mm Hg are
considered severe. Patients who have aortic stenosis should be monitored regularly, with
electrocardiography and echocardiography supplementing a thorough history and physical examination
because aortic stenosis can progress with patient growth and age. Patients who have mild aortic
stenosis, such as the boy described in the vignette, generally have no restrictions on physical activity
and exercise. Children who have moderate aortic stenosis may participate in self-limited aerobic
activities and should avoid isometric activities such as weightlifting and wrestling, which increase the
systemic afterload and workload of the left ventricle.
The boy in the vignette should be able to play football or any other competitive sport if the remainder
of his physical examination and cardiac evaluation is benign. He also may participate in weight training,
particularly the lighter weights that build endurance and muscle speed rather than the heavier weights
that build large muscle bulk. Wrestling is not contraindicated for children who have asymptomatic mild
aortic stenosis, although it may be in patients who have moderate aortic stenosis.

References:

Dickhuth H-H, Kececioglu D, Schumacher YO. FIMS Position Statement: Congenital Heart Disease and
Sports. International Federation of Sports Medicine; January 2006. Available at:
http://www.fims.org/default.asp?PageID=120975716

Maron BJ, Chaitman BR, Ackerman MJ, et al. Recommendations for physical activity and recreational
sports participation for young patients with genetic cardiovascular diseases. Circulation.
2004;109:2807-2816. Available at: http://circ.ahajournals.org/cgi/content/full/109/22/2807

Singh A, Silberbach M. Consultation with the specialist: cardiovascular preparticipation sports


screening. Pediatr Rev. 2006;27:418-424. Available at:
http://pedsinreview.aappublications.org/cgi/content/full/27/11/418

Stefani L, Galanti G, Tonicelli L, et al. Bicuspid aortic valve in competitive athletes. Br J Sports Med.
2008;42:31-35. Abstract available at: http://www.ncbi.nlm.nih.gov/pubmed/17548371

page 729
2009 PREP SA on CD-ROM

Critique: 213

In a bicuspid aortic valve, one of the commisures fails to form (arrow),


resulting in two rather than three leaflets. (Courtesy of P Lynch)

page 730
2009 PREP SA on CD-ROM

Question: 214
The mother of a 7-year-old girl who has epilepsy phones your office because her child has developed a
rash. The mother is worried that the rash may be due to her new antiseizure medication.

Of the following, the MOST appropriate next step is to

A. defer evaluation until the next health supervision visit

B. examine the child in your office promptly

C. refer the child to a dermatologist

D. refer the child to her neurologist

E. send the child to the laboratory to obtain a complete blood count

page 731
2009 PREP SA on CD-ROM

Critique: 214 Preferred Response: B


Antiseizure medications can cause a wide variety of adverse effects, including central nervous system
effects such as sedation, dizziness, personality changes, and occasionally, worsening of seizures. The
neurologist, psychiatrist, or other physician who prescribes such medications should be familiar with
adverse effect profiles of these medications. Adverse effects of antiseizure medications may occur
outside the central nervous system as well. Bone marrow suppression and liver toxicity are among the
more serious possible systemic effects.
Rashes are common in children and can occur as an adverse effect of most antiseizure medications.
Such drug rashes tend to occur during the first month of use, as described for the girl in the vignette.
Rapid assessment of the child is important because in some cases, the rash may be a harbinger of a
more global, serious allergic reaction or of Stevens-Johnson syndrome (erythema multiforme major)
(Item C214). A general pediatrician often is better trained than a neurologist in the identification of the
many rashes that can occur in childhood. Therefore, prompt assessment should be undertaken by the
primary care physician in the office. A dermatologist referral may be needed, and laboratory testing may
be useful, but these decisions should be made after the clinical assessment by the child's primary care
physician.

References:

French JA, Kanner AM, Bautista J, et al. Efficacy and tolerability of the new antiepileptic drugs I:
treatment of new onset epilepsy. Report of the Therapeutics and Technology Assessment
Subcommittee and Quality Standards Subcommittee of the American Academy of Neurology and the
American Epilepsy Society. Neurology. 2004;62:1252-1260. Available at:
http://www.neurology.org/cgi/content/full/62/8/1252

Johnston MV. Seizures in childhood. In: Kliegman RM, Behrman RE, Jenson HB, Stanton BF, eds.
Nelson Textbook of Pediatrics. 18th ed. Philadelphia, Pa: Saunders Elsevier; 2007:2457-2475

page 732
2009 PREP SA on CD-ROM

Critique: 214

Stevens-Johnson syndrome due to lamotrigine: There is involvement of


conjunctivae and oral mucosa as well as erythematous eroded patches.
(Courtesy of M Rimsza)

page 733
2009 PREP SA on CD-ROM

Question: 215
A newborn male experiences prolonged oozing following circumcision. Hematologic evaluation reveals
that he has less than 1% of factor VIII clotting activity and a prolonged partial thromboplastin time,
consistent with severe hemophilia A. His family history is negative for any individuals affected by
clotting disorders.

Of the following, the MOST accurate statement for counseling this child’s parents is that

A. another family member likely is affected, but the condition is so mild that the person has not been
diagnosed

B. in families such as this, 50% of affected boys have a spontaneous gene mutation

C. molecular genetic testing can detect mutations in 50% of affected individuals

D. severe hemophilia is the rarest of all types

E. there is an 80% chance that the mother is a hemophilia carrier

page 734
2009 PREP SA on CD-ROM

Critique: 215 Preferred Response: E


One third to one half of all males who have hemophilia A have no family history of the disorder, as
described for the infant in the vignette. In these cases, it is important to recognize that multiple genetic
scenarios can result in such an outcome.
In about 20% of the cases involving no previous family history for hemophilia, the mother is not a
carrier, in which case her son has a de novo disease-causing mutation. In some of these situations, the
son is a somatic mosaic for the mutation, ie, the mutation is not present in all of the cells of his body.
In 80% of such cases, the mother is a carrier, and she might have a de novo gene mutation. This
mutation could have occurred in the egg or sperm cell from which she was conceived (in which case it
is present in all of her cells). Alternatively, it may be due to a somatic mutation that occurred in early
embryogenesis or to germline mosaicism, in which case it is present in only some of her germ cells.
The mother could have inherited the disease-causing mutation from her mother or unaffected father (if
he is a somatic or germline mosaic). Finally, the mother could have inherited the mutation from a
previous generation, which may have been passed on only through daughters, for example, so that no
one was affected with hemophilia A.
Cases in which the mutation for hemophilia A is present in a mosaic state are unusual, and the gene
mutation runs true in families. Therefore, every boy in a family who has the same mutation will have the
same severity of disease.
Severe hemophilia (<1% normal factor VIII activity) is the most common of all types of hemophilia A.
Molecular genetic testing can detect mutations in 98% of those who have severe disease.

References:

Hamosh A. Clinical case studies illustrating genetic principles. In: Nussbaum RL, McInnes RR, Willard
HR, eds. Thompson & Thompson Genetics in Medicine. 7th ed. Philadelphia, Pa: Elsevier Saunders;
2007:268-269

Johnson MJ, Thompson AR. Hemophilia A. GeneReviews. 2005. Available at:


http://www.geneclinics.org/servlet/access?db=geneclinics&site=gt&id=8888891&key=xAcWBcrjmZrVo&
gry=&fcn=y&fw=8cy5&filename=/profiles/hemo-a/index.html

page 735
2009 PREP SA on CD-ROM

Question: 216
An 18-year-old young woman reports that she has "bumps" in her vaginal area. She recently became
sexually active with a single partner. She says that the lesions are not tender, and she has no vaginal
discharge or itching. Genital examination reveals several clusters of flesh-colored, pedunculated
lesions, primarily in the posterior fourchette, compatible with genital warts (Item Q216). You counsel her
about treatment options.

Of the following, the MOST accurate statement regarding management and treatment of genital warts is
that

A. human papillomavirus infectivity is eradicated by treatment of external warts

B. no definitive evidence supports the superiority of any of the available genital wart treatments

C. single treatment with clinician- or patient-applied methods eradicates all lesions in most patients

D. waiting for spontaneous resolution of warts is not acceptable

E. with her lesions, the patient is not currently a candidate for the human papillomavirus vaccine

page 736
2009 PREP SA on CD-ROM

Question: 216

Papules (arrow), as described for the patient in the vignette. (Courtesy of M.


Rimsza)

page 737
2009 PREP SA on CD-ROM

Critique: 216 Preferred Response: B


Human papillomavirus (HPV) is the cause of genital warts, which usually appear as flat, papular, or
pedunculated lesions of the genital mucosa (Item C216). Research data indicate that currently available
therapies for the treatment of genital warts might reduce, but probably do not eradicate, HPV infectivity.
When no treatment is given, genital warts may resolve spontaneously, remain unchanged, or increase
in number or size. An acceptable alternative for some patients is to forego treatment and wait for
spontaneous resolution. No definitive evidence suggests that any of the available treatments, including
topical patient- or clinician-applied medications (podofilox, imiquimod, podophyllin resin, or bichloro- or
trichloroacetic acid), cryotherapy, surgical removal, intralesional interferon, or laser therapy, is superior
to any other. Most patients require a course of therapy rather than a single treatment to eradicate warts.
Vaccination against HPV still is recommended for a patient who has external warts because the vaccine
can protect against other strains of HPV to which the patient may not have been exposed.

References:

Markowitz LE, Dunne EF, Saraiya M, Lawson HW, Chesson H, Unger ER. Quadrivalent human
papillomavirus vaccine: recommendations of the Advisory Committee on Immunization Practices
(ACIP). MMWR Recomm Rep. 2007;56(RR02):1-24. Available at:
http://www.cdc.gov/mmwr/preview/mmwrhtml/rr5602a1.htm

Workowski KA, Berman SM, Centers for Disease Control and Prevention. sexually transmitted diseases
treatment guidelines, 2006. MMWR Recomm Rep. 2006;55(RR11):1-94. Available at:
http://www.cdc.gov/mmwr/preview/mmwrhtml/rr5511a1.htm

page 738
2009 PREP SA on CD-ROM

Critique: 216

Filiform papules characteristic of human papillomavirus infection on mucosal


surfaces. (Courtesy of M. Rimsza)

page 739
2009 PREP SA on CD-ROM

Question: 217
A 2-year-old girl is brought to the emergency department after being found unconscious at her
grandparent’s home. Her mother reports that she was in her usual good health when she was dropped
off at her grandparents 2 hours ago and that there is no history of trauma. Of note, the grandmother
found a spilled, opened bottle of her "blood pressure" medicine in the bathroom. On physical
examination, the girl is somnolent but arouses with stimulation. There is no sign of trauma on physical
examination. Her temperature is 98.0°F (37.0°C), heart rate is 60 beats/min, respiratory rate is 25
breaths/min, and oxygen saturation is 93% on room air. Her pupils are 2 mm and reactive bilaterally.
Her mouth and mucous membranes are dry, and she has no rashes. You order serum electrolyte
measurement and a urine toxicology screen.

Of the following, the MOST appropriate additional tests to obtain are

A. chest radiography and electrocardiography

B. computed tomography (CT) scan of the abdomen and electrocardiography

C. CT scan of the head and electrocardiography

D. CT scan of the head and lumbar puncture

E. electrocardiography and skull radiographs

page 740
2009 PREP SA on CD-ROM

Critique: 217 Preferred Response: C


A child who has an altered level of consciousness can present a diagnostic dilemma. Patient age, a
clear history of the present illness, and a comprehensive past medical history can help guide the
diagnostic evaluation and narrow the differential diagnosis. A useful guide for remembering potential
causes of an altered level of consciousness is the mnemonic AEIOU TIPS (tips on the vowels) (Item
C217).
Toxic ingestions are a common cause of an acute altered level of consciousness in a toddler. Serum
and urine toxicology screens should be performed if the ingestion is not witnessed because multiple
drugs may have been ingested. The previously healthy child described in the vignette has signs and
symptoms consistent with a clonidine overdose, as evidenced by her being awakened with stimulation
and having dry mucous membranes and pupillary constriction. The onset of action of this drug is less
than 1 hour, and toxic effects include neurologic and respiratory depression, hypotonia, bradycardia,
hypotension, and cardiac arrhythmias. Treatment is supportive, with both intensive care unit monitoring
and baseline electrocardiography indicated. In this case, CT scan of the head also is indicated because
the child might have had an unwitnessed fall.
Primary pulmonary processes rarely present as an altered level of consciousness unless there is an
associated oxygen deficiency. Therefore, chest radiography is not necessary in this case. Abdominal
CT scan is indicated if abdominal trauma or an intra-abdominal infectious process is suspected. Lumbar
puncture should be undertaken if meningitis is suspected, but the child in the vignette is afebrile and
was reported in good health just 2 hours ago. Skull radiography has limited utility in the evaluation of an
altered level of consciousness.

References:

Avner JR. Altered states of consciousness. Pediatr Rev. 2006:27:331-338. Available at:
http://pedsinreview.aappublications.org/cgi/content/full/27/9/331

Frankel LR. Neurological emergencies and stabilization. In: Kliegman RM, Behrman RE, Jenson HB,
Stanton BF, eds. Nelson Textbook of Pediatrics. 18th ed. Philadelphia, Pa: Saunders Elsevier:
2007:405-412

page 741
2009 PREP SA on CD-ROM

Critique: 217

page 742
2009 PREP SA on CD-ROM

Question: 218
The parents of a 6-year-old boy are concerned because he has been developing pubic hair over the
past 6 months. On physical examination, you note a recent growth spurt, Sexual Maturity Rating 3 pubic
hair, a penis that is 8 cm in length and androgenized, and testes that are 5 mL in volume. Other findings
are normal. His bone age is 7 years. You order measurements of serum testosterone,
17-hydroxyprogesterone, dehydroepiandrosterone, luteinizing hormone, and follicle-stimulating
hormone.

Of the following, the MOST important additional test is measurement of serum

A. adrenocorticotropic hormone

B. estradiol

C. free testosterone

D. human chorionic gonadotropin

E. prolactin

page 743
2009 PREP SA on CD-ROM

Critique: 218 Preferred Response: D


The child described in the vignette has sexual precocity, with testes that are increased in volume and
definite evidence of increased phallus size and pubic hair. Therefore, it is likely that the increased
androgen is being produced by the child's testes. Such production could be related to autonomous
testicular functioning, as in "testitoxicosis," or gonadotropin-independent sexual precocity, but it is more
likely related to testicular stimulation by gonadotropins. With true central sexual precocity, luteinizing
hormone (LH), follicle-stimulating hormone (FSH), and circulating concentrations of testosterone are
elevated. However, testosterone also could be produced if the testes are stimulated by human chorionic
gonadotropin (HCG), which can mimic LH and stimulate growth of the Leydig cells that produce
testosterone. HCG may be produced by germ cell or other tumors located in the central nervous
system, the mediastinum, the liver (hepatoblastomas), and other locations. This hormone should be
measured specifically in the blood as beta-HCG. Elevation of HCG concentrations causes pubertal
change in boys but not in girls because girls require LH and FSH to stimulate ovarian estrogen
production. Boys who have sexual precocity as a result of HCG secretion have a smaller testicular
volume than expected for pubertal stage because FSH-stimulated Sertoli cell numbers do not increase.
Measurement of adrenocorticotropic hormone is not useful because concentrations of this hormone
fluctuate with stress and are elevated persistently only in the presence of Cushing disease or adrenal
insufficiency, not precocious puberty. Serum estradiol values are likely to be slightly elevated because
testosterone is converted to estrogen peripherally in fat and in the liver, so if testosterone or other
androgen concentrations are elevated, so are estrogens. Estrogen values in this child are likely to be
low because no breast enlargement is reported. Measurement of free testosterone is unlikely to be
useful in a child whose physical examination reveals so much androgen effect. Free testosterone
measurement may be useful to examine androgen effect when normal testosterone concentrations are
associated with mild clinical hyperandrogenism in women. Serum prolactin does not stimulate
production of androgen in boys. Sometimes, prolactinomas are associated with mild hyperandrogenism
and irregular menses or amenorrhea in women.

References:

Ferry RJ Jr. Precocious pseudopuberty. eMedicine Specialties, Pediatrics: General Medicine,


Endocrinology. 2007. Available at: http://www.emedicine.com/ped/topic1881.htm

Kaplowitz P. Precocious puberty. eMedicine Specialties, Pediatrics: General Medicine, Endocrinology.


2007. Available at: http://www.emedicine.com/ped/topic1882.htm

Muir A. Precocious puberty. Pediatr Rev. 2006;27:373-381. Available at:


http://pedsinreview.aappublications.org/cgi/content/full/27/10/373

Rivarola MA, Belgorsky A, Mendilaharzu H, Vidal G. Precocious puberty in children with tumours of the
suprasellar and pineal areas: organic central precocious puberty. Acta Paediatr. 2001;90:751-756.
Abstract available at:
http://www.blackwell-synergy.com/doi/abs/10.1111/j.1651-2227.2001.tb02800.x?journalCode=apa

Saenger P. Overview of precocious puberty. UpToDate Online 15.3. 2008. Available for subscription at:
http://www.uptodateonline.com/utd/content/topic.do?topicKey=pediendo/14867

page 744
2009 PREP SA on CD-ROM

Question: 219
During the health supervision visit for an infant, her mother mentions that the child has been tolerating
solid foods with no problem. When placed on her back to be examined, she brings her feet to her
mouth. Her mother holds a small mirror to the child’s face to distract her during your examination, and
the baby reaches for the mirror and pats her image.

Of the following, these developmental milestones are MOST typical for an infant whose age is

A. 2 months

B. 4 months

C. 6 months

D. 9 months

E. 12 months

page 745
2009 PREP SA on CD-ROM

Critique: 219 Preferred Response: C


Tolerating solid foods well, placing her feet to her mouth while supine, and reaching for a mirror and
patting the image are socioemotional milestones most typical of a 6-month-old child. At 2 months of
age, infants bring their hands to their mouths, swipe at dangling objects with their hands, and grasp and
shake hand toys. A 4-month-old infant has a spontaneous social smile and, when shown a mirror,
smiles and vocalizes. An infant of this age also brings items to his or her mouth, can maintain an
upright position if placed upright with the assistance of pillows, and knows the difference between
strangers and family. A 9-month-old infant can imitate nursery games in response to a mother's
demonstrations and may give a toy in response to a request. A 12-month-old infant points for requests,
may throw a toy in play or refusal, helps with dressing, may hug a stuffed animal, and may offer a toy to
an image in a mirror.

References:

Knobloch H, Stevens FM, Malone AF. The revised developmental stages. In: Manual of Developmental
Diagnosis: The Administration and Interpretation of the Revised Gesell and Amatruda Developmental
and Neurologic Examination. Albany, NY: Developmental Evaluation Materials, Inc; 1987:17-120

Whitaker T, Palmer F. The developmental history. In: Accardo PJ. Capute & Accardo's
Neurodevelopmental Disabilities in Infancy and Childhood. Volume I: Neurodevelopmental Diagnosis
and Treatment. 3rd ed. Baltimore, Md: Paul H. Brookes Publishing Co; 2008:297-310

page 746
2009 PREP SA on CD-ROM

Question: 220
You admitted a patient to the hospital yesterday who had acute onset of fever (temperature of 103.0oF
[39.4oC]), a petechial rash, meningismus, and shock. She required blood pressure support and
mechanical ventilation during the night. As per the protocol for your hospital, you placed this child into
respiratory isolation upon admission. Today you are told that her blood culture is growing Neisseria
meningitidis. The nurse taking care of her asks you how long the child needs to remain in respiratory
isolation.

Of the following, the BEST answer is until the child

A. completes 1 day of antimicrobial therapy

B. defervesces

C. is clinically stable

D. is extubated

E. is proven not to have meningitis

page 747
2009 PREP SA on CD-ROM

Critique: 220 Preferred Response: A


Standard isolation precautions should be observed for patients who have meningococcal disease, and
they should remain in respiratory isolation until 24 hours after the initiation of effective antimicrobial
therapy. The presence of continued fever, prolonged intubation, clinical instability, or meningitis is
irrelevant to the issue of isolation. Patients treated for invasive disease with any antimicrobial agent
other than cefotaxime or ceftriaxone also must receive 2 days of rifampin, one dose of ceftriaxone, or a
single dose of ciprofloxacin prior to hospital discharge to eradicate the nasopharyngeal carriage of
Neisseria meningitidis. Family members and individuals who have had close contact with the patient
also should be given chemoprophylaxis. Exposed individuals who have received either the
polysaccharide or conjugate meningococcal vaccine still should receive chemoprophylaxis because the
vaccine may not contain the serotype that is specific for the patient's illness (eg, serotype B).
Invasive disease caused by N meningitidis usually presents in one of three forms. Meningococcemia
is characterized by symptoms suggestive of an upper respiratory tract infection and complaints of
headache, lethargy, myalgias, or joint pain. Within hours, a petechial or purpuric rash (Item C220) may
develop, as might circulatory collapse. Meningococcal meningitis often begins with similar symptoms to
meningococcemia, but affected children become irritable and have signs of meningeal irritation on
physical examination. Occult bacteremia with N meningitidis also has been described. Affected children
present with a febrile illness, and a blood culture grows N meningitidis, but the children never progress
to the clinical picture of fulminant meningococcemia.

References:

American Academy of Pediatrics. Meningococcal infections. In: Pickering LK, Baker CJ, Long SS,
McMillan JA, eds. Red Book: 2006 Report of the Committee on Infectious Diseases. 27th ed. Elk Grove
Village, Ill: American Academy of Pediatrics; 2006:452-460

Bilukha OO, Rosenstein N. Prevention and control of meningococcal disease: recommendations of the
Advisory Committee on Immunization Practices (ACIP). MMWR Recomm Rep. 2005;54(RR07):1-21.
Available at: http://www.cdc.gov/mmwr/preview/mmwrhtml/rr5407a1.htm

Hugo B. Index of suspicion: case 4. Pediatr Rev. 2005;26:295-301. Available at:


http://pedsinreview.aappublications.org/cgi/content/full/26/8/295

Notice to readers: revised recommendations of the Advisory Committee on Immunization Practices to


vaccinate all persons aged 11-18 years with meningococcal conjugate vaccine. MMWR Morbid Mortal
Wkly Rep. 2007;56:794-795. Available at: http://www.cdc.gov/mmwr/preview/mmwrhtml/mm5631a3.htm

page 748
2009 PREP SA on CD-ROM

Critique: 220

Meningococcemia is characterized by a petechial and purpuric eruption.


(Courtesy of G. Schutze)

page 749
2009 PREP SA on CD-ROM

Question: 221
You are speaking to the mother of a previously healthy boy who has just broken out with chickenpox
lesions. His mother states that one of her son’s classmates also has the disease. No one else in the
household is ill. He did not receive varicella vaccine, but all of his other immunizations are up to date.
His mother asks whether her son is at risk for developing a severe case of the disease.

Of the following, the factor that is MOST likely to increase his risk for moderate-to-severe varicella
disease is

A. age younger than 12 years

B. being a secondary case in the household

C. concurrent ear infection

D. receipt of a recent course of antibiotics

E. receipt of short-term salicylate therapy

page 750
2009 PREP SA on CD-ROM

Critique: 221 Preferred Response: B


Chickenpox or varicella infection is caused by the varicella-zoster virus. Infection tends to be more
severe in adolescents and adults and in those who have immunocompromising conditions.
Complications may include bacterial superinfection of the skin lesions (Item C221A), pneumonia (Item
C221B), central nervous system involvement (cerebellar ataxia, encephalitis), thrombocytopenia,
glomerulonephritis, arthritis, and hepatitis. The decision of whether a patient should receive antiviral
therapy, the route by which it is administered, and the duration of therapy is determined by specific host
factors, the extent of infection, the timing of exposure, and initial response to therapy.
Oral acyclovir is not recommended for routine use in healthy children who have varicella. However,
oral acyclovir should be considered in immunocompetent hosts who are at increased risk for
moderate-to-severe varicella. Persons who fall into this category include: those who are older than 12
years of age; those who have chronic cutaneous or pulmonary disorders; persons receiving long-term
salicylate therapy; people receiving short, intermittent, or aerosolized courses of corticosteroids; and
secondary household cases (disease usually is more severe than in the primary case). The factor that
would increase the risk for developing moderate-to-severe varicella disease for the patient described in
the vignette is being a secondary case in the household. His age, ear infection, or receipt of antibiotics
or short-term salicylate therapy is not associated with increased risk for moderate-to-severe disease.
Intravenous antiviral therapy is recommended for immunocompromised patients who have varicella,
including patients being treated with chronic corticosteroids. To maximize efficacy, therapy should be
initiated early in the course of the illness, preferably within 24 hours of rash onset. Oral acyclovir
generally should not be used to treat immunocompromised children who have varicella because of poor
oral bioavailability. However, high-dose oral acyclovir therapy may be used in selected
immunocompromised patients who are believed to be at lower risk of developing severe varicella. Such
patients include those infected with human immunodeficiency virus who have relatively normal
concentrations of CD4+ lymphocytes and children who have leukemia in whom careful follow-up is
guaranteed. Varicella-zoster immune globulin, if available, or intravenous immunoglobulin administered
shortly after exposure can prevent or modify the course of varicella disease, but immune globulin
preparations are not effective once disease is established.

References:

American Academy of Pediatrics. Varicella-zoster infections. In: Pickering LK, Baker CJ, Long SS,
McMillan JA, eds. Red Book: 2006 Report of the Committee on Infectious Diseases. 27th ed. Elk Grove
Village, Ill: American Academy of Pediatrics; 2006:711-725

Arvin AM. Antiviral therapy for varicella and herpes zoster. Semin Pediatr Infect Dis. 2002;13:12-21.
Abstract available at: http://www.ncbi.nlm.nih.gov/pubmed/12118839

Balfour HH Jr, Rotbart HA, Feldman S, et al. Acyclovir treatment of varicella in otherwise healthy
adolescents. The Collaborative Acyclovir Varicella Study Group. J Pediatr. 1992;120:627-633. Abstract
available at: http://www.ncbi.nlm.nih.gov/pubmed/1313098

Whitley RJ. Approaches to the treatment of varicella-zoster virus infections. Contrib Microbiol.
1999;3:158-172

Whitley RJ. Therapeutic approaches to varicella-zoster virus infections. J Infect Dis. 1992;166(suppl
1):S51-S57. Abstract available at: http://www.ncbi.nlm.nih.gov/pubmed/1378081

page 751
2009 PREP SA on CD-ROM

Critique: 221

Lesions of varicella may become infected secondarily with Streptococcus


pyogenes or Staphylococcus aureus. (Reprinted with permission from
English R. Varicella. Pediatr Rev. 2003;24:372-379. Photo courtesy of J. Zenel.)

page 752
2009 PREP SA on CD-ROM

Critique: 221

The chest radiograph in varicella pneumonia is characterized by bilateral


infiltrates that, in the early stages, may have a nodular component (arrow).
(Courtesy of the Red Book® Online.)

page 753
2009 PREP SA on CD-ROM

Question: 222
A father brings in his 8-year-old son because the boy has been "feeling tired" for the past few weeks.
The remainder of the history is unremarkable. His weight is at the 5th percentile, height is less than the
5th percentile, temperature is 98.6°F (37°C), heart rate is 88 beats/min, respiratory rate is 16
breaths/min, and blood pressure is 124/84 mm Hg. Urinalysis findings include a specific gravity of
1.005, pH of 6.5, no blood, and 2+ protein. Other laboratory results are:
· Sodium, 134.0 mEq/L (134.0 mmol/L)
· Potassium, 5.4 mEq/L (5.4 mmol/L)
· Chloride, 96.0 mEq/L (96.0 mmol/L)
· Bicarbonate, 14.0 mEq/L (14.0 mmol/L)
· Blood urea nitrogen, 96.0 mg/dL (34.3 mmol/L)
· Creatinine, 8.4 mg/dL (742.6 mcmol/L)

Of the following, the MOST likely additional finding expected for this child is

A. enlarged kidneys on ultrasonography

B. hypomagnesemia

C. increased parathyroid hormone concentration

D. low insulin-like growth factor I concentration

E. reticulocytosis

page 754
2009 PREP SA on CD-ROM

Critique: 222 Preferred Response: C


The boy described in the vignette has fatigue, growth failure, and elevated blood urea nitrogen (BUN)
and creatinine values, which are consistent with a diagnosis of chronic renal failure (CRF). CRF in
children is characterized by the marked loss of excretory function. The normal glomerular filtration rate
(GFR) is 100 to 120 mL/min per 1.73 m2, and children who have severe renal impairment should be
considered for renal replacement therapy (dialysis or transplantation) when the GFR approaches 20
mL/min per 1.73 m2. Prior to reaching this degree of renal dysfunction, other abnormalities occur due to
impaired renal excretory function, reduced functional nephron mass, and impaired endocrine function.
Because of the CRF, affected children often have poor appetites and impaired growth, leading to
failure to thrive and short stature in those who have long-standing disease. More specifically, affected
children actually have elevated growth hormone and insulin-like growth factor (IGF) I concentrations,
but because of high concentrations of IGF-binding proteins, the bioavailability is reduced, resulting in
abnormal growth. Decreased urine output with impaired excretion of sodium and water can lead to
hypertension. Other minerals, including phosphorus, potassium, and magnesium, tend to accumulate in
renal failure. With decreased nephron mass, 25-hydroxyvitamin D3 exiting the liver cannot be activated
in the kidney to 1,25-dihydoxyvitamin D3 due to low concentrations of 1-alpha hydroxylase activity. Low
bioactive forms of vitamin D3 accompanied by phosphorus retention and secondary
hyperparathyroidism lead to renal osteodystrophy (renal rickets) (Item C222). The low erythropoietin
values that are seen in advanced renal disease (usually when the GFR is less than 35 mL/min per 1.73
m2) lead to anemia associated with a low reticulocyte count.
Although children who have acute renal failure often have inflamed kidneys that are enlarged on
ultrasonography, children who have CRF usually have small, shrunken kidneys on ultrasonography.
Thus, kidney size sometimes can help distinguish between acute and chronic renal failure.

References:

Fine RN, Whyte DA, Boydstun II. Conservative management of chronic renal insufficiency. In: Avner
ED, Harmon WE, Niaudet P, eds. Pediatric Nephrology. 5th ed. Philadelphia, Pa: Lippincott Williams &
Wilkins; 2004:1291-1311

Wong CS, Mak RH. Chronic kidney disease. In: Kher KK, Schnaper HW, Makker SP, eds. Clinical
Pediatric Nephrology. 2nd ed. London, England: Informa Healthcare; 2007:339-352

page 755
2009 PREP SA on CD-ROM

Critique: 222

Anteroposterior radiograph of the knees shows diffuse demineralization of


both cortical and trabecular bone and fraying of the metaphyses. (Courtesy of
D. Mulvihill)

page 756
2009 PREP SA on CD-ROM

Question: 223
A 4-year-old girl presents with a 2-week history of bilateral discolored rhinorrhea, nasal congestion, and
decreased oral intake. Her mother states that at the onset of this illness, she developed clear rhinorrhea
2 days after attending child care. Despite using over-the-counter antihistamines and decongestants, the
child’s symptoms have persisted. A quick review of her chart shows that her immunizations are up to
date, including her pneumococcal conjugate vaccine series and her annual influenza vaccination. On
physical examination, the child has appropriate vital signs for her age, infraorbital edema bilaterally, and
yellowish mucus in her nares. You suspect acute bacterial rhinosinusitis (ABRS) and discuss evaluation
and treatment options with the mother.

Of the following, a TRUE statement regarding this child’s condition is that

A. a sinus radiograph should be performed prior to initiating antibiotic therapy for ABRS

B. ABRS can be distinguished easily from a viral upper respiratory tract infection

C. allergic rhinitis is the most common risk factor for developing ABRS

D. the gold standard test for organism identification in ABRS is a nasal swab culture

E. the most likely bacterial pathogen is Haemophilus influenzae

page 757
2009 PREP SA on CD-ROM

Critique: 223 Preferred Response: E


Acute bacterial rhinosinusitis (ABRS) is defined as sinusitis symptoms lasting for fewer than 4 weeks.
Common symptoms include purulent nasal discharge and sinus pressure. Because the symptoms of
ABRS are difficult to distinguish from a viral upper respiratory tract infection (URI), symptoms of a viral
URI that persist past 7 to 10 days, as described for the girl in the vignette, generally are regarded as
indicative of bacterial sinusitis. Viral URI is the most common preceding condition for ABRS, but allergic
rhinitis is a precipitant in approximately 20% of cases.
In 2001, the American Academy of Pediatrics published management guidelines for the evaluation
and treatment of ABRS. Because the initial symptoms of ABRS usually are indistinguishable from a
viral URI, physical examination, laboratory tests, and radiographic studies such as a sinus radiograph
or sinus computed tomography scan are not helpful in distinguishing a prolonged viral URI from
bacterial sinusitis. A Waters view sinus radiograph may help confirm unilateral sinusitis by showing
maxillary opacification, but it does not need to be performed prior to initiating antibiotics.
An antral puncture is considered the gold standard for isolation of the causative organism, but it is
invasive and rarely performed today. A nasal swab culture of the nares may isolate the causative
organism, but it also will include many other normal flora. With the initiation of the pneumococcal
conjugate vaccine, nontypeable Haemophilus influenzae has replaced Streptococcus pneumoniae as
the predominant bacterial pathogen in ABRS.
When determining appropriate antibiotic choices for the treatment of ABRS, the clinician should
consider the most likely pathogen, recent antibiotic use, and local antibiotic resistance rates. Depending
on the organism, 15% to 80% of ABRS cases resolve spontaneously without treatment. Most clinicians,
however, are reluctant to use a "watch-and-wait" approach after 10 to 14 days of unremitting symptoms.
Initial recommended oral therapies include high-dose amoxicillin (90 mg/kg per day),
amoxicillin/clavulanate, cefdinir, cefpodoxime proxetil, or cefuroxime axetil. Patients who have had
recent antibiotic therapy or who attend child care may require amoxicillin/clavulanate or intramuscular
ceftriaxone as initial therapy.
Many adjunctive therapies for ABRS are available, including nasal corticosteroids, oral
decongestants, cough expectorants, pine oil, nasal saline rinses, and nasal decongestants. However,
none has shown clinical benefit.

References:

American Academy of Pediatrics Subcommittee on Management of Sinusitis and Committee on Quality


Improvement. Clinical practice guideline: management of sinusitis. Pediatrics. 2001;108:798-808.
Available at: http://pediatrics.aappublications.org/cgi/content/full/108/3/798

Brook I, Foote PA, Hausfeld JN. Frequency of recovery of pathogens causing acute maxillary sinusitis
in adults before and after introduction of vaccination of children with the 7-valent pneumococcal
vaccine. J Med Microbiol. 2006;55:943-946. Available at:
http://jmm.sgmjournals.org/cgi/content/full/55/7/943

Taylor A, Adam HM. In brief: sinusitis. Pediatr Rev. 2006;27:395-397. Available at:
http://pedsinreview.aappublications.org/cgi/content/full/27/10/395

page 758
2009 PREP SA on CD-ROM

Question: 224
A 5-year-old boy is brought to the emergency department following 2 days of headache, nausea, and
vomiting. His mother reports that he has had no fever or diarrhea and that everyone at home, "including
the dog," has the same symptoms. Physical examination demonstrates a heart rate of 120 beats/min,
respiratory rate of 24 breaths/min, blood pressure of 100/60 mm Hg, and oxygen saturation of 100% on
room air. The boy is mildly irritable, and his mucous membranes appear bright red. His lungs are clear,
and abdominal examination findings are unremarkable. As you are completing your evaluation, the
mother tells you that the furnace in their house has been malfunctioning.

Of the following, the MOST appropriate next step is to

A. administer 2 L of oxygen by nasal cannula

B. arrange for emergent hyperbaric oxygen therapy

C. monitor the patient’s oxygen saturation continuously with pulse oximetry

D. obtain a venous carboxyhemoglobin measurement

E. obtain arterial blood gases

page 759
2009 PREP SA on CD-ROM

Critique: 224 Preferred Response: D


Carbon monoxide (CO) is an odorless, colorless gas that, when inhaled in a closed space, leads to
nonspecific clinical symptoms that can be mistaken easily for those seen in a variety of unrelated
medical conditions. Because unrecognized CO poisoning can lead to significant morbidity, including
delayed cognitive and focal neurologic deficits, as well as mortality, physicians should have a high
index of suspicion for this diagnosis when examining patients who have vague, "flulike" symptoms. This
is especially important if there is no accompanying fever or others in the same environment have similar
symptoms, such as described in the vignette. Although a "cherry red" appearance of the skin and
mucous membranes traditionally was believed to be a clue to the diagnosis, this now is recognized as a
highly insensitive sign.
CO poisoning results from CO binding to hemoglobin 250 times more readily than oxygen. The
resultant carboxyhemoglobin (COHgb) is unable to transport oxygen to tissues and interferes with
oxygen dissociation from the remaining oxyhemoglobin. Tissue hypoxia results, with effects manifested
in every organ system. Central nervous system (headache, confusion, lethargy, dizziness, coma),
pulmonary (dyspnea on exertion, tachypnea), cardiovascular (tachycardia, palpitations), and
gastrointestinal (nausea and vomiting) symptoms predominate.
Evaluation and treatment of CO poisoning include removal of the patient from the contaminated
environment, patient stabilization, high-flow oxygen administration, and measurement of a COHgb
concentration. A COHgb value of greater than 3% to 5% in a nonsmoker is indicative of exposure.
Although symptomatic patients typically have values greater than 10%, such values do not correlate
well with type or magnitude of symptoms. Pulse oximetry cannot distinguish between oxyhemoglobin
and COHgb, so oxygen saturation measurements typically are normal. Arterial blood gases should be
measured in those who have severe symptoms to quantify the degree of acidosis, an indicator of
anaerobic metabolism in the face of poor tissue oxygenation.
The goal of oxygen therapy in the treatment of CO poisoning is to convert COHgb rapidly to
oxyhemoglobin. The administration of 100% oxygen using a nonrebreather mask can decrease the
half-life of COHgb from 300 minutes to 90 minutes and should be continued until the COHgb
concentration is less than 5%. Although hyperbaric oxygen can decrease the COHgb half-life further to
30 minutes, its benefit in improving patient outcome has not been demonstrated clearly. At present,
expert consensus guidelines recommend its use only for severe intoxications.

References:

Clardy PF, Manakar S. Carbon monoxide poisoning. UpToDate Online 15.3. 2008. Available for
subscription at:
http://www.utdol.com/utd/content/topic.do?topicKey=ad_tox/2932&selectedTitle=1~26&source=search_r
esult

Juurlink DN, Buckley NA, Stanbrook MB, Isbister GK, Bennett M, McGuigan MA. Hyperbaric oxygen for
carbon monoxide poisoning. Cochrane Database Syst Rev. 2005;1:CD002041. Available at:
http://www.mrw.interscience.wiley.com/cochrane/clsysrev/articles/CD002041/frame.html

Kind T, Etzel RA. In brief: carbon monoxide. Pediatr Rev. 2005;26:150-151. Available at:
http://pedsinreview.aappublications.org/cgi/content/full/26/4/150

page 760
2009 PREP SA on CD-ROM

Question: 225
A 15-year-old girl presents with a history of abdominal pain. She is a competitive runner and has
required frequent ibuprofen for treatment of knee pain. On physical examination, you note epigastric
tenderness. Fecal occult blood test results are positive.

Of the following, the test that is MOST likely to provide a definitive diagnosis is

A. abdominal computed tomography scan

B. Helicobacter pylori serology

C. serum amylase measurement

D. upper endoscopy with biopsies

E. upper gastrointestinal radiographic series

page 761
2009 PREP SA on CD-ROM

Critique: 225 Preferred Response: D


The epigastric pain, use of nonsteroidal anti-inflammatory agents, and guaiac-positive stool described
for the patient in the vignette suggest the possibility of peptic ulcer disease. Peptic ulcers are erosions
of the gastric or duodenal mucosa that disrupt the epithelium and predispose to abdominal pain or
gastrointestinal bleeding. The most common predisposing factors placing patients at risk for the
development of gastric ulcers are Helicobacter pylori infection, medications (particularly
chemotherapeutic agents or nonsteroidal anti-inflammatory drugs), Crohn disease, vasculitis, or severe
illness requiring intensive care unit hospitalization. Ulcers also may be idiopathic and without an
identifiable inflammatory or infectious cause.
If peptic ulcer is suspected and definitive diagnosis is required, the test of choice is upper
gastrointestinal endoscopy or esophagogastroduodenoscopy (EGD). EGD is superior to other imaging
modalities such as ultrasonography or abdominal computed tomography scan for identifying mucosal
lesions and allows for biopsy of the stomach to evaluate for Crohn disease or H pylori. If a patient has a
bleeding ulcer (Item C225) that has a visible vessel, therapeutic endoscopy (in which the bleeding area
is injected with epinephrine or cauterized) can be performed at the same time as the diagnostic
procedure. Because endoscopy is an invasive test that requires sedation or anesthesia, the benefits of
the procedure need to be weighed against the risks. In particular, the endoscopy should be undertaken
with caution in a patient who has a bleeding disorder or a critically ill patient who has gastrointestinal
bleeding. In such patients, empiric therapy with proton pump antagonists and hemodynamic
stabilization should be attempted before endoscopy is performed.
H pylori serology and serum amylase measurement cannot diagnose peptic ulcer disease. Upper
gastrointestinal radiographic series and abdominal computed tomography scan have poor sensitivity for
ulcers.

References:

De Giacomo C. Helicobacter pylori gastritis and peptic ulcer disease. In: Guandalini S, ed. Textbook of
Pediatric Gastroenterology and Nutrition. London, England: Taylor & Francis; 2004:73-94

Fox VL. Pediatric endoscopy. Gastrointest Endosc Clin North Am. 2000;10: 175-194. Abstract available
at: http://www.ncbi.nlm.nih.gov/pubmed/10618461

page 762
2009 PREP SA on CD-ROM

Critique: 225

Bleeding duodenal ulcer (arrow). (Courtesy of A. Bousvaros)

page 763
2009 PREP SA on CD-ROM

Question: 226
You are seeing a 30-year-old multigravid woman for prenatal counseling. She has had immune
thrombocytopenic purpura for the past 5 years, and her spleen was removed 2 years ago. She asks you
about the effects that her disease might have on her unborn child.

Of the following, you are MOST likely to tell her that

A. if her newborn has thrombocytopenia, he or she will be treated with intravenous immunoglobulin

B. maternal platelet counts predict fetal risks of intracranial hemorrhage

C. maternal platelet transfusion during pregnancy will minimize the risk for neonatal thrombocytopenia

D. operative delivery of the newborn will reduce the risk of intracranial hemorrhage

E. the newborn will require a platelet transfusion soon after birth

page 764
2009 PREP SA on CD-ROM

Critique: 226 Preferred Response: A


The fact that the mother described in the vignette has immune thrombocytopenia is noteworthy.
Because this condition is associated with immunoglobulin G (IgG) directed against maternal platelet
antigens, transplacental acquisition of these antibodies by the fetus may occur after 30 weeks'
gestation. As such, maternal-fetal surveillance is recommended, but no direct correlation exists
between maternal platelet counts and fetal platelet counts or the risk of intracranial hemorrhage.
Maternal platelet transfusion during pregnancy may help avoid concerns for the mother, but not the
fetus or newborn. Although the risk for intracranial hemorrhage is only about 5% overall, it is greatest
when fetal platelet counts are less than 20.0x103/mcL (20.0x109/L) because spontaneous intracranial
hemorrhage may occur. An operative delivery of the newborn does not reduce the risk of intracranial
hemorrhage significantly.
Treatment of the newborn generally is supportive, with attention to the risk of thrombocytopenia
significant enough to warrant platelet transfusion. Ninety percent of affected newborns require no
treatment. However, platelet transfusion may be in order for any infant of a mother who has immune
thrombocytopenic purpura when the neonate exhibits bleeding or has a platelet count of less than
20.0x103/mcL (20.0x109/L). The best treatment for affected infants is intravenous immunoglobulin
(IVIg), which blocks circulating maternal IgG directed against platelet antigens and allows the newborn's
platelet counts to rise. Platelet transfusion for the newborn is less effective than IVIg and may risk
exposure to platelet-associated antigens that could induce a postnatal immune thrombocytopenia.

References:

Buyon JP, Nugent D, Mellins E, Sandborg C. Maternal immunologic diseases and neonatal disorders.
NeoReviews. 2002;3:e3-e10. Available for subscription at:
http://neoreviews.aappublications.org/cgi/content/full/3/1/e3

Murphy MF, Bussel JB. Advances in the management of alloimmune thrombocytopenia. Br J Haematol.
2007;136:366-378. Abstract available at: http://www.ncbi.nlm.nih.gov/pubmed/17233844

Wong W, Glader B. Approach to the newborn who has thrombocytopenia. NeoReviews.


2004;5:e444-e450. Available for subscription at:
http://neoreviews.aappublications.org/cgi/content/full/5/10/e444

page 765
2009 PREP SA on CD-ROM

Question: 227
You are addressing a group of expectant mothers who are due to deliver their infants in the next few
weeks. You discuss the benefits of breastfeeding and explain that it is the best nutrition for most babies.
One woman asks you if it is acceptable to breastfeed if she has had hepatitis in the past. You explain
that there are only a few infections that would prevent a mother from being able to breastfeed her baby.

Of the following, breastfeeding is MOST likely to be contraindicated if a mother

A. has active untreated pulmonary tuberculosis

B. has genital herpes without breast lesions

C. is a cytomegalovirus carrier

D. is hepatitis B surface antigen-positive

E. is hepatitis C antibody-positive

page 766
2009 PREP SA on CD-ROM

Critique: 227 Preferred Response: A


Human milk is the optimal nutrition for infants. Benefits include transference of protective maternal
antibodies, improved bonding between mother and child, and probable improvement in cognitive and
developmental function of the infant. Most mothers can breastfeed successfully, although there are
contraindicated conditions for both infants and mothers. Infants who have galactosemia should not
receive human milk, and infants who have other forms of metabolic disease, such as urea cycle defects
or phenylketonuria, may receive only a limited amount. Mothers infected with human immunodeficiency
virus (in the United States) or human T-cell lymphotrophic virus-1 or -2 and those who have active
untreated tuberculosis or active herpes lesions on the breast should not breastfeed their infants.
However, mothers who are hepatitis B surface antigen-positive or hepatitis C antibody-positive, are
cytomegalovirus carriers, or have genital herpes without breast lesions can breastfeed safely. Maternal
medications that preclude breastfeeding include antineoplastic agents, immunosuppressants, lithium,
and radiopharmaceutical agents.
Several disorders of the breast may make breastfeeding difficult, but they are not contraindications
to breastfeeding. Previous breast surgery may cause ineffective lactation, but this varies among
mothers. Women who have inverted or flat nipples may experience difficulties with latch-on, but this can
be improved with early feedings, use of nipple shields, and lactation consultation. Use of a breast pump
also may help. Women who have breast cancer may be able to breastfeed if they are not taking
antineoplastic medications. Mastitis, inflammation of the breast usually caused by obstruction of ducts,
may make breastfeeding painful, but more frequent nursing is the best recommendation to help resolve
this condition. Sore or cracked nipples may develop, especially if the infant has oral-motor dysfunction,
and adjusting the infant's latch-on may improve these symptoms. A mother who has Candida infection
of the breast may continue to breastfeed, but both she and her infant should be treated for the infection
to avoid a cycle of reinfection.

References:

American Academy of Pediatrics Section on Breastfeeding. Breastfeeding and the use of human milk.
Pediatrics. 2005;115:496-506. Available at:
http://pediatrics.aappublications.org/cgi/content/full/115/2/496

Chandran L, Gelfer P. Breastfeeding: the essential principles. Pediatr Rev. 2006;27:409-417. Available
at: http://pedsinreview.aappublications.org/cgi/content/full/27/11/409

Powers NG, Slusser W. Breastfeeding update 2: clinical lactation management. Pediatr Rev.
1997;18:147-161. Available at: http://pedsinreview.aappublications.org/cgi/content/full/18/5/147

page 767
2009 PREP SA on CD-ROM

Question: 228
A 6-month-old girl, who was born in Nigeria, presents for an urgent visit as soon as the family arrives in
the United States because of fever and irritability. Physical examination reveals a fussy infant who has
anorexia, a temperature of 100°F (37.8°C), and swelling of all of the fingers of the right hand (Item
Q228). The remainder of the examination findings are negative.

Of the following, the MOST likely cause of this pattern of swelling in this child is

A. cellulitis

B. juvenile idiopathic arthritis

C. malaria

D. sickle cell disease

E. trauma

page 768
2009 PREP SA on CD-ROM

Question: 228

Swelling of the digits, as exhibited by the infant in the vignette. (Courtesy of


M. Rimsza)

page 769
2009 PREP SA on CD-ROM

Critique: 228 Preferred Response: D


Sickle cell disease (SCD) usually is diagnosed in the United States in early infancy because of
mandated newborn screening. If screening is not performed, the diagnosis may be delayed until the
development of symptoms from a vaso-occlusive crisis. In infants and toddlers, the first vaso-occlusive
crisis may be heralded by painful and swollen hands or feet, also known as dactylitis or "hand and foot
syndrome." The examination findings for the child described in the vignette suggest the diagnosis of
dactylitis that is most likely due to SCD.
Dactylitis due to SCD usually presents with swelling and tenderness of the hands (Item C228) or feet
that is self-limited and resolves in 5 to 31 days. Fever and leukocytosis also may be noted.
Radiographs of the hands and feet taken 7 to 14 days after the swelling begins may reveal periosteal
new bone formation or intramedullary densities. Such lesions resolve in 2 to 3 months. Diagnostic
confusion with cellulitis or osteomyelitis (which rarely may occur in conjunction with dactylitis) is
frequent, but dactylitis often involves most, if not all, of the digits equally and is not accompanied by
overlying cellulitis of the skin.
Trauma generally does not involve all digits equally without contusion or laceration of the overlying
skin. Arthritis, including juvenile idiopathic arthritis, usually affects only a few joints and is uncommon in
this age group. Malaria causes fever and anemia but does not commonly cause dactylitis or other
extremity findings.
Dactylitis may be seen in other conditions, including psoriasis, insect bites with angioedema, and
other conditions that are more common in older children and adults.

References:

American Academy of Pediatrics Section on Hematology/Oncology and Committee on Genetics. Health


supervision for children with sickle cell disease. Pediatrics. 2002;109:526-535. Available at:
http://pediatrics.aappublications.org/cgi/content/full/109/3/526

Gill FM, Sleeper LA, Weiner SJ, et al for the Cooperative Study of Sickle Cell Disease. Clinical events
in the first decade in a cohort of infants with sickle cell disease. Blood. 1995;86:776-783. Available at:
http://bloodjournal.hematologylibrary.org/cgi/reprint/86/2/776

Meremikwu MM. Sickle cell disease (updated). BMJ Clinical Evidence. 2007. Available for subscription
at: http://clinicalevidence.bmj.com/ceweb/conditions/bly/2402/2402.jsp

Miller ST, Sleeper LA, Pegelow CH, et al. Prediction of adverse outcomes in children with sickle cell
disease. N Engl J Med. 2000:342:2:83-89. Abstract available at:
http://www.ncbi.nlm.nih.gov/pubmed/10631276

Sickle-cell dactylitis. In: Wheeless' Textbook of Orthopaedics. Available at:


http://www.wheelessonline.com/ortho/sickle_cell_dactylitis

page 770
2009 PREP SA on CD-ROM

Critique: 228

Vaso-occlusive crises in infants who have sickle cell disease often are
characterized by swelling and tenderness of the fingers or toes. (Courtesy of
M. Rimsza)

page 771
2009 PREP SA on CD-ROM

Question: 229
You work as a voluntary attending pediatrician in the resident continuity clinic at your local hospital. You
are precepting a resident, who tells you that she has just evaluated a 16-year-old varsity volleyball
player. The girl’s height is 71 inches, weight is 125 lb, and blood pressure is 115/74 mm Hg. The
resident is concerned about scoliosis and a 3/6 holosystolic murmur heard at the cardiac apex with
radiation to the left axilla (Item Q229).

Of the following, the MOST likely diagnosis for this patient is

A. Ehlers—Danlos syndrome

B. infective endocarditis

C. Marfan syndrome

D. rheumatic heart disease

E. Williams syndrome

page 772
2009 PREP SA on CD-ROM

Critique: 229 Preferred Response: C


The patient in the vignette has several physical examination findings that suggest the diagnosis of
Marfan syndrome, including scoliosis, tall stature, and a holosystolic murmur that could be due to mitral
valve disease. The murmur is heard at the cardiac apex and is of the quality and location typical for
mitral regurgitation (Item C229A), a common result of the prolapsing and redundant valve. The patient
should be referred to a specialist, usually a geneticist, who will use the Ghent criteria to determine if the
child has Marfan syndrome.
Marfan syndrome is an autosomal dominant connective tissue disorder that is caused by mutations
in the fibrillin gene on chromosome 15. The abnormal fibrillin affects the organization of extracellular
myofibrils that support the formation of elastin networks in connective tissue. The abnormal fibrillin is
associated with variable cardiovascular, skeletal, and ocular features. Not all people who have Marfan
syndrome have cardiovascular involvement, but the blood vessels, cardiac valves, and myocardium all
contain substantial amounts of fibrillin. Cardiovascular involvement in children most commonly results in
dilation of the aortic root at the sinuses of Valsalva and abnormality of the atrial ventricular valves. The
mitral valve frequently is affected and demonstrates dysplasia, prolapse, and dysfunction, as may the
tricuspid valve.
The skeletal findings of Marfan syndrome may include scoliosis, a tall and thin habitus, pectus
deformities (Item C229B), arachnodactyly (Item C229C), and hypermobility of the joints. The ocular
findings may include ectopia lentis (lens dislocation) (Item C229D). Often, the arm span-to-height ratio
exceeds 1.05, and the upper-to-lower segment ratio is less than 0.88.
Patients in whom Marfan syndrome is diagnosed or highly suspected should be counseled to avoid
participation in competitive contact sports. Avoidance of contact sports is important because there is an
increased risk of cardiac, skeletal, and ophthalmologic problems resulting from injury. A specific
potential problem with the cardiovascular system is tearing of the weakened, abnormal aortic wall,
which can lead to a catastrophic outcome. It is important to remember that sports practice often can be
as competitive and intense as the games; both should be avoided. Consultation with an experienced
pediatric cardiologist is important, and echocardiographic imaging of the heart is essential to evaluate
for the possibility of cardiovascular involvement.
The patient described in the vignette does not have the findings of either Ehlers-Danlos or Williams
syndromes. She has no findings to suggest infective carditis or rheumatic fever/heart disease.

References:

Maron BJ, Chaitman BR, Ackerman MJ, et al. Recommendations for physical activity and recreational
sports participation for young patients with genetic cardiovascular diseases. Circulation.
2004;109:2807-2816. Available at: http://circ.ahajournals.org/cgi/content/full/109/22/2807

Maron BJ, Thompson PD, Ackerman MJ, et al. AHA scientific statements. Recommendations and
considerations related to preparticipation screening for cardiovascular abnormalities in competitive
athletes: 2007 update. A scientific statement from the American Heart Association Council on Nutrition,
Physical Activity, and Metabolism: endorsed by the American College of Cardiology Foundation.
Circulation. 2007;115:1643-1655. Available at: http://circ.ahajournals.org/cgi/content/full/115/12/1643

Moodie DS. AAP: health supervision for children with Marfan syndrome. Clin Pediatr (Phila).
1997;36:489

Peirpont MEM. Connective tissue diseases. In: Moller JH, Hoffman JIE, eds. Pediatric Cardiovascular
Medicine. Philadelphia, Pa: Churchill Livingstone; 2000:901-912

page 773
2009 PREP SA on CD-ROM

von Kodolitsch Y, Robinson PN. Marfan syndrome: an update of genetics, medical and surgical
management. Heart. 2007;93:755-760. Extract available at:
http://heart.bmj.com/cgi/content/extract/93/6/755

page 774
2009 PREP SA on CD-ROM

Critique: 229

Pectus carinatum in a patient who has Marfan syndrome. (Courtesy of T.


Jewett)

page 775
2009 PREP SA on CD-ROM

Critique: 229

Arachnodactyly is a feature of Marfan syndrome. (Courtesy of M. Rimsza)

page 776
2009 PREP SA on CD-ROM

Critique: 229

Subluxed lens: The lens is displaced inferiorly (arrows show the border of the
lens). In Marfan syndrome, the lens typically is displaced superiorly and
temporally. (Courtesy of the Wake Forest University Eye Center)

page 777
2009 PREP SA on CD-ROM

Question: 230
The parents of a 6-month-old previously well infant bring her to your office. She had been developing
normally, but she stopped interacting with her parents over the last 24 hours. For several days prior to
this development, she had had unusual spells during which her head and chin dropped to her chest.
Now she is having clusters of these spells involving head drop and body flexion. On physical
examination, there is no bruising. The infant is afebrile and alert, her tone is low, and she does not
make persistent eye contact or track visually. You refer her to the emergency department, where results
of a complete blood count, electrolyte panel, urinalysis, and a noncontrast head computed tomography
scan are normal.

Of the following, the test that is MOST likely to reveal the correct diagnosis is

A. electroencephalography

B. electroretinography

C. lumbar puncture

D. muscle biopsy

E. serum lactate measurement

page 778
2009 PREP SA on CD-ROM

Critique: 230 Preferred Response: A


The infant described in the vignette has an encephalopathy, a "confusional" state. It is critical to
consider focal, ischemic, ictal, infectious, and toxic/metabolic causes for any patient presenting with
mental status changes. The noncontrast head computed tomography scan and laboratory tests were
appropriate initial decisions in the emergency department to rule out emergent, treatable problems. For
this infant, the key to the diagnosis is recognition that the cluster of body spasms combined with the
loss of developmental milestones is characteristic of infantile spasms, which should be assessed by
electroencephalography (EEG). EEG is used to identify the neurophysiologic pattern that confirms the
diagnosis of infantile spasms. The characteristic diffuse, slow, disorganized, and high-amplitude pattern
is known as "hypsarrhythmia."
Infantile spasms describes a syndrome of epilepsy and encephalopathy in infancy, with a peak onset
at 6 months. The spasms (Item C230) can range from subtle, quick movements involving just the head
to dramatic "clasp-knife" full body spasms. Infantile spasms can have a number of causes or can be
idiopathic. The prognosis is often poor, particularly if the child's neurologic development was abnormal
prior to the onset of spasms. Potential causes of the infantile spasms may be structural, genetic, or
metabolic.
Electroretinography (ERG) is used to assess the neurophysiology of the retina. For example, ERG
responses are abnormal in Leber congenital amaurosis and various rod/cone dystrophies. This child's
new visual impairment associated with hypotonia and spasms is not likely related to retinal pathology,
but more likely is due to a central nervous system problem causing abnormal visual processing.
A lumbar puncture should be considered in an infant who has fever and an acute encephalopathy to
rule out infectious causes, but establishing the diagnosis of infantile spasms is more important in this
case. A lumbar puncture, if it is obtained, might be more valuable as part of a metabolic evaluation, as
would measurement of serum lactate and a muscle biopsy, if needed to assess for the possibility of a
mitochondrial disorder.

References:

Kossoff EH. Infantile spasms. In: Singer HS, Kossoff EH, Hartman AL, Crawford TO, eds. Treatment of
Pediatric Neurologic Disorders. Boca Raton, Fla: Taylor & Francis; 2005:111-116

Mackay MT, Weiss SK, Adams-Webber T, et al. Practice parameter: medical treatment of infantile
spasms. Report of the American Academy of Neurology and the Child Neurology Society. Neurology.
2004;62:1668-1681. Available at: http://www.neurology.org/cgi/content/full/62/10/1668

page 779
2009 PREP SA on CD-ROM

Question: 231
While examining an infant in the newborn nursery, you note that the pupil of one eye seems abnormally
large, and little of the iris is visible. The baby appears otherwise normal. A subsequent ophthalmologic
evaluation confirms the diagnosis of partial aniridia.

Of the following, the MOST accurate statement regarding the diagnosis is that

A. aniridia is associated with hepatoblastoma

B. few individuals who have aniridia have an affected parent

C. isolated aniridia has an autosomal recessive inheritance pattern

D. molecular testing is available to determine risk for Wilms tumor

E. routine abdominal ultrasonography should be performed every 3 months until age 5 years in affected
individuals

page 780
2009 PREP SA on CD-ROM

Critique: 231 Preferred Response: D


Aniridia is defined as complete or partial iris hypoplasia (Item C231), often associated with foveal
hypoplasia and reduced visual acuity. Sometimes it can be detected in the newborn period with the use
of direct ophthalmoscopy; the red reflex may appear unusual in shape or is too large due to the lack of
intervening iris tissue. Infants who have such a finding should be referred to ophthalmology for
confirmation and further delineation of the defect. The ophthalmologist plays a major role in the
management of affected children. Isolated aniridia is an autosomal dominant trait, and most affected
individuals have an affected parent.
It is important to note that aniridia is associated with Wilms tumor in some individuals. It is now
possible to evaluate a blood specimen from an individual who has aniridia to determine if he or she has
a PAX6 gene mutation, which is detectable in 90% of isolated cases. If a PAX6 mutation is detected,
Wilms tumor is not a concern. However, if no mutation of PAX6 is detected, the aniridia could be part of
a condition, such as aniridia-Wilms tumor or the Wilms tumor-aniridia-genital anomalies-retardation
(WAGR) syndrome. These conditions are caused by genes adjacent to PAX6, which, when deleted,
result in aniridia plus other anomalies. In such cases, it is important to follow a regular surveillance
protocol for Wilms tumor that includes regular urinalysis and routine renal ultrasonography.
Wilms tumor also occurs with increased frequency in conditions such as Beckwith-Wiedemann
syndrome (BWS) and hemihypertrophy. In these conditions, published surveillance protocols for the
development of associated neoplasms should be followed.
Aniridia is not associated with an increased risk for hepatoblastoma, although BWS and
hemihypertrophy are.

References:

Clericuzio C. Recognition and management of childhood cancer syndromes: a systemic approach. Am


J Med Genet. 1999;25:89:81-90. Abstract available at: http://www.ncbi.nlm.nih.gov/pubmed/10559762

Hingorani M, Moore A. Aniridia. GeneReviews. 2005. Available at:


http://www.geneclinics.org/servlet/access?db=geneclinics&site=gt&id=8888891&key=xAcWBcrjmZrVo&
gry=&fcn=y&fw=581E&filename=/profiles/aniridia/index.html

Lapunzina P. Risk of tumorigenesis in overgrowth syndromes: a comprehensive review. Am J Med


Genet C Semin Med Genet. 2005;137:53-71. Abstract available at:
http://www.ncbi.nlm.nih.gov/pubmed/16010678

page 781
2009 PREP SA on CD-ROM

Critique: 231

Aniridia appears as a large pupil with little iris. (Courtesy of the Media Lab at
Doernbecher)

page 782
2009 PREP SA on CD-ROM

Question: 232
A 15-year-old girl presents with vaginal pain and burning accompanied by feelings of warmth and
generalized muscle aches for the past 24 hours. She has had no previous similar symptoms. She is
sexually active and does not use barrier methods for contraception. On physical examination, you find
multiple shallow ulcers of the labia minora that are surrounded by erythema and are exquisitely tender
to touch (Item Q232). There is no vaginal discharge.

Of the following, the most appropriate treatment is

A. acyclovir 5 to 10 mg/kg intravenously every 8 hours for 2 to 7 days

B. acyclovir 400 mg orally once a day for 1 year

C. acyclovir 400 mg orally 3 times a day for 7 to 10 days

D. acyclovir 800 mg orally twice a day for 2 days

E. no antiviral chemotherapy

page 783
2009 PREP SA on CD-ROM

Question: 232

(Courtesy of M. Rimsza)

page 784
2009 PREP SA on CD-ROM

Critique: 232 Preferred Response: C


Most genital herpes simplex virus (HSV) infections are caused by HSV type 2 (HSV-2), although the
prevalence of HSV type 1 (HSV-1) appears to be increasing. Primary infection with either of the two
types may go undetected, but the labial lesions (Item C232), systemic symptoms of generalized muscle
aches, and no previous symptoms described for the girl in the vignette suggest that this is a primary
HSV infection. The clinical diagnosis should be confirmed by type-specific laboratory testing.
Recurrences and subclinical shedding are much less frequent for genital HSV-1 infection than HSV-2
infection, and the prognosis and counseling can be influenced by the type detected.
Antiviral therapy for a first clinical episode of genital herpes is recommended, even if manifestations
are mild, because some patients develop severe or prolonged symptoms at a later time. A 7- to 10-day
course of oral acyclovir (400 mg three times a day)is recommended. Famciclovir and valacyclovir are
other treatment options.

References:

American Academy of Pediatrics. Herpes simplex. In: Pickering LK, Baker CJ, Long SS, McMillan JA,
eds. Red Book: 2006 Report of the Committee on Infectious Diseases. 27th ed. Elk Grove Village, Ill:
American Academy of Pediatrics; 2006:361-371

Workowski KA, Berman SM, Centers for Disease Control and Prevention. Sexually transmitted
diseases treatment guidelines, 2006. MMWR Recomm Rep. 2006;55(RR11):1-94. Available at:
http://www.cdc.gov/mmwr/preview/mmwrhtml/rr5511a1.htm

page 785
2009 PREP SA on CD-ROM

Critique: 232

Infection of the genitalia with herpes simplex virus produces painful ulcers.
(Courtesy of M. Rimsza)

page 786
2009 PREP SA on CD-ROM

Question: 233
A 16-year-old boy presents to the emergency department with an acute change in his mental status.
According to his parents, he was previously healthy and has suffered no recent trauma. On physical
examination, he is somnolent, has pinpoint pupils and mild hypotension, and demonstrates shallow
breathing.

Of the following, the test that is MOST likely to help determine the cause of his altered level of
consciousness is

A. brainstem auditory evoked response

B. chest radiography

C. electroencephalography

D. serum amino acid measurement

E. urine toxicology screen

page 787
2009 PREP SA on CD-ROM

Critique: 233 Preferred Response: E


Evaluation of the patient who has an altered level of consciousness consists of a thorough medical
history, a detailed physical examination, and appropriate diagnostic studies. A sudden onset over
minutes in a previously healthy individual suggests trauma or a cerebrovascular accident; effects of
toxic ingestions may present over several hours; and a more gradual onset suggests infection,
metabolic disease, or a space-occupying lesion. Vital signs, level of consciousness, pupil size and
reaction, motor responses, and skin findings can be crucial in narrowing potential causes and
determining appropriate diagnostic studies. Diagnostic studies might include blood and urine testing (for
potential metabolic, infectious, and toxic causes), lumbar puncture, computed tomography scan and
magnetic resonance imaging (with or without angiography), brainstem evoked potentials,
electroencephalography, and electrocardiography.
The teenager described in the vignette has had an acute mental status change, and his
somnolence, hypotension, shallow and slow respirations, and pinpoint pupils are consistent with an
opiate ingestion. Therefore, toxicology screens for confirmation as well as to evaluate for coingestions
are indicated. Evoked potentials may be useful in testing the integrity of the brainstem and in providing
prognostic information for comatose patients, but results are nonspecific and rarely helpful for initial
diagnosis. Chest radiography generally is of little value in the assessment of a comatose patient.
Although seizures may produce an altered level of consciousness, no suggestion of seizure activity is
present in this patient that would necessitate electroencephalography. The patient's age makes an
inborn error of metabolism unlikely, and serum amino acid measurement can be deferred.

References:

Avner JR. Altered states of consciousness. Pediatr Rev. 2006:27:331-338. Available at:
http://pedsinreview.aappublications.org/cgi/content/full/27/9/331

Frankel LR. Neurological emergencies and stabilization. In: Kliegman RM, Behrman RE, Jenson HB,
Stanton BF, eds. Nelson Textbook of Pediatrics. 18th ed. Philadelphia, Pa: Saunders Elsevier;
2007:405-412

page 788
2009 PREP SA on CD-ROM

Question: 234
The parents of a 3-year-old boy in whom you recently diagnosed type 1 diabetes mellitus are anxious
about providing the best diabetes control for their son, but wish to avoid frequent fingersticks to
measure blood glucose. They have read that a hemoglobin A1c gives a measure of blood glucose
control and correlates with long-term complications of diabetes. They request that this blood test be
obtained at weekly intervals to give them assurance of good control.

Of the following, the MOST important information to provide them about hemoglobin A1c measurement
is that it

A. can replace self blood glucose monitoring

B. is not useful in children younger than 5 years of age

C. should be obtained every month

D. should be obtained every 2 to 3 months

E. should be obtained every 6 months

page 789
2009 PREP SA on CD-ROM

Critique: 234 Preferred Response: D


The hemoglobin A1c (HbA1c) is a measure of nonenzymatic glycosylation of hemoglobin A and
correlates with concentrations of circulating blood glucose over the life of the red blood cell. It is not a
useful measure of daily changes in blood glucose or of amplitude of excursion of blood glucose over the
course of a day. Because the red cell has a life of approximately 120 days, measurement of HbA1c
more frequently than every 2 months is not likely to be helpful in clinical management. In general, this
measurement is made every 2 to 3 months. Results permit broad adjustments in insulin therapy and
monitoring of large trends in management. If the HbA1c is measured less frequently, glycemic control
may deteriorate without recognition. As children have been reported to say, it is "the test that does not
lie." It is useful in a child of any age, although the presence of fetal hemoglobin in very young children
may make interpretation of the results more difficult. In addition, measurement of HbA1c in children who
have hemoglobinopathies may not be entirely accurate. Instead, measurement of total glycosylated
hemoglobin can be substituted. For children who have rapid red cell destruction, the published
relationship between serum glucose and HbA1c may be inaccurate.
Self blood glucose monitoring gives a relatively dynamic picture of blood glucose changes over the
course of a day and allows decisions to be made about adjustment of basal insulin requirements as well
as adjustments for meals, exercise, and other life activities. A combination of self blood glucose
monitoring and periodic measurement of HbA1c permits reasoned decisions and improved glycemic
control in this most difficult disorder.

References:

McCulloch DK. Estimation of blood glucose control in diabetes mellitus. UpToDate Online 15.3. 2008.
Available for subscription at:
http://www.uptodateonline.com/utd/content/topic.do?topicKey=diabetes/7913

McCulloch DK. Glycemic control and vascular complications in type 1 diabetes. UpToDate Online 15.3.
2008. Available for subscription at:
http://www.uptodateonline.com/utd/content/topic.do?topicKey=diabetes/10573

Silverstein J, Klingensmith G, Copeland K, et al, Care of children and adolescents with type 1 diabetes:
a statement of the American Diabetes Association. Diabetes Care. 2005;28:186-212. Available at:
http://care.diabetesjournals.org/cgi/content/full/28/1/186

page 790
2009 PREP SA on CD-ROM

Question: 235
A mother brings in her child for a health supervision visit. He is able to pull to stand, take a few
independent steps, and use two fingers to grasp pieces of cereal.

Of the following, these developmental milestones are MOST typical for a child whose age is

A. 6 months

B. 9 months

C. 12 months

D. 15 months

E. 18 months

page 791
2009 PREP SA on CD-ROM

Critique: 235 Preferred Response: C


The motor milestones described in the vignette are appropriate for a 12-month-old child. Such children
can pull to a stand and cruise, take a few independent steps (Item C235A), and use a neat pincer grasp
to handle a raisin or pellet. Motor milestones for a 6-month-old child include sitting with support, rolling
back to front (Item C235B), and transferring an object between hands. A 9-month-old child can sit
indefinitely, may creep on hands and knees, and can use a radial digital grasp to pick up a cube. A
15-month-old child can run stiff-legged. An 18-month-old child can walk up and down stairs with one
hand held, climb on a chair to reach items, scribble spontaneously, turn two or three pages at once in a
picture book, drink from a regular cup, and get a spoon to his or her mouth.
Parents typically first report delays in motor milestones between the ages of 6 and 12 months. A
child's motor developmental milestones are determined from both parental history and direct
observation during the neurodevelopmental evaluation. A motor quotient may be determined by dividing
the developmental age level by the chronologic age and multiplying it by 100. Gross motor quotients
below 50 usually are indicative of cerebral palsy. Motor quotients in the 50 to 75 range may be more
indicative of low muscle tone and developmental coordination disorder. Children who have milder motor
delays may have comorbid neurodevelopmental disorders such as learning disorders or
attention-deficit/hyperactivity disorder.

References:

Accardo PJ, Accardo J, Capute A. A neurodevelomental perspective on the continuum of


developmental disabilities. In: Accardo PJ. Capute & Accardo's Neurodevelopmental Disabilities in
Infancy and Childhood. Volume I: Neurodevelopmental Diagnosis and Treatment. 3rd ed. Baltimore,
Md: Paul H. Brookes Publishing Co; 20083-10

Blasco PA. Motor delays. In: Parker S, Zuckerman B, Augustyn M, eds. Developmental and Behavioral
Pediatrics: A Handbook for Primary Care. 2nd ed. Philadelphia, Pa: Lippincott Williams & Wilkins;
2005:242-247

Knobloch H, Stevens FM, Malone AF. The revised developmental stages. In: Manual of Developmental
Diagnosis: The Administration and Interpretation of the Revised Gesell and Amatruda Developmental
and Neurologic Examination. Albany, NY: Developmental Evaluation Materials, Inc; 1987:17-120

page 792
2009 PREP SA on CD-ROM

Question: 236
A 5-year-old girl presents approximately 96 hours after being bitten by a dog on her leg. Her mother
states that she developed fever and swelling of the leg around the bite site over the past 12 hours.
Physical examination reveals a nontoxic-appearing girl who has a temperature of 101.8°F (38.8°C) and
an open wound with visible purulence and surrounding erythema.

Of the following, the MOST likely pathogen responsible for these symptoms is

A. Eikenella corrodens

B. Kingella kingae

C. Pasteurella multocida

D. Staphylococcus aureus

E. Streptococcus pyogenes

page 793
2009 PREP SA on CD-ROM

Critique: 236 Preferred Response: D


Approximately 10% to 15% of patients who sustain a bite wound from a dog develop infection in the
wound. The patient described in the vignette developed an infection 4 days after sustaining her injury
and has visible purulence at the bite site. Late infections that produce purulence are usually due to
Staphylococcus aureus. Infections with Pasteurella multocida can occur following an animal bite, but
they usually develop rapidly (within 24 hours) and exhibit erythema, tenderness, and edema. Eikenella
corrodens is associated more commonly with human than animal bites. Although dogs have been found
to be colonized with group A Streptococcus (GAS), wound infections with this organism after a bite are
uncommon. In addition, GAS is less likely to produce overt purulence than S aureus. Kingella kingae
are found in the human oropharynx and are an important pathogen in bone and joint disease among
children younger than 5 years of age.

References:

American Academy of Pediatrics. Bite wounds. In: Pickering LK, Baker CJ, Long SS, McMillan JA, eds.
Red Book: 2006 Report of the Committee on Infectious Diseases. 27th ed. Elk Grove Village, Ill:
American Academy of Pediatrics; 2006:191-195

Sagerman PJ. Wounds. Pediatr Rev. 2005;26:43-49. Available at:


http://pedsinreview.aappublications.org/cgi/content/full/26/2/43

page 794
2009 PREP SA on CD-ROM

Question: 237
A 10-year-old boy was bitten by a dog 2 days ago while visiting relatives in rural Mexico. He was
playing outside with his cousin when a stray dog suddenly ran up and bit him on the arm. After the
incident, the dog ran off and could not be found. His mother washed the wound with soap and water,
but no other medical attention was sought at that time. Physical examination today reveals a
moderately deep bite wound on the boy’s right forearm that is erythematous, mildly indurated, and
tender, with seropurulent drainage. You prescribe appropriate antibiotic therapy.

Of the following, the MOST appropriate postexposure prophylaxis regimen for this patient is

A. rabies immune globulin alone

B. rabies immune globulin and rabies vaccine

C. rabies immune globulin and tetanus vaccine

D. rabies vaccine and tetanus vaccine

E. rabies vaccine alone

page 795
2009 PREP SA on CD-ROM

Critique: 237 Preferred Response: B


Infection with the rabies virus produces an acute illness that has rapidly progressive central nervous
system findings, including anxiety, dysphagia, seizures, and encephalitis that, in most cases,
progresses to death. Postexposure prophylaxis for rabies is recommended for all persons: 1) bitten by
wild mammalian carnivores, bats, or domestic animals that may be infected and 2) who report an open
wound, scratch, or mucous membrane that has been contaminated with saliva or other potentially
infectious material from a rabid animal or human. Prophylaxis should be initiated as soon as possible
after bites by known or suspected rabid animals.
The goal of postexposure prophylaxis is to prevent virus from entering neural tissue. Prompt local
treatment of the wound is critical. All wounds should be flushed thoroughly and cleaned with soap and
water. If possible, wounds should not be sutured. The need for tetanus prophylaxis and antibiotic
therapy also should be considered. After wound care is completed, concurrent use of passive (human
rabies immune globulin) (HRIG) and active (rabies vaccine) immunoprophylaxis is required for optimal
therapy. Prophylaxis should begin as soon as possible after exposure, ideally within 24 hours, but
prophylaxis still should be initiated if indicated, regardless of the interval between exposure and
initiation of therapy. Therefore, the boy described in the vignette should receive both HRIG and rabies
vaccine. HRIG should be used concomitantly with the first dose of vaccine for postexposure
prophylaxis. If vaccine is not available immediately, HRIG should be given alone and immunization
started as soon as possible. Similarly, if HRIG is not available immediately, vaccine should be
administered and HRIG given if it can be obtained within 7 days after initiating immunization. If
administration of both vaccine and HRIG is delayed, both should be used regardless of the interval
between exposure and treatment.
The recommended dose of HRIG is 20 IU/kg. As much of the dose as possible should be used to
infiltrate the wound, if present. The remainder is administered intramuscularly. A 1.0-mL dose of rabies
vaccine is given intramuscularly in the deltoid region or the anterolateral aspect of the thigh on the first
day of postexposure prophylaxis, and repeated doses are provided on days 3, 7, 14, and 28 after the
first dose for a total of five doses.
HRIG or rabies vaccine alone or in combination with tetanus vaccine is inadequate postexposure
prophylaxis.

References:

American Academy of Pediatrics. Rabies. In: Pickering LK, Baker CJ, Long SS, McMillan JA, eds. Red
Book: 2006 Report of the Committee on Infectious Diseases. 27th ed. Elk Grove Village, Ill: American
Academy of Pediatrics; 2006:552-559

Centers for Disease Control and Prevention. Human rabies prevention - United States 1999:
recommendations of the Advisory Committee on Immunization Practices (ACIP). MMWR Recomm Rep.
1999;48(RR-1):1-21. Available at: http://www.cdc.gov/mmwr/preview/mmwrhtml/00056176.htm

Rupprecht CE, Gibbons RV. Prophylaxis against rabies. N Engl J Med. 2004;351:2626-2635. Extract
available at: http://content.nejm.org/cgi/content/extract/351/25/2626

page 796
2009 PREP SA on CD-ROM

Question: 238
An 8-year-old boy presents with gross hematuria associated with intermittent right-sided flank pain.
There is no history of dysuria, urgency, frequency, or trauma. Physical examination reveals a
temperature of 98.6°F (37°C), heart rate of 76 beats/min, respiratory rate of 20 breaths/min, blood
pressure of 106/66 mm Hg, and no abdominal or costovertebral angle tenderness. Urinalysis shows a
specific gravity of 1.025, pH of 6, 3+ blood, and trace protein. Microscopy documents 20 to 50 red blood
cells/high-power field. Renal ultrasonography reveals a normal bladder with mild hydronephrosis on the
right and an echogenic focus (Item Q238) with shadowing in the right kidney.

Of the following, the MOST likely additional expected laboratory feature contributing to this patient’s
condition is

A. a positive urine culture

B. elevated urinary calcium excretion

C. elevated urinary citrate excretion

D. hypercalcemia

E. metabolic alkalosis

page 797
2009 PREP SA on CD-ROM

Question: 238

Longitudinal ultrasonography scan of the abdomen reveals an echogenic


focus with distal shadowing in a mildly dilated renal collecting system.
(Courtesy of D. Mulvihill)

page 798
2009 PREP SA on CD-ROM

Critique: 238 Preferred Response: B


The renal colic (intermittent flank pain) and gross hematuria combined with red blood cells in the urine
and hydronephrosis with a shadowing echogenic focus in the kidney described for the patient in the
vignette represent urolithiasis. Children who have urolithiasis require treatment for the stone via either
surgical extraction or extracorporeal shock wave lithotripsy under the guidance of a pediatric urologist.
The pediatrician needs to recognize the underlying risk factors for stone development to aid in the
prevention of a recurrence.
Urolithiasis in the pediatric patient usually involves the ureters and upper urinary tract, with a few
patients exhibiting stones within the bladder. The most common types of renal stones in decreasing
order of occurrence are calcium oxalate, calcium phosphate, mixed (calcium oxalate and calcium
phosphate), struvite (magnesium ammonium phosphate), cystine, and uric acid. The evaluation for
urolithiasis often begins with a 24-hour urine collection to look at urine volume and measure creatinine
as reference points to ensure adequate fluid intake and adequate 24-hour urine collection. In addition,
the 24-hour urine collection is aimed at measuring promoters of stone formation (calcium, oxalate, uric
acid, and cystine) and inhibitors of stone formation (magnesium and citrate). The most common
metabolic abnormality in a child who has stones is hypercalciuria. If hypercalciuria is present, the child
should have serum calcium, phosphorus, ionized calcium, intact parathyroid hormone, and vitamin D
(25-hydoxyvitamin D3 and 1,25-dihydoxyvitamin D3) measured.
A child who has a renal stone also might have a urinary tract infection (UTI). Some urinary
pathogens that produce urease (especially Proteus sp) can lead to an environment favorable to struvite
stone formation. Also, even calcium-containing stones can obstruct urine flow and result in the
development of a UTI. However, a UTI is not likely in a patient who has no dysuria, urgency, or
frequency, such as the boy in the vignette. Risk factors for the development of renal stones include
hypercalciuria, hyperoxaluria, decreased urinary magnesium, and decreased urinary citrate.
Although hypercalciuria is commonly associated with renal stones, hypercalcemia is not a typical
finding. One condition associated with hypercalciuria and renal stones is renal tubular acidosis.
Metabolic alkalosis is seen when calcium phosphate stones are formed, but it is far less likely than
hypercalciuria to be a risk factor for stone formation for the patient in the vignette.

References:

Alon US, Srivastava T. Urolithiasis. In: Kher KK, Schnaper HW, Makker SP, eds. Clinical Pediatric
Nephrology. 2nd ed. London, England: Informa Healthcare; 2007:539-551

Milliner DS. Urolithiasis. In: Avner ED, Harmon WE, Niaudet P, eds. Pediatric Nephrology. 5th ed.
Philadelphia, Pa: Lippincott Williams & Wilkins; 2004:1091-1111

page 799
2009 PREP SA on CD-ROM

Question: 239
A 12-year-old boy presents with a 3-year history of hay fever in the spring. He experiences daily nasal
congestion, sneezing, and rhinorrhea from March to May that worsens when he is outside. He is
asymptomatic for the remainder of the year, but his parents are concerned because his symptoms
interfere with outdoor sports activities. Use of over-the-counter first-generation antihistamines resulted
in undesirable sedation.

Of the following, the BEST initial medication to treat this patient is a(an)

A. intranasal corticosteroid

B. intranasal decongestant

C. nasal saline spray

D. oral decongestant

E. oral leukotriene antagonist

page 800
2009 PREP SA on CD-ROM

Critique: 239 Preferred Response: A


The boy described in the vignette presents with classic symptoms of allergic rhinitis. The approach to
medical management of allergic rhinitis should take into account the patient's primary symptoms, past
experiences with medications, and any coexisting medical problems. Nasal corticosteroids have
become a first-line therapy for allergic rhinitis, with studies demonstrating improvement in nasal
pruritus, sneezing, nasal obstruction, and rhinorrhea. Further, nasal steroids typically are well tolerated
and have minimal adverse effects. Nasal decongestants may be purchased over the counter and
markedly reduce nasal congestion, but recurrent or prolonged use can result in rhinitis medicamentosa.
Nasal saline is an adjunct for nasal toilet, but it is inferior to nasal corticosteroids for the treatment of
allergic rhinitis.
Oral decongestants may be taken separately or in combination with an oral antihistamine. They can
improve nasal congestion, but they may increase blood pressure or cause insomnia. Oral leukotriene
antagonists have been approved for allergic rhinitis and mild persistent asthma. Although this class of
medication generally has an excellent safety profile and is not a steroid, it is inferior to nasal
corticosteroids for treatment of nasal allergic rhinitis symptoms.

References:

Atkins D, Leung DYM. Principles of treatment of allergic disease. In: Kliegman RM, Behrman RE,
Jenson HB, Stanton BF, eds. Nelson Textbook of Pediatrics. 18th ed. Philadelphia, Pa: Saunders
Elsevier; 2007:942-948

Mahr TA, Sheth K. Update on allergic rhinitis. Pediatr Rev. 2005;26:284-289. Available at:
http://pedsinreview.aappublications.org/cgi/content/full/26/8/284

page 801
2009 PREP SA on CD-ROM

Question: 240
You are examining a 7-year-old boy who has a 2-day history of abdominal pain. The pain began 2
nights ago after he ate pizza with the rest of his family and initially was crampy and diffuse. No one else
became ill. He continued to complain of pain through the day yesterday, and this morning he vomited
once, prompting his mother to bring him to the office. The emesis was nonbilious and nonbloody, and
he has had no diarrhea, fever, or urinary symptoms. On physical examination, the boy is afebrile, has
normal vital signs, and has diminished bowel sounds with involuntary guarding in the right lower
quadrant. There are no peritoneal signs, and Rovsing, obturator, and psoas signs are negative.

Of the following, the study MOST likely to confirm the diagnosis is

A. abdominal radiograph

B. complete blood count

C. computed tomography scan of the abdomen and pelvis

D. C-reactive protein determination

E. procalcitonin determination

page 802
2009 PREP SA on CD-ROM

Critique: 240 Preferred Response: C


Appendicitis is the most common abdominal surgical condition in children. Obstruction of the appendix
with fecal matter or other debris leads to edema, bacterial overgrowth, mucosal breakdown, and
appendiceal inflammation. Early diagnosis and surgical treatment are the keys to favorable outcomes.
Diagnosing appendicitis in children can be challenging. When present, the highly sensitive signs and
symptoms of abdominal pain that migrates to the right lower quadrant, fever, and abdominal rebound or
guarding often are enough to prompt surgical exploration. However, the presentation in children
frequently is not classic. The difficulties encountered in evaluating preverbal patients, the overlap of
appendicitis with many other causes of childhood abdominal pain, and the often atypical presentations
contribute to the 28% to 57% appendicitis misdiagnosis rate in children.
Laboratory and radiologic studies often are used as diagnostic adjuncts when appendicitis is
considered in a patient who has an atypical presentation, such as the child described in the vignette.
Abdominal computed tomography scan (Item C240) is the most useful for confirming the diagnosis
because its sensitivities and specificities approach 100%. There is debate in the literature regarding the
optimal use of oral or rectal contrast for such studies. Abdominal radiographs are not helpful.
No laboratory tests are independently predictive of the presence or absence of appendicitis.
Although procalcitonin has not been studied in this setting, neither the total white blood count, absolute
neutrophil count, nor C-reactive protein measurement has been found to be adequately sensitive or
specific. However, with growing concern about the long-term effects of ionizing radiation exposure in
children, laboratory tests combined with clinical findings may prove useful in identifying low-risk
individuals for whom imaging can be avoided.

References:

Bundy DG, Byerly JS, Liles EA, Perrin EM, Katznelson J, Rice HE. Does this child have appendicitis?
JAMA. 2007;298:438-451. Abstract available at: http://www.ncbi.nlm.nih.gov/pubmed/17652298

Kwok MY, Kim MK, Gorelick MH. Evidence-based approach to the diagnosis of appendicitis in children.
Pediatr Emerg Care. 2004;20:690-698

Wesson DE. Evaluation and diagnosis of appendicitis in childhood. UpToDate Online 15.3. 2008.
Available at:
http://www.utdol.com/utd/content/topic.do?topicKey=ped_surg/4980&selectedTitle=4~150&source=sear
ch_result

page 803
2009 PREP SA on CD-ROM

Critique: 240

Computed tomography scan of the abdomen in a reformatted coronal


projection shows a fluid-filled tubular structure with a thick wall consistent
with an obstructed appendix. (Coutesy of D. Mulvihill)

page 804
2009 PREP SA on CD-ROM

Question: 241
A 12-year-old boy who has a history of recurrent abdominal pain presents to your office for an annual
health supervision visit. The boy complains of periumbilical pain, unrelated to meals, occurring twice a
month and lasting 15 minutes. Physical examination findings are normal. Fecal occult blood test results
are negative. His father, who is a physician, asks if the boy should undergo testing for Helicobacter
pylori.

Of the following, a TRUE statement about H pylori infection is that

A. all children who have positive H pylori serologies should undergo endoscopy

B. antibiotic therapy for H pylori is most effective when combined with a proton pump inhibitor

C. H pylori is difficult to detect on gastric histology without special immunofluorescence staining

D. H pylori infection is less prevalent in children from the developing world

E. H pylori organisms rarely develop antibiotic resistance

page 805
2009 PREP SA on CD-ROM

Critique: 241 Preferred Response: B


Helicobacter pylori is a curved, gram-negative bacillus that has a causative role in peptic ulcer disease.
Approximately 50% of individuals worldwide are colonized with H pylori; the organism is more prevalent
in the developing world. Approximately 20% of people infected with H pylori develop complications from
the infection, but there is a long latency period between the time of infection (usually in childhood) and
the time that symptoms develop. The complications of long-term infection may be serious, however,
and include gastric and duodenal ulcers, gastric lymphoma (mucosal associated lymphoid tissue
lymphoma [MALToma]), and atrophic gastritis.
If symptoms or clinical findings suggest the presence of H pylori, a number of diagnostic tests can
aid in establishing a diagnosis. The gold standard for H pylori diagnosis is upper gastrointestinal
endoscopy with biopsy of the gastric antrum. Endoscopy often shows nodular inflammation of the
antrum (Item C241A), and biopsy can demonstrate the characteristic organisms in the gastric
epithelium. Most often, organisms can be seen easily on conventional hematoxylin and eosin stain; in
cases involving few organisms, special staining is helpful (Item C241B). Given the invasiveness of
endoscopy, this procedure should not be performed on every child who has abdominal pain or
dyspeptic symptoms. If a clinician wants to rule out H pylori as a cause of gastrointestinal symptoms,
available tests include a serum H pylori immunoglobulin G serology, a fecal sample for H pylori antigen,
or a C13 urea breath test for H pylori. The limitation of these tests is the occurrence of both
false-positive and false-negative results. In particular, the false-positive rate of serology is about 10%.
Therefore, this test has a poor positive predictive value and probably should not be ordered if the
clinical suspicion of H pylori infection is low (eg, in a child who has periumbilical or lower abdominal
pain).
Given that chronic abdominal pain and dyspeptic symptoms are so common in children and adults,
the role of H pylori in chronic pain remains highly controversial. Even if a diagnostic test for H pylori has
a positive result, it may be hard to prove a definitive association between the patient's symptoms and
the H pylori. Therefore, the clinician and patient need to consider whether to perform definitive testing
(ie, endoscopy) or treat empirically. The "test versus treat" question remains a clinical dilemma because
empiric therapy is more cost-effective and less invasive, but endoscopy provides a definitive diagnosis
and excludes other conditions (eg, reflux esophagitis).
Treatment of H pylori infection usually involves two antibiotics (eg, amoxicillin and metronidazole or
amoxicillin and clarithromycin) in conjunction with a proton pump inhibitor (PPI) (eg, omeprazole,
lansoprazole, pantoprazole). The PPI enhances antimicrobial eradication because the organisms prefer
the normal acid medium of the stomach. Unfortunately, antimicrobial resistance is common in H pylori,
especially to metronidazole. Therefore, if H pylori is causing serious disease and has been treated,
documentation of eradication is suggested either by negative fecal antigen studies or by endoscopy.

References:

Ford A, McNulty C, Delaney B, Moayyedi A. Helicobacter pylori infection. BMJ Clinical Evidence. 2007.
Available for subscription at: http://clinicalevidence.bmj.com/ceweb/conditions/dsd/0406/0406.jsp

Gold BD, Colletti RB, Abbott M, et al; North American Society for Pediatric Gastroenterology and
Nutrition. Helicobacter pylori infection in children: recommendations for diagnosis and treatment. J
Pediatr Gastroenterol Nutr. 2000;31:490-497

Vilaichone RK, Mahachai V, Graham DY. Helicobacter pylori diagnosis and management. Gastroenterol
Clin North Am. 2006;35:229-247. Abstract available at: http://www.ncbi.nlm.nih.gov/pubmed/16880064

page 806
2009 PREP SA on CD-ROM

Critique: 241

H pylori infection produces a nodular inflammation of the gastric antrum.


(Courtesy of A. Bousvaros)

page 807
2009 PREP SA on CD-ROM

Critique: 241

Silver stain of a gastric biopsy demonstrating curved H pylori bacteria in a


gastric crypt. (Courtesy of J. Glickman)

page 808
2009 PREP SA on CD-ROM

Question: 242
A term newborn is delivered to a mother who has had a 5-day history of a nonspecific gastroenteritis,
some loose stools, generalized malaise, and low-grade fever. The infant had a seizure at 6 hours of age
and is ill, with an inspired oxygen requirement of 0.40, some petechiae, and oozing from the umbilicus
and phlebotomy sites. He is irritable on neurologic examination.
Laboratory findings include:
· White blood cell count, 7.5x103/mcL (7.5x109/L)
· Platelet count, 90.0x103/mcL (90.0x109/L)
· Hematocrit, 45% (0.45)
· Aspartate aminotransferase, 240.0 U/L
· Alanine aminotransferase, 300.0 U/L
· Fibrinogen, 90.0 mg/dL (2.6 mcmol/L)
· Prothrombin time, 20 seconds
· Partial thromboplastin time, 60 seconds
· Internationalized Normalized Ratio (INR), 1.80
· Serum glucose, 90.0 mg/dL (5.0 mmol/L)

A lumbar puncture reveals 35 white blood cells, with 50% polymorphonuclear cells and 50%
mononuclear cells; 1 red blood cell; glucose of 60.0 mg/dL (3.3 mmol/L); and protein of 100 mg/dL
(1,000 g/L). No organisms are seen on cerebrospinal fluid (CSF) Gram stain.

Of the following, a TRUE statement about this patient’s meningitis is that

A. gram-negative organisms are unlikely to be causative

B. group B streptococcal meningitis is likely to be the cause

C. infection likely is related to maternal enteroviral infection

D. the abnormal CSF glucose and protein values indicate bacterial meningitis

E. the abnormal liver function test results and CSF cell counts indicate herpes simplex virus infection

page 809
2009 PREP SA on CD-ROM

Critique: 242 Preferred Response: C


The infant described in the vignette has symptoms of respiratory distress, neurologic irritability, and
coagulopathy. The likely viral gastroenteritis suffered by his mother was of short enough duration not to
have conferred any protective antibody transfer to the fetus. His laboratory findings are most concerning
for thrombocytopenia, coagulopathy, and elevated hepatic transaminases. His lumbar puncture results
indicate a pleocytosis consistent with meningitis.
Neonatal meningitis of either viral or bacterial cause typically occurs in the face of a sepsis
syndrome that may be associated with known risk factors prenatally (maternal fever, chorioamnionitis,
prolonged rupture of membranes) or postnatally (bacteremia, urinary tract infection, or respiratory
distress), with attendant signs of apnea, lethargy, acidosis, and hypoglycemia. The infection most often
is bacterial. Physical examination findings can include a bulging or tense anterior fontanelle, irritability,
fever, emesis, coma, and seizures; neuromotor tone may be increased or decreased. CSF values in
bacterial meningitis generally include elevated protein values to more than 150 mg/dL (1,500 g/L), white
blood cell count of more than 100, and a glucose value less than 50% of that measured in the blood.
These values may differ greatly from those seen in viral (aseptic) meningitis of the newborn, such as
that affecting the infant described in the vignette. Neonatal enteroviral infection may follow maternal
infection, characteristically in the spring and summer seasons. Clinically, affected newborns manifest
illness with a sepsis syndrome, coagulopathy, and meningitis. Hepatopathy and respiratory distress
also may be significant.
The mortality rate of neonatal bacterial meningitis generally is cited at 10%, but both mortality and
late morbidity vary with the cause of meningitis in the newborn. The most common pathogens for
neonatal meningitis include group B Streptococcus, Escherichia coli, other gram-negative rods, and
Listeria monocytogenes. Although neonatal group B streptococcal meningitis is common, it generally is
associated with later onset (after 7 days), a shocklike state, and CSF findings consistent with bacterial
meningitis, as noted previously. Gram-negative meningitis varies with perinatal exposure, prior maternal
or neonatal antibiotic use, comorbidities, and neonatal intensive care unit (NICU) practices. S
pneumoniae and Haemophilus influenzae are uncommon causes of meningitis in the newborn.
Staphylococcal infections may be increasing causes of concern in the NICU. Methicillin-resistant S
aureus meningitis may require aggressive therapy and isolation. Herpes simplex viral
meningoencephalitis is associated with a hemorrhagic pleocytosis, apnea, seizures, coagulopathy, and
hepatic transaminase values generally greater than 1,000 U/L.

References:

Heath PT, Nik Yusoff NK, Baker CJ. Neonatal meningitis. Arch Dis Child Fetal Neonatal Ed.
2003;88:F173-F178. Abstract available at: http://www.ncbi.nlm.nih.gov/pubmed/12719388

Klinger G, Chin C-N, Beyene J, Perlman M. Predicting the outcome of neonatal bacterial meningitis.
Pediatrics. 2000;106:477-482. Available at:
http://pediatrics.aappublications.org/cgi/content/full/106/3/477

Miyairi I, Berlingieri D, Protic J, Belko J. Neonatal invasive group A streptococcal disease: case report
and review of the literature. Pediatr Infect Dis J. 2004;23:161-165. Abstract available at:
http://www.ncbi.nlm.nih.gov/pubmed/14872185

Moylett EH. Neonatal Candida meningitis. Semin Pediatr Infect Dis. 2003;14:115-122. Abstract
available at: http://www.ncbi.nlm.nih.gov/pubmed/12881799

page 810
2009 PREP SA on CD-ROM

Philip AGS. Neonatal meningitis in the new millennium. NeoReviews. 2003;4:e73-e80. Available for
subscription at: http://neoreviews.aappublications.org/cgi/content/full/4/3/e73

Polin RA, Harris MC. Neonatal bacterial meningitis. Semin Neonatol. 2001;6:157-172. Abstract
available at: http://www.ncbi.nlm.nih.gov/pubmed/11483021

page 811
2009 PREP SA on CD-ROM

Question: 243
A 5-month-old boy is brought to the emergency department by his mother because of decreased activity
and vomiting for 1 day. She reports occasional foul-smelling stools but no recent changes in stool
pattern. There has been no fever. As a neonate, the boy had difficulty gaining weight and prolonged
jaundice, but he has not required hospitalization. Physical examination reveals an ill-appearing child
who has mild dehydration, a heart rate of 120 beats/min, and otherwise normal vital signs. He appears
somewhat cachectic, and his weight is at the 3rd percentile. Laboratory values include a normal
complete blood count and urinalysis, sodium of 134.0 mEq/L (134.0 mmol/L), chloride of 86.0 mEq/L
(86.0 mmol/L), potassium of 3.8 mEq/L (3.8 mmol/L), and carbon dioxide of 31.0 mEq/L (31.0 mmol/L).
Blood urea nitrogen and creatinine values are within normal limits.

Of the following, the MOST likely diagnosis is

A. Bartter syndrome

B. congenital adrenal hyperplasia

C. cystic fibrosis

D. Fanconi syndrome

E. hypertrophic pyloric stenosis

page 812
2009 PREP SA on CD-ROM

Critique: 243 Preferred Response: C


Cystic fibrosis is an autosomal recessive disorder affecting many children and adolescents. It is caused
by a defect in a chloride channel, the cystic fibrosis transmembrane conductance regulator (CFTR), on
the apical membranes of the linings of the airways, intestinal tract, vas deferens, biliary tree, pancreatic
ducts, and sweat ducts. The result is ineffective secretion of fluids from affected areas and an increased
sodium and chloride sweat concentration, the latter being the basis for diagnostic testing. Affected
children have varying degrees of mucoid airway obstruction, with secondary bacterial infections, failure
to thrive, intestinal obstruction, pancreatic and biliary dysfunction, and infertility. Hypochloremic
metabolic alkalosis with dehydration, as described for the boy in the vignette, is a common feature
because of the high salt loss from the sweat glands. Foul-smelling stools, prolonged jaundice, and poor
weight gain are additional features of the disease. Therefore, the boy in the vignette should undergo
testing for cystic fibrosis (eg, sweat chloride testing and genetics testing).
Bartter syndrome results from a defect in chloride reabsorption in the loop of Henle. Clinical features
include failure to thrive, polyuria, and vomiting. Hypochloremia and metabolic alkalosis can occur, but
hypokalemia, which can be severe, is usual and due to urinary potassium wasting. Infants who have the
salt-losing form of congenital adrenal hyperplasia also may present with vomiting and failure to thrive,
but typical electrolyte abnormalities are hyponatremia, hypochloremia, and hyperkalemia. Fanconi
syndrome is characterized by abnormal proximal renal tubule function. Excessive bicarbonaturia
causes metabolic acidosis with hyperchloremia, and hypokalemia may be seen. Clinical features in
infancy can be similar to those of the previously described diseases, with failure to thrive, vomiting, and
polyuria being common. Infants who have hypertrophic pyloric stenosis often have hypochloremic
metabolic alkalosis, but this condition is seen in younger infants and is the result of substantial vomiting
of a long duration.

References:

Boat TF, Acton JD. Cystic fibrosis. In: Kliegman RM, Behrman RE, Jenson HB, Stanton BF, eds.
Nelson Textbook of Pediatrics. 18th ed. Philadelphia, Pa: Saunders Elsevier; 2007:1803-1816

Davis PB. Cystic fibrosis. Pediatr Rev. 2001;22:257-264. Available at:


http://pedsinreview.aappublications.org/cgi/content/full/22/8/257

page 813
2009 PREP SA on CD-ROM

Question: 244
A 15-month-old girl presents to the emergency department with a temperature of 103°F (39.5°C) during
respiratory virus season. Physical examination reveals rhinorrhea and mild cough but no other focus of
infection. However, she has diffuse bruises in various stages of healing on her abdomen, subscapular
area, and both extensor and flexor surfaces of her extremities. Laboratory studies reveal a white blood
cell count of 9.2x103/mcL (9.2x109/L) with a normal differential count, platelet count of 376.0x103/mcL
(376.0x109/L), hemoglobin of 13.0 g/dL (130.0 g/L), and hematocrit of 39% (0.39).

Of the following, the BEST next step in the evaluation of this child is

A. computed tomography scan of the brain

B. measurement of factor VIII

C. measurement of von Willebrand factor

D. prothrombin time and partial thromboplastin time

E. radiographic skeletal survey

page 814
2009 PREP SA on CD-ROM

Critique: 244 Preferred Response: E


Bruises on the back (Item C244), abdomen, and flexor surfaces in various stages of healing, as
reported for the child in the vignette, are suspicious for nonaccidental trauma. Thus, the child should
undergo a bone survey to look for occult fractures.
Bruising in older infants and toddlers often results from the combination of increasing mobility and
developing coordination and stability. However, bruising that relates to falls associated with creeping,
crawling, cruising, and early walking is seen primarily on the front of the body, especially on the
forehead, forearms, and pretibial areas of the legs, where the child "meets" obstacles in the
environment.
The appearance of a bruise depends on its age, location, and depth as well as the child's skin
pigmentation. A review of the literature about "dating" bruises in children reveals that a bruise may
appear red, blue, or purple at any time from onset to resolution but often contains yellow discoloration
after 18 hours from onset. However, even multiple bruises resulting from the same injury at the same
time in the same person may "age" differently, so it may be difficult to age bruises accurately.
Coagulation disorders such as hemophilia and von Willebrand disease may present with bruising but
often are associated with other physical findings and a history of mucosal bleeding. Studies that can
assess for a coagulation disorder, such as measuring prothrombin time, partial thromboplastin time, von
Willebrand factor, and factor VIII, are unlikely to yield positive results for the child described in the
vignette.
In the absence of signs of head contusion, irritability, focal neurologic signs, or developmental delay,
computed tomography scan of the brain is unlikely to be helpful and would result in unnecessary
exposure to ionizing radiation. Although not specifically addressed in this vignette, some recommend
evaluation for possible soft-tissue injury, including renal, hepatic, and splenic contusions or laceration,
with abdominal ultrasonography. Minor elevations in liver enzymes and microscopic hematuria may be
found with less severe injury. Ophthalmologic examination may detect retinal hemorrhages, which
indicates a potential need for brain imaging, preferably with magnetic resonance imaging in a
neurologically stable child.
Evaluation of the family and caretaker systems by social services and possibly a report for
evaluation by child protective services is clearly indicated when possible child abuse is suspected.
Development of a safety plan may improve the outcome for women who are in abusive relationships
with domestic partners, which is a risk factor for child abuse, and decrease episodes of violence, at
least in the short term.

References:

Barlow J, Johnston I, Kendrick D, Polnay L, Stewart-Brown S. Individual and group-based parenting


programmes for the treatment of physical child abuse and neglect. Cochrane Database Syst Rev.
2006;3:CD005463. Available at:
http://www.mrw.interscience.wiley.com/cochrane/clsysrev/articles/CD005463/frame.html

Coulter K. In brief: bruising and skin trauma. Pediatr Rev. 2000;21:34-35. Available at:
http://pedsinreview.aappublications.org/cgi/content/full/21/1/34

Kellogg ND and the Committee on Child Abuse and Neglect. Evaluation of suspected child physical
abuse. Pediatrics. 2007;119:1232-1241. Available at:
http://pediatrics.aappublications.org/cgi/content/full/119/6/1232

Klevens J, Sadowski L. Intimate partner violence towards women (update). BMJ Clinical Evidence.

page 815
2009 PREP SA on CD-ROM

2007. Available for subscription at:


http://clinicalevidence.bmj.com/ceweb/conditions/woh/1013/1013_I5.jsp

Labbé J, Caouette G. Recent skin injuries in normal children. Pediatrics. 2001;108:271-276. Available
at: http://pediatrics.aappublications.org/cgi/content/full/108/2/271

Maguire S, Mann MK, Sibert J, Kemp A. Are there patterns of bruising in childhood which are diagnostic
or suggestive of abuse? A systematic review. Arch Dis Child. 2005;90:182-186. Abstract available at:
http://www.ncbi.nlm.nih.gov/pubmed/15665178

Sugar NF, Taylor JA, Feldman KW and the Puget Sound Pediatric Research Network. Bruises in infants
and toddlers: those who don't cruise rarely bruise. Arch Pediatr Adolesc Med. 1999;153:399-403.
Available at: http://archpedi.ama-assn.org/cgi/content/full/153/4/399

page 816
2009 PREP SA on CD-ROM

Critique: 244

Bruising on the back may be the result of nonaccidental trauma. (Courtesy of


D. Krowchuk)

page 817
2009 PREP SA on CD-ROM

Question: 245
You receive a telephone call from the physician mother of a 1-week-old patient who was born at 24
weeks' gestation. He is being treated in the neonatal intensive care unit and has been stable on the
ventilator. She is concerned because when she visited him this morning, his blood pressure was 44/26
mm Hg. His mean arterial pressure was 30 mm Hg. She is worried that his blood pressure is low and
that this may be harmful.

Of the following, the MOST accurate statement regarding blood pressure in the preterm infant is that

A. blood pressure values for preterm infants should be compared with those for term infants

B. blood pressure values vary indirectly with gestational age

C. mean arterial pressure should be no less than the corrected gestational age in weeks

D. patent ductus arteriosus narrows the pulse pressure by raising the diastolic pressure

E. systemic hypertension typically occurs coincidentally with pulmonary hypertension

page 818
2009 PREP SA on CD-ROM

Critique: 245 Preferred Response: C


Preterm birth can be associated with a number of morbidities, including hyper- or hypotension. There
are well-described normative values for blood pressure in a healthy term newborn, but normal blood
pressure values for the preterm infant are much less clear. In fact, there may be no true correct or
expected blood pressure for the preterm infant, particularly the child who is born extremely preterm.
Blood pressure increases during the first few days and weeks after birth for each gestational age.
Neonatologists generally agree that the mean arterial blood pressure for a preterm infant should not be
less than the corrected gestational age in weeks. Thus, for example, a 26-week-gestation infant should
have a mean arterial blood pressure in excess of 26 mm Hg. Beyond this simple rule, a given blood
pressure value can be considered adequate if there is no evidence of metabolic acidosis, elevated
lactate concentration, or inadequate end-organ perfusion.
Just as infant and child blood pressure norms are not compared with adult norms, the normative
blood pressure for a term infant is not used to assess the blood pressure of the preterm infant. As noted
previously, blood pressure varies directly, not indirectly, with advancing gestational age. A patent
ductus arteriosus widens the blood pressure by allowing blood to be diverted away from the
higher-resistance systemic circulation and toward the lower-resistance pulmonary circulation. As this
"steal" becomes greater, the diastolic pressure decreases and the pulse pressure (difference between
the systolic and diastolic pressure) increases. Finally, systemic hypertension usually does not occur
with pulmonary hypertension. In fact, it is not uncommon to note systemic hypotension with pronounced
pulmonary hypertension as the failing right ventricle leads to a decrease in cardiac output.

References:

Padbury JF. Neonatal hypotension and hypovolemia. In: Rudolph C, Rudolph A, eds. Rudolph's
Pediatrics. 21st ed. New York, NY: McGraw Hill Medical Publishing Division; 2003:137-140

Nwankwo MU, Lorenz JM, Gardiner JC. A standard protocol for blood pressure measurement in the
newborn. Pediatrics. 1997;99:E10. Available at:
http://pediatrics.aappublications.org/cgi/content/full/99/6/e10

Weindling AM, Subhedar NV. The definition of hypotension in very low-birthweight infants during the
immediate neonatal period. NeoReviews. 2007;8:e32. Available for subscription at:
http://neoreviews.aappublications.org/cgi/content/full/8/1/e32

Zubrow AB, Hulman S, Kushner H, Falkner B. Determinants of blood pressure in infants admitted to
neonatal intensive care units: a prospective multicenter study. Philadelphia Neonatal Blood Pressure
Study Group. J Perinatol. 1995;15:470-479. Abstract available at:
http://www.ncbi.nlm.nih.gov/pubmed/8648456

page 819
2009 PREP SA on CD-ROM

Question: 246
A 17-year-old boy who receives carbamazepine for epilepsy presents to the emergency department
after a 40-minute generalized tonic-clonic seizure. He has been well, and there is no history of trauma.
On physical examination, he answers a few questions, but he is sleepy and confused. He is afebrile,
and his vital signs are normal. Although he is uncooperative, he moves all limbs spontaneously with
good strength.

Of the following, the diagnostic test that is MOST likely to explain this seizure is

A. magnetic resonance imaging

B. noncontrast head computed tomography scan

C. prolonged electroencephalography

D. serum anticonvulsant measurement

E. serum electrolyte measurement

page 820
2009 PREP SA on CD-ROM

Critique: 246 Preferred Response: D


The adolescent described in the vignette has a prior diagnosis of epilepsy. He now is in a postictal state
after status epilepticus, but during the assessment in the emergency department appears to be
returning to normal consciousness. The most likely causes for a prolonged seizure in this setting are his
epilepsy and failure to take his medication. Accordingly, measuring his serum drug concentrations
should be most helpful in explaining the seizure.
Brief or prolonged seizures can be symptomatic of intracranial lesions such as ischemic strokes,
trauma, hemorrhages, neoplasms, or focal or generalized brain infections. They also can occur due to
metabolic derangements involving glucose or electrolytes. It is important to consider this broad range of
causes in any child presenting to the emergency department with seizure, particularly one in status
epilepticus, defined as any seizure lasting 30 minutes or longer. If the seizure has a focal onset, there is
a residual focal deficit, or the seizure is accompanied by fever and prolonged confusion, imaging
studies should be ordered emergently. For infants or for those children whose histories suggest
electrolyte disturbance, glucose and electrolyte testing can be helpful. Electroencephalography (EEG)
may be helpful if confusion is prolonged because occasionally the prolonged confusion may be due to
nonconvulsive status epilepticus. In this case, however, the child's postictal state is resolving.
Therefore, EEG will not add anything useful to initial medical decisions.
For the child or adolescent who has a prior diagnosis of epilepsy, the most likely cause of the
prolonged seizure is noncompliance with medications. Illicit drug use also may be involved and is worth
considering in the differential diagnosis. In addition, excessively high serum drug concentrations of
carbamazepine, sometimes due to drug-drug interactions, occasionally can precipitate seizures. Newer
antiseizure medications are used more frequently in children. Unfortunately, many hospital laboratories
are unable to measure these rapidly. A history obtained from the family ("He just ran out of medicine,
and we couldn't get to the pharmacy") or a phone call to the patient's neurologist ("This has happened
before - he doesn't take his medication and misses appointments") may help clarify the diagnosis in
these cases.

References:

Riviello JJ Jr, Ashwal S, Hirtz D, et al. Practice parameter: diagnostic assessment of the child with
status epilepticus (an evidence-based review). Report of the Quality Standards Subcommittee of the
American Academy of Neurology and the Practice Committee of the Child Neurology Society.
Neurology. 2006;67:1542-1550. Available at: http://www.neurology.org/cgi/content/full/67/9/1542

page 821
2009 PREP SA on CD-ROM

Question: 247
A mother brings in her 4-month-old baby because she is concerned about the infant’s head shape. The
baby is growing and developing normally. Physical examination findings are normal except for a flat
occiput and a wide biparietal diameter with a flat forehead. The head circumference is normal, and the
anterior fontanelle is small but patent.

Of the following, this baby’s findings MOST likely are caused by

A. bilateral coronal suture synostosis

B. bilateral lambdoid suture synostosis

C. metopic suture synostosis

D. positional plagiocephaly

E. sagittal suture synostosis

page 822
2009 PREP SA on CD-ROM

Critique: 247 Preferred Response: A


Craniosynostosis, the premature fusion of one or more cranial sutures, may have a pre- or postnatal
onset. The causes are varied and include mechanical forces, suboptimal brain growth, gene mutations,
and metabolic derangements. Craniosynostosis may be primary, in the case of a genetic cause, or
secondary, as can be seen with vitamin D deficiency or hyperthyroidism.
Sutural formation and skull growth are exceedingly complex. Histologically, craniosynostosis begins
at a single point and spreads along the suture. Skull growth typically occurs in a direction perpendicular
to each of the major sutures (Item C247A).
The infant described in the vignette demonstrates a short anteroposterior (AP) dimension to the skull
and a wide biparietal dimension with flattening of the forehead. These findings are consistent with
bilateral coronal synostosis, in which growth of the skull perpendicular to the coronal sutures is limited
(Item C247B). Coronal suture synostosis may be isolated but frequently is associated with large thumbs
or great toes and often is due to dominant mutations in fibroblast growth factor receptor (FGFR) genes.
For infants who have this presentation, it is imperative to take a family history to determine if there are
similarly affected relatives.
Metopic suture synostosis results in trigonocephaly, or a "keel-shaped" skull (Item C247C). Sagittal
suture synostosis causes the head shape to be scaphocephalic (long in the AP dimension and narrow
in the biparietal dimension) (Item C247D).
Posterior plagiocephaly (flattening of one or both sides of the occiput) that results from premature
closure of the lambdoid suture(s) is rare and often is confused with positional plagiocephaly (Item
C247E). Since the advent of the recommendation that infants be placed on their backs to sleep, there
has been a dramatic increase in positional plagiocephaly, which is characterized by unilateral flattening
of the occiput and compensatory prominence of the forehead on the same side. Positional
plagiocephaly can be ameliorated by the use of a helmet molded to fit the infant's skull; ideally, this
therapy should be initiated prior to 9 months of age.

References:

Cohen MM Jr. Fibroblast growth factor receptor mutations. In: Cohen MM Jr, MacLean RE, eds.
Craniosynostosis: Diagnosis, Evaluation, and Management. 2nd ed. New York, NY: Oxford University
Press; 2000:77-94

Cohen MM Jr. History, terminology, and classification of craniosynostosis. In: Cohen MM Jr, MacLean
RE, eds. Craniosynostosis: Diagnosis, Evaluation, and Management. 2nd ed. New York, NY: Oxford
University Press; 2000:103-111

Robin NH, Falk MJ, Hlademan-Englert CR. FGFR-related craniosynostosis syndromes. GeneReviews.
2007. Available at:
http://www.geneclinics.org/servlet/access?db=geneclinics&site=gt&id=8888891&key=xAcWBcrjmZrVo&
gry=&fcn=y&fw=MR4U&filename=/profiles/craniosynostosis/index.html

page 823
2009 PREP SA on CD-ROM

Critique: 247

Skull growth normally occurs in a direction perpendicular to each of the


sutures. (Courtesy of A. Johnson)

page 824
2009 PREP SA on CD-ROM

Critique: 247

Premature fusion of the coronal sutures (blue) results in brachycephaly, a


prominent and flattened frontal bone, flattening of the occiput, and anterior
displacement of the vertex. (Courtesy of A. Johnson)

page 825
2009 PREP SA on CD-ROM

Critique: 247

Metopic synostosis (blue) results in a triangular-shaped forehead and bossing


of the parieto-occipital regions. (Courtesy of A. Johnson)

page 826
2009 PREP SA on CD-ROM

Critique: 247

Sagittal suture synostosis (blue) causes the skull to be elongated


(scaphocephaly). (Courtesy of A. Johnson)

page 827
2009 PREP SA on CD-ROM

Critique: 247

In positional plagiocephaly (left), when viewed from above, the head has a
parallelogram shape, with unilateral occipitoparietal flattening, displacement
of the ipsilateral ear anteriorly, and bossing of the ipsilateral frontal skull. In
contrast, in unilateral lambdoidal synostosis (right, [blue]), the head has a
trapezoidal shape, with unilateral occipital flattening, posterior displacement
of the ipsilateral ear, and bossing of the contralateral frontal skull. (Courtesy
of A. Johnson)

page 828
2009 PREP SA on CD-ROM

Question: 248
The parents of a 14-year-old girl are concerned about her weight loss. Her weight today is 20 lb less
than a documented weight obtained 1 year ago at her camp physical examination. She complains of
frequent nausea, decreased appetite, and early satiety, even after eating very small portions. She has
no vomiting or diarrhea, but frequent constipation. She complains of increased fatigue but is still able to
participate in diving 5 days a week. She is doing well in school academically. She attained menarche at
12 years of age and had monthly periods for about 18 months, but she has had no menses for the past
7 months. She has been a vegetarian for the past 18 months and feels she is at a good weight
currently. On physical examination, her body mass index is 17.0. Her urine pregnancy test result is
negative.

Of the following, the MOST likely diagnosis is

A. anorexia nervosa

B. depression

C. hypothalamic tumor

D. hypothyroidism

E. inflammatory bowel disease

page 829
2009 PREP SA on CD-ROM

Critique: 248 Preferred Response: A


The diagnosis of an eating disorder is not always straightforward, but the decreased appetite and early
satiety, large weight loss, amenorrhea, and disturbed body image with a low body mass index reported
for the girl in the vignette strongly suggest anorexia nervosa. Participating in a body-conscious sport
such as diving, becoming a vegetarian as a way to restrict the diet, and the presence of constipation
also may be associated with this condition.
The lack of diarrhea or hematochezia makes inflammatory bowel disease less likely. Weight loss,
doing well in school, and maintaining a strong exercise program are uncommon in hypothyroidism.
Depression can accompany an eating disorder, but the early satiety, constipation, amenorrhea, and
good school performance are unlikely to occur if depression is the sole diagnosis. A hypothalamic
tumor typically presents with neurologic symptoms or signs of increased intracranial pressure.
A limited laboratory evaluation that includes a complete blood count; metabolic panel; urinalysis;
measurement of free thyroxine, thyroid-stimulating hormone, luteinizing hormone, follicle-stimulating
hormone, and prolactin; and electrocardiography may be useful in ruling out other possible organic
causes of this patient's weight loss and symptoms.

References:

Fisher M. Treatment of eating disorders in children, adolescents, and young adults. Pediatr Rev.
2006;27:5-16. Available at: http://pedsinreview.aappublications.org/cgi/content/full/27/1/5

Rome ES, Ammerman S, Rosen DS, et al. Children and adolescents with eating disorders: the state of
the art. Pediatrics. 2003;111:e98-e108. Available at:
http://pediatrics.aappublications.org/cgi/content/full/111/1/e98

Rosen DS. Eating disorders in children and young adolescents: etiology, classification, clinical features,
and treatment. Adolesc Med. 2003;14:49-59. Abstract available at:
http://www.ncbi.nlm.nih.gov/pubmed/12529190

page 830
2009 PREP SA on CD-ROM

Question: 249
You are evaluating an 8-month-old infant in preparation for administering chloral hydrate to perform a
sedated brainstem auditory evoked potentials test.

Of the following, a CONTRAINDICATION to proceeding with the planned sedation is

A. absence of intravenous access

B. history of delivery at 36 weeks estimated gestational age

C. ingestion of infant formula 3 hours ago

D. ingestion of apple juice 3 hours ago

E. no previous anesthetic administration

page 831
2009 PREP SA on CD-ROM

Critique: 249 Preferred Response: C


The goal of procedural sedation is to provide anxiolysis and analgesia and minimize patient movement.
Sedation should be viewed as a continuum; the American Academy of Pediatrics has defined four
levels of sedation. Minimal sedation (formerly anxiolysis) is defined as a drug-induced state in which the
patient retains the ability to respond normally to verbal commands, and cardiovascular and respiratory
functions are unaffected. Moderate sedation (formerly conscious sedation) is a drug-induced
depression of consciousness in which patients still should respond to verbal and physical stimulation.
Cardiovascular status is maintained, but the clinician must be able to recognize and respond to
potential airway compromise. Deep sedation is defined as further depression of the level of
consciousness, with partial or complete loss of protective airway reflexes and the need for assistance
with airway maintenance. General anesthesia is the deepest level of sedation that is characterized by
loss of consciousness and airway protective reflexes. Impairment of respiratory and cardiovascular
function is common.
Provision of safe and effective sedation requires appropriate patient selection, evaluation, and
preparation. Appropriate monitoring and resuscitation equipment, thorough documentation, and trained
personnel are essential. A contraindication to proceeding with the planned sedation for the patient in
the vignette is the ingestion of formula 3 hours ago, which increases the risk of aspiration due to
potential loss of airway reflexes. Children should not ingest formula or food within 6 hours or human
milk within 4 hours of sedation. Clear liquids such as juice are allowed up to 2 hours prior to a planned
elective sedation and may help reduce gastric contents. Neither the history of birth at 36 weeks
estimated gestational age nor the lack of previous anesthetic administration are contraindications, but
the preanesthetic history should explore any complications at birth as well as any family history of
difficulty with sedation or anesthesia. Because preterm infants are at increased risk of apnea and
bradycardia following sedation until they have achieved a postconceptual age of 52 weeks, they
deserve additional monitoring. Pediatric sedations frequently are achieved with oral medications, but
personnel and equipment to obtain urgent intravenous access should be readily available.

References:

American Academy of Pediatrics, American Academy of Pediatric Dentistry, Coté CJ, Wilson S, AAP
Work Group on Sedation. Guidelines for monitoring and management of pediatric patients during and
after sedation for diagnostic and therapeutic procedures: an update. Pediatrics. 2006;118:2587-2602.
Available at: http://pediatrics.aappublications.org/cgi/content/full/118/6/2587

Koh JL, Palermo T. Conscious sedation: reality or myth? Pediatr Rev. 2007:28:243-248. Available at:
http://pedsinreview.aappublications.org/cgi/content/full/28/7/243

Wetzel R. Anesthesia and perioperative care. In: Kliegman RM, Behrman RE, Jenson HB, Stanton BF,
eds. Nelson Textbook of Pediatrics. 18th ed. Philadelphia, Pa: Saunders Elsevier; 2007:460-474

page 832
2009 PREP SA on CD-ROM

Question: 250
You observe a child entering the waiting room, accompanied by her mother. She looks at the
receptionist and says "Hi." While holding her doll, the child turns to her mother and says "juice." The
mother gives her a cup of juice, and the child says "doll" and tries to give the doll a drink. The mother
shakes her head, and the child says "no." The child then points to her own mouth, smiles, and says
"mouth." The mother takes a tissue to clean the doll’s face. The child says "me" and begins to imitate
her mother’s action with another wipe. The child looks at her mother, says "ma ma," and gives her
mother a hug.

Of the following, these developmental milestones are MOST typical for a child whose age is

A. 12 months

B. 15 months

C. 18 months

D. 24 months

E. 30 months

page 833
2009 PREP SA on CD-ROM

Critique: 250 Preferred Response: C


The developmental milestones described in the vignette are appropriate for a child of 18 months of age.
A healthy 18-month-old child may speak 7 to 10 words spontaneously and correctly, use words for
wants or needs, identify one or more body parts, and imitate household tasks. Typically developing
infants of 12 months of age can understand that a particular set of sounds represents a certain object or
action and may be just beginning to say their first words. A 15-month-old child can say four to six
specific words, use jargon, and indicate some desire or need by pointing. An 18-month-old child may
have a vocabulary of 20 to 29 words and will point to four body parts on request. A 24-month-old child
typically has a vocabulary of more than 50 words, is beginning to use two- to three-word sentences, and
will point to seven body parts and name at least one part. A 30-month-old child can speak in eight- or
nine-word sentences, identify his or her sex, name seven body parts, and relate events that occurred 2
to 3 days ago.
When evaluating a child's language, cognitive, and behavioral development, the clinician needs to
determine if the language development is appropriate to the cognitive development and if there are any
atypical social behaviors.

References:

Dedrick C. Developmental milestones. Developmental Behavioral Pediatrics Online. 2005. Available at:
http://dbpeds.org/articles/detail.cfm?TextID=%20701

Knobloch H, Stevens FM, Malone AF. The revised developmental stages. In: Manual of Developmental
Diagnosis. Albany, NY: Developmental Evaluation Materials, Inc; 1987:17-120

Page Glascoe F. Developmental screening. In: Parker S, Zukerman B, Augustyn M, eds.


Developmental and Behavioral Pediatrics: A Handbook for Primary Care. 2nd ed. Philadelphia, Pa:
Lippincott Williams & Wilkins; 2005:41-50

Whitaker T, Palmer F. The developmental history. In: Accardo PJ. Capute & Accardo's
Neurodevelopmental Disabilities in Infancy and Childhood. Volume I: Neurodevelopmental Diagnosis
and Treatment. 3rd ed. Baltimore, Md: Paul H. Brookes Publishing Co; 2008:297-310

page 834
2009 PREP SA on CD-ROM

Question: 251
The hospital laboratory calls your office to tell you that the rapid plasma reagin (RPR) test on the cord
blood of a newborn you saw yesterday in the hospital is positive at 1:4. You recall that the physical
examination findings for the infant were normal.

Of the following, the MOST appropriate approach to this infant is to

A. begin penicillin therapy

B. obtain further maternal history

C. place the infant in contact isolation

D. repeat the RPR in 3 months

E. report the case to the Department of Health

page 835
2009 PREP SA on CD-ROM

Critique: 251 Preferred Response: B


Two serologic tests are used routinely to screen a patient for syphilis. The rapid plasma regain (RPR)
and the Venereal Disease Research Laboratory (VDRL) tests both are based on the serologic detection
of immunoglobulin G (IgG) antibody titers against lipoidal antigen from Treponema pallidum. These
tests are inexpensive, provide rapid results, and can be quantitated. The quantitation tends to correlate
with disease activity. Results of both of these nontreponemal screening tests should be confirmed with
a treponemal test such as the fluorescent treponemal antibody absorption (FTA-ABS) or the T pallidum
particle agglutination (TP-PA). The treponemal tests are specific for the diagnosis of syphilis, are
nonquantitative, and usually remain positive for life.
Maternal passage of IgG antibody begins after 28 weeks of pregnancy. Therefore, results of cord
blood testing of a newborn, such as in the infant described in the vignette, reflect the mother's status.
Because the infant has a positive cord blood titer at 1:4 without signs or symptoms of congenital
syphilis, more maternal history is required. The RPR or VDRL can be falsely positive due to
concomitant viral infections (eg, Epstein-Barr virus), connective tissue disease, malignancy, intravenous
drug abuse, endocarditis, laboratory error, contamination of the specimen by Wharton jelly when cord
blood is used, or even pregnancy, thus necessitating the need for more history.
Until more history is obtained, there is no indication for penicillin therapy. Unless the mother has a
history of untreated syphilis, there is no indication to make a report to the health department. Newborns
who have open skin lesions or snuffles (chronic rhinorrhea) (Item C251) due to syphilis can be
contagious, but because the patient in the vignette is asymptomatic, there is no need for isolation.
Waiting 3 months and repeating the testing is not the best approach because if the mother does have
syphilis without a history of treatment, the infant requires therapy before leaving the hospital. For
patients who acquire syphilis through sexual contact, a positive RPR or VDRL with a positive
confirmatory FTA-ABS or TP-PA confirms the diagnosis.

References:

American Academy of Pediatrics. Syphilis. In: Pickering LK, Baker CJ, Long SS, McMillan JA, eds. Red
Book: 2006 Report of the Committee on Infectious Diseases. 27th ed. Elk Grove Village, Ill: American
Academy of Pediatrics; 2006:631-644

Hyman EL, Adam HM. In brief: syphilis. Pediatr Rev. 2006;27:37-39. Available at:
http://pedsinreview.aappublications.org/cgi/content/full/27/1/37

page 836
2009 PREP SA on CD-ROM

Critique: 251

Rhinorrhea (snuffles) in an infant who has congenital syphilis. (Courtesy of


M. Rimsza)

page 837
2009 PREP SA on CD-ROM

Question: 252
You are speaking to a group of neonatal nurses about the laboratory methods that can be used to make
the diagnosis of human immunodeficiency virus infection/acquired immune deficiency syndrome in
high-risk infants.

Of the following, the test that is MOST likely to confirm the diagnosis is

A. cord blood culture

B. DNA polymerase chain reaction

C. neonatal specific immunoglobulin

D. p24 antigen

E. RNA polymerase chain reaction

page 838
2009 PREP SA on CD-ROM

Critique: 252 Preferred Response: B


Diagnosing human immunodeficiency virus (HIV) infection in neonates and infants, especially in the
perinatal period, is complicated by the prolonged persistence of maternal antibody following delivery.
Standard serologic tests for the detection of HIV are not reliable until infants are approximately 18
months of age. In the neonatal period, infected infants produce only small amounts of HIV-specific
immunoglobulin G antibodies to a restricted number of antigens, and these antibodies are obscured by
maternal antibody, making them very difficult to measure.
The preferred test for the diagnosis of HIV infection in infants is HIV nucleic acid detection by
polymerase chain reaction (PCR) assay of DNA extracted from peripheral blood mononuclear cells.
About 30% of infants who are infected with HIV have a positive DNA PCR assay result in samples
obtained before 48 hours of age. A positive result identifies infants who were infected in utero.
Approximately 93% of infected infants have detectable HIV DNA by 2 weeks of age, and 100% of
infected infants have positive HIV DNA PCR assay results by 1 month of age. A single HIV DNA PCR
assay has a sensitivity of 95% and a specificity of 97% on samples collected from infants 1 to 36
months of age. The HIV DNA PCR assay is more sensitive on a single assay than viral culture.
Several alternative diagnostic methods have been suggested. Viral isolation by culture is expensive,
has limited availability, and requires up to 28 days for positive results. This test no longer is
recommended and has been replaced by the DNA PCR assay. The p24 antigen detection test is
substantially less sensitive than HIV DNA PCR assay or culture. Results may vary, and the test is not
recommended. HIV RNA PCR is not recommended for routine testing of infants and children younger
than 18 months of age because a negative result does not exclude HIV infection definitely, especially if
infection is acquired at the time of delivery, when HIV RNA concentrations in the plasma may be quite
low.

References:

American Academy of Pediatrics. Human immunodeficiency virus infection. In: Pickering LK, Baker CJ,
Long SS, McMillan JA, eds. Red Book: 2006 Report of the Committee on Infectious Diseases. 27th ed.
Elk Grove Village, Ill: American Academy of Pediatrics; 2006:378-401

Maldarelli F. Diagnosis of human immunodeficiency virus infection. In: Mandell GL, Bennett JE, Dolin R,
eds. Mandell, Douglas, and Bennett's Principles and Practice of Infectious Diseases. 6th ed.
Philadelphia, Pa: Elsevier Churchill Livingstone; 2005:1506-1526

Pollack H, Zhan MX, Ilmet-Moore T, Ajuang-Simbiri K, Krasinski K, Borkowsky W. Ontogeny of


anti-human immunodeficiency virus (HIV) antibody production in HIV-1-infected infants. Proc Natl Acad
Sci USA. 1993;90:2340-2344. Available at: http://www.pnas.org/cgi/reprint/90/6/2340

Read JS and the Committee on Pediatric AIDS. Diagnosis of HIV-1 infections in children younger than
18 months in the United States. Pediatrics. 2007;120:e1547-e1562. Available at:
http://pediatrics.aappublications.org/cgi/content/full/120/6/e1547

page 839
2009 PREP SA on CD-ROM

Question: 253
The parents of a 3-year-old boy who has polyuria and polydipsia ask if anything can be done for their
child’s symptoms and what the prognosis is for toilet training. After confirming a normal serum glucose
value and a negative urine culture, you arrange for a water deprivation test at the hospital. The test
begins at 9 am, and assessments are made hourly. At 0900, the patient weighs 14.1 kg, the serum
osmolality is 290 mOsm/kg H2O, and the urine osmolality is 120 mOsm/kg H2O. The measurements
made over the course of the test are summarized in Item Q253. Per protocol, the patient is given no
food or fluids intravenously or orally. Aqueous vasopressin is administered subcutaneously at 1101,
immediately after the 1100 laboratory samples are taken. After 4 hours, the test is stopped, and the
patient is allowed to drink to prevent hypovolemia.

Of the following, the MOST likely diagnosis is

A. central diabetes insipidus

B. nephrogenic diabetes insipidus

C. no abnormalities in urinary concentrating ability

D. primary (psychogenic) polydipsia

E. inconclusive results necessitating repeat testing

page 840
2009 PREP SA on CD-ROM

Question: 253

page 841
2009 PREP SA on CD-ROM

Critique: 253 Preferred Response: B


When a child presents with polyuria and polydipsia, it is imperative for the clinician to exclude diabetes
mellitus. Once excluded by a normal glucose value, as occurred in the vignette, the clinician should test
for normal urinary concentrating ability, which usually is measured on a first morning urine specimen,
making it difficult to obtain in the incontinent child. In this situation, a water deprivation test may be
necessary to assess urinary concentrating ability appropriately. The test is conducted in the hospital
due to the risks of dehydration in the patient who has diabetes insipidus (DI) and cannot concentrate
urine adequately. Younger children who are not yet toilet trained require a urinary catheter to monitor
urine output and allow for urine osmolar testing.
In a healthy person, the water deprivation test results in weight loss, increased serum osmolality,
and progressive increase in urine osmolality. The effective urinary concentrating ability demonstrates
normal antidiuretic hormone (ADH) production and normal renal response to hypovolemia. In contrast,
when the 4-hour test was administered to the boy in the vignette, he lost weight, developed a
progressive increase in serum osmolality, but maintained a fixed urine osmolality. These findings are
consistent with DI. A trial of aqueous vasopressin (VP) was given in an effort to differentiate between
central DI (VP-responsive) and nephrogenic DI (VP-unresponsive). The patient's lack of response to VP
indicates that he has nephrogenic DI (NDI).
NDI is an X-linked disorder in 90% of cases and is caused by mutations in the V2R gene (located on
the X chromosome), which encodes for the arginine vasopressin type 2 receptor (V2R) that is
expressed on the basolateral membrane of the collecting tubules. When circulating antidiuretic
hormone is presented to the V2 receptor, water channels (aquaporin-2 [AQP2]) translocate to the
luminal surface of the collecting tubule, resulting in water reabsorption. Ten percent of NDI cases are
autosomal recessive and caused by mutations in the AQP2 gene.
Alternative clinical possibilities in the setting of polyuria and polydipsia include primary (psychogenic)
polydipsia, a diagnosis that is excluded readily by a water deprivation test. Affected patients have
normal ADH production and normal tubular response. Accordingly, when the plasma osmolality reaches
295 mOsm/kg H2O, the urine osmolality exceeds 800 mOsm/kg H2O. Results of the water deprivation
test for the boy in the vignette are conclusive, demonstrating an abnormality in urinary concentrating
ability that failed to respond to volume depletion (excluding primary polydipsia) and aqueous
vasopressin (excluding central DI).

References:

Goodyer P. Disorders of tubular transport. In: Kher KK, Schnaper HW, Makker SP, eds. Clinical
Pediatric Nephrology. 2nd ed. London, England: Informa Healthcare; 2007:317-336

Knoers NVAM, Monnens LAH. Nephrogenic diabetes insipidus. In: Avner ED, Harmon WE, Niaudet P,
eds. Pediatric Nephrology. 5th ed. Philadelphia, Pa: Lippincott Williams & Wilkins; 2004:777-787

Rose BD, Post TW. Hyperosmolal states-hypernatremia. In: Clinical Physiology of Acid-base and
Electrolyte Disorders. 5th ed. New York, NY: McGraw-Hill Medical Publishing Division; 2001:746-793

page 842
2009 PREP SA on CD-ROM

Question: 254
An 18-year-old boy who has mild persistent asthma presents to the emergency department with a
2-week history of coughing and wheezing that has not improved with twice-daily use of his beta2
agonist metered dose inhaler (MDI). On physical examination, the teenager is breathing comfortably but
often coughs and has audible expiratory wheezing. His vital signs are appropriate for age, but a room
air pulse oximetry reading is 95%. Chest radiography shows some peribronchial streaking but no
infiltrate, no consolidation, and a normal cardiac silhouette. His only other medication is a medium-dose
inhaled corticosteroid.

Of the following, the MOST appropriate management step is to

A. begin a course of outpatient oral steroids

B. change his steroid inhaler to one that combines a steroid and long-acting beta2 agonist

C. double his steroid MDI dose

D. increase his rescue beta2 agonist to every 4 hours

E. observe the patient

page 843
2009 PREP SA on CD-ROM

Critique: 254 Preferred Response: A


The primary goal of managing asthma exacerbations is early treatment, with special attention paid to
infants and those who are at high risk for fatal asthma. Early treatment strategies include use of a
written asthma plan, recognition of symptoms, appropriate intensification of therapy, and removal of
possible allergens or irritants.
The teenager described in the vignette has a history of persistent asthma and presents with a typical
asthma exacerbation of moderate severity. The 2007 Expert Panel Report 3: Guidelines for the
Diagnosis and Management of Asthma (available at www.nhlbi.nih.gov/guidelines/asthma/asthgdln.pdf)
provides an excellent resource to determine the appropriate course of action for such patients. Two
primary therapies advocated for asthma exacerbations are short-acting beta2 agonists (SABAs) and
oral corticosteroids (evidence level A). For patients who have mild asthma exacerbations, a 24- to
48-hour course of a SABA is recommended. Patients who respond completely to this intervention need
no further therapy. Patients who have moderate, severe, or life-threatening exacerbations should seek
medical attention, and corticosteroids should be initiated promptly.
Neither increasing the patient's SABA to every 4 hours nor observing him is an appropriate option
because he has had symptoms for 2 weeks without improvement. Initiation of a combination
steroid/long-acting beta2 agonist inhaler (eg, fluticasone/salmeterol or budesonide/formoterol) would
not benefit him because it would only add a longer-duration beta2 agonist to his current medication
regimen.
Some clinicians advocate doubling the inhaled steroid dose at the onset of asthma symptoms.
However, this approach has not been shown to be effective in reducing the severity or preventing
progression of exacerbations. An emergency department study has supported quadrupling the dose of
an inhaled corticosteroid for 7 days at the onset of worsening symptoms. This may obviate the need for
oral steroids, but does not have the level of evidence as is present for oral corticosteroids.

References:

Harrison TW, Oborne J, Newton S, Tattersfield AE. Doubling the dose of inhaled corticosteroid to
prevent asthma exacerbations: randomised controlled trial. Lancet. 2004;363:271-275. Abstract
available at: http://www.ncbi.nlm.nih.gov/pubmed/14751699

Keeley D, McKean M. Asthma and other wheezing disorders in children. BMJ Clinical Evidence. 2006.
Available for subscription at: http://clinicalevidence.bmj.com/ceweb/conditions/chd/0302/0302.jsp#Q1

National Heart, Lung, and Blood Institute. National Asthma Education and Prevention Program. Expert
Panel Report 3: Guidelines for the Diagnosis and Management of Asthma. 2007. Available at:
www.nhlbi.nih.gov/guidelines/asthma/asthgdln.pdf

page 844
2009 PREP SA on CD-ROM

Question: 255
A 15-year-old boy presents to the emergency department after falling off of his skateboard. He was
skating downhill at high speed when he hit a bump and fell off his board. He fell onto his left shoulder
and struck his abdomen on the curb. He now complains of left shoulder pain. On physical examination,
his heart rate is 110 beats/min, respiratory rate is 24 breaths/min and shallow due to pain, and blood
pressure is 130/75 mm Hg. He refuses to move his left shoulder. His lung sounds are clear, and his
abdomen is diffusely tender. Radiographs of his left shoulder are reported as normal. You order an
abdominal computed tomography (CT) scan.

Of the following, the CT scan is MOST likely to demonstrate

A. abdominal aortic dissection

B. duodenal hematoma

C. pancreatic transection

D. retroperitoneal hemorrhage

E. splenic laceration

page 845
2009 PREP SA on CD-ROM

Critique: 255 Preferred Response: E


Blunt abdominal trauma in children is second only to head injury as a cause of injury-related mortality.
The most commonly injured intra-abdominal organs are the liver and spleen, comprising 60% of
childhood blunt abdominal injuries, with renal, pancreatic, and bowel injuries occurring much less
frequently. Most pediatric solid organ injuries are diagnosed by computed tomography (CT) scan,
although ultrasonography is being used increasingly as an initial screening test. Most solid organ
injuries are managed nonoperatively.
The patient described in the vignette demonstrates classic signs and symptoms of a splenic injury,
with left upper quadrant tenderness and left shoulder pain due to diaphragmatic irritation. Blunt force
from the fall onto the curb is the likely mechanism. The CT scan is the diagnostic modality of choice
because it can demonstrate the injury and extent of splenic damage (Item C255). With the exception of
patients who have massive splenic disruption and hemodynamic instability, spleen preservation is
preferred to splenectomy, with 90% to 98% of patients recovering fully without surgery.
Although thoracic aortic dissection may be seen following blunt chest trauma (steering wheel versus
chest) or a high-velocity deceleration mechanism, abdominal aortic dissection is an uncommon injury,
especially in children, following blunt trauma.
Duodenal hematomas typically are seen after a focal blow to the upper abdomen (the so-called
"handlebar injury") or after a deceleration mechanism in a restrained passenger following a high-speed
motor vehicle crash. Accounting for only 2% of blunt abdominal injuries in one study, these injuries are
notoriously difficult to diagnose. They often are missed on initial, noncontrast abdominal CTs, and
affected patients may present days after the initial trauma with vomiting and diffuse abdominal pain. A
high index of suspicion related to the mechanism of injury is the key to timely diagnosis.
Pancreatic injuries are uncommon and occur from similar mechanisms as intestinal injuries. These
injuries also may be missed on initial CT scans. Elevated pancreatic enzyme values and persistent
upper abdominal pain should prompt evaluation for a pancreatic injury. Conservative treatment in
patients without pancreatic duct injury usually leads to full recovery.
Retroperitoneal hemorrhage may result from significant renal trauma. Physical examination findings
include flank ecchymoses and back pain. A CT scan should demonstrate both the presence of
retroperitoneal blood as well as its source in the trauma victim.

References:

Holmes JF, Sokolove PE, Brant WE, et al. Identification of children with intra-abdominal injuries after
blunt trauma. Ann Emerg Med. 2002;39:500-509. Abstract available at:
http://www.ncbi.nlm.nih.gov/pubmed/11973557

Wegner S, Colletti JE, Van Wie D. Pediatric blunt abdominal trauma. Pediatr Clin North Am.
2006;53:243-256. Abstract available at: http://www.ncbi.nlm.nih.gov/pubmed/16574524

page 846
2009 PREP SA on CD-ROM

Critique: 255

Axial computed tomography scan of the abdomen shows disruption of the


spleen with a large hematoma. (Courtesy of D. Mulvihill)

page 847

Potrebbero piacerti anche